Nothing Special   »   [go: up one dir, main page]

Untitled

Download as pdf or txt
Download as pdf or txt
You are on page 1of 340

Third Edition

Mayur Arun Kulkarni MD, DNB (Radiology)


Director
Shree Diagnostics and Shashwat Imaging Clinic, Pune, Maharashtra

Saurabh S Patil MD (Radiology)


Director
Sonomed Clinic, Alibag, Maharashtra

Amit M Shetty
MD, DNB (Radiology), FRCR (London), EDIR, Fellow MSK Imaging, MGH (Boston, USA)
Chief Consultant
Nutek Medical Centre, Andheri, Mumbai, Maharashtra

CBS Publishers & Distributors Pvt Ltd


• New Delhi • Bengaluru • Chennai • Kochi • Kolkata • Mumbai
• Hyderabad • Nagpur • Patna • Pune • Vijayawada
DISCLAIMER
This book contains questions based on important topics
frequently asked in previous years National Level PG
Entrance Examinations & State Level Examinations in India.
Often repeated topics and sub-topics have been included
for students’ benefit. We do not claim that these questions
are exact or similar to questions asked in any recent
examinations in India. If any such similarity is found, it is
purely coincidental and by chance.

ISBN: 978-81-945783-4-5

Copyright © Authors & Publishers

Third Edition: 2020

All rights reserved. No part of this book may be reproduced or transmitted in any form or by any means, electronic or
mechanical, including photocopying, recording, or any information storage and retrieval system without permission, in
writing, from the authors and the publishers.

Published by Satish Kumar Jain and produced by Varun Jain for


CBS Publishers & Distributors Pvt Ltd
4819/XI Prahlad Street, 24 Ansari Road, Daryaganj, New Delhi 110 002, India.
Ph: +91-11-23289259, 23266861, 23266867  Website: www.cbspd.com
Fax: 011-23243014
e-mail: delhi@cbspd.com; cbspubs@airtelmail.in.
Corporate Office: 204 FIE, Industrial Area, Patparganj, Delhi 110 092
Ph: +91-11-4934 4934 Fax: 4934 4935
e-mail: feedback@cbspd.com; bhupesharora@cbspd.com

Branches
• Bengaluru: Seema House 2975, 17th Cross, K.R. Road Banasankari 2nd Stage, Bengaluru 560 070, Karnataka
Ph: +91-80-26771678/79 Fax: +91-80-26771680 e-mail: bangalore@cbspd.com
• Chennai: 7, Subbaraya Street, Shenoy Nagar, Chennai 600 030, Tamil Nadu
Ph: +91-44-26680620, 26681266 Fax: +91-44-42032115 e-mail: chennai@cbspd.com
• Kochi: 68/1534, 35, 36-Power House Road, Opp. KSEB, Cochin-682018, Kochi, Kerala
Ph: +91-484-4059061-65 Fax: +91-484-4059065 e-mail: kochi@cbspd.com
• Kolkata: 6/B, Ground Floor, Rameswar Shaw Road, Kolkata-700 014, West Bengal
Ph: +91-33-22891126, 22891127, 22891128 e-mail: kolkata@cbspd.com
• Mumbai: 83-C, Dr E Moses Road, Worli, Mumbai-400018, Maharashtra
Ph: +91-22-24902340/41 Fax: +91-22-24902342 e-mail: mumbai@cbspd.com

Representatives

• Hyderabad +91-9885175004 • Patna +91-9334159340


• Pune +91-9623451994 • Vijayawada  +91-9000660880

Printed at:
From the Publisher’s Desk
Dear Readers,
I extend my warm welcome and convey my heartfelt thanks for appreciating the CBS Exam
Books for another successful year. It has been an amazing journey so far and I am highly
grateful for your support and cooperation to help us achieve various milestones in this whole
span of time. The mission with which we started in the year 2015 was to bring nothing but the
best of everything to our target audience and today I can proudly say that we have maintained
that standard and are committed to continue the same in future as well.
Every single title under the banner of CBS Exam Books has been developed and nurtured
like an infant. The authors and our entire team work day and night to bring the best in
everything for you. Be it content, presentation, social media contests and offers, we strive
to meet your expectations with every passing year. Your trust has motivated us to maintain
and upgrade ourselves during this period. I am extremely thankful to all our authors who are
the real pillars of the complete series of CBS Exam Books. The contributions of our esteemed
authors have laid the foundations of CBS Exam Books.
At this juncture, I can recall these lines by Drake,
“Sometimes it’s the journey that teaches you a lot about your destination”.
We have grown and changed with the passage of time to upgrade our ways of providing our readers with maximum benefits and
help them manage their time and efforts in effective manner. Previous year was the year of great achievements. Let me show you a
glimpse of our successful journey:
•• Most of the titles of CBS Exam Books received wide acceptance and recognition by the readers of proving their usefulness and
supremacy. To mention a few, SARP Anatomy, CRISP, Surgery Sixer, Complete Review of Pathology, Conceptual Review of
Pharmacology, SOCH, Forensic Medicine, Complete Review of Medicine, Conceptual Review of PSM, MICRONS, My PGMEE Notes,
AIIMS MedEasy, and PRIMEs. With your constant support and our consistent efforts, I am sure that we will together witness an
exponential acceptance of all CBS Exam Books in coming future as well.
•• The presence of CBS Exam Books has broadened through our various social media platforms. We have received great appreciation
for our regular Facebook activities such as online test series, giveaways, scientific content for knowledge enhancement, authors’ live
sessions, and various contests, like Bid 2 Win, Fastest Finger First, Book Fair and Facebook Community Awards. Join us on all these
platforms to avail and enjoy our exciting offers and benefits.

A book is incomplete if it does not have the right readers. We value you and your feedback. Please share your feedback and suggestions
directly with me at bhupesharora@cbspd.com. We promise to deliver in our books, what you desire to see.
I would like to sum up with these eternal lines of Robert Frost:
Woods are lovely dark and deep,
But I have promises to keep.
And miles to go before I sleep,
And miles to go before I sleep!
Wishing you success in all your endeavors!

Bhupesh Arora
Vice President – Publishing & Marketing
(PGMEE and Nursing Division)
Email: bhupesharora@cbspd.com
Mobile: (+91) 9555590180
Preface
RADIOLOGY is a black-and-white, obscure terrain of difficult way to combat them is to mug them up. This makes the subject very
images and modalities for each and every PG aspirant. The irony of difficult to remember and ultimately daunting and intimidating.
this subject is that it is considered as most neglected subject during A few examples of such factual topics are:
the UG curriculum and is never actually taught to students. The ƒƒ General Radiology topics including Physics
scenario changes suddenly when students clear their UG course and ƒƒ Radiation exposure values/adverse effects of Radiation/Radiation
take a step forward to the PG entrance exams. Examiners now assume Syndromes
(wrongfully so) that students know everything about all imaging ƒƒ Silhouette Sign
modalities and confront them with X-ray, USG, CT, MRI, PET scan ƒƒ Named Signs and Appearances
images... and the result is... well, unwarranted and extreme phobia of ƒƒ Investigation-of-Choice/Gold Standard Investigation
this subject! ƒƒ Contrast Media
The primary intent of this book is to address this injustice meted ƒƒ Diagnostic Radionuclide Studies
out to students. By the end of this book and with every meticulous Fact: Radiology is a Conceptual subject with most of the facts
revision that you perform, this huge “GAP” between what you are based on beautiful Core Concepts. Understanding these Core
taught and what is expected from you in your exams – WILL NOT Concepts would help us remember all the factual topics very easily.
EXIST. This is a promise. This is the absolute truth! Believe it or not, almost all factual topics in
This treatise on Radiology, therefore, begins with a very truthful Radiology – including those listed above – are based on “Core Concepts”.
assumption that “UG students know very little about Radiology” The correct way to approach Radiology is to understand these concepts
– and this indeed is an actual fact. Therefore, our approach toward first and then practice them. In this way Radiology will not only become
Radiology would be a very fundamental approach starting from easy to understand and remember but also fun to study!
the very basics of modalities, images and their interpretation, and A few examples of this unique way of learning Radiology include:
extending up to their applications in diagnostic Radiology. We will
question each and every aspect of these black-and-white modalities Factual Topic Underlying Core Concept
with “Why? What? When? Where? How?” and eventually seek all the •• Sillhoutte Sign in Chest •• Differential Radiographic
answers until satisfaction. Radiology is Based on Density Theory
� Myth: Radiology is a Short Subject •• Radiological Syndromes are •• Law of Radiobiology
Look at the statistics: Based on
Radiology MCQs Total MCQs •• Each Diagnostic Radionuclide •• Physiology of the
Use is Based on Radionuclide
NEET - PG 12 300
•• Named Signs and •• Differential Radiographic
FMGE 10 200 Appearances in Radiology Density Theory
Thus statistically, 12/300 and 10/200 are in favor of Radiology being (at least 90% of them) are
Based on
called a short subject. But this is not true. Just speak to your colleagues
and seniors and they will let you know that subjects like Medicine, •• Investigation of Choice/Gold •• Basic Understanding of Each
Surgery, OBG, Pediatrics, Orthopedics and ENT have image-based Standard Investigations is Modality and its Physics
questions based on radiological investigations…. How can you leave Based on
them out? Thus the total number of radiology-related questions in
Although we speak and focus primarily on the PG Entrance
your examinations is now at least 15–20 thus making it a major chunk
examination (NEET/DNB/AIIMS/PGI/JIPMER, etc.), remember
of questions. Hence, we have made an attempt to make this book as
that as Doctors we have an entire lifetime of medical practice ahead.
comprehensive as possible with 1000+ original images, illustrations,
This examination is just the 1st step towards that journey and I hope
concept boxes and tips-and-tricks for your preparation.
that even beyond this examination, the book will surely help you in
Fact: Radiology is a major subject and needs to be studied your practice irrespective of the specialty you choose.
comprehensively considering the current examination pattern and
trends.
� Myth: Radiology is a Factual Subject
Mayur Arun Kulkarni
This is what most of us believe about this subject and hence we are Saurabh S Patil
afraid of it. We start dealing with these factual topics and the only Amit M Shetty
Acknowledgements
“Intelligence is the ability to adapt to change – Stephen Hawking” It is a great pleasure to present this third edition after the success
of previous editions of Conceptual Review of Radiology. The number of
Look at the AIIMS, NEET pattern exams in 2019-2020 and you
Radiology related questions asked in recent NEET exams and success
would realize how rapidly the exam pattern is changing. A smart
of students who referred to CRR speaks for itself. The efforts we took
teacher (and a student too) is the one who can recognize these
in shaping the third edition based on your inputs, was no mean task
changes early and incorporate them into the learning process. Gone
either. Lastly, it is the interest and affection shown by PG aspirants that kept
are the days when most of the questions were single line factual
questions. Now the questions are clinically oriented, with patient us motivated in finishing the book well within time. My best wishes
history, examination findings, investigation reports/images/videos to all students for their future.
and what not! Answering such questions require an integrated  Saurabh S Patil
clinical approach that now has to be practiced right from the 1st day My dear wife Dr Anusha Shetty was always there for support in this
of learning. Every topic in this 3rd edition begins with a recently super-busy time. My family, mom-dad and my brother Manish have
asked “Clinical Quiz” question with its solution discussed at the supported me unconditionally throughout this endeavor. Special
end of that topic. Another new highlight of this new edition is high- thanks to my boss Dr Sharad Sancheti for his motivation and
yield “Clinical Pearls” that will cover important topics in other encouragement. I must mention my dear friend Dr Pooja Deshpande
subjects related to a particular diagnosis. In addition to these we have – for being a close and special confidante – in testing times.
continued with our conceptual approach using the popular “Concept  Amit M Shetty
Boxes”, Mnemonics and an Image-Based Approach. The book is now
more concise, full of new updated images, recently asked questions We are extending our special thanks to Mr Satish Kumar Jain
and is truly a Clinical Integrated book for Radiology! (Chairman) and Mr Varun Jain (Managing Director), M/s CBS
This book is dedicated to the two little angels in my life Spruha Publishers and Distributors Pvt Ltd for their wholehearted support
and Hrida – the newest and cutest little addition to our family! With in publication of this book. We have no words to describe the role,
the completion of the book I now pledge to spend more time with efforts, inputs and initiatives undertaken by Mr Bhupesh Arora (Vice
them and indulge in the little pleasures of parenting. My dear wife President - Publishing & Marketing, PGMEE and Nursing Division) for
Kavita is my pillar of support. It is impossible being supportive to a helping and motivating us.
husband, who is in work-mode 24 × 7, for 365 days. I wonder how
she manages to do it. Thank you for being a solid lighthouse and We sincerely thank the entire CBS team for bringing out the book
making me realize the real priorities in life. I can teach and write with utmost care and attractive presentation. We would like to thank
and work tirelessly only because she covers up for me on all fronts! Dr Mrinalini Bakshi (Editorial Head & Content Strategist) for her
My parents have been a constant source of inspiration in my life. editorial support and Ms Nitasha Arora (Production Head & Content
Thank you both Aai-Baba for being so supportive. Shardul-Sharmila, Strategist), Dr Anju Dhir (Project Manager & Senior Scientific
Amey-Savita also deserve a special mention for their unconditional Coordinator), Mr Shivendu Bhushan Pandey (Senior Editor),
support throughout my career. My co-authors Dr Saurabh and Mr Ashutosh Pathak (Senior Proof Reader) and all the production
Dr Amit – my dear friends as well – are indeed the pillars of this project. team members Mr Chaman Lal, Mr Prakash Gaur, Mr Phool Kumar,
They have worked tirelessly and I admit, in fact more extensively Mr Bunty Kashyap, Ms Tahira Parveen, Ms Manorama Gupta,
than me, on this book and it was indeed impossible without their Ms Babita Verma, Mr Chander Mani, Mr Raju Sharma, Mr Manoj
selfless support. Dr Swapnil Yewalkar, my dear friend and partner at Chaudhary, Mr Vikram Chaudhary, Mr Manoj Malakar, Mr Arun
Shree Diagnostics and Shashwat Imaging Clinic has also supported Kumar and Mr Rahul Negi for devoting laborious hours in designing
me throughout this endeavor. Being a part of the Marrow team this and typesetting of the book.
year has brought a new vigor and enthusiasm in my teaching. I extend
We would like to acknowledge our ultimate source of inspiration –
my sincere thanks to Dr Deepu Sebin, Dr Shujad and the entire
our Students. It is for them that we can endure all the hectic travelling,
team at Marrow including my fellow esteemed faculties. Besides,
the nonstop extensive hours of teaching, the sleep deprivation and all
I am thankful to Dr Deepak Marwah sir, Dr Sarvajeet Singh sir,
the sacrifices of personal life – so that they get what they deserve in
Dr Aman Setiya sir, Ravi sir, Shivlok, Dr Ramana, Charles, Selva,
their lives.
Shiva, Venkatesh, ManiKandan, and Prakash.
 Mayur Arun Kulkarni This book is for you. Let’s rock Radiology – together!
Contents

RECENT PATTERN QUESTIONS 2020


Radiology – though a distinct specialty now – is indeed inseparable from the clinical sciences. Hence, we have taken some extra effort and
designed this book in a particular manner; grouping together related conditions in a logical order. This topic-wise table of contents will
not only help you cruise through this book when you study Radiology, but also when you study your other clinical subjects. Keep this book
handy when you study Medicine, Surgery, Pediatrics and OBG and easily find a specific condition in this table – complete your preparation
of any topic by going through its imaging findings then-and-there. Welcome to what we call the – Integrated learning experience!

Recent Pattern Questions 2020  ix ƒƒ Congenital Anomalies of Genitourinary System  170


COVID-19: High-Yield Snapshot  xii ƒƒ Prostate Imaging  173
ƒƒ Miscellaneous Gut  175
ƒƒ Multiple Choice Questions  179
1. IMAGE-BASED QUESTIONS 1-19 ƒƒ Explanations to Questions  181
2. IMAGING SIGNS IN RADIOLOGY 21-34 8. MUSCULOSKELETAL IMAGING 183-211
3. GENERAL RADIOLOGY 35-74 ƒƒ Bone and Joint Infection Bone Tumors  184
ƒƒ X-rays 36 ƒƒ Arthritis 190
ƒƒ Radiation Exposure, Protection and Guidelines  43 ƒƒ Bone Infections and Systemic Bone Disorders  196
ƒƒ Computed Tomography  46 ƒƒ Skeletal Dysplasia and Miscellaneous
ƒƒ Ultrasound Imaging  52 Musculoskeletal Imaging  203
ƒƒ MRI Basics  58 ƒƒ Multiple Choice Questions  207
ƒƒ Contrast Media in Radiology  66 ƒƒ Explanations to Questions  211
ƒƒ Multiple Choice Questions  72 9. CENTRAL NERVOUS SYSTEM IMAGING 213-244
ƒƒ Explanations to Questions  74 ƒƒ Stroke Imaging  214
4. RESPIRATORY SYSTEM 75-116 ƒƒ Head Injury  221
ƒƒ Normal Findings on a Chest Radiograph  76 ƒƒ CNS Infections  225
ƒƒ Silhouette Sign and Lung Infections  81 ƒƒ CNS Neoplasms  229
ƒƒ Pleural Abnormalities  87 ƒƒ White Matter Diseases, Phakomatoses and
ƒƒ Lobar Collapse and Airway Disorders  93 Miscellaneous 233
ƒƒ Mediastinum 97 ƒƒ Phakomatoses 234
ƒƒ Lung Tumors  102 ƒƒ Multiple Choice Questions  240
ƒƒ HRCT Thorax  106 ƒƒ Explanations to Questions  244
ƒƒ Miscellaneous Topics  108 10. WOMEN’S IMAGING 245-264
ƒƒ Multiple Choice Questions  112 ƒƒ Breast Imaging  246
ƒƒ Explanations to Questions  116 ƒƒ Obstetric Imaging  250
5. CARDIOVASCULAR IMAGING 117-129 ƒƒ Gynecologic Imaging 257
ƒƒ Imaging in Congenital Heart Disease  118 ƒƒ Multiple Choice Questions  261
ƒƒ Imaging in Acquired Heart Disease  122 ƒƒ Explanations to Questions  264
ƒƒ Imaging in Pulmonary and Aortic Diseases  124 11. RADIONUCLIDE IMAGING AND RADIOTHERAPY 265-283
ƒƒ Multiple Choice Questions  127 ƒƒ Radionuclide Imaging  266
ƒƒ Explanations to Questions  129 ƒƒ Radiotherapy 274
6. GASTROINTESTINAL TRACT IMAGING 131-162 ƒƒ Multiple Choice Questions  282
ƒƒ Peritoneal Abnormalities  132 12. HIGH-YIELD TOPICS 285-309
ƒƒ Imaging in Foregut  136 ƒƒ Tuberculosis 286
ƒƒ Small and Large Bowel Abnormalities  140 ƒƒ Hydatid Cyst  292
ƒƒ Hepatobiliary and Pancreatic Imaging  147 ƒƒ Skeletal Trauma  295
ƒƒ Miscellaneous Abdominal Imaging  154 ƒƒ Multiple Choice Questions  307
ƒƒ Multiple Choice Questions  157 ƒƒ Explanations to Questions  309
ƒƒ Explanations to Questions  161
13. NORMAL IMAGING ATLAS—BRAIN, CHEST AND ABDOMEN 311-322
7. GENITOURINARY TRACT IMAGING 163-181 ƒƒ Brain Sectional Anatomy  312
Answers
ƒƒ Urolithiasis Imaging  164 ƒƒ CT Chest Anatomy  316
ƒƒ Renal Parenchymal Abnormalities Including ƒƒ CT Anatomy Abdomen  320
Infections 166
ƒƒ Renal Masses  168 APPENDIX: Scientists and Important Discoveries in Radiology 323-323
vii
Recent Pattern Questions 2020

RECENT PATTERN QUESTIONS 2020


1. Identify the diagnosis in this child: 4. A 65-year-old female with chronic backache revealed the
following appearance on a lumbar radiograph. What is
the most likely cause?

A. Epiglottitis A. Paget’s disease B. Renal osteodystrophy


B. Adenoid enlargement C. Osteoporosis D. Ankylosing spondylitis
C. Acute laryngotracheobronchitis 5. A 10-month-old girl child presented with pain in
D. Retropharyngeal abscess abdomen, blood in stool and lump felt over abdomen. A
2. Identify the diagnosis: barium enema was done and revealed this image. What is
the most likely diagnosis?

A. Meckel’s diverticulum B. Volvulus


A. TGA B. TOF C. Intussusception D. Diverticulitis
C. TAPVC D. Ebstein’s anomaly
6. A 42-year-old patient with sudden onset headache, neck
3. A child with a known skeletal deformity was being rigidity without any obvious history of trauma. What is
evaluated and an angle was measured as shown. Can you the most likely diagnosis based on this CT image?
name the measurement being made here? 

A. Cobb’s angle B. Bohler’s angle A. Meningitis B. SAH


C. Pauwel’s angle D. Pott’s angle C. Intraparenchymal bleed D. SDH

Answers
1. C. Acute laryngotracheobronchitis 4. C. Osteoporosis
2. B. TOF 5. C. Intussusception
3. A. Cobb’s angle 6. B. SAH ix
7. Barium swallow was performed in a patient with dysphagia. 10. A HSG was performed for Infertility evaluation in a
What is the most likely diagnosis? patient. What is the most likely diagnosis?
CONCEPTUAL REVIEW OF RADIOLOGY

A. Normal study B. Cornual block


A. Achalasia cardia B. Esophageal stricture C. Hydrosalpinx D. Uterine synechiae
C. Carcinoma esophagus D. GERD 11. A patient being treated for infertility was treated with
8. Patient presented with neck swelling and regurgitation injection HMG following which she presented with
with gurgling sound when pressed over the neck. A abdominal distension and vomiting. Most likely diagnosis
barium swallow was performed and is revealed here. Most based on this USG image is:
likely diagnosis will be:

A. OHSS B. PCOS
A. Esophageal stricture B. Zenker’s diverticulum
C. Theca lutein cysts D. Ovarian torsion
C. Esophageal web D. Dysphagia lusoria
12. This appearance seen in M-mode USG thorax is suggestive
9. A patient complains of incomplete evacuation of urine
of:
and frequent micturition. Identify the diagnosis in this
image? 

A. Bulbar urethral stricture A. Stratosphere sign


B. Urethral block B. Sea shore sign
C. Urethral rupture C. A line sign
D. Urethral diverticulum D. Lung point identification sign

Answers
7. C. Carcinoma esophagus 10. C. Hydrosalpinx
8. B. Zenker’s diverticulum 11. A. OHSS
x 9. A. Bulbar urethral stricture 12. B. Sea shore sign
13. Following image of FAST in a trauma patient suggests: 16. Pulmonary plethora is a feature of:
A. TOF C. Right heart failure
B. Ebstein’s anomaly D. TAPVC
17. A patient with abdominal trauma was brought to the

RECENT PATTERN QUESTIONS 2020


Emergency Room. IV fluid were started and the patient was
found to be hemodynamically stable. The next step in the
management of this patient would be:
A. E-FAST C. Laparotomy
B. CE-CT D. CXR PA view
18. A 9-year-old female child with history of head ache and
visual disturbance. A CT image revealed a calcified solid-
cystic suprasellar lesion as show here. What could be your
possible diagnosis?

A. Positive FAST B. Negative FAST


C. Equivocal FAST D. None of the above
14. A 32-year-old patient undergoes a routine CXR PA view
for pre-employment screening. Realizing that she has
just missed her periods she does a urine pregnancy –
that comes as positive. A USG scan done on the same day
reveals that she is 5 weeks 6 days pregnant. Extremely
anxious, she comes to you for consultation. Best advice
you can give her is:

A. Pituitary macroadenoma B. Rathke’s cleft cyst


C. Craniopharyngioma D. Hypothalamic hamartoma
19. Which of the following is the etiological agent for the
given case below (Hemivertebra)?

A. Immediate termination of pregnancy


B. Mandatory invasive testing for genetic defects
C. Counselling, reassuring advice to continue with the preg-
nancy
D. Obstetric MRI
15. A neonate presents with cyanosis at birth. A CXR is done
and is shown here. What is the most likely diagnosis?

A. Congenital B. Neurofibromatosis
C. Degenerative arthritic D. Idiopathic

A. TOF C. TAPVC
B. TGA D. Ebstein’s anomaly

Answers
13. A. Positive FAST 16. D. TAPVC 19. A. Congenital
14. C. Counselling, reassuring advice... 17. A. E-FAST
15. A. TOF 18. C. Craniopharyngioma xi
COVID-19: High-Yield Snapshot
Important Timeline Parameters: Microbiology and Epidemiology:
• Originated in Wuhan, Hubei, China • Etiological agent: Novel Coronavirus – SARS – CoV – 2
• 1st case reported on December 1st, 2019 • Category B agent – Has a high infectivity but a low/moderate
• nCoV – Novel Corona virus identified on January 7th, 2020 mortality potential
• January 9th, 2020—1st death reported in China • Enveloped ss-RNA virus with petal/club shaped/crown like
• January 13th, 2020—Thailand reported 1st case outside China peplomer spikes – Solar corona appearance
• January 30th, 2020—1st case reported in India in Kerala • Origin – Bats and Snakes to human and now from Human-to-
• January 30th, 2020—WHO declares it as Public Health Emergency Human transmission
of International Concern (PHEIC) • Incubation period: 2.2 – 11.5 days – Median time 5.5 days
• February 11th, 2020—Term “COVID-19: Corona Virus Disease – 2019” • Serial interval = 5 – 6 days
coined • Mode of transmission: Droplets, Contact & Fomites
• March 11th, 2020—WHO declares it as global pandemic • Reproductive number – Number of secondary infections gen-
• March 13th, 2020—COVID-19 declared as a national disaster in erated from one infected person = 2.5 – 4.5
India • Case Fatality Rate:
� Overall—2–3.7%
� >10% in age group >80 years
• Laboratory Diagnosis: Achieved by RT-PCR - IOC
• Specimen type is Nasopharyngeal/Oropharyngeal swabs
Imaging: � Group 4–3rd week after symptom onset: GGO & Reticular
patterns are predominant findings—suggestive of interstitial
Chest radiograph: changes. Bronchiolectasis, Pleural effusion/thickening &
• May be done as 1st investigation but has poor sensitivity – lymphadenopathy may be seen.
especially in early stages • Most common imaging finding overall: Bilateral, subpleural,
HRCT Thorax: ground-glass opacities with air bronchograms & ill-defined margins
• Is IOC for imaging • Most common imaging finding in early stages of infection:
• May be done for primary diagnosis as well as follow-up Imaging Multifocal sunpleural ground glass opacities
• Imaging findings on CT: • Most common imaging finding in late stages of infection:
� Group 1—Preclinical: Unilateral, multifocal ground glass Multifocal consolidation & crazy paving pattern
opacification (GGO) • Most common lobe affected: Right lower lobe
� Group 2–1st week after symptom onset: Bilateral, diffuse • Abnormal lung CT findings can be present even in asymptomatic
ground glass opacities (GGO). Pleural effusion & Lymphadeno­ patients, and lesions can rapidly evolve into a diffuse ground-
pathy may be seen glass opacity predominance or consolidation pattern within 1–3
� Group 3–2nd week after symptom onset: GGO still the pre- weeks after onset of symptoms, peaking at around 2 weeks after
dominant finding, but consolidation may be seen onset.

Subpleural multifocal ground glass opacities (GGOs) with mild Extensive bilateral subpleural consolidations with a round area
septal thickening of cavitation on right side
Pharmacological Therapy:
• Remdesivir: On May 1st, the US FDA approved it for emergency use in hospitalised patients with severe COVID-19 only—this approval
xii is temporary, as it had to be expedited to help patients who are in critical condition. The drug may reportedly be used in 5-day or
10-day treatment durations, depending on the severity of the disease. However, this authorization is not permanent. Remdesivir is still
an investigational drug that needs to get authorization from the FDA.
• Favilavir—Approved in China
• Empirical therapy: Consists of a combination of Chloroquine + Oseltamivir + Anti-HIV Protease Inhibitors
IMAGE-BASED
QUESTIONS
1. Identify the diagnosis in this child: (Recent Pattern 2020) 4. A 65-year-old female with chronic backache revealed the
following appearance on a lumbar radiograph. What is the
most likely cause? (Recent Pattern 2020)
CONCEPTUAL REVIEW OF RADIOLOGY

A. Epiglottitis
B. Adenoid enlargement
C. Acute laryngotracheobronchitis A. Paget’s disease B. Renal osteodystrophy
D. Retropharyngeal abscess C. Osteoporosis D. Ankylosing spondylitis

2. Identify the diagnosis: (Recent Pattern 2020) 5. A 10-month-old girl child presented with pain in abdomen,
blood in stool and lump felt over abdomen. A barium
enema was done and revealed this image. What is the
most likely diagnosis? (Recent Pattern 2020)

Ans.
1. C
2. B
3. A
4. C
5. C
6. B A. TGA B. TOF
C. TAPVC D. Ebstein’s anomaly A. Meckel’s diverticulum B. Volvulus
C. Intussusception D. Diverticulitis

3. A child with a known skeletal deformity was being 6. A 42-year-old patient with sudden onset headache, neck
evaluated and an angle was measured as shown. Can you rigidity without any obvious history of trauma. What is
name the measurement being made here? the most likely diagnosis based on this CT image?
Image-Based Questions

 (Recent Pattern 2020) (Recent Pattern 2020)

A. Cobb’s angle B. Bohler’s angle A. Meningitis B. SAH


C. Pauwel’s angle D. Pott’s angle C. Intraparenchymal bleed D. SDH
2
7. Barium swallow was performed in a patient with dysphagia. 10. A HSG was performed for Infertility evaluation in a patient.
What is the most likely diagnosis? (Recent Pattern 2020) What is the most likely diagnosis? (Recent Pattern 2020)

CONCEPTUAL REVIEW OF RADIOLOGY


A. Achalasia cardia B. Esophageal stricture A. Normal study B. Cornual block
C. Carcinoma esophagus D. GERD C. Hydrosalpinx D. Uterine synechiae

8. Patient presented with neck swelling and regurgitation 11. A patient being treated for infertility was treated with
with gurgling sound when pressed over the neck. A barium injection HMG following which she presented with
swallow was performed and is revealed here. Most likely abdominal distension and vomiting. Most likely diagnosis
diagnosis will be: (Recent Pattern 2020) based on this USG image is: (Recent Pattern 2020)

Ans.
7. C
8. B
9. A
10. C
11. A
12. B
A. Esophageal stricture B. Zenker’s diverticulum A. OHSS B. PCOS
C. Esophageal web D. Dysphagia lusoria C. Theca lutein cysts D. Ovarian torsion

9. A patient complains of incomplete evacuation of urine and 12. This appearance seen in M-mode USG thorax is suggestive
frequent micturition. Identify the diagnosis in this image? of: (Recent Pattern 2020)
 (Recent Pattern 2020)

Image-Based Questions

A. Bulbar urethral stricture A. Stratosphere sign


B. Urethral block B. Sea shore sign
C. Urethral rupture C. A line sign
D. Urethral diverticulum D. Lung point identification sign
3
13. Following image of FAST in a trauma patient suggests: 16. A 67-year-old hypertensive and diabetic male presents
 (Recent Pattern 2020) with left-sided hemiplegia. Diagnosis based on this CT is:
(Recent Pattern Jan 2019)
CONCEPTUAL REVIEW OF RADIOLOGY

A. Positive FAST B. Negative FAST A. Hypertensive bleed B. Ischemia


C. Equivocal FAST D. None of the above C. SAH D. Lacunar Infarct

14. Identify the structure marked in the image: 17. Diagnosis based on this image is:
 (Recent Pattern Jan 2019) (Recent Pattern Jan 2019)

Ans.
13. A
14. B.
15. D.
16. B.
17. C.
18. D.
A. Cerebrum B. Cerebellum A. Basal ganglia hemorrhage B. Intraventricular bleed
C. Brainstem D. Corpus callosum C. Lacunar infarct D. Multiple sclerosis

A patient presented with recurrent abdominal pain.


15. 18. A patient presented with history of trauma followed by
Identify the investigation in this image: periorbital swelling and diplopia. Diagnosis based on this
(Recent Pattern Jan 2019) CT image is: (Recent Pattern Jan 2019)
Image-Based Questions

A. Barium meal B. Barium swallow A. Le Fort fracture B. Fracture zygoma


C. Barium enema D. Barium meal follow-through C. Tripod fracture D. Blow-out fracture
4
19. Incorrect statement regarding this condition is: 22. Patient presented to the emergency department with
(Recent Pattern Jan 2019) severe abdominal pain – an erect radiograph was done.
Based on the findings – the management should be:
(Recent Pattern Jan 2019/AIIMS Nov 2018)

CONCEPTUAL REVIEW OF RADIOLOGY


A. Meta-epiphyseal location
B. Eccentric location
C. Chemotherapy is treatment of choice A. Gastric lavage B. ICD insertion
D. Has distinct margins C. Laparotomy D. USG FAST

20. The most likely diagnosis on this IVU image is: 23. Patient presents with low grade fever. Diagnosis is:
(Recent Pattern 2013/Jan 2019) (Recent Pattern Jan 2019)

Ans.
19. C.
20. C.
21. C.
22. C.
23. B.
24. B.
A. VUR B. Transitional cell carcinoma
C. Ureterocele D. Ureteric calculus A. Atypical pneumonia B. Miliary TB
C. Bronchopneumonia D. ILD

21. A known case of pancreatitis presents on day 4 with 24. A 55-year-old non-smoker lady presented with on and off
breathlessness, basal crepitations. Diagnosis is: hemoptysis and productive cough for 1 year. There was no
(Recent Pattern Jan 2019/June 2018) fever or constitutional symptoms. Physical examination

Image-Based Questions
showed clubbing of fingers and coarse crepitations over
the lung base. Blood tests were essentially normal and an
initial CXR was performed. CT scan was also performed,
which is shown below. What is the radiological diagnosis?
(Recent Pattern June 2018)

A. Lung collapse B. Pleural effusions A. Pneumoconiosis B. Bronchiectasis


C. ARDS D. Aspiration pneumonia C. Emphysema D. Lung abscess
5
25. A 35-year-old male with history of 4 weeks of immobiliza- 28. Elderly female presents with a history of fall and is unable
tion for fracture femur developed sudden onset of chest to walk. The attitude of the leg in this patient would be:
pain and hemoptysis. ECG shows S1 Q3 T3 pattern. Diag- (AIIMS May 2017)
nosis is: (Recent Pattern 2018)
CONCEPTUAL REVIEW OF RADIOLOGY

A. Shortened and externally rotated


B. Shortened and abducted
A. Acute myocardial infarct B. COPD C. Lengthened and internally rotated
C. Pulmonary embolism D. Cor pulmonale D. Flexed and adducted

26. Identify the liver condition as represented in the CT scan 29. This diagnosis is: (Recent Pattern 2018/AIIMS May 2018)
picture here: (Recent Pattern June 2018)

Ans.
25. C.
26. C.
27. B.
28. A.
29. D.
30. B.
A. Amoebic liver abscess B. Pyogenic liver abscess A. Rheumatoid arthritis B. Scapholunate dissociation
C. Hydatid disease D. Ascending cholangitis C. Lunate dislocation D. Scaphoid fracture

27.
A patient suffering from morning stiffness >1 hour, 30. Identify this fracture: (AIIMS May 2017)
presents with the following abnormality in the X-ray. What
can be the most possible diagnosis?
Image-Based Questions

(Recent Pattern June 2018)

A. Psoriatic arthritis B. Rheumatoid arthritis A. Monteggia fracture B. Galeazzi fracture


C. Osteoarthritis D. Relapsing polychondritis C. Smith’s fracture D. Barton’s fracture
6
31. Identify the type of investigation done: 34. Identify the condition shown on mammography in photo-
(AIIMS Pattern Nov 2018) graph: (Recent Pattern June 2018)

CONCEPTUAL REVIEW OF RADIOLOGY


A. Angiography B. Fluoroscopy A. Carcinoma B. Abscess
C. Myelography D. Neurography C. Cyst D. Fibroadenoma

32. What is the most probable diagnosis based on the findings 35. A 30-year-old Primigravida complains of vaginal bleeding,
in the CT scan given here? pain in abdomen and vomiting. Uterus is enlarged, soft,
(AIIMS Pattern May 2018/FMGE Pattern Dec 2019) nontender. Based on the USG most likely diagnosis is:
(AIIMS May 2016)

Ans.
31. C.
32. A.
33. B.
34. D.
35. A.
A. Extradural hemorrhage
36. C.
B. Subdural hemorrhage
C. Intracerebral hemorrhage A. H. mole B. Missed abortion
D. Subarachnoid hemorrhage C. Blighted ovum D. Ectopic pregnancy

33. A new-born male baby presented with congestive heart 36. CT Image shows a:
failure. On examination enlarged fontanelles, a loud (Recent Pattern Jan 2018/AIIMS May 2018)
cranial bruit and following radiological finding was noted–

Image-Based Questions
the most likely diagnosis is: (Recent Pattern 2018)

A. Sinus pericranii A. Ascending aortic aneurysm


B. Vein of Galen malformation B. Ascending aortic dissection
C. Arachnoid cyst C. Abdominal aortic dissection
D. Dandy walker malformation D. Descending aortic thrombosis
7
37. An 8-year-old child presented with pain at shoulder and 40.
A middle-aged woman with history of backache
arm. An X-ray reveals a uniform well-defined lucent lesion underwent a Schober’s test and came as positive. She had
at the metaphysis. Most likely diagnosis is: hyperpigmented nose and ears. Looking at this radiograph,
(AIIMS May 2016) most likely diagnosis is: (AIIMS May 2017)
CONCEPTUAL REVIEW OF RADIOLOGY

A. Simple bone cyst B. Aneurysmal bone cyst A. Ankylosing spondylitis B. Degenerative disc disease
C. Giant cell tumor D. Osteoid osteoma C. Ochronosis D. Fluorosis

38. A child presented with history of fever and respiratory 41. The most likely diagnosis in this child is: (AIIMS Nov 2016)
distress. With suspicion of pneumonia an X-ray was done.
Most likely causative agent in this condition:
(AIIMS May 2017)

Ans.
37. A.
38. B.
39. B.
40. C.
41. B. A. Pneumococcal pneumonia
42. C. B. Staphylococcal pneumonia
C. Klebsiella pneumonia A. Scurvy B. Rickets
D. Mycoplasma pneumonia C. Salter-Harris injury D. Greenstick fracture

39. 35-year-old complains of fever, severe abdominal pain in 42. A 9-year-old female child presented with history of
epigastrium radiating to the back with a history of binge headache and visual disturbance. What could be your
drinking. Pulse is 120 bpm, BP is 90/60 mm Hg. CT Abdomen possible diagnosis? (Recent Pattern 2018)
Image-Based Questions

(P+C) was done. Most likely diagnosis is: (AIIMS May 2016)

A. Peptic ulcer B. Acute pancreatitis A. Pituitary macroadenoma B. Rathke’s cleft cyst


C. Chronic calcific pancreatitis D. Acute cholecystitis C. Craniopharyngioma D. Hypothalamic hamartoma
8
43. A patient was admitted with severe retrosternal pain, 46. USG of a 34-year-old man depicted a complex cystic lesion
profuse sweating and one episode of vomiting. Following in right kidney with an exophytic mass. CT image is shown
an abnormal ECG he was wheeled in for a primary here. Most likely diagnosis is: (AIIMS May 2017)

CONCEPTUAL REVIEW OF RADIOLOGY


angiography. Looking at the following image what is your
most likely inference?

A. Coronary artery aneurysm


B. Main left coronary stenosis
C. LAD stenosis A. Renal cyst B. Renal parapelvic cyst
D. Left circumflex artery stenosis C. Renal cell carcinoma D. Renal oncocytoma

44. A patient presented with fever and cough. CXR here shows 47. Identify the condition shown in this image:
obscuration of right heart border. Most likely diagnosis is: (NEET Pattern 2012)

Ans.
43. C.
44. B.
45. B.
46. C.
A. RUL consolidation A. Undescended left kidney 47. C.
B. Medial segment of RML consolidation B. Renal cyst 48. A.
C. Lateral segment of RML consolidation C. Horse shoe kidney
D. RLL collapse D. Cross fused ectopic kidney

45. Identify the structure marked with the arrow: 48. True statement regarding this diagnosis is:
(Recent Pattern Jan 2019) (Recent Pattern 2016)

Image-Based Questions

A. Recurrence is a common complication


B. Common in children and rare in adults
A. Third ventricle B. Fornix C. May be overlooked as clinically occult
C. Corpus callosum D. Pineal gland D. Is not associated with other bony/cartilaginous injury
9
49. This classical ‘Ice-cream cone’ appearance of this intensely 52. This bone tumor is most likely to be a:
enhancing lesion at the right CP angle recess suggests: (Recent Pattern 2013)
CONCEPTUAL REVIEW OF RADIOLOGY

A. Epidermoid cyst B. Arachnoid cyst A. Osteosarcoma B. Ewing’s sarcoma


C. Acoustic schwannoma D. High grade glioma C. SBC D. Osteoid osteoma

50. This ultrasound image is that of a very important scan done 53. This patient has presented with an acute onset of left-
during early gestation where a translucency thickness is sided hemiparesis. A DWI MRI image is shown here
measured in the nuchal region. This scan must be done shows an area of restricted diffusion in the right cerebral
when the Crown-Rump Length (CRL) is in the range of: hemisphere and is hence most likely suggestive of:
 (Recent Pattern 2014)

Ans.
49. C.
50. B.
51. C.
52. B.
53. B.
A. Acute intracranial hemorrhage
54. B.
A. 40–80 mm B. 45–84 mm B. Acute ischemic infarct
C. 50–100 mm D. 40–84 mm C. Multiple sclerosis
D. Herpes encephalitis

51. Fracture shown in this radiograph is: 54. The most likely diagnosis in this child is:
(Recent Pattern 2013, 2017) (Recent Pattern 2014)
Image-Based Questions

A. Congenital lobar emphysema


B. Diaphragmatic hernia
A. Colles’ fracture B. Smith’s fracture C. Pneumothorax
C. Bennett’s fracture D. Rolando’s fracture D. Hyaline membrane disease
10
55. This appearance in a 60-year-old man is suggestive of: 58. Patient presented with blood in stools and weight loss.
(Recent Pattern 2018) This CE-CT image points to which of the following probable
diagnosis is: (JIPMER May 2016)

CONCEPTUAL REVIEW OF RADIOLOGY


A. Eosinophilic granuloma B. Multiple myeloma A. Ca bladder B. Ca cervix
C. Hyperparathyroidism D. Acromegaly C. Ca rectum D. Perirectal abscess

56. The most likely cause of persistent snoring and mouth 59. Diagnosis is: (AIIMS May 2016)
breathing in this child is: (AIIMS Nov 2015)

Ans.
55. B.
56. B.
57. C.
58. C.
59. A.
60. C.
A. Tonsillar enlargement B. Adenoid enlargement
C. Croup D. Paranasal sinus polyp A. Small bowel obstruction B. Large bowel obstruction
C. Perforation D. Gallbladder ileus

57. A child presents with intermittent fever and pain in the 60. What is this appearance – seen in early pregnancy known
proximal tibia. Based on this radiograph, the most likely as: (AIIMS Nov 2016)
diagnosis is:

Image-Based Questions

A. Yolk sac sign B. Double decidual sac sign


A. Acute osteomyelitis B. Chronic osteomyelitis C. Double bleb sign D. Twin peak sign
C. Brodie’s abscess D. Garre’s osteomyelitis
11
61. Most likely diagnosis on this Barium enema image is: 64. This typical brain anomaly is known as:
(Recent Pattern 2018)
CONCEPTUAL REVIEW OF RADIOLOGY

A. Crohn’s disease B. Diverticulosis A. Lissencephaly B. Schizencephaly


C. Carcinoma colon D. Lead pipe colon C. Arnold Chiari malformation D. Hydranencephaly

62. Diagnosis on mammography: (JIPMER May 2017) 65. Identify the abnormality marked with the arrow:

Ans.
61. B.
62. C.
63. B.
64. B.
65. B. A. Bridging syndesmophytes
66. B. B. OPLL
A. Fibroadenoma B. Cyst
C. IV Disc calcification
C. Carcinoma D. Diffusely dense breast
D. Vertebral end plate sclerosis

63. A 22-week pregnant female for routine obstetric ultra- 66. A 42-year-old male presents with weight loss, intermittent
sound and the following appearance was seen during low-grade fever and back pain. A lumbar spine MRI was
evaluation of the fetal heart. Most likely fetal cardiac done, after reviewing the findings contrast was injected.
Image-Based Questions

anomaly seen here is: Shown here is a postcontrast sagittal image with disc
enhancement extending up to the epidural space. The
most likely diagnosis here is:

A. Tetralogy of Fallot B. Hypoplastic left heart A. Spine metastasis B. Pott’s spine


C. VSD D. Fetal pericardial effusion C. Degenerative disc disease D. Ankylosing spondylitis
12
67. This MR Image points to a very specific diagnosis, that is: 70. Look at CT axial image given below and identify the
structure pointed by the arrow: (Recent Pattern 2018)

CONCEPTUAL REVIEW OF RADIOLOGY


A. Scaphoid fracture
B. Scapholunate dislocation
C. Kienböck’s disease A. Aorta B. IVC
D. Radiocarpal osteoarthritis C. SMA D. SMV

68. A patient presents with dysphagia. This abnormality on a 71. Spot diagnosis:
Barium swallow study-filling phase most likely suggests:

Ans.
67. C.
68. B.
69. B.
70. B.
71. C.
72. C.
A. Unicornuate uterus B. Arcuate uterus
A. Zenker’s diverticulum B. Esophageal web C. Bicornuate uterus D. Septate uterus
C. Esophageal stricture D. Feline esophagus

69. A patient presented with left lumbar region pain. After 72. This special investigation is known as:
a screening examination an abdominal radiograph was
obtained and is shown here. The most likely diagnosis here is:

Image-Based Questions

A. CBCT
A. Right renal cystine calculus B. Scanogram
B. Left renal struvite calculus C. Orthopantomogram
C. Right renal struvite calculus D. Stenvers view
D. Left renal calcium-oxalate calculus
13
73. Painless swelling in a 25-year-old man. Diagnosis: 75. Spot diagnosis is:
CONCEPTUAL REVIEW OF RADIOLOGY

A. GCT B. Osteoma A. Fibrous dysplasia B. Cleidocranial dysplasia


C. Osteochondroma D. Osteosarcoma C. Butterfly vertebra D. Paddle ribs

74. This round hyperdense lesion on NC-CT located in the 3rd 76. Diagnosis is:
ventricle is most likely to be a:

Ans.
73. C.
74. C.
75. B.
76. C.
A. Right pneumothorax
B. Left pneumothorax
A. Arachnoid cyst B. Epidermoid cyst C. Right hydropneumothorax
C. Colloid cyst D. Giant cell astrocytoma D. Left hydropneumothorax
Image-Based Questions

14
Answers of Image-Based Questions

CONCEPTUAL REVIEW OF RADIOLOGY


1. Ans. (c) Acute laryngotracheobronchitis chronic SAH is MRI. MC site of berry aneurysm rupture is
Anterior communicating artery (AComA).
This is the classical Steeple appearance/Wine bottle
appearance of Croup/Acute laryngotracheobronchitis seen 7. Ans. (c) Carcinoma esophagus
due to subglottic edema. It is due to viral infection of the
upper airway by parainfluenza virus or respiratory syncytial This barium swallow image shows an abrupt irregular short
virus (RSV). Although imaging findings are not required for segment narrowing involving the esophagus – suggests “Rat
the diagnosis, classic findings of narrowing of the subglottic tail appearance” of Carcinoma esophagus. Birds beak sign – is
airway and dilatation of the hypopharynx are supportive of the smooth narrowing in Achalasia cardia.
the diagnosis.
8. Ans. (b) Zenker’s diverticulum
2. Ans. (b) TOF A posterior outpouching from the esophagus as seen here
This is the peculiar Boot shaped/Couer-en-sabot appearance with a history of neck swelling, regurgitation – suggests
of heart seen in Tetralogy of Fallot. The 4 components of this Zenker’s diverticulum. It is a false diverticulum arising at
tetralogy are VSD, Overriding of aorta, PS and RVH. RVH is the Killian’s dehiscence above the Cricopharyngeus muscle.
responsible for boot shaped configuration of heart and the A anterior/anterolateral diverticulum that arises below
heel of the shoe is formed by the RV. the cricopharyngeus muscle is known as Killian-Jamieson
diverticulum.
3. Ans. (a) Cobb’s angle
9. Ans. (a) Bulbar urethral stricture
The thoracic spine here shows a lateral curvature suggestive
of scoliosis. This is a Retrograde urethrogram (RGU) study. Contrast has
Scoliosis is defined as a lateral spinal curvature with a been introduced into the urethra by a cannula passed into
Cobb angle of >10°. The Cobb angle is the most widely used the external urethral meatus at tip of penis. A tight narrowing
measurement to quantify the magnitude of spinal deformities. is seen at the level of bulbar urethra – suggests a urethral
Lines are drawn along the end plates of end vertebrae of the stricture.
curvature followed by perpendiculars to these lines. The
10. Ans. (c) Hydrosalpinx
vertical angle between these perpendiculars is the Cobb’s Ans.
angle (using the Cobb-Lippman method). Another method The HSG spot image shows bilateral dilated Fallopian tubes
to measure the angle of curvature is the Risser Ferguson with a typical “retort shaped appearance” suggestive of
method – is less commonly used. Hydrosalpinx. This is most likely due to fimbrial level tubal
block – as no free peritoneal spill is seen here.
4. Ans. (c) Osteoporosis
11. Ans. (a) OHSS
This is the typical Fish-mouth/Cod-fish appearance of
vertebrae seen in osteoporosis. Both superior and inferior The ovaries look like polycystic ovaries. But the history of

Answers of Image-Based Questions


endplates are concave due to the intervertebral discs infertility and HMG injections – suggest follicle stimulation
impinging on the poorly mineralized bone. Paget’s disease is – and hence the appearance is suggestive of Ovarian
associated with picture frame vertebra. Renal osteodystrophy/ hyperstimulation syndrome. Abdominal painQ and distensionQ
Hyperparathyroidism with Rugger jersey spine. Bamboo are most common symptoms. USG reveals enlarged ovaries
spine/Poker spine is seen in Ankylosing spondylitis. with multiple folliclesQ – predisposed to torsion/hemorrhagic
cysts/rupture of cysts. Mild to moderate free fluid in
5. Ans. (c) Intussusception abdomen +/– pleural effusions. Treatment – Conservative
The barium enema reveals a “Coiled spring appearance” due with admission + IV fluid/colloids + Paracentesis of tense
to barium outlining the lumen of the intussusceptum and ascites + Prophylactic heparin – to prevent thromboembolic
the lumen of the intussuscepiens around it. Various signs phenomena secondary to hemoconcentration
described in intussusception include Sandwich sign, Target/
12. Ans. (b) Sea shore sign
Doughnut/Bull’s eye sign, Pseudokidney sign, Crescent in
a doughnut sign, Sausage sign, Claw sign. Most common This is the Sea shore sign seen on M-mode USG of using thorax in
type of intussusception in children is Ileo-colic. IOC for Normal individuals. The waves are form by the chest wall layers
intussusception in children is USG while in adults is CE-CT. and the granular sand is formed by the lung parenchyma. Rest
all the options here are seen in Pneumothorax on ultrasound.
6. Ans. (b) SAH
13. Ans. (a) Positive FAST
This NC-CT image shows extensive hyperdense areas in the
basal cisterns of brain – suggestive of extensive subarachnoid The hypoechoic (black) appearance you can see here in the
haemorrhage. Given the history of sudden onset headache Morrison’s (hepatorenal) pouch is suggestive of free fluid in
– the most likely underlying cause must be rupture if Berry the abdomen. In a patient of abdominal trauma in whom FAST
eneurysms. IOC for Acute SAH is NC-CT. IOC for sub-acute/ is done – this findings is suggestive of Positive FAST – implies
15
presence of Hemoperitoneum. If patient is hemodynamically 23. Ans. (b) Miliary TB
stable – CE-CT abdomen should be done to diagnose solid
Multiple tiny randomly scattered nodules in both lungs –
organ injury. If the patient is hemodynamically unstable –
seen due to hematogenous dissemination of TBQ. Other
emergency exploratory laparotomy needs to be done.
CONCEPTUAL REVIEW OF RADIOLOGY

differentials for such pattern are metastases, berylliosis,


14. Ans. (b) Cerebellum hemosiderosis.

Answer is obvious. In this sagittal T1W image anterior to 24. Ans. (b) Bronchiectasis
cerebellum lies the triangular-shaped fourth ventricle and
brainstem further anterior to it. Thick walled air-filled clustered appearance on CXR – HRCT is
known as Cluster of grapes signQ – seen in BronchiectasisQ.
15. Ans. (d) Barium meal follow-through Tram-track signQ, Signet ring signQ, String of pearls signQ,
Lobulated gloved fingerQ/Y/V-shaped densitiesQ seen. CXR –
This is BMFT as small bowel is opacified (look for the valvulae
initial investigationQ. HRCT – is BESTQ.
conniventesQ of jejunum in left upper abdomen). BMFT is
used in diagnosis of intestinal tuberculosis, Crohn’s disease, 25. Ans. (c) Pulmonary embolism
Whipple disease, etc. Nowadays CT/MR enteroclysisQ is
preferred. Nonenhancing hypodense filling defect in the right main
pulmonary artery on axial CECT. Deep venous thrombosis is
16. Ans. (b) Ischemia most common source for PTE. Polo mint signQ may be seen on
Identify the wedge-shaped hypodensity in right hemisphere CE-CT. CT Pulmonary angiography is IOCQ.
with loss of gray-white matter differentiation. There is also
26. Ans. (c) Hydatid disease
significant mass effect and midline shift. Although CT is 1st
investigationQ – DWI-MRI is earliest to detect acute infarctsQ. This sign is known as Serpent signQ – floating separated
membranes within the cyst. Liver is most commonly infected
17. Ans. (c) Lacunar Infarct organQ in human followed by lungs. CE-CT is IOCQ.
A hypodense (black) small area seen in the left thalamus.
Lacunar infarcts are areas of ischemia smaller than 20 mmQ 27. Ans. (b) Rheumatoid arthritis
and are due to thrombosis in small lenticulostriate arteriesQ, Classic Swan neck deformityQ (DIP joint flexion and PIP joint
thalamoperforatingQ and pontine perforating arteriesQ. They extension) with marked changes at the radiocarpal joint.
lead to specific Lacunar stroke syndromes.
Ans. 28. Ans. (a) Shortened and externally rotated
18. Ans. (d) Blow-out fracture
Radiograph shows displaced fracture neck left femur. There is
Typical fracture of roof of maxillary sinus/base of orbit which break in Shenton’s lineQ. Radiologically, Garden classificationQ is
leads to herniation of orbital contents into the sinus causing used. The affected limb is usually shortened, externally rotated
entrapment of inferior rectus muscle (most common)Q and and mildly abducted – the helpless attitudeQ of lower limb.
resultant diplopia. Teardrop signQ and Black eyebrow signQ
are seen. 29. Ans. (d) Scaphoid fracture
Answers of Image-Based Questions

19. Ans. (c) Chemotherapy is treatment of choice Frontal radiograph with ulnar deviation of wristQ – is best
radiograph for demonstration of scaphoid fractures. Often
Diagnosis is GCT (subarticular expansile lytic lesion in adult
radiologically occult fractureQ. Cast is applied in glass holding
skeleton) – Surgery is treatment of choice. Has soap bubble
positionQ. At risk of Avascular necrosisQ – more proximal is the
appearanceQ.
fracture – more the risk of AVNQ.
20. Ans. (c) Ureterocele
30. Ans. (b) Galeazzi fracture
Repeatedly asked question. Ureterocele is congenital
It is fracture distal third of radius with dislocation of distal
dilatation of distal end of ureter. Typical Adder/Cobra-head
appearanceQ on IVU. It is associated with ectopic insertion or radioulnar joint with an intact ulna. If ulna is also fractured it
ureter and duplex uretersQ. Adder head singQ seen. is known as a Galeazzi equivalent fracture.

21. Ans. (c) ARDS 31. Ans. (c) Myelography

Perihilar fluffy opacities – Bat wing opacitiesQ seen in Myelography is an invasive, indirect technique used for
pulmonary edema (cardiogenic/non-cardiogenic), ARDS. visualization of spinal cord and nerve roots done by instillation
Common pulmonary complications of pancreatitis are ARDS, of iodinated contrast in sub-arachnoid space. Now obsolete
pleural effusion (mostly left sided) and consolidation. with advent of MRI.

22. Ans. (c) Laparotomy 32. Ans. (a) Extradural hemorrhage


Radiograph shows free gas under diaphragm suggestive Biconvex/lentiform-shapedQ hyperdense bleed on right side.
of Perforation – hence answer is Laparotomy. X-ray chest Commonly-associated with skull fractures and injury to
erect – is best radiographic projectionQ whereas CT scan is Middle meningeal artery radicalQ. Most commonQ type of
the most sensitiveQ/overall best investigationQ to identify hemorrhage due to trauma. Associated with Talk-and-Die
16 pneumoperitoneum. syndromeQ.
33. Ans. (b) Vein of Galen malformation 43. Ans. (c) LAD stenosis
Congenital malformation of median prosencephalic vein Q This is a left coronary artery angiography spot image. There is
which shows dilated midline vascular channel in posterior short segment narrowing seen in the left anterior descending

CONCEPTUAL REVIEW OF RADIOLOGY


part of cranium. Most common cause of high output cardiac artery. On angiography images trace the entire arteries and
failure in pediatric age groupQ. try to find a narrowing (stenosis)/dilatation (aneurysm)/
vascular nidus (A-V malformations).
34. Ans. (d) Fibroadenoma
44. Ans. (b) Medial segment of RML consolidation
Well defined coarse macro-calcification/Popcorn calcificationQ
– s/o Fibroadenoma. It is the most common solid breast mass. Silhouette signQ - Right middle lobe is in direct contact with
On USG appear as circumscribed oval hypoechoic masses. right heart border, hence the right cardiac margin is obscured
They are typically wider than tallerQ in appearance and may in RML consolidation – more so involving its medial segment.
show posterior acoustic enhancementQ. Similarly, consolidation in right lower lobe will obscure
margins of right dome of diaphragm.
35. Ans. (a) H. mole
45. Ans. (b) Fornix
Snowstorm signQ on USG. Seen in complete mole. In partial
mole a gestational sac or fetal parts can be seen. MRI in Fornix is a C-shaped structure better seen on sagittal section.
modality of choiceQ for invasive mole or choriocarcinomaQ. It acts as major outflow for tracts of hippocampus.

36. Ans. (c) Abdominal aortic dissection 46. Ans. (c) Renal cell carcinoma
There is a thin slit-like structure which is dividing abdominal aorta RCC (yellow arrow) – with IVC invasion (blue arrow)
into true and false lumen in this image – suggestive of Aortic Ghost signQ: Organ which should be seen in normal position is
dissection. HypertensionQ is most common cause of dissection. not seen here. That is right kidney is not visualized and hence
it is organ of origin of the mass. Metastases from RCC are
37. Ans. (a) Simple bone cyst expansile, aggressive and vascular. MRI is IOC for IVC invasionQ.
Lucent geographic lesion seen in humeral metaphysis in a 47. Ans. (c) Horse shoe kidney
child is most commonly a simple bone cyst. Trapdoor signQ/
Fallen fragment signQ/Hinged fragment signQ may be seen. IVU image demonstrates kidneys/pelvicalyceal systems
lying close to the midline medially directed lower poles and
38. Ans. (b) Staphylococcal pneumonia anterior facing renal pelvis–suggests Horse shoe kidneys. Ans.
Pneumatoceles are transient intrapulmonary air-filled cystic 48. Ans. (a) Recurrence is a common complication
spaces that can have varied appearances. These are most
commonly associated with Staphylococcal pneumoniaQ. There is incongruency between Glenoid (red arrow) and
Humeral head (yellow arrow) which is lying inferior to
39. Ans. (b) Acute pancreatitis Coracoid process (blue) – suggests Subcoracoid/Anterior
shoulder dislocation. Anterior shoulder dislocation is very
Observe a bulky pancreas with fuzzy margins and adjacent
common, often seen in adults and is recurrent. Associated

Answers of Image-Based Questions


fat stranding – s/o Acute pancreatitis. CE-CT scan is IOC.
with Bankart’s lesionQ and Hill-Sachs defectQ.
Balthazar grading systemQ and CT Severity Index (CTSI)Q are
used for prognosis. 49. Ans. (c) Acoustic schwannoma
40. Ans. (c) Ochronosis Large chunk of tumor in the CP angle recess is the ice-cream
whereas extension into internal auditory canal forms the cone.
Image shows Intervertebral disc calcification commonly
Bilateral Acoustic schwannomas suggest – Neurofibromatosis
seen in alkaptonuria due to deposition of homogentisic acid.
type 2 – MISME syndrome – Multiple Inherited Schwannoma,
Schober’s test is nonspecific test that denotes loss of elasticity
Meningioma and Ependymoma. The various CP angle masses
of spine. Hyperpigmented cartilage is another clue that points are schwannoma, epidermoid, dermoid, meningioma.
toward Alkaptonuria/Ochronosis.
50. Ans. (b) 45–84 mm
41. Ans. (b) Rickets
This CRL corresponds to gestational age of 11 weeks to 13
Classical WideningQ, FrayingQ and CuppingQ of metaphysis weeks 6 daysQ - when NT scan should be done. An increase
with widened epiphyseal plate of cartilage - suggests Rickets. in NT value is associated with risk of Trisomy 21,13,18Q. Apart
Appearance of band of metaphyseal calcificationQ – is earliest from NT, other aneuploidy markers are Nasal boneQ, Ductus
radiological sign of healing rickets. venosus flowQ and Tricuspid regurgitationQ.
42. Ans. (c) Craniopharyngioma 51. Ans. (c) Bennett’s fracture
A calcified, solid-cystic, mass in the suprasellar region in a There is a 2-part fracture of base of thumb (first metacarpal)
child – is Craniopharyngioma unless proven otherwise! It is with intra articular extension. Similar intra articular 3 part
a clincher. Visual loss is due to mass effect on optic chiasm. or comminuted fracture is known as Rolando’s fractureQ.
Arises from Rathke’s pouch epitheliumQ.

17
Abductor pollicis longus tendonQ – creates an unopposed pull and is seen between 5 and 6 weeks. It corresponds with the
on the fracture fragments hence Bennett’s fracture requires appearance of the fetal poleQ – tiny dot seen between the
internal fixationQ. two blebs.
CONCEPTUAL REVIEW OF RADIOLOGY

52. Ans. (b) Ewing’s sarcoma 61. Ans. (b) Diverticulosis


Diaphyseal ill-defined lesion with onion peel or lamellar type Multiple barium-filled outpouchings seen in the colon
of periosteal reaction in pediatric/unfused skeleton – most creating a - Saw tooth appearanceQ. Various signs described
likely suggests Ewing’s sarcoma. include Bowler hat appearanceQ/Champagne glass signQ and
Cortical saucerizationQ/Groomed or Trimmed whisker’s Arrowhead sign on CE-CTQ . CE-CTQ is IOC for diverticulitis.
effectQ/Skip lesionsQ are seen in Ewing’s sarcoma.
62. Ans. (c) Carcinoma
53. Ans. (b) Acute ischemic infarct
Mammogram shows an irregular appearing mass with
Hyperintense appearance on DWI and Hypointense spiculated margins and adjacent architectural distortion – s/o
appearance on ADC map suggest restricted diffusion – s/o BIRADS V (highly suggestive of malignancy). MammographyQ
acute infarct. Restricted diffusion on DWI is most sensitive is the screening IOC for Ca-breast whereas Dynamic contrast
for acute infarct and becomes positive within minutes of enhanced MRIQ is overall the IOC.
ischemic event. Diffusion perfusion mismatch is used to
estimate the penumbra or salvageable neuroparenchyma 63. Ans. (b) Hypoplastic left heart
before attempting thrombolysis.
This is antenatal fetal ultrasound/echocardiography showing
54. Ans. (b) Diaphragmatic hernia 4 chamber view – with hypoplastic left-sided chambers.
There is herniation of bowel loops in left hemithorax and 64. Ans. (b) Schizencephaly
the left dome of diaphragm is not well appreciated with
shift of mediastinum toward right – suggests Congenital There is a gray matter lined cleft seen in neuroparenchyma on
diaphragmatic herniaQ. The most common cause of death in coronal MRI extending from the brain surface up to the lateral
congenital diaphragmatic hernia is pulmonary hypoplasiaQ. ventricle. It can be open lip type or closed lip type. Most often
the cleft involves the posterior frontal or parietal lobes. It is
55. Ans. (b) Multiple myeloma associated with Septo-optic dysplasiaQ, heterotopiaQ, absent
Radiograph of skull shows multiple punched out uniform septum pellucidumQ.
Ans. lytic lesions – peculiar of Multiple myeloma – described as
65. Ans. (b) OPLL
Rain drop skullQ. Other imaging signs include Salt pepper
appearance of vertebraQ on MRI/Vertebra planaQ/Wrinkled This is Ossified posterior longitudinal ligament–seen
vertebraQ. Lesions show cold spots on 99mTc – MDP bone commonly in Asian population and mostly asymptomatic
scanQ. but sometimes can cause compression on spinal cord. It
is associated with diffuse idiopathic skeletal hyperostosis
56. Ans. (b) Adenoid enlargement (DISH)Q.
Answers of Image-Based Questions

Lateral radiograph of neck shows soft tissue opacity in


posterosuperior nasopharynx causing moderate narrowing 66. Ans. (b) Pott’s spine
of the nasopharyngeal airway. Similar appearance in adult is Disc enhancement with involvement of both adjacent
seen in nasopharyngeal neoplasms. vertebrae – peculiar of spinal tuberculosis/Pott’s spine. CE-
MRI is IOCQ. If untreated, leads to vertebral collapseQ, anterior
57. Ans. (c) Brodie’s abscess
wedging leading to kyphosisQ and gibbus formationQ.
Well-defined oval lucent lesion with surrounding sclerosis in
tibial metaphysis – is Brodie’s abscess. TibiaQ is most common 67. Ans. (c) Kienböck’s disease
site and Staph. aureusQ is commonest organism implicated. Coronal MRI T1W image shows small sclerosed (hypointense)
58. Ans. (c) Ca rectum appearing Lunate – suggests Osteochondritis. Most common
cause is trauma followed by surgery, hemoglobinopathies,
Axial CECT image of pelvis shows an enhancing intraluminal steroid and alcohol consumption.
mass within the rectum (yellow arrow) causing luminal
narrowing. Apple core signQ may be seen on barium enema. 68. Ans. (b) Esophageal web

59. Ans. (a) Small bowel obstruction Barium swallow shows a slit-like constriction in the cervical
esophagus suggesting an Esophageal web. Feline esophagus
Multiple air fluid levelsQ seen in dilated bowel loops in central show multiple thin hair-like impressions on mucosa, stricture
abdomenQ with classical valvulae conniventesQ– s/o Small will show narrowing while diverticulum will be seen as an
bowel obstruction. This appearance has also been described outpouching.
as the Step ladder appearanceQ.
69. Ans. (b) Left renal struvite calculus
60. Ans. (c) Double bleb sign
KUB radiograph shows a large staghorn calculus. These are
Antenatal ultrasound image in first trimester – termed as most commonly Struvite/triple phosphate calculiQ which
18 the double bleb signQ. It is formed by Amnion and Yolk sac initially originates in pelvis then gradually enters the calyces.
Repeated UTI (caused by Urease producing bacteria like osteochondroma. It is covered with a cartilage cap which is
Proteus, Klebsiella, pseudomonas) is common implicated best demonstrated on MRI. Increase in thickness of cartilage
factor. cap is most sensitive sign for malignant transformation. Coat
hanger exostosisQ–is a term used for large exostosis projecting

CONCEPTUAL REVIEW OF RADIOLOGY


70. Ans. (b) IVC away from a joint.
In this axial CT scan of abdomen at level of liver the arrow
74. Ans. (c) Colloid cyst
points to a round structure on right side and anterior to
vertebra which is opacified with intravenous contrast Axial NC-CT scan shows a well-defined hyperdense round
representing the IVC. Left lateral to it lies the aorta and lesion in the 3rd ventricle which is classic for colloid cyst.
lateral to aorta are multiple splenic vein collaterals. Moderate This cyst can sometimes lead to obstruction of CSF flow with
ascites is seen – hypodensity (water density) surrounding the hydrocephalous. Appears hyperintense on T1W MRI due to
liver and spleen. high protein content.

71. Ans. (c) Bicornuate uterus 75. Ans. (b) Cleidocranial dysplasia


HSG image shows 2 widely divergent uterine horns suggesting This is the severest form of abnormality – evident as absent/
bicornuate uterus. The other differential is septate uterus hypoplastic claviclesQ. In most cases of CCD–the lateral thirds
where the intercornual distance is less than 4 cm and depth of clavicles may be hypoplastic. It involves mainly membranous
of the V formed between 2 horns is less. bonesQ and has an autosomal dominant inheritanceQ.

72. Ans. (c) Orthopantomogram 76. Ans. (c) Right hydropneumothorax


It is a panoramic or wide view radiograph of the lower face or CXR shows lucent (black) lateral aspect of right hemithorax with
dental skeleton. Apart from pathologies of teeth it is also used air fluid level in it–s/o hydropneumothorax. An expiratory CXR
to study the temporomandibular joint. better than any inspiratory CXR for a diagnosis of pneumothorax.
CT Thorax is IOC for pneumo/hydro-pneumothorax.
73. Ans. (c) Osteochondroma
An exophytic diaphyseal lesion continuous with the cortex
and directed away from the adjacent joint – is exostosis/

Ans.

Answers of Image-Based Questions

19
_ __________________________________________________________________________

_ __________________________________________________________________________

_ __________________________________________________________________________

_ __________________________________________________________________________

_
Note __________________________________________________________________________

__________________________________________________________________________

_ __________________________________________________________________________

____________________________________________________________________________________________________

____________________________________________________________________________________________________

____________________________________________________________________________________________________

____________________________________________________________________________________________________

____________________________________________________________________________________________________

____________________________________________________________________________________________________

____________________________________________________________________________________________________

____________________________________________________________________________________________________

____________________________________________________________________________________________________

____________________________________________________________________________________________________

____________________________________________________________________________________________________

____________________________________________________________________________________________________

____________________________________________________________________________________________________

____________________________________________________________________________________________________

____________________________________________________________________________________________________

____________________________________________________________________________________________________

____________________________________________________________________________________________________

____________________________________________________________________________________________________

____________________________________________________________________________________________________

____________________________________________________________________________________________________

____________________________________________________________________________________________________
IMAGING SIGNS
IN RADIOLOGY
One of the most fascinating things about studying Radiology is the highly imaginative and exciting world of Named/
Eponymous signs/appearances. When your study schedule becomes long, tedious and tiring, these signs can really help
you in getting refreshed a bit! The language of radiology is rich with descriptions of imaging findings, often metaphorical,
CONCEPTUAL REVIEW OF RADIOLOGY

which are used commonly in the day-to-day practice. These “classic signs” give us confidence in our diagnosis. Use this
section as a refresher. When you get tired going through your other subjects/systems, come to these pages and have a look
at these classical images and be amazed! Each of these images is a SPOTTER and will help you in solving numerous MCQs
in exams. Let’s Rock Radiology!

Respiratory System and Cardiac Imaging

3. Visceral Pleural Line and Deep Sulcus Sign


Visualization of the sharp visceral pleural surface (Blue arrow) due to
presence of air in pleural space – Differential radiographic density theoryQ
– in PneumothoraxQ.
1. Split Pleura Sign Unusually deep ipsilateral costophrenic angle (yellow arrow) seen in
pneumothorax due to mass effect on the dome of diaphragm pushing it
Refers to the enhancing appearance of both visceral and parietal pleura
downwards.
on CE-CT seen in empyema. Helps to distinguish it from lung abscess. An
Other signs seen in Pneumothorax include – Hyperlucent hemithorax
abscess is usually round – oval and within the lungs. Non-inflamed pleura
with absent vascular markingsQ and Mediastinal shift toward opposite
in simple pleural effusion does not enhance like empyema.
sideQ.
Imaging Signs in Radiology

4. Golden S Sign
Named after R Golden, refers to the typical S shape of Horizontal fissure
2. Pleural Meniscus Sign seen in right upper lobe collapseQ (mostly due to central hilar mass). The
Homogenous opacity in the left lower zone with an upward ill- defined lateral concavity (yellow arrow) is seen due to RUL collapse as it pulls the
concavity – looks like a Meniscus on the surface of water in a test-tube/ horizontal fissure upwards, whereas the medial convexity (blue arrow) is
capillary tube—Classical of in pleural effusion. A clear horizontal air-fluid seen due to horizontal fissure outlining the centrally located mass lesion.
levelQ is seen in hydropneumothorax. Other sign seen in RUL collapse is Juxtaphrenic peak signQ.

22
CONCEPTUAL REVIEW OF RADIOLOGY
5. Luftsichel Sign
German term meaning air sickle/crescent. Seen in Left upper lobe
collapseQ. Crescent-shaped lucency is seen outlining the aortic knuckle 8. CT Halo Sign
(black arrow) and is due to compensatory over-expansion of the superior Refers to central areas of hyper attenuation/consolidation with surrounding
segment of Left lower lobe – that tries to compensate for the left upper ground glass opacities on high-resolution computed tomography (HRCT).
lobe (LUL) collapse. LUL collapse is seen as a veil/curtain-like opacity in Earlier considered classical for Angioinvasive pulmonary aspergillosisQ
left upper and mid zone (yellow arrow). Juxtaphrenic peak signQ is also but also seen in Pulmonary TBQ, AdenocarcinomaQ, MetastasesQ,
seen in LUL collapse. Granulomatous infectionsQ.

9. Atoll Sign/Reverse Halo Sign


Refers to central ground glass opacity (haziness–yellow arrow) and
6. Air Bronchogram Sign surrounding hyperattenuation/consolidation (dense white appearance—
Visualization of air-filled bronchi (as linear lucencies) due to opacification black arrow) seen on HRCT Chest. See it is exactly the reverse of the
of surrounding alveoli and lung parenchyma. Seen on CXR and HRCT. above-mentioned Halo sign!
Differentials include Infective consolidationQ (Most common), pulmonary Considered specific for Cryptogenic organizing pneumoniaQ but some-
edemaQ, Hyaline membrane diseaseQ, Lung contusionQ, ARDSQ, times also seen in MucormycosisQ, Wegener’s granulomatosisQ, Sarcoido-
Bronchoalveolar carcinomaQ, LymphomaQ. sisQ, TBQ, etc.

Imaging Signs in Radiology

7. Monod Sign/Air Crescent Sign


Refers to thin crescent-shaped air density (yellow arrows) surrounding
a round opacity (aspergilloma – blue arrow) in a cavitary lesion. 10. Bunch/Cluster of Grapes Sign
Demonstrated on CXR or CT Chest. Commonly seen in Aspergilloma but This refers to the clustered appearance of dilated cystic bronchiectasis
also seen in necrotic tumors. Correct sign in Aspergilloma is Monod Sign. seen on HRCT Chest.
Air crescent sign is actually seen in angioinvasive aspergillosis but has The other signs seen in bronchiectasis are Signet ring signQ, Tram track
been used misleadingly in Aspergilloma in literature! signQ. Associated with cystic fibrosis.
23
CONCEPTUAL REVIEW OF RADIOLOGY

14. Continuous Diaphragm Sign


Seen in PneumomediastinumQ as the free air in mediastinum lines
the superior surface of the central tendon of diaphragm. Differential
11. Honeycombing Sign
radiographic density theory ensures that a sharp margin of the diaphragm
Appearance similar to a honeycomb due to stacking of cysts one over is seen in its entire extent. A corollary of this sign is Cupola signQ –
other seen on HRCT. Seen in usual interstitial pneumonia/idiopathic when air lines the inferior surface of central tendon of diaphragm in
pulmonary fibrosis. Suggests end stage interstitial lung disease. pneumoperitoneumQ.

12. Hampton’s Hump


Peripheral wedge-shaped radiopacity with base directed toward pleura
seen on chest X-rays (CXR) or HRCT. Suggests pulmonary infarct and is 15. Corona Radiata Appearance
seen in pulmonary thromboembolism. Other signs seen in pulmonary Multiple fine strands radiating into the lung from a central mass, is highly
thromboendarterectomy (PTE) are Fleischner signQ, Westermark signQ, suggestive of bronchial carcinoma. Central tumors may also involve the
Palla signQ, Knuckle signQ, Chang signQ, Melting ice cube signQ, Felson signQ. phrenic nerve and cause ipsilateral hemidiaphragm elevation.
Imaging Signs in Radiology

16. White-Out Lung/Opaque Hemithorax


Complete radiopaque appearance on an entire hemithorax on CXR. Seen in:
• Massive pleural effusion (in this image) – associated with contralateral
13. Inferior Rib Notching/Roesler Sign mediastinal shift (blue arrow)Q
Inferior rib notching seen on CXR. Seen in Coarctation of aortaQ, • Complete lung collapse - associated with ipsilateral mediastinal shiftQ
pulmonary arteriovenous malformation (AVM), superior vena cava (SVC) • Massive lung consolidation - associated with no mediastinal shiftQ
obstruction, Pulmonary/chest wall AV malformations. Neurofibromatosis • Other causes include - postpneumonectomy statusQ, unilateral lung
causes both superior and inferior rib notching. agenesisQ.
24
GI Tract Imaging

CONCEPTUAL REVIEW OF RADIOLOGY


17. Double Bubble Sign
Clincher of Duodenal atresiaQ. 20. Double Duct Sign
Two bubbles are represented by air distended stomach and duodenal Visualization of both dilated pancreatic duct and dilated CBD on either
bulb. Also seen in annular pancreasQ, duodenal stenosisQ. This finding can be USG, CT scan or MRI. This sign is seen in Periampullary malignancyQ/
demonstrated on antenatal USG also. Associated with Down’s syndromeQ. Carcinoma head of pancreasQ. Other signs in Carcinoma pancreas include
Single bubble sign – Pyloric stenosisQ. widening of C loop of duodenumQ, Reverse 3 signs of FrostbergQ, Tear
Triple bubble sign – Jejunal atresiaQ. drop signQ.

21. Apple Core Deformity


This refers to the appearance on barium studies resembling an apple core
18. Bird Beak Sign after it is eaten from all sides. Seen in ulceroproliferative neoplasms of
Refers to smooth elongated tapering of distal esophagus in Achalasia colonQ, resulting from the mass lesion causing narrow core with overlying
cardiaQ. Associated with Chagas diseaseQ. Rat tail signQ is seen in irregular mucosa and with overhanging margins. Other signs in carcinoma
Carcinoma esophagus. colon include ShoulderingQ, Napkin ring signQ.

Imaging Signs in Radiology

19. Chilaiditi Sign 22. Saw Tooth Appearance


Interposition of colon between right dome of diaphragm and superior Refers to appearance seen in barium enema when contrast fills in the
margin of liver – is a cause of pseudopneumoperitoneumQ. When present outpouchings from the colon – DiverticulosisQ. Here the colon is body
with pain it is known as Chilaiditi syndromeQ. Careful look at haustral of saw and the diverticuli are the teeth of saw. Other signs seen in
pattern of colon in the subdiaphragmatic area will help to differentiate Diverticulitis include – Champagne glass signQ, Bowler hat appearanceQ,
from true pneumoperitoneumQ. Arrowhead signQ on CT.
25
CONCEPTUAL REVIEW OF RADIOLOGY

23. Esophageal Web with Jet Phenomenon 26. Lead Pipe Colon
Barium swallow image showing thin horizontal shelf-like filling defectsQ Loss of haustral pattern of colon seen on barium enema – suggestive of
around lumen of esophagus seen on barium swallow in the 1st spot Ulcerative colitisQ. In early UC Granular mucosaQ and PseudopolypsQ are
image. On fluoroscopy this web shows sudden distal propulsion of seen. Button hole ulcersQ, Collar stud ulcersQ, Undercut T-shaped ulcersQ
contrast known as Jet phenomenonQ in the 2nd spot image. Post-cricoid are also seen. Complications like Toxic megacolonQ can be identified on
web is associated with Plummer Vinson syndromeQ. plain radiographs with massive dilatation of transverse colon >6 cmQ.

24. Claw Sign 27. Coffee Bean Sign


Seen on barium enema where the contrast column falls short in a claw- Coffee bean-like appearance of the folded over-distended sigmoid colon
like pattern—classical for Intussusception. Other signs in Intussusception on plain radiograph of abdomen seen in Sigmoid volvulus. Other signs
include Coiled spring appearanceQ (barium), Target signQ, Bull’s eye signQ seen are Bent inner tube signQ, Omega signQ, Liver overlap/Splenic
Doughnut with crescentQ and Pseudokidney signQ (USG), Sausage signQ overlap/Left flank overlap signQ, Frimann-Dahl signQ, Northern exposure
(CT). Ileocolic – most common site in childrenQ. signQ. Sigmoid is the most common site of volvulusQ in GIT.
Imaging Signs in Radiology

25. Mexican Hat Sign


Seen in Pedunculated colonic polypsQ.
These polyps have discrete stalks and can be observed “en face”
hanging from the nondependent wall of the colon on double contrast 28. Step Ladder Appearance
barium enema study. Formed by the appearance of 2 concentric rings: Appearance of multiple air fluid levels stacked one upon another seen in
• Outer ring (yellow arrows)—“en face” visualization of barium coating abdominal radiograph. It is seen in small bowel obstruction. Other signs
the surface of the head of a pedunculated polyp seen are String of beads appearanceQ, Gasless abdomenQ (in proximal
• Inner ring (blue arrows)—meniscus of barium surrounding the stalk of high obstruction). Transition pointQ and small bowel feces signQ are seen
the polyp visualized through the head. on computed tomography (CT).
26
GU Tract Imaging

CONCEPTUAL REVIEW OF RADIOLOGY


32. Cobra Head Sign/Adder Head SignQ (Recent Pattern Jan 2019)
29. Small Bowel Feces Sign
Seen on intravenous pyelogram (IVU) in UreteroceleQ - the dilated ureter
Presence of particulate fecal matter with air interspaced between it at level of vesicoureteric junction (VUJ) protrudes into the bladder and
seen in the small bowel on CT scan. Has high specificity for small bowel is outlined like a cobra head. Similar appearance can be identified on
obstructionQ. ultrasonography (USG).

30. Football Sign 33. Keyhole Sign


Seen in PneumoperitoneumQ. Seen on supine abdominal radio­ graphs Seen on oblique Micturating cystourethrography film where there is
due to air accumulating beneath the anterior abdominal wall creating an dilated posterior urethra (black arrow) as a result of a slit-like Posterior
oval lucency all around the abdomen. Other signs in pneumoperitoneum urethral valveQ (white arrow). PU valves when seen on antenatal
include Falciform ligament signQ, Leaping dolphin signQ, Doge’s cap signQ, ultrasound have a spinning top appearanceQ. It is most common cause of
Rigler’s double wall signQ, Cupola signQ, Inverted V signQ, Tell-tale signQ, etc. bladder outlet obstruction in neonatesQ and exclusively seen in malesQ.

Imaging Signs in Radiology

34. Staghorn Calculus


31. Gastrointestinal String Sign (of Kantor) Large radiopaque shadow resembling horns of a stag seen on kidney,
Refers to string-like narrowed appearance of a barium-filled bowel ureter, and bladder (KUB) radiograph. This appearance is seen in the
segment. Classically described in TBQ or Crohn’s diseaseQ at level of IC struvite stones which originate in the renal pelvis and gradually extend
junction. But also seen in pyloric/duodenal stenosisQ (as in this case), into two or more calyces. These patients typically have history of recurrent
Carcinoma colonQ, Carcinoid tumorQ. urinary tract infection (UTI).
27
Musculoskeletal System Imaging
CONCEPTUAL REVIEW OF RADIOLOGY

38. Rain Drop Skull AppearanceQ-(Recent Pattern 2018)


35. Steinstrasse Multiple, well defined, uniform sized, punched out, lytic lesions seen
German word meaning street of stones. Radiologically it is seen as collection in lateral skull radiograph – suggests Multiple myelomaQ. The closest
of multiple small stones/fragments of larger stone which collect around a DJ differential is metastases which are however variable shaped and may
stent after Extracorporeal Shock Wave Lithotripsy procedure. Considered as involve mandible. Multiple myeloma deposits appear cold spotsQ on
a complication of extracorporeal shock wave lithotripsy (ESWL). 99m
Tc-MDP bone scan, whereas metastases appear as hot spotsQ.

36. J-shaped/S-shaped Ureter 39. Elevated Fat Pad Sign at Elbow


Seen in Retrocaval ureter . It is a variant in IVC development the result
Q Seen following trauma - prominence of posterior (and occasionally
being proximal ureter passing posterior to the IVC and emerging to the anterior) fat pads at elbow suggest HemarthrosisQ and points toward a
right of aorta. Resultant dilatation of proximal ureter occurs. possibility of an intraarticular fractureQ.
Imaging Signs in Radiology

40. Bamboo Spine/Poker Spine Appearance


37. Pine Cone Bladder/Fir Tree/Christmas Tree Bladder Fused spine due to bridging vertebral syndesmophytesQ – seen on
Appearance on IVU seen in Neurogenic bladderQ where the inverted Ankylosing spondylitisQ
triangular-shaped bladder with the multiple small diverticula resembles Other signs seen include Dagger signQ, Trolley track signQ, Romanus
a fir tree with its leaves. signQ, Shiny corner signQ, Anderson’s lesionsQ, Barrel vertebraeQ.
28
CONCEPTUAL REVIEW OF RADIOLOGY
41. Fluid-Fluid Levels on CT/MRI
Appearance seen in the Aneurysmal bone cyst (ABC)Q on magnetic
resonance imaging (MRI) due to presence of fluid of variable composition 44. Trident Hand
within each cyst. Fluid levels can also be seen in GCTQ, Simple bone cyst Refers to the approximately equal sized ring, index and middle fingers with
– post fractureQ, Fibrous dysplasiaQ, Malignant fibrous histiocytomaQ and divergent ends giving a Trident (Trishul)-like appearance. Though seen in
Telangiectatic osteosarcomaQ. various skeletal dysplasias it is considered classical for AchondroplasiaQ.

42. Codman’s Triangle and Sunburst Periosteal Reaction


Refers to the divergent thin bony spiculations arising from the long bone 45. Vertebra Plana/Dollar Vertebra/Wafer Vertebra/Coin Vertebra
like sun-rays. These spiculations also lead to elevation of periosteum Solitary flattened vertebral body seen in multiple conditions. If seen in
resulting in a Codman triangle. Commonly seen in aggressive conditions pediatric age group it is classical for Eosinophilic granulomaQ. In adults it
like OsteosarcomaQ, Ewing’s sarcomaQ but also in OsteomyelitisQ. can be seen in Paget’s diseaseQ Multiple myelomaQ.

Imaging Signs in Radiology

46. Radiodense Metaphyseal Bands


Appearance of sclerotic linear lines which follow the metaphysis, especially
in growing bones (pediatric age group) – seen in multiple conditions.
43. Licked Candy Stick Appearance
(Mnemonic DENSE LINES)
Gradual tapering of shafts of the metacarpals or metatarsals with pointed
ends giving appearance of licked ice candy. Seen in atrophic type of
• D vitamin excess/Deficiency (Rickets) • Estrogen
Neuropathic jointQ – due to impaired pain and proprioception. Common
• Elemental poisoning – especially lead • Leukemia
causes include Diabetes (most common), Leprosy, Tabes dorsalis, Charcot
poisoning • Infection (TORCH)
Marie tooth disease. Other findings in Neuropathic joints are Bag of
• Normal – constitutional • Early cretinism
bonesQ, Jigsaw vertebraQ, Tumbling building block spineQ.
• Systemic illnesses • Scurvy
29
CONCEPTUAL REVIEW OF RADIOLOGY

50. Coat Hanger Appearance


47. Shepherd Crook Deformity Seen on radiograph of long bone in pedunculated variety of osteochon-
Refers to the appearance of femur similar to the long stick with curved droma which is a bony outgrowth with distal bulbous end projecting away
end used by shepherds to catch hold of legs of their sheep. Commonly from adjacent joint.
seen in disease like Fibrous dysplasiaQ but can also be seen in Paget’s Diaphyseal aclasis – is multiple exostosis and has a higher risk of
diseaseQ, Osteogenesis imperfectaQ, MetastasesQ. malignant transformation (20% as against just 1% in a solitary exostosis).

51. Double Posterior Cruciate Ligament (PCL) Sign


Two-linear curved hypointense structures seen – Double PCL sign of
Bucket handle tear of medial meniscus at knee. A fragment of torn
meniscus, which appears as a low-signal intensity, longitudinally oriented
48. Subperiosteal Resorption Involving Metacarpals/Phalanges band (blue arrow) lying beneath and parallel to the normal PCL (yellow
Hallmark of HyperparathyroidismQ. Resorption occurs at the outer cortex arrow), creating a double PCL configuration. An intact anterior cruciate
at insertional points of ligaments and tendons. Radial margins of middle ligament (ACL) is required to prevent the flipped fragment from migrating
and proximal phalanges of index fingerQ – Most common site. further laterally – thus it is seen only in those patients with an intact ACL.
Imaging Signs in Radiology

52. Butterfly Vertebra


49. Hair on End Appearance Also known as Cleft vertebraQ, Sagittal cleft vertebraQ, Anterior
Refers to widening of diploic space with thickening of trabecula. The rachischisisQ, Anterior somatoschisisQ, and Anterior spina bifidaQ.
thickened trabeculae are oriented such that they appear to radiate Vertebral fusion defect of 2 lateral halves due to persistence of intervening
from skull inner table like erect hair. Seen in Thalassemia majorQ and notochord remnant, such that there is depression in between when seen
less commonly in other conditions like Sickle cell anemiaQ, Hereditary in frontal view on radiograph or coronal CT/MRI giving appearance of
spherocytosisQ. butterfly with open wings.
30
CONCEPTUAL REVIEW OF RADIOLOGY
53. Soap Bubble Appearance 56. Cumulus Cloud Appearance
Expansile lytic lesion in bone with thin internal septations–very peculiar OsteosarcomaQ is a malignant mineralizing neoplasm that breaches and
appearance seen only in few conditions, namely breaks through the cortex early in the course of the disease resulting in a soft
• GCT/Osteoblastoma – in this case tissue mass. This soft tissue mass is unmineralized initially but later shows
• Aneurysmal bone cyst ill-defined bone within. Mineralization of the tumor osteoid leads to a dense
• Telangiectatic osteosarcoma cloud-like appearance – Cumulus cloud appearanceQ (yellow arrows).
• Malignant fibrous histiocytoma Cumulus cloud appearance has also been described in Bone islandsQ/
• Bone angiosarcoma. EnostosisQ.

54. Knock Knees/Genu Valgum 57. Question Mark Sign


A condition where knees are directed medially and often touch Also known as buckling of posterior cruciate ligament (PCL)Q and refers to
each other. Commonly associated with rickets during the age of the question mark like configuration of the posterior cruciate ligament.
2–5 years when the child has started walking and bearing weight. But can Presence of this sign is an indirect indicator of a complete anterior
also be idiopathic or congenital. cruciate ligament tearQ.

Neuroimaging

Imaging Signs in Radiology

58. Dural Tail Sign


55. Bone-within-bone Appearance/Endobones
This refers to the enhancement of the dural lining adjacent to mass on
Seen in OsteopetrosisQ – also known as Marble bone diseaseQ/Chalk bone contrast enhanced MRI Brain. A dural tail is an MRI finding that strongly
diseaseQ/Albers – Schonberg diseaseQ. suggests that the lesion is a meningiomaQ. Studies suggest that the actual
Other features of Osteopetrosis include – Sandwich vertebraQ, “tail” is due to dural hypertrophyQ, rather than tapering of the actual men-
Generalized osteosclerosis (increased bone density)Q, Curved line in iliac ingioma. It is not specific for menin­giomas and is also seen in Pleomorphic
bone parallel to iliac crestQ, Erlenmeyer flask deformityQ. xanthoastrocytomaQ, GlioblastomaQ and other dural based lesionsQ.
31
CONCEPTUAL REVIEW OF RADIOLOGY

62. White Cerebellum Sign/Dense Cerebellum Sign


59. Dawson’s Fingers Abnormal hyperdense appearance of the both cerebellar hemispheres due
Diverging fluid attenuated inversion recovery (FLAIR) hyperintensities to relative hypodensity in the cerebral hemispheres.
seen arising perpendicular to the corpus callosum and seen in parasagittal  It indicates very severe hypoperfusion of the supratentorial
images. Indicates demyelination in pericallosal white matter and specific neuroparenchyma and a poor prognosis.
for Multiple sclerosisQ. Other appearances seen in multiple sclerosis (MS)  Seen in Global hypoperfusion/Anoxia and is also associated with
are T1 black holesQ, Venus necklaceQ, Ependymal dot-dash signQ. Reversal signQ.

63. Mount Fuji Sign


In a patient of Tension pneumocephalusQ (air within the cranium -
mostly post-traumatic) while scanning in supine position air collects in
the most nondependent (Frontal region). This air takes up a heaped-up
appearance giving the silhouette-like shape of Mount Fuji. It suggests
60. Puff-of-smoke Appearance that the pressure of the gas is at least greater than that of the surface
Seen in Moyamoya diseaseQ. Refers to the angiographic appearance of tension of cerebrospinal fluid (CSF) in between the frontal lobes. A similar
multiple tiny abnormal intracranial collateral vessels. These collaterals peaking sign – pointed tips of frontal lobes is a sign of raised induction
are formed due to progressive narrowing of supraclinoid parts of internal chemotherapy (ICT). Both these signs are considered as indications for
carotid arteries which is characteristic of Moyamoya disease. emergent neurosurgical intervention.
Imaging Signs in Radiology

61. Dense MCA Sign


64. Cyst-with-a-dot Sign
Refers to the hyperdense cord-like appearance of the proximal portions
of middle cerebral artery on noncontrast CT scan. This hyperdense It refers to a small cyst with slightly hyperdense wall with an eccentrically
appearance is due to presence of acute thrombus within the lumen and is placed hyperdense dot (scolex of the parasite) within it seen on CT scan/
an indicator of Acute strokeQ in appropriate setting when even other signs MRI.
of acute infarct are not visible on CT scanning.  This appearance is classical for vesicular stage of NeurocysticercosisQ.
32
CONCEPTUAL REVIEW OF RADIOLOGY
65. Eye of Tiger Sign
Refers to the abnormal low signal intensity on T2 axial images in the
Globus pallidus due to accumulation of iron with a small hyperintensity 68. Eiffel by Night Sign
(due to gliosis). It is abnormally enhancing thickened dural/pachymeninges seen in
It is seen in Pantothenate kinase associated neurodegenerationQ/ postcontrast coronal MRI images resembling the Eiffel tower illuminated
Hallervorden Spatz diseaseQ. But also seen in Wilson diseaseQ and Organ- at night. It is seen in Hypertrophic pachymeningitisQ the common causes of
ophosphorus poisoningQ. which are CNS TBQ, NeurosyphilisQ, SarcoidQ, Granulomatous polyangiitisQ.

69. Empty Delta Sign


It refers to the triangular non-enhancing area seen within enhancing
66. Molar Tooth Sign walls of dural lining in the superior sagittal sinus in case of venous sinus
Appearance of the midbrain due to abnormal elongation of the superior thrombosis in CE-CT/CE-MRI.
cerebellar peduncles. It is seen in Joubert syndromeQ which has an Similar appearance when seen on NC-CT scan is known as Pseudo-
underlying cerebellar vermis agenesisQ. The other sign associated with delta sign and is seen in Subarachnoid hemorrhage or empyema – causing
this syndrome is Bat wing/Umbrella-shaped 4th ventricle signQ. the hyperdense walls of the sinus with a relatively hypodense center.

Imaging Signs in Radiology

67. Dumbbell Shaped Tumor 70. Figure of 8 Sign


Dumbbell like appearance of a soft tissue neoplasm which has a Appearance of brain seen in Lissencephaly-pachygyria spectrum –
constriction at the level of neural foramina and a bulbous intraspinal and especially Type 1 – Classical LissencephalyQ.
extra spinal component. This shape is typical of Spinal nerve sheath tumors In this type few poorly formed gyri (Pachygyria) and a smooth outer
– Schwannoma (most common)Q, NeurofibromaQ, GanglioneuromaQ, surface – result in a figure of 8 appearance. It is usually associated with
NeuroblastomaQ. band heterotopia.
33
CONCEPTUAL REVIEW OF RADIOLOGY

71. Viking Helmet Sign (Left Image) Colpocephaly (Right Image)


74. Retort Shaped Fallopian Tubes(Recent Pattern 2020)
Prominent bilateral frontal horns of lateral ventricles with midline falx
touching roof of ventricle on coronal MRI. Seen in HydrosalpinxQ – due to a fimbrial blockQ. The fallopian tubes
Occipital horns of bilateral lateral ventricles are dilated and oriented blocked at fimbrial end appear dilated at one end (fimbrial) and show
parallel to each other on axial MRI Brain. gradual tapering at the other end (cornual) taking shape of Retort (an
Both these signs are seen in Corpus callosal agenesisQ. equipment in chemistry lab).

72. Starry Sky Appearance


Seen in Neuro cysticercal encephalitisQ – especially in HIV patients. 75. Breast-within-breast Appearance
Described on NC-CT – multiple calcified tiny cysts scattered in both Mammographic appearance of breast hamartomas (Fibroadenolipomas)Q.
cerebral hemispheres – Starry sky appearance. These benign lesions are well-circumscribed and contain a mixture
Also descried on T2W MRI – due to multiple hyperintense fluid filled of fibrous, glandular and fatty tissue (just like normal breast), and they
cysts seen in brain parenchyma bilaterally. appear very much like a second smaller breast sitting within a breast.

Women’s Imaging
Imaging Signs in Radiology

73. Banana-Shaped UterusQ (Recent Pattern Jan 2018)


Refers to banana-like shape of unicornuate uterus on hysterosalpingo­
graphy.
34
GENERAL RADIOLOGY
C hapter O utline
• X-rays • MRI Basics
CONCEPTUAL REVIEW OF RADIOLOGY

• Radiation Exposure, Protection and Guidelines • Contrast Media in Radiology


• Computed Tomography • Multiple Choice Questions with Explanations
• Ultrasound Imaging

X-rays
1
CLINICAL QUIZ
1. Laurel & Hardy were brought in for a Chest radiograph. The
technician did Laurel’s radiograph using exposure factors as
80 kV and 6 mAs. Considering Hardy’s body habitus which
of the following would be the most appropriate change in
exposure factors that is required to be done?
A. Decrease kV, Decrease mAs
B. Increase kV, Increase mAs
C. Decrease kV, Increase mAs
D. Increase kV, Decrease mAs

X-rays
ƒƒ X-rays are a part of Electromagnetic spectrumQ
ƒƒ Electromagnetic spectrumQ:
•• Spectrum comprising energy components/photonsQ
that are propagated in space by a combination of
electric and magnetic fields
•• In increasing order of frequencies/energies this
spectrum includes—Radio waves (Least frequency
and energy),Q Microwaves, Infra-red, Visible light,
Ultraviolet, X-rays and Gamma rays (Maximum
frequency and Energy).Q
•• All components travel in space at same speedQ—Speed
of light–3 × 108 m/sQ
•• All components travel in space by the same type of
waveQ, though the individual wave properties (like
amplitude, wavelength, frequency) may differ.
This is the 1st X-ray image that he took—an image of his wife’s—
Discovery of X-rays Mrs Bertha Roentgen’s - hand, with their engagement ring on it!

ƒƒ X-rays were discovered by Sir Wilhelm Conrad RöentgenQ,


on 8th Nov 1895Q. Atomic Structure—2 Basic Concepts
X-rays

ƒƒ Hence, he is designated as the Founding Father of ƒƒ In order to understand X-rays better we need to know
RadiologyQ and this day—8th November is celebrated as a few basic concepts about the structure of an atom.
International Radiology dayQ. Consider a Tungsten atom, for example:

36
CO Basic Concept 1
NC E P T

GENERAL RADIOLOGY
• Every shell in an atom has its own fixed Energy level associated
with it.Q
• All electrons in that shell lie at that fixed Energy level
• Energy levels for inner shells in a Tungsten atom are:

K - shell –70 keV

L - shell –12 keV

M - shell –2 keV
ƒƒ An atom consists of
•• Central nucleus—comprised of protonsQ and neutronsQ ƒƒ Now look what happens after the removal of the K-shell
•• Electrons orbiting around the nucleus in Shells— electron. In the K-shell there is just one electron left. Because
(K, L, M, N)… and so forth. Each shell can contain a of a deficient electron, an electron void/empty space is
fixed number of electrons—2, 8, 18, 32 and so forth created in innermost shell, thus making the atom unstable.
respectively. When all shells are filled with the entire The atom wants to become stable, but how? Think.
full set of electrons—the atom is considered a stable ƒƒ What happens is that an outer shell electron jumps to the
atom. inner shell that is L-shell to K-shell. Subsequently outer shell
ƒƒ Consider the K-shell electron in the image above (Yellow electrons jump to the underlying inner shells and finally a
circle). This electron—if you watch carefully—is a bound free electron in space fills the outermost shell—making the
electron, in the sense that it is bound by the positive charge atom stable again.
of the Nucleus and hence must stay in the K-shell. In fact, all ƒƒ Look at the jumping L shell electron—
electrons are bound by the positive charge of the nucleus in •• L-shell energy level is –12
their respective shells. The inner-shell electrons are tightly •• K-shell energy level is –70
bound whereas the outer-shell electrons are loosely bound. ƒƒ Thus if an electron has to jump from L-shell (–12) to K-shell
ƒƒ Scientists decided to conduct an experiment and tried to (–70) it will have to lose some energy so that it comes to
remove this innermost shell electron from a tungsten atom. an energy level of –70. Thus the energy lost is equal to the
Because it was bound by the nucleus, when they tried to pull energy difference between the shells, here it is 58 keV.
out this electron they had to spend some energy in pulling it
out—precisely around 70 keVQ.
ƒƒ Thus when they spent an energy of 70 keV, they pulled out
the K-shell electron. The following mathematical equation
just summarizes the above experiment: CO Basic Concept 2
NC E P T
 X (initial energy of electron) + 70 keV = Free electron
in space (has energy level of Zero) • Whenever an electron jumps from an outer shell to inner shell,
it has to lose some of its energyQ
So
• The amount of energy lost is equal to the difference between
X + 70 = 0
the energy levels of the two shells
Thus
X = –70 K-shell energy level –70 keV
ƒƒ X = –70 thus represents the initial energy level of the K-shell L-shell energy level –12 keV
electron. It is also the energy level of the K shell itself. If electron jumps from L shell to K shell 58 keV of energy
ƒƒ Remember that each and every shell in an atom has its own it will have to release
fixed energy level. All electrons in that shell are at exactly
that energy level, which is in K shell of tungsten atom. The
These 2 concepts will help us understand X-ray better!
energy level of all electrons is –70.
ƒƒ If amount of energy required to remove K-shell electron was
Tungsten
70 keV, the amount of energy required to remove an L-shell
electron would be less than that (as it is located away from Because Tungsten is a very important component of an X-ray
the nucleus) and it was found to be 12 keV. Similarly, for tube, let us quickly review few important aspects of tungsten:
M shell it was further less at 2 keV. ƒƒ Symbol—WQ
ƒƒ Atomic number—74Q
X-rays

ƒƒ Atomic mass—183.8Q
ƒƒ Atomic mass number—184Q
ƒƒ Classified as Transitional metalQ in the Periodic table

37
X-ray Tube Structure
From outside to inside the X-ray tube consists of:
ƒƒ Tube housingQ: Made up of a lead lined material and is
CONCEPTUAL REVIEW OF RADIOLOGY

filled up with an oil bath – inside which the Tube envelope


is kept.
ƒƒ Tube envelopeQ: Made up of glass/ceramic material –
inside which is vacuum and 2 electrodes – cathode and
anode.
ƒƒ Cathode: Given a very high negative potentialQ
•• Made of Tungsten filament/coilQ—hence called as
Cathode filament
ƒƒ Anode: Given a very high positive potentialQ
•• Conventional X-ray tubes—TungstenQ
ƒƒ Large potential difference—is applied across the anode
•• Recent X-ray tubes
and cathode—very important in production of X-rays
Tungsten (90%) + Rhenium (10%)Q/molybdenumQ
ƒƒ Anode may be stationary or Rotating anode Q (has better Production of X-rays
heat tolerating capacity Q)
It will be very easy to understand and remember using the
following Flow chart:

Mechanism of Production of X-rays


1. Continuous spectrum radiation/BremsstrahlungQ: Based on Law of thermodynamics—Energy can never be created, nor can it be
destroyed. But it can be converted from one form into another.
X-rays

38
Thus kinetic energy of the electron—is converted into energy of X-rays
2. Characteristic spectrum radiation: Based on the 2 basic concepts regarding structure of atom that we have discussed

GENERAL RADIOLOGY
Thus the amount of energy lost when an outer shell electron jumps to an inner shell is converted into energy of X-rays

Mechanisms of X-ray Production: Comparison


CO
NC E P T

Continuous spectrum radiation/Bremsstrahlung Characteristic spectrum radiation


Also known as White radiation/Braking radiation No specific synonym
Contributes to 70–80% of X-rays used in diagnostic radiologyQ Contributes to 20–30% of X-rays used in diagnostic radiologyQ
Consists of X-ray beam of variable energy—creates a Continuous Consists of X-ray beams of characteristic energy content—that is
spectrum of energies on a graph—hence the name equal to difference between energy levels of 2 shells
Used predominantly in most of diagnostic imaging Used predominantly in MammographyQ

Exposure Factors: Kilovoltage peak (kVp) ƒƒ Milliampere second (mAs)Q


•• Combination of:
and Milliampere second (mAs)  mAmpQ—Current passed through the cathode
ƒƒ Kilovolt peak (kVp): filament
•• Voltage applied across the cathode and anode in the  TimeQ—Time of exposure
X-ray tube ƒƒ Determines the number of X-ray photonsQ in the X-ray
•• Higher is the voltage, higher is the kinetic energy of beam
electrons hitting the Anode, greater is the penetrating ƒƒ Directly affects the contrast—Increased mAs—Increased
power of the emerging X-raysQ contrastQ
•• Thus High kVp—results in higher penetrating power of
X-raysQ Interaction of X-rays with Matter—What
•• kVp also affects Radiographic Contrast– Happens in the Patient’s BodyQ
 Low kVp—High contrastQ—called as Short scale
contrastQ ƒƒ These are ways in which X-ray beams interact with
 High kVp—Low contrastQ—called as Long scale the patient’s body, when they enter inside. Various
contrastQ interactions are known:
•• Compton effectQ—Most common interactionQ.
Discussed in detail here
X-rays

•• Photoelectric effectQ—Discussed as follows:

39
Compton Effect and Photoelectric Effect can be Confusing,
Hence Let us Study them in a Comparative Manner
CONCEPTUAL REVIEW OF RADIOLOGY

Compton effectQ Photoelectric effectQ


• Interaction of X-ray photon with outer shell electronQ • Interaction of X-ray photon with inner shell electronQ

• Most common interaction of X-rays with matterQ • Less common


• Is a mid-energy phenomenonQ • Is a low-energy phenomenonQ
• Most important outcome is scatter radiationQ. It causes image • There is no scatter radiationQ—hence image quality is better.
distortion, blurring and decreased diagnostic quality of the • Because there is no Scatter radiation, we try to maximize
radiographic image Photoelectric effectQ
• Hence, to minimize scatter radiation, we try to minimize Compton • Maximizing Photoelectric effect—
effectQ ƒƒ Using Low energy X-raysQ
• Minimizing Compton effect— ƒƒ High atomic number targetQ
ƒƒ Using High energy X-rays Q

•• Coherent scatteringQ/Classical scatteringQ/ •• PhotodisintegrationQ –


Thompson’s scatteringQ/Rayleigh scatteringQ –  Part of the nucleus of an atom (Alpha particle/
 Low energy X-ray photon transfers its energy to an neutron/combination) is ejected by a high energy
electron in an atom - the excited electron vibrates incident photon
thus releasing the excess energy as a scatter X-ray  Occurs only if incident photon is >7 MeV—not in
beam. No ionization occurs, only direction of X-rays diagnostic RadiologyQ
is changed •• Pair productionQ—
 If a single electron of atom is involved—  High energy photon hits a nucleus and photon
X-rays

Thompson’s scatteringQ disappears


 If all atomic electrons are involved together—  The photon (energy) is converted into 2 particles
Rayleigh scatteringQ (matter)—one electron, one positron
 Not important in Diagnostic radiology.  Occurs only if incident photon is >1.02 MeV—not
in diagnostic Radiology
40
Radiation Units Equivalent DoseQ—The letter of highlight here is E
It is a very factual topic and is very difficult to remember.
Let us try to apply some Mnemonic Secret strategies to remem-

GENERAL RADIOLOGY
ber them.

Radiation ExposureQ—(Keyword to focus on is Exposure)

• Conventional unit—REMQ
ƒƒ Rem—stands for Radiation Equivalent in Man
ƒƒ E is letter of highlight—so both units contain E prominently!
• SI unit—SievertQ
ƒƒ E is letter of highlight—so both units contain E prominently!

• Conventional unit—RoentgenQ
ƒƒ Whenever you read the word exposure just remember the Other Radiation Units
1st person exposed to X-rays was Roentgen—the unit.
ƒƒ Radio-activityQ:
• SI unit—Coulomb/KgQ
•• CurieQ—Conventional unit
•• BecquerelQ—SI unit
ƒƒ KermaQ:
Absorbed DoseQ—Just look at this term, and pick up the 1st
•• Kinetic energy released per unit mass of tissue
letter of each word

Effects of Radiation on Human Body


Acute Radiation SyndromesQ/Radiation
SicknessQ/Radiation ToxicityQ
Three classical types exist -
ƒƒ Hematopoietic syndromeQ/Bone marrow syndromeQ:
•• 1st organ system to be affected in the bodyQ
•• Threshold dose: Around 1–2 GrayQ
•• LD 50/60 is 2.5–5 GrayQQ
•• Drop in all blood cell counts occurs for several weeks.
•• Primary cause of death is infection and hemorrhage.
•• Survival decreases with increasing dose.
•• Most deaths occur within a few months after exposure.
ƒƒ Gastrointestinal syndrome:
•• 2nd organ system affected
• Conventional unit—RadQ •• Threshold dose is 6–10 GrayQ
ƒƒ Rad—stands for Radiation absorbed dose •• Symptoms are malaise, severe diarrhea, electrolyte
ƒƒ Just looking at Absorbed Dose you get A and D, so the unit imbalance
is Rad! •• Survival is extremely unlikely, death occurs within
• SI unit—GrayQ 2 weeks
ƒƒ Red—is a color, that in a typical Indian accent of English may ƒƒ CardiovascularQ/CNS syndromeQ:
be sometimes pronounced as Rad (No offence, I am a proud •• Threshold dose is around 20 GrayQ
Indian myself) •• Death is likely due to collapse of the circulatory system
ƒƒ If Rad is a color-resembling term, the SI unit that goes along as well as increased pressure in the confining cranial
with it is another color-resembling word Gray (similar to
X-rays

vault as the result of increased fluid content caused by


Grey) edema, vasculitis, and meningitis.
•• Death occurs within 3 days of exposure

41
CO Acute Radiation Syndromes (ARS)—Why do They Occur in a Particular Order?
NC E P T
CONCEPTUAL REVIEW OF RADIOLOGY

• ARS—is a factual topic you are supposed to remember. If you try ƒƒ Yes it is—Bone marrow. It contains undifferentiated stem
to remember it by mugging it up, you will find it difficult. Let us cells constantly involved in Haematopoiesis. Hence the
look at its Conceptual basis. Haematological/Bone marrow syndrome—is the earliest
• Law of Bergonié and TribondeauQ: syndrome to occur.
ƒƒ Basic concept in Radio-biology ƒƒ Next is GIT syndrome—as the mucosal layer is the cell
ƒƒ Whatever tissue/organ/region in the body has the maximum population undergoing active mitosis—the superficial layers
proportion of undifferentiated cells/cells in active mitosis will of cells is being replaced from basal layers
be more sensitive to radiation. ƒƒ Last is CVS/CNS—minimal proliferation—hence relatively high
ƒƒ Thus can you guess which tissue/organ will be the most doses are required.
sensitive?

Deterministic and Stochastic Effects of Radiation


SMART-WORK STRATEGY TIP: Remembering the various properties of these effects can be difficult. Hence let us study them in a
comparative manner. Also try to remember their examples first so that using those examples we can remember the individual properties.

Deterministic Effects Stochastic Effects


Examples • Acute radiation syndromes (discussed above)
Q
Radiation induced carcinogenesisQ
• CataractQ Genetic mutationsQ
• Skin changesQ—Erythema, ulceration Chromosome aberrationsQ
• SterilityQ
• Radiation myelitis
• Fibrosis
• Teratogenesis/Fetal death
Onset • AcuteQ and Sub-acuteQ effects Chronic effectsQ
Threshold dose • YesQ No threshold doseQ
Severity of effect • Directly proportional to doseQ Not related to doseQ
Risk of occurrence • Non-linear relationship with thresholdQ Linear relationship with no threshold (LNT)Q

Deterministic effects and dose relationship

Stochastic effects and dose relationship


1
CLINICAL QUIZ - SOLUTION
• Coming back to Laurel and Hardy! But kVp also affects contrast of the image inversely, isn’t it?
• Look at Laurel – he is so thin and fragile. So the X-ray technician So increase in kVp will decrease the image contrast. We can
adjusted the exposure factors accordingly. The radiographic compensate for it by changing mAs.
exposure achieved was perfect! • mAs: We have studied that mAs affects image contrast in a
• Now came Hardy. He is…well healthy and chubby! So the direct manner. So in order to increase the image contrast we
technician must adjust his exposure factors accordingly. We have must increase the mAs.
X-rays

studied the most important factors kVp and mAs and how they • So the answer to the Quiz will be B. Increase kV (to increase
affect the exposure/contrast. Let’s consider them individually. penetrating power) and Increase mAs (to improve image
• kVp: Affects penetrating power directly. It’s logical that for the contrast).
X-ray beams to penetrate through the huge Hardy- they would • If you feel stressed anytime this year….go watch an episode of
require high penetrating power. So kVp must be increased. “Laurel & Hardy” – it’s refreshingly funny!
42
Radiation Exposure,
Protection and Guidelines

GENERAL RADIOLOGY
2
CLINICAL QUIZ
2. A 32-year-old patient undergoes a routine CXR PA view for pre-
employment screening. Realizing that she has just missed her
periods she does a urine pregnancy – that comes as positive. A
USG scan done on the same day reveals that she is 5 weeks 6 days
pregnant. Extremely anxious, she comes to you for consultation.
Best advice you can give her is: (NEET 2020 Pattern)
A. Immediate termination of pregnancy
B. Mandatory invasive testing for genetic defects
C. Counselling, reassuring advice to continue with the pregnancy
D. Obstetric MRI

The following table denotes radiation exposures in common


imaging modalities:
Radiation Exposures–How to
Investigation Radiation exposure Type of Modality CO
NC E P T Remember this Table?
(Effective dose
measured in Milli- • Well do not expect some magic concept here…!
Sieverts–mSv) • But I have tried to put in some visual clues that may help you in
solving MCQs based on this table.

Radiation Exposure, Protection and Guidelines


CT AbdomenQ 10 CT scan/
• Green band: Spot radiographs: least radiation exposures
CT Thorax 8–9 Radionuclide
ƒƒ Patient is exposed to radiation only one/twice
studies
Dynamic Cardiac 6 ƒƒ Hence this group has least radiation exposure
scintigraphy ƒƒ CXR–Most common radiological investigation done, so
remember its value: 0.02 mSv
PETQ 5
ƒƒ Remember the Lowest in this group: Limb/Joint X-rays <0.01
Bone scan 4 mSv, and the Highest in this group Lumbar spine 1–1.5 mSv
CT Head Q
2–3.5 • Yellow band: Diagnostic procedures: intermediate exposure
ƒƒ Since multiple views (AP/Lateral/Obliques/Fluoroscopic
Barium enema Q
7.2 Diagnostic
screening) done, hence has exposure greater than Spot
Barium meal follow 3 procedures–
radiographs
through multiple views/
ƒƒ Remember the Lowest is MCU–1.2–1.4 mSv and the Highest
X-ray exposure to
Barium meal 2.6 Barium enema–7.2 mSv
patient
• Red band: CT/Radionuclide studies: highest exposure
Barium swallow 1.5 ƒƒ Tertiary diagnostic modalities like CT/radionuclide studies
MCUQ 1.2–1.4 have high radiation exposures
Lumbar spineQ 1.0–1.5 Spot radiographs: ƒƒ Remember lowest in this group is CT Brain–2–3.5 mSv,
Only 1 or 2 X-ray Highest is CT Abdomen (Highest of all modalities)–10 mSv
Abdomen X-ray 0.7 ƒƒ Also remember PET (commonly done now)–5 mSv
exposures to the
Hip joint 0.4 patient
Skull X-ray 0.06
CXR PA viewQ 0.02 PRINCIPLES OF RADIATION
Limb X-rays /Joint
Q
<0.01 PROTECTION
X-raysQ
ICRP–International commission on radiological protection
Reference: Values may differ across texts. I choose to rely on a guidelines includes the following 3 basic principles:
Standard reference book and trust it!
Grainger and Allison’s Diagnostic Radiology i. JustificationQ—means that no practice resulting in
Royal College of Radiologists: Making the best use of a department exposure to ionizing radiation should be adopted unless
of clinical radiology: guidelines for doctors, 5th edn. London, it results in sufficient net benefit to exposed individuals or
Royal College of Radiologists, 2003 society to offset the detriment. 43
ii. OptimizationQ—requires that the individual dose, the radioactive materials into the environment by employing all
number of people exposed and the likelihood of inadvertent “reasonable methods”
exposure should be kept as low as reasonably achievable ƒƒ ALARA is not only a sound radiation safety principle, but
(ALARA). also a regulatory requirement for all “radiation protection
CONCEPTUAL REVIEW OF RADIOLOGY

iii. LimitationQ—the exposure of individuals should be subject programs”


to dose limits designed to ensure that no individual is
exposed to an unacceptable radiation risk. 10–Day RuleQ
ƒƒ A rule proposed by the International Commission on
ALARA Principle Radiological Protection (ICRP)Q
ƒƒ As Low as Reasonably AchievableQ ƒƒ Applies to women of child bearing age
ƒƒ The ALARA radiation safety principle is based on the ƒƒ “Whenever possible, one should confine the radiological
minimization of radiation doses and limiting the release of examination of the lower abdomen and pelvis to the 10-
days following onset of menstruation”Q

  Clinical Pearls
When to do a Hysterosalpingography (HSG)? after the bleeding stops but before the 11th day of the cycle - so it
HSG is done by inserting a cervical cannula and injecting iodinated is Day 6 – 10 of the menstrual cycle.
contrast through it. When seen on a Fluoroscopy the contrast will
outline the uterine cavity, bilateral Fallopian tubes and then spill
into the peritoneal cavity. The image shown here is a spot image
of an HSG study showing bilateral dilated retort shaped tubes
and no peritoneal spill – suggestive of bilateral hydrosalpinx.
(BTW this was the image asked in your recent NEET 2020 pattern
examination!) So HSG is done in infertility evaluation to look for:
1. Tubal patency/Hydrosalpinx
2. Müllerian duct anomalies – like Unicornuate/Bicornuate/
Septate uterus
3. Uterine adhesions and other abnormalities
What would be the best time to do an HSG study then?
Radiation Exposure, Protection and Guidelines

Just use the 10-day rule! It can be done in the 1st 10 days of the
menstrual cycle. But it is impractical to do it during menses as it
would introduce infection. Hence the BEST TIME TO DO HSG is just

LATEST GUIDELINES
The system of radiation protection that is used across Europe and worldwide is based on the recommendations of: the International
commission for radiation protection (ICRP)Q; and the International commission on radiation units and measurements
(ICRU)Q. The conceptual framework adopted by the ICRP was substantially revised and updated in 2007 with the publication of ICRP
103 (ICRP 2007)Q.
Hence for all practical purposes we should follow these 2007 guidelines.

Recommended dose limits in planned exposure situations:


Type of Dose Occupational exposure Public exposure
Effective Dose • 20 mSv per yearQ, averaged over defined 5-year periods • 1 mSv in a yearQ
(100 mSv in 5 years) • A higher per year exposure may be allowed in
• Or provision that the effective dose should not exceed a single year, provided that the average over
50 mSvQ in any single year with the total dose at end of 5 defined 5-year periods does not exceed 1 mSv
years should be <100 mSv. per yearQ
Annual equivalent dose in:
Lens of eye 150 mSvQ 15 mSvQ
Skin 500 mSvQ 50 mSvQ
Hands and feet 500 mSv Q

Dose limits do not apply in emergency exposure situations where an informed, exposed individual is engaged in volunteered life-saving
actions or is attempting to prevent a catastrophic situation
Reference: ICRP Publication 103 The 2007 Recommendations of the international commission on radiological protection
44
The ICRP and AERB guidelines are exactly similar with just one difference:
ƒƒ ICRP–allows maximum exposure to occupational workers in any one year to be a maximum of 50 mSv, provided that the total
dose at end of 5 years should be <100 mSv.

GENERAL RADIOLOGY
ƒƒ AERB–allows maximum exposure to occupational workers in any one year to be a maximum of 30 mSv, provided that the total
dose at end of 5 years should be <100 mSv.

  Clinical Pearls
Imaging in Pregnancy
X-ray imaging
• For pregnant radiation workers, after declaration of pregnancy - 1 mSv dose to the embryo/fetus should not be exceededQ.
• “No single diagnostic procedure results in a radiation dose that threatens the well-being of the developing embryo and
fetus.”— American College of Radiology.
• “Fetal risk is considered to be negligible at 5 rad or less when compared to the other risks of pregnancy, and the risk of malformations
is significantly increased above control levels only at doses above 15 rad.”— National Council on Radiation Protection
• Women should be counselled that X-ray exposure from a single diagnostic procedure does not result in harmful fetal effects.
Specifically, exposure to less than 5 rad has not been associated with an increase in fetal anomalies or pregnancy loss.”— American
College of Obstetricians and Gynecologists
MRI
• “Although there have been no documented adverse fetal effects reported, the National Radiological Protection Board arbitrarily
advises against its use in the first trimester.”—  American College of Obstetricians and Gynecologists  and  National Radiological
Protection Board7
USG
• “There have been no reports of documented adverse fetal effects for diagnostic ultrasound procedures, including duplex Doppler
imaging.” “There are no contraindications to ultrasound procedures during pregnancy, and this modality has largely replaced X-ray as
the primary method of fetal imaging during pregnancy.”— American College of Obstetricians and Gynecologists

Radiation Exposure, Protection and Guidelines


Basic Three Factors for Radiation Protection (Working Personnel and Public)
1. TimeQ
2. DistanceQ
3. ShieldingQ

Thermoluminescent dosimeter (TLD) BadgeQ - NEET 2020 pattern question


• Thermoluminescent dosimeter (TLD)Q is a passive radiation detection device that
is used for personal dose monitoring or to measure patient dose.
• TLDs are composed of phosphor crystalsQ [lithium fluoride (LiF)Q, lithium
borate (Li2B4O7)Q, beryllium oxide (BeO)Q, and magnesium borate (MgB4O7)Q]
that measure ionizing radiation primarily by trapping propagated gamma and
neutron exposure.
• Incident energy is absorbed by some of the crystal’s atoms when exposed to
ionizing radiation, thereby producing free electrons and corresponding electron
holes. Free electrons are trapped in the band gap by the imperfect lattice
structure of the crystal that is created due to doping impurities.
• The crystal is heated, the crystal vibrates to release the free electron back to
its ground state. Trapped ionization is released as light, which is measured by
photomultiplier tubes. This value is in ratio with the ionizing radiation captured
by the phosphor, and represents the dosage administered to a personQ, provided
the equipment was mounted properly.
• TLDs can measure doses between 0.01 mGy to 10 GyQ

45
2
CLINICAL QUIZ - SOLUTION
• So the patient (who was unaware of her pregnancy) was exposed • The vast majority of radiodiagnostic procedures involve fetal
CONCEPTUAL REVIEW OF RADIOLOGY

to radiation when she underwent a CXR. Is it a big deal? Let’s radiation doses that are below the threshold of 5 rad.
look at some facts. • The need for radiodiagnostic procedures during pregnancy
• Direct exposure of a fetus to radiation occurs when the fetus is should be carefully considered and the risks should be weighed
located within the field being imaged. Indirect exposure is due against the benefits. A radiodiagnostic procedure should not
to scattered radiation from maternal tissues. be withheld from a pregnant woman if the procedure is clearly
• Fetal doses resulting from radiological examination of the indicated and if it can affect her medical care. Unnecessary
mother’s skull, head, neck, chest and extremities are extremely procedures (e.g., pre-employment screen, routine periodic
low (<0.01 rad) because of the relatively low maternal radiation check-up) should be delayed because these procedures do
dose, beam direction and distance between the primary field not immediately contribute to patient’s health care and might
and the fetus. provoke anxiety.
• The teratogenic effects of in utero exposure to ionizing radiation • So in this case the exposure from a CXR PA view is extremely low
are dose-dependent with a well-defined threshold. There is and hence the correct next step would be to counsel the patient,
no indication that radiodiagnostic doses of ionizing radiation re-assure her and ask her to continue with her pregnancy with
during pregnancy increase the incidence of gross congenital routine care! (Answer is C)
malformations, intrauterine growth retardation or abortion. The
risks of such exposure are far below the spontaneous risks.

Computed Tomography
3
CLINICAL QUIZ
3. Which of the following has the maximum radiation exposure?
(NEET Pattern 2016)
A. X-ray LS spine C. PET scan
B. Barium enema D. CE-CT abdomen

4. The walls of CT scan room are coated with: (AI 2015 Pattern)
A. Glass C. Iron
B. Tungsten D. Lead
Computed Tomography

ƒƒ Computed tomography is basically a fusion of 2 technolo- ƒƒ His machine was called the EMI scannerQ (Electrical and
gies: Musical Instruments Company) or the 1st generation
i. TomographyQ—X-ray-based imaging technique devel- scannerQ
oped to acquire sectional images of the body. ƒƒ There is also a unit scale named in his honor—Hounsfield
ii. ComputersQ—Brought in to deal with the complex unit scaleQ
mathematical algorithms and iterations in the image
reconstruction Basic Principle of CT Scan
X-rays were Discovered. CT Scan was “The internal structure of an object can be reconstructed
from multiple projections of that object”Q
Invented
ƒƒ Sir Godfrey Hounsfield—Inventor of CTQ. Known as
Founding father of CT technology. He was awarded the
46 Nobel Prize in physics jointly with Allan Cormack in 1979.
• An X-ray tube at Position A emits a beam that cuts through a slice of
the patient’s brain
• An image is acquired on detectors located diametrically opposite to

GENERAL RADIOLOGY
Point A—this image is Projection 1 (P1)
• P1—is just like a spot radiograph/Routine X-ray image
• Now the tube is moved to point B along the circumference of the circle
around the patient. The detectors are also moved to a diametrically
opposite position
• Same process is repeated—Projection 2 (P2) is acquired
• Realize that P1 and P2 will appear different as they are acquired from
different angles around the patient
• This process is repeated such that P3, P4, P5……….Pn projection
images are acquired
• All these images are transmitted to a Computer. Computer will
process all these images and will reconstruct the actual slice image
of the brain from all the projection data.
• This image is showing us the “Internal Structure” of the head region.

Read the CT principle again: “The internal structure of an object can be reconstructed from multiple projections of that object”

CT Scan Generations
Ist Generation: Translate—RotateQ Scanner
• Thin pencil-shaped X-ray beam
• Single detector
• Tube-Detector complex travels horizontally across the patient’s
body to cover the entire cross section—Translation
• 1 Translation = 1 Projection (P1)
• After every translation the entire setup rotates around the patient
• Again translation occurs resulting in P2
• Thus alternate translation and rotation occur
• Slow working machine

2nd Generation: Translate—RotateQ Scanner

Computed Tomography
• Similar to 1st generation except:
ƒƒ Fan-shaped X-ray beam
ƒƒ Row of detectors
• Lesser Translation and Rotation is required due to fan-shaped
X-ray beam
• Faster machine than 1st generation

Contd…

47
3rd Generation: Rotate—RotateQ Scanner
• Wide fan-shaped X-ray beam covering the entire patient cross
section—hence translation is eliminated
CONCEPTUAL REVIEW OF RADIOLOGY

• There are 2 Rotation movements:


i. Rotation of X-ray tube
ii. Rotation of Detectors
• Faster than 2nd generation

4th Generation: Rotate—FixedQ Scanner


• Entire 360° detector ring is present around patient’s body, hence
detector motion is eliminated
• There is only 1 Rotation movement –
ƒƒ Rotation of X-ray tube around the patient
• Faster than 3rd generation
• But requires a large number of detectors—added cost for similar
resolution to that of 3rd generation machines

5th Generation: Electron BEAMQ Scanner


• No moving part in entire machine
• Electron beam is fired on a tungsten ring around the patient’s
body—X-ray production occurs
• X-ray beam passing through patient’s body is detected by a
complete 3600 detector ring around patient’s body
• Fastest developed machine till date
• Still in experimental stages
Computed Tomography

Helical/Spiral CTQ
ƒƒ The X-ray tube in the CT scan machine – with so many wires attached to it – could not be rotated around the patient’s body. It
was possible to rotate it only for 1 rotation – then bring it back to starting point – patient table moved ahead – again tube rotated
for one rotation.
ƒƒ If the tube could be rotated around the patient continuously, non-stop data acquisition would be possible thus saving precious
time.
ƒƒ This was made possible by introduction of Slip Ring TechnologyQ. In order to conceptually understand how it works, let us
compare it with a daily example we often come across:

48
CO Slip Ring Technology and Chennai Express!
NC E P T

GENERAL RADIOLOGY
Chennai Express: The train not the movie!
• Travels from Mumbai to Chennai, covering a distance of around
1300 km in around 22 hours.
• How does the engine (of any electrically powered engine) receive its
electrical supply?
• High voltage Overhead cables—are laid above the track from
Mumbai to Chennai. There is a Pantograph on top of the train,
with horizontal metal rod above it, that maintains constant physical
contact with the overhead cable.
• Not even a single external fixed wire is attached to engine from
outside, so it is free to go wherever there are overhead cables.

CT Scan Slip Ring Technology: It is exactly similar to the above-


discussed mechanism:
• Overhead cables = Slip-Rings
• Railway engine = X-ray tube
• Here instead of 1 ring around the patient there are 2 rings:
ƒƒ Outer Ring: Stationary—like the overhead cables. All Electrical
wires are attached to it.
ƒƒ Inner Ring: Rotating with the X-ray tube mounted on it.
ƒƒ Now like the Pantograph and metal rod on the top of the engine,
there are brush contactors on the inner ring that maintain
constant electrical contact with the outer ring
• Thus the X-ray tube is rotated freely without the fear of any wires
being stretched/pulled.

• Now to cover the entire abdomen/thorax/brain:


ƒƒ The tube is rotated continuously around the patient

Computed Tomography
ƒƒ The table is also continuously moved in one direction

ƒƒ Thus if you imagine both these motions occurring simultaneously, it appears as if the tube is tracing the path of a Spiral/Helix
around the patient’s body—hence the name Spiral/Helical CTQ

Multidetector CTQ
ƒƒ In all the above CT machines, we have seen that only one slice of image is obtained at a time. That is because the beam is a fan-
shaped beam that cuts through only a single slice of the patient’s body in one go.
49
CONCEPTUAL REVIEW OF RADIOLOGY

Please note that each tissue has a range of corresponding HU values.


I have mentioned single values here so that it is easier for you to
remember.
The range is usually a range of 50–100 on either sides of the value
mentioned above.

ƒƒ Cone BeamQ: It is a 3-D X-ray beam shaped like a cone, so CO CT Physics—Practical Importance
NC E P T
that in one rotation around the patient it can cover a large
thickness (multiple slices) of the patient’s body, depending Do we really need to know all this physics at your level?
upon its third dimension. Well, like it or not the answer is yes for 2 reasons
ƒƒ To detect all the X-ray data, multiple rows of detectors 1. Questions have been asked in your examinations regarding HU
are placed beyond the patient’s body—hence the name values, CT generations and other related topics.
2. Practically it will help you in CT image interpretation in solving
Multidetector CT.
MCQs.
•• Depending upon how many slices of the body are ƒƒ Early scanners used to take around 25–30 minutes for a Brain
obtained in one cut, it may be called 16 Slice/64 study. With recent Multidetector 4th generation scanners this
Slice/128 Slice CT scanners. time has come down to a few seconds. This has made CT
the Imaging modality of choice in Emergency conditions like
CT Value ScaleQ/Hounsfield Unit (HU) Acute strokeQ, Head injuryQ etc. When time is not a constraint
ScaleQ MRI is better than CT for brain evaluation.
ƒƒ HU Values help us definitively arrive at a tissue level diagnosis.
ƒƒ In CT scan the appearance of a tissue is predominantly Just by putting a measurement pointer at a site and looking
based on “Density”Q of that tissue. at the HU value there helps us know what tissue it may be,
ƒƒ Because computers are involved, a huge advancement of thus making the assessment more accurate and objective.
this technology is that the appearance of each and every
tissue in the human body is allotted a specific numerical
value. This value allotted to a particular tissue is known as
CT: Descriptive Terminology
Computed Tomography

its Hounsfield Unit (HU) value/CT value.Q


ƒƒ It is calculated based on the following formula:

µx–µw
CT number = 1000 × µw
• µx = Linear attenuation coefficient of a tissue “x”
• µw = Linear attenuation coefficient of Water
• Main determinant of Linear attenuation coefficient of a
tissue—is its DensityQ

ƒƒ H.U. Value of Water:


•• Substitute µx as µw in the above equation and you will
get the HU value of water as 0—ZEROQ.
ƒƒ Look at the Hounsfield Unit scaleQ:

50
Right versus Left ƒƒ You see that the Cortical rim of gray matter is appearing
mildly hyperdense in comparison to mildly hypodense
ƒƒ Imagine that when the patient is lying in the CT machine,
central core of white matter. This distinction is also seen in
you are standing at his foot end and looking at the imaging
the central basal ganglia region, especially on right side.

GENERAL RADIOLOGY
slice from inferior aspect. Hence the Right—Left of the
ƒƒ So CT scan can differentiate between them, but it is not a
patient is as shown here by convention
very good distinction—as their densities are very close to
each other!
Densities
ƒƒ Hypo-denseQ—means Black/dark. Air (around the patient’s CT Room Shielding
skull), Water (CSF in ventricles) will appear hypodense
ƒƒ “Lead”Q is used the wall of CT scan room to prevent leakage
ƒƒ Iso-denseQ—means Gray. Generally all soft tissues (Brain)
of radiation outside the CT room. At least 1/16-Inch lead
in the body will appear isodense
shielding or equivalent is required for the walls, doors,
ƒƒ Hyper-denseQ—means White/bright. Acute hemorrhage
floors, ceilings, and operator’s barrier.
(Left basal ganglia hemorrhage here) and Bone (Skull)
ƒƒ The concrete equivalence of 1/16-inch thick lead would be
appear hyperdense.
about 4 to 6 inches of standard-density concrete.
Gray-White Matter Differentiation on CT
ƒƒ Though we have mentioned that Brain appears isodense,
can you differentiate the appearance of Gray and White
matter on the image above?

Special CT Applications
HRCTQ • High resolution computerized tomography.
• It is a technique in which, axial (cross sectional) images of lung are obtained using very thin slices.
• Lungs and Temporal bones are the two body structures which are imaged using HRCTQ.
• Evaluating a HRCT Thorax includes a pattern approach and is said to provide biopsy level diagnosis.
• A bone algorithmQ that uses high spatial resolutionQ to increase the contrast between 2 widely differing
densities is used, like air and vessels in thorax.
• Typically slice thickness of 0.625 to 1.25 mmQ are usedQ.
• Good patient breath-hold is necessary for it.
CT PerfusionQ • Perfusion—means capillary level blood flow
• CT Perfusion imaging helps us determine the actual tissue level/capillary level blood flow in a particular
tissue
• Iodinated contrast medium is injected rapidly and as it reaches the tissues, the rapid changes in tissue
density are detected by the machine and various parameters are plotted graphically.
• These include:
ƒƒ Mean transit time (MTT)Q or Time to peak (TTP)Q of the deconvolved tissue residue function (Tmax)
ƒƒ Cerebral blood flow (CBF)Q
ƒƒ Cerebral blood volume (CBV)Q
• Usually used in:

Computed Tomography
ƒƒ Ischemic stroke—to identify the penumbra
ƒƒ Brain/Head and neck Tumors
CT AngiographyQ • Involves injection of iodinated contrast for opacification and 3D image reconstruction of vessels.
• In many cases CT has been used in conjunction with catheter angiography, and in a few cases such as
imaging the aorta and the pulmonary arteries, CTA has supplanted catheter angiography as the gold
standard.
CT DensitometryQ/Q-CTQ • Quantitative CT is different from DEXA in that it provides separate estimates of trabecular and cortical
(Quantitative CT)Q bone BMD as a true volumetric mineral density in milligrams per cubic centimeter
• It can be performed at axial sites (like lumbar vertebrae) as well as peripheral sites (like distal radius)
• Quantitative CT is excellent for predicting vertebral fractures and serially measuring bone loss, generally
with better sensitivity than projectional methods (such as DEXA) because it selectively assesses the
metabolically active and structurally trabecular bone in the center of the vertebral body.
Contd…

51
Dual Source CTQ/Dual • Uses two separate energy sets to examine the different attenuation properties of matter.
Energy CTQ Applications:
• Virtual non-contrast image can be obtained. This only one set of CT images have to be acquired saving
CONCEPTUAL REVIEW OF RADIOLOGY

time and energy


• Fatty liver quantification for liver donors
• Assessment of stent stenosis in coronary arteries
• Post CT Angiography bone subtraction
• Renal calculi composition assessment
• Bone densitometry
Cardiac CT/CT Coronary • Assessment of Coronary arteries post stenting/Bypass procedure
AngiographyQ • Evaluation of coronary artery anatomy variants
• Being evaluated as a screening tool for Coronary artery imaging
• Calcium scoring of coronaries may be done
CT EnterographyQ: • Contrast medium is given per-orally, CT images acquired when the contrast reaches the small bowel. These
techniques however do not demonstrate mucosal abnormalities
CT Enteroclysis :
Q
• A contrast medium is introduced directly into small bowel through a nasojejunal tube and CT images are
acquired
CT ColonoscopyQ: • Following bowel preparation, the large bowel is distended with air/CO2 and Supine and Prone images are
obtained. 3D endoluminal projections are used to review the mucosal surface of colon.

3
CLINICAL QUIZ - SOLUTION
• We have studied the Radiation exposures in various modalities group. USG is used in most clinical situations like appendicitis,
(Green-yellow-Red zones) before. So if you look at the options & intussusception – this was a question asked in your AIIMS Nov
the values in the table the answer is obvious. 2014 and May 2015 examinations.
• Remember that amongst all modalities CT, PET and Radionuclide • There is always a lead lining around the CT equipment room – to
studies have the highest radiation exposures. Moreover CT prevent leakage of radiation outside the room. You must have
Thorax and Abdomen have the highest exposures of all. In seen that there is a glass window between the console room
contrast CT studies another round of acquisition is done after and the equipment room. Even this glass is a lead incorporated
contrast injection – hence the overall exposure increases further. glass to prevent leakage of radiation through it.
• Hence remember on account of this high radiation exposure –
CT Thorax and abdomen are usually avoided in the pediatric age

Ultrasound Imaging
4
CLINICAL QUIZ
5. In a 45-year-old patient of right sided breast mass, USG
examination revealed an equivocal appearance – that was
Ultrasound Imaging

graded as BI-RADS 3 lesion. A special application of ultrasound


technique was used (shown here) based on estimation of
hardness of tissues. Identify the technique:
A. High Intensity Focussed Ultrasound (HIFU)
B. Contrast enhanced ultrasound
C. Elastography
D. Spatio-temporal image correlation (STIC)

6. This special application has been proven to be useful in which


of the following indications?
A. Breast mass lesions C. NASH
52 B. Liver cirrhosis D. All of the above
Basics of Sound Spectrum ƒƒ This is how the USG probe works:
ƒƒ Sound—is just mechanical energy travelling through matter
as a wave. It cannot travel through vacuum

GENERAL RADIOLOGY
ƒƒ The velocity of sound beam in human body is—1540 m/sQ
ƒƒ Sound wave frequencies: Based on wave frequencies the
sound spectrum is broadly divided as shown in the image—

Simply put–
• Less than 20 Hz—InfrasonicQ
• 20–20000 Hz—Audible range of soundQ
• 20000 Hz—Ultrasound spectrumQ
Watch the image carefully. It tells us that anything >20000 Hz is
ultrasound, but the part of the Ultrasound spectrum that is used in
diagnostic imaging is the part above a frequency value of 1 MHzQ

Production of Ultrasound Beam—


The Piezoelectric EffectQ
ƒƒ This is the basic mechanism of production of ultrasound
beam
ƒƒ Piezoelectric crystalsQ/PiezoceramicsQ—Crystals having
property of Piezoelectricity
•• Natural crystals—Quartz
•• Artificial crystals—Lead zirconate-titanateQ (PZT)Q
—Most common crystal used in USG machines
ƒƒ Ultrasound Probe:
•• Consists of a Piezoceramic at its tip, fixed between 2
electrodes, and placed on a backing block. A wire is
attached to the probe and brings in the electric current
ƒƒ Piezoelectric effect: It is the property of such crystals to
convert one form of energy into another and vice versa.
Thus: Ultrasound Imaging
Ultrasound Probe and Frequencies
ƒƒ Probe is the primary device that is placed on the patient’s
body while doing an USG examination.
ƒƒ It emits ultrasound beams into the patient’s body and
receives the reflected ultrasound beam– this is used in
image generation.
ƒƒ Various types of Ultrasound probes are used as follows:

53
Frequency Probe name Applications Frequency Probe name Applications
(Mhz) (Mhz)
2–4 Pencil probeQ • Trans-cranial 7.5–12 Linear probeQ • Superficial organ
CONCEPTUAL REVIEW OF RADIOLOGY

Doppler (TCD)Q imagingQ—Neck/


probes thyroid/breast/
scrotum/shoulder/
tendons etc.
• Peripheral
vascular
DopplerQ—Upper/
lower limb—
arterial/venous
Doppler studies
>10 Endoluminal probesQ • Transvaginal
sonography (TVS)Q
• Transrectal
ultrasound (TRUS)Q
• Transesophageal
Echocardiography
(TEE)Q
3.5–5 ConvexQ/CurvilinearQ probe • Routine • Endoscopic
abdomen + pelvic ultrasound (EUS)Q
ultrasoundQ • Intravascular
• Routine obstetric ultrasoundQ
ultrasoundQ 50 Specialized UBM probes • Ultrasound
• Obstetric Doppler biomicroscopy
studiesQ/Renal (UBM)Q of eye
Doppler studiesQ • Please note that these Frequency values are just approximate
values. The actual values may differ from book-to-book/machine-
to-machine and should not create unwarranted confusion/
Contd… controversy!

Principle of USG Image Formation—The Pulse-Echo principleQ


ƒƒ A very simple concept is used, that was used in olden days for a technique called SONAR technique

Sonar technique Human ultrasound imaging


Ultrasound Imaging

54 Contd…
Sonar technique Human ultrasound imaging
Common formula is used:
Velocity (V) = Distance (D)/Time (T)

GENERAL RADIOLOGY
so
D=V×T
Here:
• D = Depth of ocean/Depth at which the organ is located from skin surface
• V = Velocity of sound in water/human body
• T = Time required to travel this distance. Since we can easily measure the time between emission of pulse and reception of echo
(by ship or by probe), if we use just half of this time we will get T. This is because between emission and reception the sound
beam has travelled the distance D twice, once as a pulse and once while returning as echo.
Thus by using this simple principle and formula we now know at what depth the organ is located from the patient’s body surface!
Important point to note here is that as the Ultrasound beam enters the body tissues, it may be reflected, refracted, absorbed or
attenuated. Of these fates of the beam it is Beam Reflection—that is used in image generation

Methods of Display of Ultrasound: How the Above Information is Displayed?


A—ModeQ • A—AmplitudeQ
• It is a very crude/basic way of representing data
• It is just a graph of:
ƒƒ Amplitude of returning echoesQ on Y axis
ƒƒ Depth of originQ on X axis
• Was used in Sonar technique
• A-Scan of eyeQ—It is the only current practical application in
human USG imaging

B—ModeQ • B—Brightness modeQ


• Most common method of display of ultrasoundQ
• The amplitude value is given a brightness value on a brightness
scale—and an image is created
• The brightness of every point on this image corresponds to the
amplitude of returning echoes arising from that point in the
body
• High amplitude = High brightness
• Low amplitude = Low brightness

Ultrasound Imaging
M—ModeQ • M—MotionQ
• Moving structures are described along a single line of the
ultrasound beam
• Rapid sampling of around 1000 times/second—ensures rapid
motion assessment—hence is used in Cardiac and Fetal cardiac
imaging

55
  Clinical Pearls
B-Mode USG Descriptive Terminology
CONCEPTUAL REVIEW OF RADIOLOGY

Hypoechoic:
• Low amplitude of returning echoes from a tissue results
in its appearing Dark/Black—Hypoechoic
• Seen in cases of fluid filled structures in the body—like
GB/UB/Cysts/Free fluid, etc.

Look at the hypoechoic appearing simple cyst at lower pole of left kidney
Isoechoic:
• Intermediate amplitude of returning echoes results in
Gray appearance—Isoechoic
• Seen in most of the solid organs—Liver/spleen, Muscles

Look at the relatively isoechoic appearing Liver parenchyma. While the


GB—being fluid filled is appearing hypoechoic
Hyperechoic:
• High amplitude of returning echoes results in White
appearance—Hyperechoic
• Seen in Air/Calculi in the body

Look at the hyperechoic appearing calculus at lower pole of left kidney

Doppler UltrasoundQ
Ultrasound Imaging

Routine • Emitter—USG probe on body Doppler


ƒƒ Extended application of Ultrasound—based on the Doppler B mode surface—Stationary effect—does
effect Ultrasound of • Reflector—Liver parenchyma- not occur
ƒƒ Used for Vascular imaging—detection of blood flow in Liver Stationary
blood vessels USG of Carotid • Emitter—USG probe on body Doppler
ƒƒ Doppler EffectQ (Named after the physicist Christian arteries surface—Stationary effect—
Doppler)—Change in frequency of sound whenever there • Reflector—RBCs in carotid occurs
is relative motion between the emitter of sound and the arteries—Moving
reflector of sound. The absolute change in frequency of • Thus there is relative motion
sound that occurs is called as Doppler shift (Fd)Q between emitter and
reflector of sound.

56
ƒƒ Doppler EquationQ Biologic Effects of Ultrasound on
Fd =
2FoVCosθQ
Human Body
C
•• Fd = Doppler shiftQ—that absolute change in frequency Ultrasound imaging is remarkably safe for imaging, with no

GENERAL RADIOLOGY
•• Fo = Transmitting frequencyQ—constant value major known adverse effects demonstrated in human exper-
•• V = velocity of blood flow in vesselsQ iments. However there may be 2 potential hazards associated
•• C = velocity of sound in human body (1540 m/s)Q with it -
ƒƒ Doppler angleQ:
•• Defined as angle between the ultrasound beam and Thermal EffectsQ
plane of blood flow ƒƒ Because Ultrasound is a form of energy, it may be deposited
•• Used in Doppler equation as Cosine value of that angle into the body tissues, resulting in heating of tissues.
•• In routine practice Doppler angle should be between ƒƒ This is the Ultrasound beam that is attenuated by the tissues
45 to 60 degrees
that gets deposited in the tissues and causes heating
ƒƒ Color DopplerQ:
ƒƒ Minimal rise in temperature noted in diagnostic ultrasound
•• Subjective methodQ of Doppler interpretation
imaging, with greater heating in Doppler applications.
•• The information is color coded based on direction of flow:
ƒƒ Thermal Index (TI)Q: Ratio of
 Blood towards the probe—Red colorQ
 Blood away from the probe—Blue colorQ Power produced by Probe
•• This Red-Blue allotment is arbitrary, can be reversedQ— Power required to raise tissue temperature by 1°C
hence subjective method
•• Also the color flow can be seen, but the flow cannot be
measuredQ—hence subjective method. Mechanical EffectsQ
•• Has low sensitivityQ for flow detection. Can miss ƒƒ The sound wave being comprised of alternate Compression
detection of low volume flow. and Rarefaction create rapid rise and fall in tissue pressures
ƒƒ Power DopplerQ: along the wave path
•• Subjective methodQ of Doppler interpretation ƒƒ Especially in organs that have a lot of air-fluid interfaces
•• Directional assessment is not doneQ (lungs, bowel), these pressure changes may cause physical
•• The color display is based on amplitude of returning echoes tissue damage due to rapid oscillations of tissue compo-
•• High sensitivityQ—Even low flow volume can be nents.
detected—Advantage over Color Doppler
ƒƒ This is termed as CavitationQ/Micro-cavitationQ
•• Color flow is seen, but the flow cannot be measured—
ƒƒ Mechanical index (MI)Q: Attempts to quantify the likeli-
hence subjective method
hood of cavitation. Value must always be less than 1.9.
ƒƒ Spectral DopplerQ/Pulsed DopplerQ
•• Objective methodQ of Doppler interpretation
•• The flow information is plotted as a graph of Velocity (Y Special Applications of Ultrasound
axis)Q and Time (X axis)Q
ElastographyQ • Discussed below
•• Thus accurate velocity measurements like Peak systolic
velocity (PSV)Q, End diastolic velocity (EDV)Q can be HIFU Q
• High Intensity Focused UltrasoundQ—
obtained—Hence Objective method. Non-invasive focused thermal ablation
technique
• Lethal heat is produced at the focus point—
causing thermal ablation of tissue
CO Ultrasound Gel—Why is it used? • Applications:
NC E P T
ƒƒ Uterine fibroid ablationQ
• US coupling gel is used to help transmit US waves to and from
ƒƒ Breast lesion ablationQ
the transducer.
ƒƒ Liver lesion ablation
• Reflection of sound waves occurs at interfaces where there is Ultrasound Imaging
a difference in the speed of propagation of sound waves, and, Contrast • Involves administration of IV contrast
as Ultrasound waves travel relatively slowly through air, the air- enhanced containing “microbubbles”Q
skin interface has potential to reflect a great deal of the waves ultrasound • These microbubbles when exposed to rapid
we would like to use for imaging. (CE-US) compression- rarefaction affect the sound
• Placing US coupling gel between the transducer and skin greatly reflection, and may result in enhancement
reduces this effect, so that maximum Ultrasound beam enters of tissues similar to that seen in CT/MR
the patient’s body. • Microbubbles measure around 6–8
• System is designed in such a way that both the transducer face micrometersQ so do not cause any risk of
and the coupling gel have acoustic impedances similar to that of air embolism
skin, thus minimizing reflection from skin surface
• USG Coupling Gel consists of:
ƒƒ Polyethylene glycolQ
ƒƒ GlycerineQ
ƒƒ Phenoxyethanol and Polyacrylamide gel 57
4
CLINICAL QUIZ - SOLUTION
• A specialized application of ultrasound that estimates the Hardness/ ƒƒ Strain elastography (also known as static or compression elas-
CONCEPTUAL REVIEW OF RADIOLOGY

Elasticity of tissues is known as Elastography (Ans to 1st Quiz is C). tography)


Elastography methods take advantage of the changed elasticity ƒƒ Shear wave elastography (also known as transient elastogra-
of soft tissues resulting from specific pathological or physiological phy)
processes. For example, many solid tumors are known to differ • The proven applications of USG Elastography include:
mechanically from surrounding healthy tissues. Similarly, fibrosis ƒƒ Breast tumorsQ—Benign versus Malignant
associated with chronic liver diseases causes the liver to become ƒƒ Assessing liver fibrosisQ
stiffer than normal tissues. Elastography methods can hence be ƒƒ Liver steatosisQ – NASH
used to differentiate affected from normal tissue for diagnostic • Thus answer to Quiz question is D. All of the above
applications. It can be of 2 types

MRI Basics
5
CLINICAL QUIZ
7. A patient comes to the casualty with a seizure and was advised
an MRI Brain for evaluation. He gives a history of surgery
1 month back and the adjoining radiograph. In view of the
internal fixation implant what would be the further manage­
ment of this patient?
A. MRI is contraindicated
B. MRI may be done with a cast immobilization of the lower
limb
C. MRI may be done routinely today
D. MRI may be done after 6 months

History of MRI resistance of the wire. At 4 Kelvin (-269ºC) electric wire loses
its resistance. Once a system is energized, it won’t lose its
ƒƒ Felix BlochQ—Elucidated Nuclear Magnetic Resonance
magnetic field.
(NMR)Q
ƒƒ In few 0.3 T machines a Permanent magnet may be used.
ƒƒ Raymond DamadianQ—
•• Discovered that malignant tissue had different NMR ƒƒ Magnets used in our hospitals range from 0.2 Tesla to 3 TeslaQ.
parameters than normal tissue. ƒƒ Stronger is the magnet—better is the Image resolutionQ.
•• Based on this discovery he produced the first ever NMR ƒƒ Remember that MRI has no Radiation exposureQ as X-rays
image of a rat tumor in 1974. are not involved.
•• In 1977 Damadian and his team constructed the first “Human MRI is based on Gyromagnetic propertyQ
super conducting NMR scanner (known as The Indom- of Hydrogen nucleusQ”
itableQ) and produced the first image of the human
Let us try and understand this statement
body, which took almost 5 hours to scan
ƒƒ Paul LauterburQ –
•• He was awarded the Nobel Prize in 2003Q for his contri- Structure of an Atom
butions for MRI along with Peter MansfieldQ. ƒƒ We have seen in X-ray discussion that an atom consists of
MRI Basics

a central nucleus and electrons around it. Inside any atom


MRI Magnet there are 3 types of motions occurring continuously –
ƒƒ Most commonly used magnets in our MRI machines are ƒƒ Electrons rotating around Nuclei
superconducting magnetsQ ƒƒ Electrons rotating around themselves
ƒƒ The magnetic field is generated by a current, which runs ƒƒ Nucleus (protons) rotating around themselves—Most
through a loop of wire. The wire is surrounded with a important for MRIQ
coolant, such as liquid helium, to reduce the electric
58
ƒƒ This Nuclear spin is called—Net nuclear magnetic spin— Why is Human MRI Based on
and is a pre-requisite for MRI. This spin gives rise to a
property of the nucleus called as Gyro-magnetic propertyQ
Hydrogen Nucleus?
ƒƒ Atoms with a strong nuclear magnetic spin are called MRI ƒƒ It is most abundantly present atomQ in the body. 60–70% of

GENERAL RADIOLOGY
active nuclei. The MRI active nucleiQ (and their atomic mass body weight is formed by water (H2O)—each molecule has
numbers) in the body are: 2 hydrogen atoms
ƒƒ Hydrogen–1, Carbon–13, Nitrogen–15, Oxygen–17, Fluo- ƒƒ Maximum Gyromagnetic ratioQ: In quantum physics, there
rine–19, Sodium–23, Phosphorus–31 is an entity called “Gyromagnetic Ratio”. It is beyond the
scope of this discussion. It is enough to know that this ratio is
different for each proton, being maximum for hydrogen.

Basic Steps of MRI Functioning


Step 1 Step 3

• Under the influence of Earth’s magnetic field (weak magnetic • While aligned the nuclei begin to Precess/Wobble along an axis.
field), all the Hydrogen nuclei in our body are randomly oriented, • They process at a particular frequency called Larmor frequencyQ
thus cancelling out each other. Our bodies do not have any given by the equation:
significant magnetism ω = gBoQ
where:
ω is Precessional/Larmor frequencyQ
g is gyromagnetic ratioQ
Bo is Magnetic field strengthQ
Step 2 Step 4

• When placed under a strong external magnetic field (like an MRI • Now back to the Net magnetization vector.
magnet) all the nuclei get aligned along the plane of the external • The NMV is also called as Longitudinal magnetizationQ—along
MRI Basics

magnetic field (Bo) the longitudinal axis of Bo—the external magnetic field
• Majority align parallel to Bo, few align antiparallel.
• After cancelling out a few anti-parallel ones, the human body
now develops a Net Magnetization Vector (NMV)Q along the
direction of Bo

Contd…

59
Step 5
CONCEPTUAL REVIEW OF RADIOLOGY

• An excitation Radio-frequency (RF) pulse is applied at the exact


Larmor frequency of H-nucleus
• This transfers energy to the NMV and flips it into the Horizontal
plane—Transverse magnetization
• This process is called excitation

Step 6 • T1 relaxationQ:
ƒƒ Occurs along Z axis
ƒƒ Is also called Spin-Lattice relaxation
ƒƒ T1 recovery timeQ—is the time the nucleus
takes for 63% of longitudinal magnetization
to recover
ƒƒ Determines T1 contrastQ
• T2 relaxation/decayQ:
ƒƒ Occurs along X-Y axis
ƒƒ Also called Spin-Spin relaxation
ƒƒ T2 relaxation timeQ - is the time the nucleus
takes for 63% of transverse magnetization
to be lost
ƒƒ Determines T2 contrast
• Because every tissue has different and peculiar
T1/T2 relaxation properties, each tissue will
appear different on T1/T2—MRI thus has
excellent soft tissue contrast resolution

Step 7 Image acquisition

• During T1/T2 relaxation the excess energy acquired from the RF


pulse is being emitted out
• This energy is received by the Receiver coils of the machine at
particular times—and is used for Image formation.
• Thus the MRI image is formed.
MRI Basics

• I had warned you MRI is bizarre! Now let us move into relatively easy things in MRI!!

60
  Clinical Pearls
MRI—Descriptive Terminology and Important Sequences

GENERAL RADIOLOGY
Since in MRI we detect the intensity of signal arising from the nuclei/protons, the appearance is described as:
• HypointenseQ: Dark
• IsointenseQ: Intermediate
• HyperintenseQ: Bright

T1 Weighted sequence (T1W) T2 Weighted sequence (T2W)

CSF/Water: HypointenseQ (dark) CSF/Water: HyperintenseQ (bright)


Fat: HyperintenseQ (bright) Fat: HypointenseQ (dark)
Gray-White matter appearanceQ (As per their names) Gray-White matter appearance (Opposite to their names)
• Gray matter—is GrayQ (isointense) • Gray matter—is hyperintenseQ
• White matter—is WhiteQ (hyperintense) • White matter—is hypointenseQ
Better for depiction of anatomyQ Better for depiction of pathologyQ

CO The FAT Controversy


NC E P T
Which of the following appears hyperintense (bright) on both FAST SPIN ECHO/TURBO SPIN ECHO sequencesQ:
T1W and T2W images? The technique of acquiring images was slightly modified and this
A. Gallbladder is what we use in our daily practice today. These are acquired in
B. CSF a much shorter time than conventional sequences and hence are
C. Kidney preferred. But because the acquisition technique is slightly modified
D. Fat the appearance of FAT on FAST/TURBO SPIN ECHO sequences is
slightly different.
Answer: D. Fat (How????)
• That is Fat appears Hyperintense (Bright) on both T1 and T2W
Do not panic. There is a reasonable answer to it. images.
This is a Controversial question - rather because it has been To summarize remember:
framed incorrectly. • Conventional sequences (Theoretical, not used because time
On Conventional MRI SequencesQ: consuming)
MRI Basics

• T1W: Fat is Bright, Water is Dark. ƒƒ T1: Fat - hyperintense,


• T2W: Fat is Dark, Water is Bright. ƒƒ T2: Fat - Hypointense.
But Conventional T1/T2 sequences are very time consuming and BUT
hence are not used in routine practice. • Fast Spin Echo/Turbo Spin Echo (Used in clinical practice)
ƒƒ Fat is Hyperintense on both T1 and T2.
No more confusion. I think the examiners also need to be reasonable,
show some courtesy and set the questions correctly!

61
  Clinical Pearls
The T2 FLAIR Sequence
CONCEPTUAL REVIEW OF RADIOLOGY

T2 Weighted (T2W) T2 Fluid Attenuated Inversion RecoveryQ (FLAIR)Q

CSF/Water: Hyperintense CSF/Water: Hypointense—FLUID signal is attenuatedQ—hence


the name!
Gray-White matter appearance (Opposite to their names) Gray-White matter appearance (Opposite to their names)
• Gray matter—is hyperintense • Gray matter—is hyperintense
• White matter—is hypointense • White matter—is hypointense
Better for depiction of pathology Can detect even the smallest of lesions

Why is T2-FLAIR needed?


Almost all abnormalities in the brain appear bright on T2W images. But CSF also appears bright on T2W images. This bright background
(of CSF) makes the bright spot in the brain (the actual abnormality) very inconspicuous. But in FLAIR the CSF signal intensity is deliberately
suppressed. So now against a dark background of CSF (because it is FLAIR) the abnormality in the brain is very well appreciated. Look
at the following example:

  T2W FLAIR

Lesion Hyperintense (look at the left frontal region) Hyperintense


CSF Background Hyperintense Hypointense
MRI Basics

Lesion Not good Very good. FLAIR increases the conspicuity of the lesion
prominence (like movie screen in a dark theatre hall!!)

62
Other Important MRI Sequences
STIRQ: Short tau Inversion RecoveryQ

GENERAL RADIOLOGY
• Inversion recovery sequence similar to FLAIR
• FatQ - Signal intensity suppressed
• Great use in Musculoskeletal imagingQ
• Most bone lesions appear hyperintense on STIR
• STIR is excellent to identify Marrow edema

GREQ: Gradient Echo sequencesQ/T2* sequenceQ


Or SWIQ: Susceptibility weighted imagingQ

• Susceptible to minute magnetic field inhomogeneity.


• BloomingQ/Intense loss of signalQ/Black spots are seen in
presence of:
ƒƒ Blood productsQ or
ƒƒ CalcificationQ
• Known as Blood sensitive sequenceQ
ƒƒ SWI has better sensitivity than GRE

DWIQ: Diffusion weighted imagingQ


• Based on diffusion of molecules in tissues due to random
thermal motion
• Abnormalities described as Restricted diffusion/Facilitated
diffusion
• Restricted diffusion in Brain is seen inQ:
ƒƒ Acute infarct
ƒƒ Abscess/Empyema
ƒƒ Diffuse axonal injury
ƒƒ Lymphoma
ƒƒ Epidermoid
ƒƒ ADEM
ƒƒ Carbon monoxide poisoning

PWIQ: Perfusion weighted imagingQ

• Assessment of tissue perfusion


• Involves injection of Gadolinium contrast
Parameters measured include:
1. Mean transit time (MTT)Q or Time to peak (TTP)Q of the
deconvolved tissue residue function (Tmax)
2. Cerebral blood flow (CBF)Q
MRI Basics

3. Cerebral blood volume (CBV)Q


Usually used in:
• Ischemic strokeQ
• Brain/Head and neck TumorsQ

Contd…
63
Fat suppressed images
CONCEPTUAL REVIEW OF RADIOLOGY

• May be T1 or T2W
• Used to differentiate fat from other lesions
• Postcontrast MRI images are always Fat-suppressed T1W
images

PDQ: Proton Density imagesQ

• Nuclear magnetic resonance of protons (hydrogen ions)


forms the major basis of MRI. Hence it is used such that
signal is weighted to reflect the actual density of protons; an
intermediate sequence sharing some features of both T1 and
T2.
• Proton density images were extensively used for brain
imaging, however they have largely been replaced by FLAIR.
• PD however continues to offer excellent signal distinction
between fluid, hyaline cartilage and fibrocartilage - makes it
ideal in the assessment of joints.Q

MRI AngiographyQ

• Various types:
ƒƒ Time-of-flight (TOF) MRAQ—Most commonly used. Does
not require injection of any contrast
ƒƒ Contrast enhanced (CE) MRAQ—requires injection of
Gadolinium compounds
ƒƒ Phase contrast (PC) MRAQ—Can detect direction and
velocity of flowQ

MR VenographyQ

• Most commonly used for suspected intracranial venous sinus


thrombosis
• Does not require injection of contrast
MRI Basics

Contd…
64
MR SpectroscopyQ

GENERAL RADIOLOGY
• Detects chemical composition of tissuesQ
• Produces a spectrumQ rather than images
• May be single/multivoxel typeQ

Magnetic Resonance Cholangiopancreatography (MRCP)Q

• Heavily T2W images suited for demonstration of fluid filled


structures
• Used for demonstration of biliary tree and pancreatic ducts.
Entire biliary tree is visualized.
• Equivalent to ERCP

Blood Oxygen Level Dependent (BOLD) Imaging/Functional MRI • Echo planar imaging (EPI), a fast MR image acquisition
imaging (fMRI) technique is used.
• During the fMRI image acquisition, the patient is asked to
alternatively perform several tasks or is stimulated to trigger
several processes or emotions..
• The detection of brain areas which are used during a condition
is based on the Blood Oxygenation Level Dependent (BOLD)
effect. When neurons are activated, the resulting increased
need for oxygen is overcompensated by a larger increase in
perfusion. As a result, the venous oxyhemoglobin concen-
tration increases and the deoxyhemoglobin concentration
decreases. As the latter has paramagnetic properties, the
intensity of the fMRI images increases in the activated areas.

Lesions/Tissues Appearing Hyperintense/Bright Contraindications for MRIQ:


MRI Basics

on T1W ImagesQ (Remember as PF2M4) ƒƒ Absolute contraindicationsQ—


ƒƒ Proteinaceous fluid •• Cardiac pacemaker
ƒƒ Fat •• Metallic foreign body of eye
ƒƒ Slow Flow •• Ferromagnetic hemostatic aneurysm clips in CNS
ƒƒ Melanin ƒƒ Relative contraindicationsQ –
ƒƒ Methemoglobin—in Subacute hemorrhage •• Cochlear implants
ƒƒ Mineralization—copper (Wilson’s), Manganese deposition •• Claustrophobia
ƒƒ Contrast Medium—Gadolinium •• Insulin pumps/Nerve stimulators 65
•• 1st trimester pregnancy •• To prevent electromagnetic radiation generated by
•• Epidural catheters/CNS Ventricular catheters the MR scanner from causing interference in nearby
•• Spinal fixation devices medical devices.
•• Feeding tubes ƒƒ The RF-shield must encircle the entire room - walls, floor,
CONCEPTUAL REVIEW OF RADIOLOGY

•• Prosthetic heart valves and ceiling. Such a conductive box used to shield out stray
•• Orthopedic External fixators electromagnetic interference is also known as a Faraday
cageQ.
Faraday CageQ ƒƒ Virtually any type of metal can be used, including aluminum
ƒƒ Radiofrequency (RF) shielding of an MR scanner is and galvanized steel. However, the most common RF-
mandatory: enclosure consists of wood panels wrapped with copperQ.
•• To prevent extraneous electromagnetic radiation from
contaminating/distorting the MR signal

5
CLINICAL QUIZ - SOLUTION
• Doing an MRI in a patient with a metallic fixation device seems Vascular stents and coils:
dangerous, isn’t it? • These rely on granulation tissue ingrowth to provide retentive
• Well its not! strength. This process takes 6–8 weeks, after which the device
Internal fixation devices: can be safely scanned without risk of loosening. However CNS
• All contemporary implants show no deflection towards the aneurysm clips are considered an absolute contraindication for
magnetic field in the laboratory and at the 1.0 Tesla MR machine MRI.
portal. The composite alloy of stainless steel is effectively Anterior Cruciate ligament reconstruction screw:
nonmagnetic and hence does not raise any safety issues. • This screw has significant ferromagnetic property.
Implant quality titanium contains a trace amount of iron and, • But as it is tightly implanted into the bone – it des not move/
in the annealed state, its structure is completely nonmagnetic. loosen at contemporary strength MRI magnets.
Moreover the implants are fixed into bones with screws and  Another concern with metallic implants is the Heating effect. Metals
nails, with the body’s weight acting on them and hence are are efficient conductors, and may therefore become preferentially
rendered immobile and fixed. heated due to eddy current and RF effects. However This rise in
External fixators: temperature of orthopedic implants after excessive bombardment
• The external fixators exhibit significant ferromagnetism. While with RF pulses is deemed negligible
the bulk of the clamp is nonmagnetic, the bolts (less than  So you can freely do an MRI after orthopedic internal fixation
10% weight of the clamp) are highly ferromagnetic. Strong devices. Do it with after a gap of 6–8 weeks after coronary stenting.
magnetic attraction of these components of external fixators Never do it with CNS aneurysm clips in situ or with external
contraindicates the use of MRI orthopedic implants!

Contrast Media in Radiology


Contrast Media in Radiology

6
CLINICAL QUIZ
8. A patient underwent left hemi-thyroidectomy for a thyroid nodule. On the
2nd day after surgery patient complained of difficulty in swallowing water
followed by mild swelling and pain in neck. Suspecting iatrogenic esophageal
injury, the surgeon did an oral contrast study – which is shown here. Which
of the following contrast material is the best one to be used in this clinical
situation? (NEET 2018 Pattern)
A. Barium
B. Gadolinium
C. Gastrografin
D. Iohexol

66
CO Definition of Contrast Media Classification
NC E P T

GENERAL RADIOLOGY
To be honest, no exact definition of Contrast medium exists in
literature. Here is my perspective of what a contrast medium is, and
it helps us understand it conceptually.
“Contrast medium is any agent used to enhance the appearance of
a structure on a radiological image”
For example:
1. On a routine Chest radiograph, we see the mediastinal soft tissues
as a midline opacity. But can you see the esophagus separately
in the mediastinum? No. Now ask the patient to swallow some
barium, and as soon as he does, take another chest X-ray. Now the
esophagus would be outlined by the barium contents. Thus Barium
enhances the appearance of esophagus—it is a contrast medium!
2. Similarly on an Abdomen X-ray we cannot see the kidneys/
ureters separately from other soft tissues. Inject some Urografin
and after same time if we repeat the abdomen radiograph we
can see the renal outline and the ureters filled with white/dense
material. Thus Urografin (Iodinated compound) has enhanced
the renal/ureteric structures—it is a contrast medium!

X-RAY AND CT CONTRAST MEDIA

Negative versus Positive Contrast Media


Negative contrast mediaQ Positive contrast mediaQ
There is an Intussusception seen in the right iliac fossa. But seen Barium-filled esophagus with a smooth elongated tapering at its
how the fluid within the dilated proximal small bowel loops helps distal end—is Achalasia cardiaQ
us recognize these bowel loops separately from other soft tissues.

Contrast Media in Radiology


Examples: Examples:
AirQ—used to distend bowel loops for Double contrast Barium BariumQ/Iodinated contrast mediaQ used in Bowel-related studies
enema (DCBE)/CT Colonography Iodinated contrast mediaQ used in IVU/MCU/RGU
WaterQ/MannitolQ—Used to distend bowel loops prior to CT
abdomen

Barium Contrast Media •• BaSO4—is extremely InertQ. It does not cause any
irritation/damage to bowel mucosa. It does not
ƒƒ Barium sulfateQ—BaSO4Q: Most common barium com-
interfere with normal process of digestion/absorption.
pound used
•• It does not itself gets absorbedQ (being water
ƒƒ Why is it used?
insoluble)—remains in the bowel lumen where we
•• Barium has a High atomic number—56. Hence it is want it to be. 67
highly radio-opaque
•• Is non-toxic
Used for endoluminal bowel fluoroscopic/spot studies like:
•• Coats the mucosa uniformly thus revealing mucosal
patterns/abnormalities Barium enemaQ For large bowel and
ƒƒ Where is it used? ileocecal junction
CONCEPTUAL REVIEW OF RADIOLOGY

evaluation
Used for endoluminal bowel fluoroscopic/spot studies like:

Barium swallowQ For evaluation


of hypopharynx,
esophagus and
gastroesophageal
junction

Barium distal loopogramQ For evaluation of the


distal loop of bowel
after a colostomy,
prior to colostomy
closure. Iodinated
contrast medium is
usually preferred
Barium mealQ For evaluation
of stomach and
proximal duodenum

ƒƒ How is it used?
•• Used as a BaSO4 suspensionQ—not a solutionQ because
Barium meal follow-through (BMFT)Q For small bowel BaSO4 is water insoluble
evaluation
•• High density suspensions—preferred for Fluoroscopic
studiesQ
Contrast Media in Radiology

•• Low density suspensions—used for CT studiesQ


ƒƒ Contraindications for use of Barium:
•• Absolute contraindicationQ: Perforation of bowelQ–
as it causes severe inflammation/chemical peritonitis/
pleuritis/mediastinitis—which may be fatal
•• Relative contraindications:
 Suspected tracheoesophageal fistula/Rectovagi-
nal/Rectovesical fistula—Iodinated contrast is
better alternative in such cases
 Prior allergic reactions
 Left sided colonic obstructionQ—If it remains
static in an obstructed colon it may get inspissated
and hard leading to severe constipation
Contd…

68
Iodinated Contrast Media
Iodine – A Water-soluble Contrast
MediumQ NEET 2019 Pattern

GENERAL RADIOLOGY
ƒƒ Atomic number—53—has excellent radio-opacity, hence
seen better separately from soft tissues.
ƒƒ Iodine content of the molecule determines its radiographic
density. More is Iodine, more is density—whiter/denser it
appears—so that it is better differentiated from adjacent soft
tissues
ƒƒ Has low toxicity profile in the body

Iodine: Particle Ratio—Another Boring


CO
NC E P T Ratio
Classification of Water-soluble Iodinated
This topic of Contrast media is a very factual topic. We are dealing
with classifications, names, lists and hence it is also very volatile/ Contrast Media
difficult to remember. To add upon it is a Numerical ratio here that Ionic Monomers: I/P Ratio = 3:2Q—
could add to our woes. But what if we understood what it means
conceptually rather than just mugging it up. Let us try!
Lowest and Worst RatioQ
ƒƒ These are sodium/meglumine salts of Triazoic acid/
• Iodine-Particle ratio/Contrast agent ratio =
Iothalamic acid
Number of Iodine atoms ƒƒ I/P ratio is 3:2—not a good ratio
Number of particles per mL solution ƒƒ Examples:
•• Diatrizoic acid salts—UrografinQ, GastrografinQ, Angio-
Numerator: Number of Iodine atoms—Determines Attenuation
graffinQ, TrazograffQ, UrovisionQ, UrovideoQ
• We have seen above that the Iodine content will determine the •• Iothalamic acid salts—ConrayQ, TriovideoQ
radiographic density of the contrast medium ƒƒ Have very high osmolarity (5–8 times that of human plasma)
• This is because it is Iodine that is responsible for blocking/ ƒƒ Ionic contrast media are contraindicated for use in
attenuating the X-ray beam myelographyQ
• More is Iodine, more is density—whiter/denser it appears—so
that it is better differentiated from adjacent soft tissues Ionic Dimers: I/P Ratio = 6:2Q
• So ideally for a contrast medium, we would want the Iodine
ƒƒ Slightly better I/P ratio than ionic monomers
content (Numerator) to be as high as possible
ƒƒ Examples:
Denominator: Number of particles per mL—Deter­mines Osmo- •• Salts of Ioxaglic acidQ, Iocarmic acidQ
larity ƒƒ Ionic contrast media are contraindicated for use in
• If we dissolve a powder in water and create a solution, there MyelographyQ
may be osmotically active particles per mL of that solution—
Nonionic Monomers: I/P Ratio = 3:1Q

Contrast Media in Radiology


that will determine its osmolarity.
• For contrast media, the Ideal osmolarity—is as low as possible. ƒƒ Most common group of contrast medium used clinicallyQ
Because the High osmolar contrast agents have an osmolarity ƒƒ Safest of the clinically used group of contrast mediaQ
of around 5-times that of blood/serum. Hence they will be ƒƒ I/P ratio = 3: 1Q—is almost similar to the Ionic dimers.
associated with adverse effects. ƒƒ Osmolarity is around twice that of human plasma
• So ideally for a contrast medium, we would want the particles/ ƒƒ Examples:
mL (Denominator) to be as low as possible
•• Metrizamide (Amipaque)Q—1st member of this group
Now let us review the Ratio again: to be developed. Is not used now
• I/P = Iodine (Ideally as high as possible)/Particles/mL (Ideally as •• Iohexol (Omnipaque)Q—Most common and Safest of
low as possible) = Ideal ratio should be as high as possible the clinically used individual contrast media
• Hence remember that •• Iopamidol (iopamiro)Q
An ideal I/P ratio—is as high as possible •• Ioversol (Conray)Q
•• Iopromide (Ultravist)Q

69
Nonionic Dimers: I/P Ratio = 6:1Q— •• Patient’s with CKD in the setting of diabetes mellitus
Highest and Best RatioQ have a 4-fold increaseQ in the risk of CIN.
•• Treatment of CIN
ƒƒ Iso-osmolar groupQ of contrast media—Osmolarity is  HemodialysisQ can efficiently remove contrast
CONCEPTUAL REVIEW OF RADIOLOGY

similar to that of human plasma from the blood stream


ƒƒ Theoretically the Safest contrastQ—but due to cost constraint •• Prevention of CIN:
not very commonly used  Pre-contrast hydration using IV normal salineQ—
ƒƒ Highest and Best I/P ratio: 6: 1Q is most important step
ƒƒ Examples:  High dose StatinsQ—Rosuvastatin
•• IotrolQ  Bicarbonate therapyQ
•• IotrolanQ (Isovist)Q  N-Acetyl cysteineQ—Has sulfhydryl groups that
•• IodixanolQ/(Visipaque)Q act as anti-oxidants and free radical scavengers
 Ascorbic acid/Vitamin CQ
Toxicity/Adverse Effects of Contrast Media  Metformin, though not nephrotoxic, should be
ƒƒ Hyperosmolarity related adverse effectsQ: used prudently, because if renal failure does
•• More with Ionic monomers/dimers occur, there is risk of concomitant lactic acidosis.
ƒƒ Chemotoxicity related adverse effectsQ: Therefore, metformin should be stopped at the
•• Most likely due to the Cations (Na+, K+) time of the procedure and resumed 48 hours later
•• Contrast Induced Nephropathy (CIN)Q—Considered if renal function remains normal.
in detail later •• Prognostically, it is a transient process, with renal
ƒƒ Immunological/Allergic adverse effects: function may reverting to normal within 7-14 days of
•• May be either Anaphylactic/Anaphylactoid reactionsQ contrast administration, provided good supportive care
ƒƒ Idiosyncratic Adverse effects: is administered and comorbidities are well managed
•• Pyrogenic/Febrile reactions due to unsterile injections/
Vasovagal shock/Hypertensive attacks
ƒƒ Low osmolar contrast media have a higher risk of MRI CONTRAST AGENTS
Thromboembolic phenomenaQ associated with them—
HeparinizationQ is required if they are used for angiography
T1 Relaxation AgentsQ
Contrast Induced Nephropathy (CIN) Q
ƒƒ Gadolinium compoundsQ: Most commonly used MRI
ƒƒ All iodinated contrast media are nephrotoxic. High osmolar contrast mediumQ
are more nephrotoxicQ, low osmolar are less nephrotoxicQ •• Paramagnetic substancesQ—Affect the magnetic
ƒƒ Diagnostic criteria: properties of adjacent molecules
•• Impairment of renal function, measured as: •• Act as extracellular contrast agent a cause shortening
 25% increase in serum creatinine from baselineQ or of T1 relaxation timeQ—hence appear bright on T1W
 0.5 mg/dL (44 µmol/L) increase in absolute serum MRIQ
creatinine valueQ •• But fat also appears bright on T1W images, and would
 within 48-72 hoursQ after intravenous contrast cause confusion whether the hyperintense (bright)
administration (following contrast exposure, appearance is due to contrast enhancement or fat. So the
serum creatinine levels peak between 2 and 5 days post contrast images acquired are always Fat-suppressed
Contrast Media in Radiology

and usually return to normal in 14 days) T1W imagesQ—so anything that is hyperintense is
ƒƒ Markers of CINQ: contrast enhancement.
•• Serum creatinineQ—as described above •• FDA approved agents:
•• Estimated GFR (eGFR)Q (estimated GFR [eGFR] <60  Gd—HP—DO3A—Gadoteridol/ProHance
mL/min/1.73 m2)—predictive marker of CIN  Gd—DTPA—Magnevist
•• Serum Cystatin C levelsQ  Gd—DTPA—BMA—Omniscan
•• Plasma neutrophil gelatinase-associated lipocalin
(NGAL), also known as human neutrophil lipocalin, is T2 Relaxation AgentsQ
an early predictive biomarker ƒƒ Superparamagnetic iron oxide (SPIO)Q/Ultrasmall—SPIO
•• Risk factors for CIN: (USPIO)Q—
 Elderly age •• SPIO is a unique MRI contrast medium that undergoes
 Diabetes mellitus selective phagocytosis by reticuloendothelial system
 CKD—Known case of chronic kidney disease cellsQ (Kupffer cells)Q
(estimated GFR [eGFR] <60 mL/min/1.73 m2) •• Uptake causes hypointense appearance on T2W
 Hypovolemia imagesQ
 Multiple myeloma •• Specific for FNH—shows around 60–70% signal loss on
•• The single most important patient-related risk factor is T2W imagesQ
pre-existing Chronic Kidney DiseaseQ •• Hepatic adenomas show only 15–20% signal loss.
70
Liver Specific Contrast Agents ULTRASOUND CONTRAST MEDIA
Q
ƒƒ Manganese—DPDP ƒƒ USG contrast agents are echo enhancers that boost the
ƒƒ Gd—BOPTAQ echogenicity of blood.

GENERAL RADIOLOGY
ƒƒ Gd—EOB—DTPAQ ƒƒ These consist of microscopic gas filled bubbles, their
surfaces reflecting large amount of USG beams. The
Nephrogenic Systemic FibrosisQ—Unique backscattering effect they create increases the echogenicity
Adverse Effect of MRI Contrast Agents of blood.
ƒƒ It is caused by gadolinium exposure used in imaging in
patient’s who have renal insufficiencyQ Generations of USG Contrast Agents
ƒƒ Also known as Nephrogenic fibrosing dermopathyQ ƒƒ 1st GenerationQ—Unstablized bubbles. Cannot survive
ƒƒ Subcutaneous edema and firm, indurated, erythematous through the pulmonary circulation, hence used for Large
skin plaques seen. They progress to flexure contractures vein and Cardiac studies (intra-cardiac shunt identification)
with restricted movements. Liver/spleen/lungs also affected ƒƒ 2nd GenerationQ—Longer lasting bubbles coated with shells
ƒƒ Pathophysiology: of Protein, Lipids, Synthetic polymers
•• Seen due to transmetalationQ, (replacement of the gad- ƒƒ 3rd GenerationQ—Encapsulated emulsions/bubbles
olinium from the chelate and forming a free gadolinium
ion). Free gadolinium ions may then deposit in different Types of USG Contrast Agents
tissues and result in inflammation and fibrosis. ƒƒ Tissue specific USG contrast agentsQ:
•• Toll-like receptors (TLR)Q, in particular TLR4Q and •• Used for Liver, kidney, pancreas, prostate, ovary
TLR7Q, play a role in the development of nephrogenic •• Improve the acoustic differences between normal and
systemic fibrosis abnormal tissues
ƒƒ Associated gadolinium compounds: •• Bubble rupture creates a characteristic mosaic pattern–
•• OmniscanQ/GadodiamideQ—Most commonly impli- Induced acoustic emission
cated •• Examples—
•• MagnevistQ/Gadopentetate dimeglumineQ—2nd most  LevovistQ—1st generation agent used for Cardiac/
common Liver imaging
•• OptiMARK/GadoversetamideQ  SonovistQ, SonazoidQ
ƒƒ Progressive condition with increased morbidity and mor­ ƒƒ Vascular USG contrast agentQ:
tality and no definite treatment •• Gas microbubbles less than 5–10 micrometers, so that
they can pass through pulmonary circulation into the
  Clinical Pearls systemic circulation
•• Examples—AlbunexQ, InfosanQ
Can MRI contrast studies be done safely in renal impairment ƒƒ Recent advances—
patients as an alternative to CE-CT? •• SonosalpingographyQ—may be done using contrast
In the early days when MRI was new, it was used as an
agents to assess tubal patency
alternative to CE-CT when CE-CT could not be done in renal
•• Reflux sonographyQ done similar to MCU—for VUR
impairment patients. The thought was that Gadolinium – not
assessment
being nephrotoxic – could be safely injected in CKD patients and
diagnosis could be made. However this backfired when cases
•• Gastric distension by contrast—resulting in better
pancreas visualization.

Contrast Media in Radiology


of Nephrogenic systemic fibrosis (NSF) were reported. Hence
today if the renal function tests are impaired neither CE-CT nor
CE-MRI can be performed. Clinicians are stuck because they
need a diagnosis but Radiologists refuse to do contrast studies.
There is indeed an impasse. Let’s see what the future beholds!

6
CLINICAL QUIZ - SOLUTION
Esophageal injury/leak is a dreaded complication of neck surgery and is the surgeons nightmare!
 When doing a contrast study for this condition we need to check if there is any extravasation of contrast into neck soft tissues. The
spot image shows you an extraluminal collection of contrast in the neck soft tissues with horizontal fluid level – suggesting esophageal
leak. So what is the contrast material to be used? Barium is contra-indicated in suspected perforation/leaks. Gadolinium is an MR
contrast agent and has no role in radiography. Gastrografin is a high osmolar iodinated contrast – used specifically for GI indications
and may be used here. Iohexol is a low osmolar iodinated contrast that is extremely safe and may be used here. When you have to
choose between Gastrografin (High osmolar) and Iohexol (Low osmolar) contrast – always choose low osmolar contrast – it is overall
safer and much better tolerated by patients. Hence in this case the most appropriate contrast agent to be used is Iohexol.

71
Multiple Choice Questions
CONCEPTUAL REVIEW OF RADIOLOGY

X-rays
1. X-rays were discovered by: 14. SI unit of Radiation exposure is: (JIPMER May 2016)
A. Ian Donald B. Godfrey Hounsfield A. Joule/kg B. Rad
C. Wilhelm Röntgen D. Marie curie C. Roentgen D. Coulomb/kg
2. International radiology day is celebrated on: 15. The 10 day rule applies to:
A. Children B. Young females
A. 1st May B. 22nd August
C. Elderly males D. Elderly females
C. 8 November
th
D. 1st December
3. The main difference between X-rays and Light is:  (AI 2010)
Computed Tomography
A. Energy B. Mass
16. The first generation CT scanner developed by Sir Godfrey
C. Speed D. Type of wave
Hounsfield was what type:
4. X-rays are produced when:  (AIIMS Nov 2002) A. Translate-Rotate B. Rotate-Rotate
A. Electron beam strikes nucleus. C. Rotate—Fixed D. Electron beam type
B. Electron beam strikes anode. 17. Continuous helical acquisition of data in CT scanners, with
C. Electron beam reacts with electromagnetic field. 360 degree continuous rotation of the gantry was possible
D. Electron beam strikes cathode. because of introduction of:
5. Atomic number of Tungsten:  (JIPMER May 2016) A. Advanced X-ray tubes B. Slip-ring technology
A. 42 B. 181 C. Spring technique D. Mobile detectors
C. 74 D. 82 18. CT Number of water and bones respectively: 
6. Which of the following X-ray interaction is the most common  (JIPMER May 2016)
of all X-ray interactions and is responsible for almost all A. 100 and 0 B. 0 and - 1000
scatter radiation: C. 0 and + 1000 D. + 1000 and - 100
A. Photo-electric effect B. Compton effect 19. Walls of CT scanner room are coated with:  (AI 2010)
C. Pair production D. Photodisintegration A. Lead B. Glass
7. Atoms with same mass number and atomic number but with C. Tungsten D. Iron
different nuclear energies are called as: (JIPMER 2016) 20. Calcification is best detected by:
A. Isomers B. Isotones A. USG B. CT
C. Isobars D. Isotopes C. MRI D. Fluoroscopy
8. Contrast of a radiographic image is determined by: 21. 128 slice CT scanner is a:
A. kVp B. mAs A. 1st generation scanner B. 2nd generation scanner
C. Both D. None C. Multi-detector CT scanner D. Portable CT scanner
9. Which of the following is/are present inside nucleus of an 22. Which of the following appears hypodense on CT?
atom? (PGI May 2018) A. Acute blood B. Bone
A. Electron B. Proton C. Calcification D. CSF
C. Neutron D. Photon
E. None of the above Ultrasound Imaging
10. Unit of absorbed radiation is/are: (PGI NOV 2017) 23. Harmonic imaging is used in:
Multiple Choice Questions

A. Rem B. Gray A. USG B. CT


C. Curie D. Rad C. MRCP D. Nuclear imaging
E. Becquerel 24. The part of sound spectrum, known as Ultrasound, is one
above a frequency value of:
11. Which of the following statements about ‘Stochastic effects’
A. 20 Hz B. 20,000 Hz
of radiation is true?
C. 1 MHz D. 20 MHz
A. Severity of effect is a function of dose
25. The part of sound spectrum, used in diagnostic imaging is the
B. Probability of effect is a function of dose part above a frequency value of:
C. It has a threshold A. 20 Hz B. 20,000 Hz
D. Erythema and cataract are common examples C. 1 MHz D. 20 MHz
12. Mammography uses:  (JIPMER May 2016) 26. Most commonly used Piezoelectric crystal is:
A. Continuous spectrum radiation/Bremsstrahlung A. Quartz
B. Characteristic radiation B. Cane sugar
C. Electrons C. Lead zirconate
D. Gamma rays D. Barium zirconate
13. Acute Hematological syndrome occurs at an exposure of: 27. Doppler effect is the change in:
A. 2 Gy B. 10 Gy A. Velocity B. Frequency
72 C. 20 Gy D. 50 Gy C. Temperature D. Reflection
28. In routine clinical practice the Doppler angle should equal to 44. Which of the following indicates radiograph contrast induced
or less than: nephropathy?  (AIIMS Nov 2013)
A. 0 degrees B. 45 degrees A. Increased creatinine levels

GENERAL RADIOLOGY
C. 60 degrees D. 90 degrees B. Decreased urine output
29. Probe used for thyroid ultrasound is: C. Increased bilirubin
A. Convex probe B. Linear probe D. Decreased bilirubin
C. Endoluminal probe D. Cardiac probe 45. Which of the following contrast agent is preferred in a patient
30. Most commonly used method of display of Ultrasound is: with decreased renal function to avoid contrast nephropathy:
A. A mode B. B mode A. Acetylcysteine B. Fenoldopam
C. M mode D. D mode C. Mannitol D. Low osmolar contrast
31. Mode/Method of display used in ophthalmic ultrasound is: 46. Heparinization is required to prevent the risk of thrombo-
A. A mode B. B mode embolic phenomena when using:
C. Doppler mode D. All of above A. High osmolar contrast
32. All of the following appear hypoechoic on USG except: B. Low osmolar contrast
A. GB B. UB C. Gadolinium
C. Simple cyst D. Calculus D. USG contrast
MRI Basics 47. Absolute contraindication for use of Barium is:
33. FLAIR images in MRI selectively suppress the signal arising from: A. Obstruction B. Hernia
A. Gray matter B. White matter C. Perforation D. Ascites
C. CSF D. Skull vault 48. USG Contrast media are:
34. Which of the following is contraindication for MRI? A. Gel microbubbles
A. Presence of cardiac pacemakers B. Air microbubbles
B. Cochlear implants C. Nano-colloid particles
C. Claustrophobia D. All of the above D. None of above
35. Gyromagnetic property of Hydrogen nucleus/proton is used in: 49. Most important risk factor for development of CIN is:
A. USG B. CT A. Elderly age B. Pre-existing CKD
C. MRI D. PET C. Diabetes mellitus D. Anemia
36. Which of the following imaging modality is/are used to 50. Best preventive strategy for CIN is:
image white matter disease of brain: (PGI May 2018) A. Precontrast hydration B. N-Acetyl cysteine
A. MR Spectroscopy B. MRI C. Bicarbonates D. Statins
C. CT D. PET 51. Markers of CIN include:
E. Skull Radiography A. Serum Cystatin C B. Serum creatinine
37. MRI rooms are shielded completely using a continuous sheet C. eGFR D. All of above
or wire mesh of copper/aluminum known as: 52. Gadolinium is:
A. Maxwell cage B. Faraday cage A. Ferromagnetic B. Diamagnetic
C. Edison cage D. Ohm’s cage C. Paramagnetic D. Supermagnetic
38. Advantages of MRI is/are: (PGI NOV 2017) 53. Most common contrast agent associated with NSF is:
A. Multiplanar imaging A. Gadopentetate B. Gadodiamide
B. No ionizing radiation C. GadoDTPA D. Gadovistate
C. Blood vessel can be seen without contrast 54. Which of the following appears bright on both T1 and T2 MRI
D. Lung pathology better seen than CT scan image:  (PGI May 2019)
39. Which of the following is a non-ionizing radiation modality? A. Fat B. CSF

Multiple Choice Questions


A. Angiography B. CT (AIIMS NOV 2018) C. Melanin D. Muscles
C. Mammography D. MRI E. Hemorrhage
40. MRI magnet is a: 55. Assertion: Multi detector CT is gold standard test for aortic
A. Ferro-magnet B. Para-magnet dissection
C. Simple magnet D. Superconducting-magnet Reason: In case of skilled person, even TEE is most sensitivity
41. Which of the following element in the human body has the and specificity as multidetector CT  (AIIMS May 2019)
maximum Gyromagnetic ratio: A. Both Assertion and Reasons are true and the Reason is the
A. Hydrogen B. Oxygen correct explanation for the Assertion
C. Iron D. Nitrogen B. Both Assertion and Reasons are true, but the Reason is
not the correct explanation for the Assertion
Contrast Media in Radiology C. Assertion is true, but the Reasons is false
42. Water soluble contrast media is: (Recent Pattern 2019) D. Assertion is false, but the Reasons is true
A. Barium B. Iodine E. Both Assertion and Reasons are false
C. Bromium D. Calcium 56. Uses of USG: (AIIMS May 2019)
43. Which of the following contrast may be used for a case of A. Is helpful in Insertion of CV-Line
suspected esophageal perforation? (Recent Pattern 2018) B. To dissolve coronary plaque
A. Barium B. Gadolinium C. Diaphragmatic movements
C. Iohexol D. Gastrografin D. All of the above 73
57. Most important advantage of Transcranial USG:  59. Wavelength of X-rays:  (JIPMER May 2019 Pattern)
 (AIIMS May 2019/JIPMER May 2019 Pattern) A. 1–10 nm
A. Detect AV malformation B. 0.1–10 nm
CONCEPTUAL REVIEW OF RADIOLOGY

B. Detect emboli C. 0.01–10 nm


C. Detect vasospasm D. 0.0001–10 nm
D. Detect brain blood vessels stenosis
58. The order of imaging methods (from worst to best) with
respect to visibility of detail (resolution) is: 
 (JIPMER May 2019 Pattern)
A. Gamma camera, fluoroscopy, CT
B. Ultrasound, fluoroscopy, radiography
C. Gamma camera, fluoroscopy, MRI
D. Radiography, fluoroscopy, MRI

Answers
1. C 9. B,C 17. B 25. C 33. C 41. A 49. B 57. C
2. C 10. B,D 18. C 26. C 34. D 42. B 50. A 58. D
3. A 11. B 19. A 27. B 35. C 43. C 51. D 59. C
4. B 12. B 20. B 28. C 36. A,B,C,D 44. A 52. C
5. C 13. A 21. C 29. B 37. B 45. D 53. B
6. B 14. D 22. D 30. B 38. A,B,C 46. B 54. A,E
7. A 15. B 23. A 31. D 39. D 47. C 55. B
8. C 16. A 24. B 32. D 40. D 48. B 56. D

Explanations to Questions
3. Both light and X-rays are electromagnetic radiation and thus 40. Magnets used for MRI are of three types: permanent,
have same speed and type of wave. Being energies they do resistive and superconductive. These days almost all higher
not have mass and hence cannot differ in terms of mass. They strength magnets are superconductive magnets. Both
however have differing energies and frequencies. permanent and resistive MRI scanners are limited to low-field
12. In mammography we need good differentiation of low contrast applications, primarily open MRI and extremity scanners.
Explanations to Questions

structures and very high spatial resolution for micro-calcifications These magnets are useful for claustrophobic patients.
Hence we use material that produces Characteristic X-rays 41. Hydrogen has the highest gyromagnetic ratio and is abundantly
with energies of 17-20 keV (20-30 keV for larger breasts) to seen in the human body and is hence used for MR imaging.
produce the best contrast. 45. In esophageal perforation there is risk of mediastinitis while
The commonly used material is Molybdenum (characteristic performing any contrast study. Here an iodinated contrast is
X-rays at 17.5 and 19.6 keV). Lower energy photons have a preferred and amongst ionic and non-ionic the latter one is
higher probability of interacting with matter and, therefore, preferred for less risk of aspiration pneumonia
produces better contrast. In mammography characteristic 46. Low osmolar contrast has actually osmolarity similar to
radiation forms 80% of the beam. human blood and thus exerts no extra osmolar load on
22. Out of the given options CSF which has water like composition kidneys and thus considered to be least nephrotoxic of all.
appears hypodense while rest all options will appear as 47. Administration of barium in a case of perforation will result in
hyperdense. This density is measured with respect to density dreaded complication of barium (chemical) peritonitis – it is
of gray matter in brain and that of skeletal muscles in rest of absolute contra-indication for use of barium.
the body.
23. Harmonic imaging is an advancement in ultrasound
technology where only the harmonic sets of images are used
for image creation resulting in a better quality image.

74
RESPIRATORY SYSTEM
C hapter O utline • Mediastinum
• Normal Findings on a Chest Radiograph • Lung Tumors
• Silhouette Sign and Lung Infections • HRCT Thorax
CONCEPTUAL REVIEW OF RADIOLOGY

• Pleural Abnormalities • Miscellaneous Topics


• Lobar Collapse and Airway Disorders • Multiple Choice Questions with Explanations

Normal Findings on a Chest Radiograph


CLINICAL QUIZ
1
1. A 65-year-old man was admitted to the ICU with breathlessness and
pedal edema. A CXR was advised as a part of basic evaluation. Identify
the parameter being measured here on this CXR:
A. Hilar ratio
B. Cardiomediastinal ratio
C. Cardiothoracic ratio
D. Hyperinflation ratio
2. On a CXR AP view the cut-off value of this ratio to diagnose cardiomegaly
is:
A. 0.50
B. 0.55
C. 0.60
D. 0.65

NORMAL FINDINGS
Normal Findings on a Chest Radiograph

Radiographic Exposure vertebral body projected between the clavicles. If this is not
the case then the patient is rotated, either to the left or to
ƒƒ On a good quality radiograph, the margins of lower thoracic the right.
vertebral bodies should just be visible through the heartQ.
ƒƒ Whatever side the distance between these landmarks is
ƒƒ Under-exposed film: Occurs if an insufficient number of more – the patient is rotated toward that same sideQ.
X-ray photons have passed through the patient - vertebral ƒƒ Rotation causes Unilateral radiolucencyQ (blackness).
bodies are not seen. The film will look ‘whiter’Q leading to Usually the side to which the patient is rotated appears
potential ‘over-diagnosis’ of pathology. blacker.
ƒƒ Overexposed film: Occurs if too many photons have
resulted in overexposure of the X-ray film. The film will
appear too ‘black’Q, resulting in pathology being less
conspicuous and may lead to ‘under-diagnosis’. Rotation in Pediatric CXRs
CO
NC E P T
Patient Centering/Rotation
• Whenever a ‘bachoo’ X-ray is being done the parents/
ƒƒ It is assessed by defining anatomic landmarks. technicians are always required to hold the child down for the
•• Midline anatomic landmark: The spinous process of exposure while the child cries, shouts, wiggles, turns, kicks,
upper dorsal vertebrae bites…as he is afraid. Hence almost all of the pediatric CXRs are
•• Lateral anatomic landmarks: Medial ends of clavicles always shot in a state of rotation.
on either side. • This rotation causes significant difference in the densities
of either side lung fields, and may be misinterpreted as
ƒƒ On a well-centered CXR, the medial ends of both clavicles
abnormality unless we take into account rotation.
should be equidistant from the spinous processQ of the

76
RESPIRATORY SYSTEM
NORMAL CXR:
•• Well exposed – Lower thoracic vertebral borders just visualized
•• No rotation
•• Good Inspiration -10 posterior and 6 anterior ribs seen RIGHT and LEFT Mediastinal borders on a CXR
•• Central Trachea
•• No asymmetric lung opacities
•• Left dome slightly lower than right
•• Left hilum higher than right
The left mediastinal marginQ is formed from the top-to-bottom
by the following structures:
Degree of Inspiration–Appearance of Ribs ƒƒ Aortic knuckle
ƒƒ The degree of inspiration/expiration is assessed by counting ƒƒ Main pulmonary trunk
the ribs. ƒƒ Left atrial appendage Left ventricle
ƒƒ How to identify the Posterior Vs Anterior ends of ribs? Occasionally the ill-defined margin of the left superior
Remember the simple rule: mediastinum may be formed by the subclavian artery – above
Posterior ends of • Closer to midline the aortic knuckle.

Normal Findings on a Chest Radiograph


RibsQ • Horizontally oriented
Anterior ends of • Away from midline On a Left Anterior Oblique CXR
RibsQ • Obliquely oriented ƒƒ Anterior heart margin is formed by RVQ
ƒƒ Posterior heart margin is formed by LVQ
ƒƒ Criteria for good Inspiration: On a CXR there must be
visible above the dome of diaphragm:
On a Right Anterior Oblique CXR
•• At least 10 posterior ends of ribsQ
•• At least 6 anterior ends of ribsQ ƒƒ All the 4 chambers are seen at margins of the heart shadow.
ƒƒ If lesser number of ribs are seen above the dome of ƒƒ Posterior border is formed by LA – RAO view is used for LA
diaphragm then the radiograph is termed as being in Mid size assessmentQ
inspiration/Expiration;
ƒƒ Radiographic features on an Expiratory film: Trachea
•• Heart shadow appears largerQ: As the diaphragm is ƒƒ Seen as a vertical linear band like lucency
higher up in expiration the heart is obviously spread
ƒƒ Is located in the midline in its upper half. Deviated slightly
out and appears broader
to the right at the level of aortic knuckleQ
•• Bilateral basal lung opacitiesQ: As this basal lung is least
filled with air, there is resultant blood vessel crowding,
giving rise to basal opacities. In Inspiratory phase the lung is   Clinical Pearls
well distended, so no vessel crowding, hence no opacities.
Identifying the Carina
Mediastinum • Carina is the point where the trachea divides into the right and
left bronchi. Carefully tracing the tracheal lucency downwards
The right mediastinal marginQ is formed from the top-to-bottom will bring you to the carina. Identifying it is important clinically.
by the following structures: • Normal carinal angle is 60°–75°.
ƒƒ SVC • Carinal widening occurs in enlarged left atrium (in mitral
ƒƒ Right atrium stenosis)Q and enlarged subcarinal lymph nodesQ
ƒƒ IVC – sometimes, not always seen 77
Hilum Angles Cardiophrenic Angles
• Sharp angles at the interface of Heart
Normal Each hilum has a
and Diaphragm. May not always be
Configuration • Short and indistinct upper limb: Formed by sharp as this is a common site for
CONCEPTUAL REVIEW OF RADIOLOGY

of Hilum superior pulmonary veinQ on that side


epicardial fat pads, or pericardial
• Long and prominent lower limb: Formed by cysts which may obscure these
lower lobe pulmonary arteryQ on that side
angles.
Contributors Major contributors: Costophrenic Angles
to Hilar • Upper lobe pulmonary veins – form the short • Sharp, acute angles at the junction
shadows indistinct upper limb of hilum point of Diaphragm and the chest
• Lower lobe pulmonary vein – forms the long wall laterally.
prominent lower limb of hilum • Always sharp and acute. Blunting
Minor contributors: of these angles is seen in pleural
• Central bronchi effusionsQ.
• Lymph nodes
Lower lobe pulmonary veins – DO NOT
contribute to hilar shadows   Clinical Pearls
Hilar Point & The point where the upper and lower limbs of Diaphragmatic Levels
Hilar levels the hilum meet with each other – is the Hilar Left dome of diaphragm is lower than the right. This is because of
point. If you join the hilar points on either side, the weight of the HeartQ being transmitted onto the left dome,
you will realize that: and NOT DUE TO LIVER PUSHING THE RIGHT DOME UPWARD.
• The Left hilum is HIGHERQ than the right – in This knowledge of the correct mechanism is essential as it will
97% of normal patients help us solve many MCQs such as. Which dome of diaphragm will
be lower in a case of isolated DextrocardiaQ?
• Both hila at the same level – in 3% of normal
Answer will be Right dome of diaphragm.
patients
• Right higher than Left – Always abnormal.
• The left hilum is higher because – the left Fissures
pulmonary artery arch above the left main Main Fissures
bronchus. Whereas on the right side the
right pulmonary artery passes anterior to the ƒƒ Right lung has 2 fissures: Major/Oblique and Minor/
right main bronchus. Hence the left vessels Horizontal fissure
are higher and as the hila are mainly formed ƒƒ Left lung has just one fissure: Major/Oblique fissure
by blood vessels – LEFT IS HIGHERQ.
Major/Oblique Fissure
Normal Findings on a Chest Radiograph

Normal Shape • If you draw a continuous curve along the Best seen on a lateral film as it runs ‘obliquely’ from Postero-
of Hilum upper and lower limbs of the hilum the superior to anteroinferior. It begins posteriorly at level of T4/
hilum is always CONCAVE facing laterallyQ. T5 level, passes through the hilum and comes down antero-
• If this concavity is lost, or if the hilum margin inferiorly. Left oblique fissure is steeper and ends around 5 cm
is Convex facing laterally, then in India – the posterior to anterior CP angle. The right oblique fissure ends
most common cause of this finding is TB further anteriorly.
lymphadenopathyQ.
Minor/Horizontal Fissure
Diaphragm Runs ‘horizontally’ on the right side from the hilum to the region
of 6th rib in the mid-axillary line.
Contour • The diaphragmatic contour is formed
by the right and left domes of
Bilateral
diaphragm.
pulmonary hila
• Parts of Diaphragm NOT SEEN on a
– Yellow circle
CXR PA:
represents the
ƒƒ Central tendon of diaphragmQ
Hilar point
ƒƒ Small medial most portion of left
dome of diaphragmQ
Levels • The left dome of diaphragm is always
lower than the rightQ.
• In few normal patients, both the
domes may be at the same level.
• A difference in the levels of the
domes of diaphragm of >3 cm
is significantQ – and must be
investigated for any abnormality.
78
Diaphrag- Oblique
matic levels fissures
–determined (Black) and

RESPIRATORY SYSTEM
by the Horizontal
weight of fissure
the Heart. (Yellow)
See the superim-
difference posed on
in a Normal faded CXR
case and a images
case of Situs
inversus

Hidden AreasQ
The following areas are considered to be hidden on a CXR PA
LATERAL CHEST RADIOGRAPH
view: It was very commonly done in olden days – however now a CT
ƒƒ Lung ApicesQ is preferred.
ƒƒ Mediastinum and HilaQ The side of chest where the abnormality is suspected,
ƒƒ Retrocardiac lung should be close to the cassette.
ƒƒ Apparent infradiaphragmatic lungQ: Posterior and
basal segments of bilateral lower lobes and the posterior “3 Normal Findings on a Lateral CXR” Q
costophrenic sulcus are located deep down posteriorly and ƒƒ Retrosternal Lucency
are overlapped by the abdominal soft tissues. •• Lucency/blackness just behind the sternum, where the
ƒƒ BonesQ: Overlap between lung lesions and bone lesions may 2 lungs meet.
create confusion. Oblique/Lateral/Expiratory films may help. •• Obliteration of this space occurs with Anterior
mediastinal massesQ, Ascending aorta aneurysmsQ
  Clinical Pearls and lymph nodal massesQ and occasionally with RA
Kerley Lines on a CXR enlargement
ƒƒ Retrocardiac Lucency:
Kerley lines/Septal lines are seen when the interlobular septa
•• Similar area of blackness behind the heart
in the pulmonary interstitium becomes prominent due to
•• Lost in Lower lobe pathologiesQ, pleural effusionQ,

Normal Findings on a Chest Radiograph


lymphatic engorgement or edema of the connective tissues of
the interlobular septa. They appear when pulmonary capillary descending aortic aneurysmQ
wedge pressure reaches 20-25 mm Hg and are of 3 types: ƒƒ Vertebral Translucency:
•• Examine the overlapping soft tissue density on the
Kerley A linesQ • Non-branching lines radiating centrally vertebral column.
from the hilum to the periphery •• Normally as you travel from the top to the bottom the
• 1–2 mm thick, 2–6 cm long vertebral bodies become progressively translucent/
blackQ.- this is known as the Spine signQ
• Formed by thickening of deep interlobular
•• This gradient may be lost in lower lobe/posterior basal
septae
consolidationQ - where the lower portion appears
Kerley B lines Q
• Horizontal, transverse short lines in the whiter/denser.
peripheral lung field perpendicular to the
overlying pleural surface
• Formed by thickening of interlobular
septae
Kerley C linesQ • Reticular opacities at lung base
• Represent Kerley B lines seen enface on a
lateral film
Causes of • Pulmonary edema
Kerley LinesQ • Infections
• Mitral stenosis
• Congenital heart disease
• Sarcoidosis Normal findings on a Kerley A and Kerley B lines –
• Neoplasms: Alveolar cell carcinoma, Lateral CXR – Schematic drawing for your
Lymphoma, Lymphangitis carcinomatosis, 1. Retrosternal lucency understanding
2. Retrocardiac lucency
• Pneumoconiosis
3. Lucency gradient along
the vertebral column
79
  Clinical Pearls
Incidental Findings and Normal Variants on a CXR
CONCEPTUAL REVIEW OF RADIOLOGY

Before we jump into studying abnormalities on a CXR, let us have a look at a few interesting incidental findings and normal variants.
Though asymptomatic in most of the cases, recognizing these findings is clinically important as they may affect clinical decisions. Let us
try and form a visual impression.

Cervical Rib: Left-sided cervical rib is seen here. These are Situs inversus: Always pay attention to the side marker (R/L)
supernumerary ribs arising from C7 vertebra. Though asymptomatic placed on the radiograph. Dextrocardia is isolated right sided
in most cases they may cause “Thoracic outlet syndrome” causing cardiac apex. Here if you watch carefully the lucency of the
compression of Brachial plexus nerve roots or the Subclavian vessels gastric/fundic bubble is seen below the dome of diaphragm on
right side–suggesting Situs Inversus
Normal Findings on a Chest Radiograph

Scoliosis: Look beyond the chest and you can see that the
Left Mastectomy: Well-defined soft tissue density is seen dorsal spine has a lateral curvature – with a convexity toward
overlapping the lower part of the right hemithorax – suggestive of right in its upper half and convexity toward left in lower part –
breast shadow. It is missing on the left side – suggests possibility of suggests Scoliosis. Though idiopathic in most of the cases the
Left mastectomy. In an elderly female most of the times it suggests diagnosis once established will help in the follow-up evaluation
surgery for Carcinoma breast. to determine if it is progressive.

Chilaiditi syndrome: Colon interposed between the liver and the


right hemidiaphragm – is a cause of pseudopneumoperitoneum. Azygos fissure: Most common accessory fissureQ. The portion
The colonic wall markings are seen here and hence this is not true of the lung medial to this fissure is called as Azygos lobeQ – is
pneumoperitoneum. Just the presence of this imaging findings is not a true lobeQ as it does not have its own bronchus. When
known as Chilaiditi sign. If the patient has pain along with it, it is left-sided it contains an accessory hemiazygos vein
termed as Chilaiditi syndrome.
80
1
CLINICAL QUIZ - SOLUTION
So, the CXR here is showing measurement of maximum cardiac diameter (A) and the maximum inner thoracic diameter (B). The
ratio A/B is known as the cardiothoracic ratio – and is an objective criterion for diagnosis of cardiomegaly. Hence the answer to the

RESPIRATORY SYSTEM
1st quiz is C.
Assessment of Heart on CXR:
Qualitative
On a well-centered CXR, 2/3rds of the heart is on the left of the spine, whereas 1/3rd is on the right.
Quantitative
Maximum transverse cardiac diameter:
Should be
<14.5 cm in FemalesQ
<15.5 cm in MalesQ
An increase of 1.5 cmQ in transverse diameter on comparable serial films – is significant.
Cardiothoracic Ratio
Ratio of Maximum Transverse diameter of heart: Maximum transverse diameter of Inner thorax
Diagnosis Based on Cardiothoracic Ratio
On a CXR PA VIEW
<0.50 Normal
0.50 – 0.55 Borderline
>0.55 CardiomegalyQ
On a CXR AP VIEW/Pediatric CXR
<0.60 Normal
>0.60 CardiomegalyQ
Remember that on an AP view there is false cardiomegaly – that is the heart appears magnified. Hence, the cut-off for cardiomegaly
on an AP view is 0.60 as against that on PA view (0.55). Hence, the answer to the 2nd quiz is C.

Silhouette Sign and Lung Infections

Silhouette Sign and Lung Infections


2
CLINICAL QUIZ
3. Child presented with h/o fever and respiratory distress. Blood
investigations revealed an elevated CRP and a total leukocyte count of
15000/mm3 With suspicion of pneumonia a X-ray was done. Most likely
causative agent in this condition: (AIIMS May 2017 Pattern)
A. Pneumococcal pneumonia
B. Staphylococcal pneumonia
C. Klebsiella pneumonia
D. Mycoplasma pneumonia

81
DIFFERENTIAL RADIOGRAPHIC seen because of 2 tissues of differing density lying adjacent
to each other… this is indeed a very useful insight into image
DENSITY THEORY: TRUE interpretation.
CONCEPTUAL BASIS OF THE
CONCEPTUAL REVIEW OF RADIOLOGY

Silhouette Sign
SILHOUETTE SIGN Now let us climb one step higher on the concept.
ƒƒ The human body is comprised of literally ‘n’ number of Whenever you look at a chest radiograph (or any other
tissues like skin, fat, ligaments, tendons, muscles, synovium, radiograph) you can see certain sharp margins/silhouettes. On a
gray matter, white matter, bones….just to name a few. Can CXR these silhouettes are:
a X-ray image help us identify each and every such tissue ƒƒ Right upper mediastinal margin
separately from each other…..the answer is NO. ƒƒ Right heart border
ƒƒ The limitation of the radiography technique and the human ƒƒ Right dome of diaphragm
visual system together means that only a few discrete ƒƒ Aortic knuckle
tissue densities are seen separately from each other on a ƒƒ Lateral margin of descending thoracic aorta
radiograph. ƒƒ Left heart border
ƒƒ These densities from the less to more dense are: ƒƒ Left dome of diaphragm
All these margins, if you think about it, are formed by soft
tissues – and hence are water density on a radiograph. As already
AIR Fat Water Bone Metal
discussed these are seen as sharp margins because they are
surrounded by a different density – Air density of lungs. Rather
•• Air density: Least dense tissue component in the body. each of these structures are lined by a specific lobe/segment of
Comprises Lungs, Air-filled cysts, air within bowel lung. For example, the right heart border is lined on its lateral
loops, etc. Appears black on a radiograph. aspect by the right middle lobe (its medial segment) that lies in
•• Fat density: Slightly denser than air. Will appear dark the immediate right paracardiac area.
gray. Now imagine a situation where the RML is consolidated.
•• Soft tissue/Water density: This is the largest group of Consolidation is a condition in which the air in the alveoli
density. All soft tissues other than fat are water density. is replaced with fluid. So, the air density of normal lungs is
Includes literally all the soft tissues in the body – converted to water density when it gets consolidated. Now let us
muscles, ligaments, tendons, heart, diaphragm, etc. what will happen alongside the right heart border. The right heart
•• Bone: One of the most densest in the body. Appears border is water density, will now be lined by the consolidated
white and seen separately from the adjacent soft tissues. RML which is water density – hence both being of the same
•• Metal: This was a category that was added later because density the sharp margin of the right heart will be lost. This is
orthopedic implants were being used in the body and called as POSITIVE SILHOUETTE SIGNQ.
these appeared bright white – even denser than and THE SILHOUETTE SIGN may be stated (by Benjamin
shinier than bones. Felson) as:
ƒƒ So, because these densities can be seen separately from “An intrathoracic lesion touching a border of the heart, aorta
Silhouette Sign and Lung Infections

each other, it is logical that the plane where any 2 different or diaphragm will obliterate that border on the radiograph.
densities lie immediately adjacent to each other – would An intrathoracic lesion not anatomically contiguous with a
be the plane where maximum contrast is seen—in-fact a border of one of these structures will not obliterate that border.
SHARP MARGIN is seen at such interface. The term SILHOUETTE SIGN is applied to indicate the loss of
Thus, if you look at each and every margin on a chest the silhouette of any of these borders by an adjacent disease.”
radiograph there are 2 tissues of different densities at these In recalling the lung lobes involved in obscuring the
margins. For example: borders, just try to remember the lobar distribution in 3D so that
•• Right and Left mediastinal margins/heart borders: it is easier to visualize and remember.
Mediastinum/Heart (Water density) is surrounded by Silhouette Lost/ Lung Lobe/Segment Involved
Lungs (Air density) Obliterated
•• Right and left domes of diaphragm: Diaphragm (Water
Right upper Anterior segment of right upper lobeQ
density) is lined by Lungs above (Air density)
mediastinal margin
•• Humerus/Bone surface: Bone density surrounded by
muscles (Water density) Right heart border Right middle lobe (medial segment)Q
ƒƒ Similarly see that where the densities are same you cannot Right dome of Right lower lobe – Most commonlyQ
see a sharp margin. diaphragm Right middle lobe – OccasionallyQ
•• Interventricular septum/Cardiac valves are not seen: Aortic knuckle Left upper lobe (Apicoposterior
Myocardium (Water density) lined by Blood (Water segment)Q
density)
Lateral margin of des- Left lower lobe (superior and posterior
•• Inferior surface of Diaphragm is not seen: Diaphragm
cending thoracic aorta basal segment)Q
(Water density) lined by Peritoneal fluid/Liver/Spleen
(Water density) Left heart border Left upper lobe (Lingular segment)Q
82 ƒƒ So to conclude whenever you look at a radiograph and see Left dome of Left lower lobeQ
sharp margins/silhouettes you now know that these are diaphragm
bronchial wall is too thin to be seen on a routine
Differential Radiographic Density radiograph. Hence, we have Air density – Air density
(DRDT) and Silhouette Sign: Mother adjacent to each other, thus you cannot see bronchioles
CO
NC E P T of All Concepts within a normal lung.

RESPIRATORY SYSTEM
ƒƒ Consolidated lung:
• The above discussion we have had is indeed the Mother of all
Concepts.
•• Air-filled terminal/respiratory bronchioles are now
surrounded all around by fluid filled alveoli. So now
• Over time as you read the rest of this book you will realize that
almost all the named signs you come across in all the systems we have Air density – Water density adjacent to each
of radiology – are indeed applications of DRDT/Silhouette sign. other, thus the bronchioles now become visible in an
• Air bronchogram sign of consolidation, Continuous diaphragm area of consolidation as linear lucencies – THE AIR
sign of pneumomediastinum, Visceral pleural line of BRONCHOGRAM SIGNQ.
pneumothorax, Rigler’s double wall sign of pneumomediastinum
are just a few examples Air Bronchogram Sign is seen in
• Now let us study few extended applications of this concept
Common ConditionsQ Occasionally SeenQ

Air Bronchogram SignQ • Consolidation • Alveolar cell carcinoma


• Pulmonary edema • Lymphoma
This is based entirely on the concepts we have studied so far, • Hyaline membrane disease • Alveolar proteinosis
and is an important sign showing that an opacity is intra- • Expiratory film • ARDS
pulmonaryQ. • Sarcoidosis
ƒƒ Normal Lung: • Fibrosing alveolitis
•• Air-filled terminal/respiratory bronchioles are sur­ • Radiation fibrosis
rounded all around by air-filled alveoli. The intervening

Opacity in the Ill-defined


right upper haziness in
zone – causing the right
obscuration of paracardiac
the right upper area, with
mediastinal obscuration
border – of the
suggests RUL right heart
Consolidation border –

Silhouette Sign and Lung Infections


suggests RML
consolidation

Coronal Lung
window CT
image showing
extensive
bilateral
consolidation,
with multiple air
bronchograms
(black arrows)

83
LUNG INFECTIONS
Alveolar vs
Alveolar versus Interstitial Pattern of Interstitial lung
opacities
CONCEPTUAL REVIEW OF RADIOLOGY

Opacities
ƒƒ There are certain diseases that selectively affect the Air-
spaces (alveoli) whereas few affect the Interstitium creating
specific imaging patterns.
ƒƒ While assessing any opacity on a CXR we must try to
differentiate into alveolar or interstitial opacities and
honestly this can be a bit difficult sometimes. But this
approach can help us identify the underlying etiology in a
particular case.
Alveolar/Air-space Disease Interstitial Disease
Alveoli are filled with abnormal Interstitial tissues are
materialQ – fluid/blood/ thickened/fibrosedQ along with
pus/protein/cell debris/ involvement of alveolar walls Extensive
combination of these. Staphylococcal
Imaging findings on CXRQ: Imaging findings on CXRQ: pneumoniae with a
• Fluffy/blobby ill-defined • Multiple small nodules – Pneumatocele
opacities tiny opacities (black arrow)
• Cotton wool like opacities • Linear reticular pattern –
• Coalescing opacities mesh like fine or coarse lines
• Segmental/lobar distribution • Reticulonodular –
• Air bronchogram sign combination of small
nodules and fine lines
• Reduced lung volume in
extensive disease
• Honey comb pattern in end
stage disease
Differential diagnosis: Differential diagnosis:
• Pulmonary edema – • Viral pneumonia
cardiogenic/non-cardiogenic • Atypical/Mycoplasma CXRs showing
• Lobar pneumonia pneumonia a round cavity
• Pulmonary hemorrhage • Pneumocystis pneumonia within the lungs
• Lymphoma • Sarcoidosis with a horizontal
air-fluid level –
Silhouette Sign and Lung Infections

• Bronchoalveolar carcinoma • Idiopathic pulmonary


Lung Abscess
• ARDS fibrosis
• Aspiration pneumonia • Lymphangitis carcinomatosis
• Hyaline membrane disease • Rheumatoid lung ƒƒ Lobar Pneumonia:
• Transient tachypnea • Scleroderma •• Usually unifocalQ
• TB •• Streptococcus pneumoniae – is MOST COMMON causeQ
•• Occasionally rounded lesions with ill-defined margins
  Clinical Pearls appear especially in children under the age of 8 years,
Pneumonia versus Pneumonitis producing the so called – “Round Pneumonia”Q
•• Airways are not involved, so they remain patent,
PneumoniaQ:
hence there is little or no volume loss. Hence - Air
• Usually implies an infection by pathogenic organisms resulting
bronchograms are commonQ.
in consolidation of lungs
ƒƒ Bronchopneumonia:
PneumonitisQ: •• Also called Lobular pneumoniaQ
• Inflammatory processes that primarily involve the alveolar •• Usually multifocalQ and centered to distal airways
wall – e.g. Fibrosing alveolitis/Usual interstitial pneumonia •• Large heterogeneous, scattered opacities distributed
along the course of airways
Pneumonia Classification •• As the disease worsens the opacities coalesce and
become homogenous
Based on distribution: Based on the imaging pattern pneumo- •• Air bronchogram is usually ABSENTQ
nias may be classified as: ƒƒ Atypical pneumonia: Discussed below.

84
Based on Etiology
Now that we have studied the Imaging classification of pneumonias, let us quickly review the specific points associated with specific
etiologies that may be asked as MCQs.

RESPIRATORY SYSTEM
Causative Agent of Specific Imaging Features
Pneumonia
Streptococcus pneumoniae • MC cause of community acquired lobar pneumonia in adultsQ
Staphylococcus aureus • Common in debilitated patients/as superinfection in Influenza/secondary to IV drug abuseQ
• Cavitation is commonQ – in later radiographs
• PneumatocelesQ – usually seen in children in the recovery phase
Klebsiella pneumoniae • Occurs in elderly and in alcoholics– involving the RUL
• Involved lung volume is Increased – Bulging of fissuresQ
Legionnaire’s disease: • Rapidly spreading pneumonia occurring from the organism contaminating water from ACs, coolers,
Legionella pneumophila showers
• Also causes diarrhea, Mental confusion (hyponatremia), shockQ
Actinomycosis • Appearance may mimic Bronchogenic carcinomaQ
• Extension of disease to contiguous soft tissues/bones (periostitis) – differentiating featureQ
• CT: Peripheral homogenous consolidation with central low attenuation and adjoining pleural
thickeningQ – diagnostic of Actinomycosis

Atypical pneumonia: • Mycoplasma pneumonia is termed atypical because:


Most Common cause is ƒƒ Atypical organism - They lack rigid cell walls, hence not susceptible to Penicillins
Mycoplasma pneumoniaeQ ƒƒ Fine reticular/nodular shadows appear in the perihilar region early which may progress to
consolidation (rarely lobar involvement)
ƒƒ WBC counts usually normal
ƒƒ Non-productive cough, absent/minimal sputum
ƒƒ Not caused by aspiration
ƒƒ Usually seen in poorly maintained air-conditioned areas
• Viruses – Influenza A and B, RSV, Adenovirus, Chlamydia psittaci, Coxiella burnetii (Q fever)
Varicella pneumonia • Predominantly nodular opacities 5 – 10 mm in size
• When persistent may get calcified – Miliary shadowsQ on a CXR

Lung Abscess

ƒƒ Most commonly due to aspiration from upper respiratory ƒƒ CXR Findings:

Silhouette Sign and Lung Infections


tract, associated with poor dentition and periodontal •• Cavity containing a gas-fluid levelQ – most classical
infection finding.
ƒƒ Hence the superior segment of right lower lobe is – MOST •• Usually abscesses are round in shapeQ and appear
COMMON siteQ similar in both frontal and lateral projections.
ƒƒ CE-CT is Most sensitive and specific imaging modalityQ - ƒƒ CT Findings:
Investigation of choiceQ •• Round lesions with thick walls and internal gas-fluid
level and surrounding consolidation

CE-CT Thorax - CE-CT Thorax


Rounded cavitary - Typical
lesion in the peripherally
right lower lobe located crescentic
with peripheral collection with
enhancement and enhancing walls
air-fluid level – – Empyema. The
Suggests a Lung enhancing walls
Abscess are the parietal
and visceral
pleura – Split
pleura sign

85
Lung cavity
Empyema versus Lung Abscess with central
CO
NC E P T opaque
CONCEPTUAL REVIEW OF RADIOLOGY

contents
Empyema Lung Abscess surrounded by
Lentiform shape Usually round in all projections air – Monad
sign/Air
Forms an obtuse angle with Forms an acute angle – Claw
crescent sign –
the costal/pleura surface sign
Aspergilloma
Located peripherally – close Located centrally within the
to the chest wall lungs
On CT the lung vessels The bronchovascular
will be distorted and lung structures will be interrupted
parenchyma compressed at the margins CT Thorax – Lung
Split pleural sign: Thick Thick enhancing walls of an window image
enhancing pleural walls intrapulmonary lesion showing bilateral
predominantly
surrounding a fluid collection
central
on post contrast CT
Bronchiectasis –
Smooth walls Thick irregular walls a case of ABPA

  Clinical Pearls
Mendelson syndrome – a type of Aspiration pneumonia
• Chemical pneumonia due to aspiration of acid gastric
Immunocompromised
contents during anesthesiaQ
patient with
• Intense bronchospasm occurs followed by flood of pulmonary
symmetric ground
edema
glass opacities (white
• CXR – Sudden massive pulmonary edema pattern
arrows) and confluent
consolidations in both
lower lobes (black
Miscellaneous Lung Infections arrows) – suggests
Aspergillosis Pneumocystis
pneumonia
ƒƒ A spectrum of mycotic diseases caused by the Aspergillus
species, usually A fumigatusQ.
Silhouette Sign and Lung Infections

ƒƒ CXR is the initial investigation of choiceQ. However, CT is ƒƒ Allergic Bronchopulmonary Aspergillosis (ABPA):
best overall investigationQ. •• The primary radiologic criteria for ABPA include
ƒƒ Aspergilloma/Saprophytic aspergillosis: fixed or transitory pulmonary infiltratesQ and central
•• Most common formQ bronchiectasisQ as a late manifestation
•• Non-invasive •• Finger in glove appearanceQ. Segmental/lobar
•• Due to colonization of a chronic lung cavity (usually collapse may occur due to mucoid impaction.
from TB/Histoplasmosis/Sarcoidosis) by the fungus •• On CT: Fleeting pulmonary alveolar opacitiesQ
•• On CXR - Rounded or ovoid soft tissue attenuating ƒƒ Angioinvasive Aspergillosis:
masses located in a cavity and outlined by a crescent •• Most severe and aggressive formQ - can be life threatening
of air. Altering the position of the patient usually shows ƒƒ Halo signQ: On HRCT, Consolidation surrounded by a halo
that the fungal ball is mobile – is a diagnostic feature. of ground glass opacity is seen – is highly suggestive
•• On CT - well-formed cavity is seen with a central soft ƒƒ Eventually as cavitation results the central necrotic lung
tissue attenuating rounded mass surrounded by a separates from the adjacent lung – creating an Air Crescent
crescent of air – this is the characteristic MONOD SignQ - resembles an Aspergilloma
SIGNQ. Sometimes the mass may entirely fill the cavity,
thus taking on the shape of the cavity, obliterating the Pneumocystis Jiroveci (Formerly Carinii)
surrounding air crescent and no longer being mobile. Pneumonia
•• Air crescent signQ: is actually used to describe a
recovering case of invasive pulmonary aspergillosis. ƒƒ Most common opportunist infection in AIDS patientsQ
It is wrongly used for the air crescent around an ƒƒ CXR may be normal in early cases. Peculiar finding is
aspergilloma. The correct sign in Aspergilloma is Perihilar and mid and lower zone bilateral interstitial/
Monod sign. ground glass opacitiesQ that may progress and involve the
86 entire lung
ƒƒ On HRCT: Bilateral ground glass opacities/infiltratesQ (GGOs) spreading from the hila to periphery.
ƒƒ Dark bronchus sign/Black bronchus signQ: Dark/black appearing bronchus relative to the GGO involving lungs
ƒƒ Cavities/Pneumatoceles/Subpleural blebsQ may develop leading to PneumothoraxQ

RESPIRATORY SYSTEM
Egg Shell Calcification of Lymph NodesQ – Discussed in
Middle Mediastinal Mass Lesions Later
2
CLINICAL QUIZ - SOLUTION
So, there is a dense area of consolidation in the CXR that you see. This finding along with the history of fever, respiratory distress,
elevated CRP and elevated TLC – suggests infective etiology. So the treatment would be to start with empirical antibiotics.
 Now rarely though the imaging findings can point toward a specific underlying etiological agent. Out of the very few such clues,
one is seen here – an air-filled cystic lucency just adjacent to a consolidation – suggests a Pneumatocele. A pneumatocele in turn is
suggestive of staphylococcal infection ! Hence the answer is B. Staphylococcal pneumonia.
 Here is a challenge for you. Can you quickly go through the above text and identify such clinchers? Here let me start by enlisting a
few (you fill in the rest of the blanks):

Causative agent Imaging clincher


Staphylococcal pneumonia Pneumatocele
Klebsiella Bulging fissures
Mycoplasma/Atypical pneumonia
Actinomycosis
Varicella
Aspergilloma
Pneumocystis


Pleural Abnormalities
3
CLINICAL QUIZ
4. A patient presented with sudden onset breathlessness. His SpO2 is dropping
and his respiratory rate is 32/min. On examination the left hemithorax is
hyper-resonant on percussion & has absent breath sounds over the left
hemithorax. As soon as you look at this radiograph – what should be your 1st
step in the management of this patient? (AIIMS Nov 2016)
A. Secure the airway C. ICD insertion
B. BP measurement D. Left-side needle thoracotomy
5. A patient of RTA with injury over chest and limbs has low SpO2. M mode US
of right upper part of chest shows stratosphere sign. What is the diagnosis?
Pleural Abnormalities
 (AIIMS Nov 2013)
A. Hemothorax C. Cardiac tamponade
B. Pneumothorax D. Pulmonary embolism

PLEURAL EFFUSION
Ultrasound • BEST investigationQ for detection of minimal pleural effusion (Remember- Fluid is Friend of UltrasoundQ)
• Can detect even minimal quantities – 3–5 mLQ
• Fluid is seen as Anechoic/hypoechoic with fine internal echoes in the pleural space
• Fluid-Color signQ: (Presence of color signal in the fluid collection). Because of the transmitted respiratory and
cardiac movements
Contd... 87
Lateral Decubitus • BEST Radiograph projectionQ for detection of minimal pleural effusion
Radiograph • Can detect around 10–25 mLQ of fluid as it settles down to the dependent lateral chest wall in this position
Erect Lateral • Next best for detection of pleural effusion - Shows the fluid accumulated in the posterior CP angle recessQ
CONCEPTUAL REVIEW OF RADIOLOGY

Radiograph • Can detect around 75 mLQ of fluid


Erect PA/AP • Usually PA is preferred over AP
Radiograph • Can detect around 150–200 mLQ of fluid
CT Scan • Can detect minimal fluid.
• More sensitive than radiographsQ, almost same sensitivity as USG
• Especially useful for loculated collections
• Can be used for guided interventions

Blunting of costo-   Clinical Pearls


phrenic angle – Opaque Hemithorax/White-out lung – How to differentiate
Earliest finding of between the underlying causes?
pleural effusion on a
Opaque hemithorax is a very striking imaging abnormality. One
CXR PA view
entire hemithorax appearing as opaque – is something you
cannot miss. But what lies inside that opaque hemithorax. Of
the various pathologies and their combinations that can cause
this appearance the 3 major conditions include 1. Massive
pleural effusion 2. Complete lung collapse and 3. Complete lung
consolidation.
Believe it or not it is indeed possible to distinguish between them
based on just 1 finding on the radiograph. Yes! Just 1 finding! and
that is “Mediastinal shift”
Massive Pleural effusion Mediastinal shift to opposite
side – due to mass effect
Complete lung collapse Mediastinal shift to same
side – due to volume loss
Complete lung consolidation No mediastinal shift – as
Band/Layer of fluid consolidation neither causes
rising up vertically mass effect nor volume loss.
along the left latera
chest wall – Lamellar
pleural effusion CXR Supine AP View
ƒƒ Least sensitive for detecting pleural effusionsQ
ƒƒ The fluid accumulates in the posterior aspect of the
Imaging Findings on CXR Erect PA View hemithorax along the posterior chest wall–as in supine
position this is the most dependent part.
ƒƒ Earliest finding is blunting of lateral CP angle recessQ –
ƒƒ Initially there is a generalized hazy homogeneous opacity
suggests around 100 – 200 mL of fluid that has accumulated
with ill-defined margins over the lower lung zones.
in the posterior CP angle recess and has just spilled into the ƒƒ With further fluid accumulation, the opacity of the entire
lateral CP angle recess – causing its blunting. hemithorax increases, and obscuration of the diaphragm
Pleural Abnormalities

ƒƒ With further fluid accumulation the opacity rises up and has margin occurs.
a typical lateral upward sloping slightly ill-defined margins
– Meniscus signQ. The fluid will obscure the diaphragmatic Atypical Patterns of Pleural Effusion
marginQ – Silhouette sign. (Refer to Image 2 in "Imaging
Signs in Radiology") Lamellar Pleural EffusionQ
ƒƒ A very large pleural effusion appears as an Opaque ƒƒ Represents band like/thick layer of pleural fluid that rises up
hemithorax/White-out lungQ with a mediastinal shift vertically along the lateral lung surface
to the contralateral side. The mediastinal shift can be less
prominent or even absent in the presence of underlying
Subpulmonic Pleural EffusionQ
lung pathology (e.g. collapse) ƒƒ Fluid collection between the base of lung (Visceral pleura)
and Dome of diaphragm (Parietal pleura)Q.
ƒƒ More common on right sideQ

88
CT Signs to Distinguish between Pleural
Right pleural effusion Effusion and Ascites
on a Supine AP film
Displaced crus On a horizontal axial image–
– Haziness in right

RESPIRATORY SYSTEM
signQ Pleural fluid collects posterior to
hemithorax but
vascular markings are diaphragmatic crus and hence displaces the
clearly seen crus anteriorly.
Ascites collects anterior to the crus and may
cause posterior displacement
Diaphragm signQ Any fluid that is on the exterior of the dome
of diaphragm is in the pleura. Any fluid
within the dome is ascites.
Interface signQ Interface between liver/spleen and pleural
fluid is less sharp than that between the
liver/spleen and ascites
Bare area signQ The peritoneal coronary ligament prevents
ascitic fluid from extending over the entire
posterior surface of the liver, whereas in a
free pleural space, pleural fluid may extend
over the entire posterior costophrenic recess
behind the liver

  Clinical Pearls
White-out lung/ Side Predilection of Pleural Effusion
Opaque hemithorax
Predominantly Left-sided Predominantly Right sided
- Gross right sided
pleural effusion is seen in: pleural effusion is seen in:
pleural effusion
• Pancreatitis • Heart failure
causing tracheal
• Aortic dissection • Liver abscess
shift toward left
• Esophageal rupture • Ascites
side
• Pericarditis

ƒƒ On Erect CXR PA view and Lateral view, subpulmonary


effusion presents as an elevated hemi-diaphragm–termed Loculated
as pseudo diaphragmatic contour. pleural
ƒƒ On Right side: Diaphragmatic contour is alteredQ–its apex effusion along
the Horizontal
lies more laterally than normal. Mild blunting of CP angle
fissure
may be seen with tracking of fluid in fissures.
ƒƒ On Left side: Seen as increased distance between gastric air
bubble and lung base (>2 cm)Q.

Loculated/Fissural Pleural Effusion


ƒƒ May occur in fissures – as they have 2 layers of pleura and
hence a potential space or as a result of pleural adhesions/
lung disease.
Pleural Abnormalities
ƒƒ Loculated effusions usually have little depth but
considerable width–like a biconvex lensQ
ƒƒ Fissural effusion is usually oval/elliptical with long axis of
the oval being parallel to the fissure involved.
ƒƒ Horizontal fissure loculated effusion: Seen as oval opacity
on PA and Lateral view with its axis parallel to the fissure
ƒƒ Oblique fissure loculated effusion: Poorly-defined opacity
on PA film but well seen on lateral film since the fissure is
seen tangentially.
Loculated
ƒƒ They vanish rapidly with treatment–hence termed “Vanishing
pleural effusion
lung tumorsQ/Phantom lung tumorsQ” along the
Oblique fissure
89
USG image – The CT Thorax
anechoic (black) area shows right
marked as “???” Pleural effusion
CONCEPTUAL REVIEW OF RADIOLOGY

above the Liver/Right seen as


hemidiaphragm – is hypodensity in
mild pleural effusion dependent right
hemithorax

T2W MRI Coronal Right


image showing Pneumothorax –
hyperintense fluid in Look at the Visceral
right pleural cavity pleural line,
with underlying Hyperlucency with
collapsed lung absent vascular

PNEUMOTHORAX Tension PneumothoraxQ


Pneumothorax is presence of air in pleural cavity. Imaging Findings
Most common cause of Pneumothorax is – Spontaneous
pneumothorax due to rupture of congenital pleural blebQ. Most important complication of Pneumothorax.
Of the above 4 classical imaging findings, if the last 2 findings
CXR Imaging Findings are prominently seen then Tension pneumo should be considered.
ƒƒ CXR is the initial examination of choiceQ. ƒƒ Mediastinal ShiftQ: Towards contralateral side is greater in
ƒƒ An Expiratory Chest X-rayQ is much better for detection Inspiration – is diagnostic.
of minimal pneumothorax. Refer to the Concept highlight ƒƒ Deep Sulcus SignQ: as a result of diaphragmatic depression.
later in this discussion. Sometimes the dome of diaphragm may be inverted.
ƒƒ Visceral pleural lineQ: ƒƒ Expansion of the Rib cageQ: Due to hyperdistended
•• As air collects in the pleural space it separates the thoracic cavity on the affected side.
parietal and visceral pleura, with the latter being
pushed inwards. Thus, there is air density lining the
visceral pleura (water density) – this gives rise to a
sharp margin of the visceral pleura becoming visible –
DRDS/Silhouette sign.
Pleural Abnormalities

Left Tension
ƒƒ Hyperlucency with absent vascular markingsQ: Pneumothorax
•• The pleural space filled with air appears hyperlucent – Look at the
(excessively black) with absent vascular markings–as prominent right-
there are no blood vessels in the pleural space. ward Mediastinal
•• Lung fields though lucent have internal blood vessel shift and Deep
markings – this point helps to distinguish between sulcus sign
Pneumothorax versus Normal lung
ƒƒ Mediastinal shiftQ:
•• Pleural air causes mass effect and results in mediastinal
shift that is always toward the OPPOSITE SIDEQ
ƒƒ Deep Sulcus SignQ:
•• Mass effect may also be exerted on the ipsilateral dome
of diaphragm which is pushed down and hence the CP
angle sulcus appears deep down
90
• So, the amount of air (pneumothorax) remains constant in
inspiration and expiration, but in expiration there is a significant
decrease in the surface area of the inner chest wall and the lung
surface – so the same amount of Butter/Pneumothorax appears

RESPIRATORY SYSTEM
as a thicker layer on the Smaller bread slice/Chest wall.
• Feeling Hungry (for more Concepts)?

CT for Pneumothorax
Right Tension ƒƒ CT is considered the gold-standard in the diagnosis of
Pneumothorax – Look at
pneumothoraxQ.
the prominent left-ward
ƒƒ Best diagnosed on lung window imagesQ when air is seen in
Mediastinal shift and
the non-dependent part
Deep sulcus sign
USG for Pneumothorax
The normal lung interface with pleura shows lung sliding with
  Clinical Pearls vertical comet tails running down from the pleural surface.
Treatment of Tension Pneumothorax In pneumothorax:
Tension pneumothorax is a potentially life-threatening condition ƒƒ Absent sliding of the pleuraQ: Due to air in between the
with death occurring within a few minutes of onset. A lot of parietal and visceral pleura, preventing lung from sliding.
questions have been asked regarding the management of ƒƒ Absent comet tail artifactsQ: This sliding is absent and so
tension pneumo. are the comet tail artifacts from the pleura.
Needle Thoracotomy ƒƒ Lung point identification signQ: Visualizing the junction
• 1st step in treatmentQ between sliding lung and absent sliding is known as the
• Putting a needle in the pleural space creates a communication lung point sign - 100% specific for pneumothorax. Also gives
between the pleura and atmosphere, thus under pressure an indication of pneumothorax size by its location. Though
gradient the air comes out of the pleural cavity and this specific, it is not sensitive
continues until pressure equalization is achieved—thus the ƒƒ Stratosphere signQ: Seen on M mode ultrasound. Normal
TENSION component is relieved sliding movement of lung produces Seashore signQ: whereas
• Needle is inserted at: absence of lung sliding results in Barcode/Stratosphere signQ
ƒƒ In adults—5th intercostal space slightly anterior to mid ƒƒ Sea-shore signQ-AIIMS Nov 2019 Pattern – is a sign seen on M mode
axillary line ultrasound in normal USG Thorax. The chest wall move­
ƒƒ In Children—2nd intercostal space in the midclavicular line ments form the waves while the sand is formed by the
ICD Insertion normal lung. Refer to Image-based Question 12 for the
• 2nd step – after the tension component is relieved image.
• For the residual air remaining inside the pleural cavity an inter-
costal drain with a water seal is placed, which will gradually Other Complications of Pneumothorax
resorb the air
• Inserted in the triangle of safety comprising of: ƒƒ Hydropneumothorax: Air and fluid in the pleural cavity –
ƒƒ Anteriorly—Anterior axillary fold results in a sharp horizontal fluid levelQ. In pleural effusion
ƒƒ Posteriorly—Posterior axillary fold the upper margin of fluid is usually Ill-defined and meniscus
ƒƒ Apex—Axilla like in shape.
ƒƒ Base—5th intercostal spaces ƒƒ Re-expansion Pulmonary edemaQ:
•• Results from rapid drainage of a large pneumothorax
•• May also complicate rapid drainage of a large pleural
Expiratory CXR is Better for Diagnosis
Pleural Abnormalities
effusion
CO of Minimal Pneumothorax •• Extensive consolidation develops throughout ipsilater-
NC E P T al lung.
• Let us consider a common day-to-day example: •• Usually resolves in 24–48 hours.
ƒƒ You have 2 blocks of Butter – both exactly of the same volume.
ƒƒ You have 2 Bread slices – one is regular size and the other is Unilateral Hypertranslucent HemithoraxQ
half of the regular size.
(Mnemonic is PACS)
• One fine early morning you decide to coat both slices with
butter. What will happen? You will agree that the smaller slice ƒƒ Pneumothorax
will have a much thicker layer of butter on it, isn’t it? ƒƒ Patient rotation
• Now consider the following: ƒƒ Patient position not proper – as in Scoliosis
ƒƒ Butter – is amount of air in pneumothorax. ƒƒ Poland syndrome – absent Pectoralis major/surgical
ƒƒ Bread slices – represent the chest wall and lung surface in removal
Inspiration (regular size) and Expiration (half of regular size). ƒƒ Post-mastectomy
Contd... ƒƒ Pleural effusion – contralateral
91
ƒƒ Pulmonary embolism – Westermark sign ƒƒ Most commonly involve the lower portions of the chest,
ƒƒ Pulmonary emphysema - asymmetric sparing the apices and costophrenic angles.
ƒƒ Air trapping: Bronchial obstruction/Foreign body aspiration, ƒƒ Incomplete Border sign/Pregnant lady sign on CXRQ: The
Obliterative bronchiolitis inner margin is often well-defined because it is tangential
CONCEPTUAL REVIEW OF RADIOLOGY

ƒƒ Congenital lobar emphysema (CLE) to the X-ray beam and the adjacent lung is a good contrast
ƒƒ Swyer-James syndrome medium. The tapering outer margin is indistinct as it is en
face to the X-ray beam and the chest wall provide less tissue
Bilateral Hypertranslucent LungQ contrast.
ƒƒ Over-penetrated film ƒƒ Rarely short linear regions of fibrosis are seen extending
ƒƒ Thin body habitus radially away from the plaque - Hairy plaque SignQ
ƒƒ Bilateral mastectomy
ƒƒ Right to left shunt congenital cardiac disease Pleural Malignancies
ƒƒ Over-expansion of lungs – Emphysema, Asthma, Acute ƒƒ Metastasis – Most commonQ
bronchiolitis ƒƒ Malignant mesothelioma:
•• Associated with asbestos exposure – particularly
CrocidoliteQ
MISCELLANEOUS PLEURAL •• Latent period is usually 20–40 yearsQ
ABNORMALITIES •• Chest radiography is the initial screening examination,
while computed tomography (CT) scanning is preferred
for staging the tumor.
Bilateral Pleural Calcifications •• Nodular pleural thickening around all or part of lung
Usually seen in asbestosis and some other pneumoconiosis. This with hemorrhagic pleural effusion– is usual presentation
calcification is termed as Pleural Plaques: •• Mediastinum is usually central/shifted to same side –
ƒƒ Pleural plaquesQ are the most common manifestation of due to volume loss of underlying lung
asbestos-related disease •• Positron emission tomography (PET) is becoming
ƒƒ Holly leaf sign: The holly leaf sign refers to the appearance useful in two clinical settings:
of pleural plaques on chest X-rays. Their irregular  Differentiating between benign and malignant
thickened nodular edges are likened to the appearance of asbestos-related pleural thickening
a holly leaf.  Assessing for nodal metastases
ƒƒ CT–is Best investigationQ.  There is a correlation between the degree of FDG
ƒƒ Arises from the parietal pleuraQ. uptake and the biological aggressiveness of the
tumor, which may help to guide treatment

Right Hydro- Right Hydro-


pneumothorax Pneumothorax
– Look at the CT – Look at the
horizontal horizontal fluid
fluid level and level and
pneumothorax pneumothorax
on right side
with an ICD
in situ. Mild
pleural effusion
also noted on
left side.
Pleural Abnormalities

Malignant
Mesothelioma –
Thick enhancing
nodular pleural
thickening with
Dense sheet like pleural effusion on
pleural-plaques right side
92 (Black arrows) (Yellow arrows)
3
CLINICAL QUIZ - SOLUTION
The radiograph is showing a typical hyperlucency with absent vascular markings, visceral pleural line, mediastinal shift toward right
side and deep sulcus sign. These imaging findings along with the clinical findings of falling SpO2 and absent breath sounds suggest

RESPIRATORY SYSTEM
Tension pneumothorax. The 1st step in management of tension pneumothorax is Needle thoracotomyQ (hence ans is D.). ICD insertion
may be done after the tension component is relieved and will treat the residual pneumothorax over the next few days/weeks.
Stratosphere sign on M mode USG is suggestive of pneumothorax. Other findings of pneumothorax on USG includes absence of pleural
slidingQ, visualization of A linesQ, Lung point identification signQ. Sea-shore signQ is seen in normal patients on M mode USG.

Lobar Collapse and Airway Disorders


4
CLINICAL QUIZ
6. A patient presented with recurrent sinusitis, lower respiratory tract infec-
tions and fever. He has been a known case of male factor infertility with
oligospermia with poor sperm motility. A CXR was done and shows a typical
combination of abnormalities. What is the most likely diagnosis here?
A. Scimitar syndrome C. Kartagener syndrome
B. Macleod syndrome D. Lady Windermere syndrome
7. The IOC for diagnosis of Bronchiectasis is: (JIPMER May 2017)
A. CXR C. HRCT, 
B. Fluoroscopy D. Bronchoscopy

LOBAR COLLAPSE

Lobar Collapse and Airway Disorders


CT is IOC/Best Investigation for Lobar Collapse
Radiological Signs of Collapse

Direct Signs
Displacement of • Most reliable sign
interlobar fissures • Extent of collapse will determine degree of displacement
Loss of aeration • Will result in increased density of lung field – is due to retained secretions
• As air density is converted to water density - adjacent water density margins like heart/diaphragm will be obscured
Vascular and • Crowding of blood vessels
Bronchial signs • Air bronchogram sign – if visible will reveal crowding of vessels

Indirect Signs
These are seen due to compensatory changes
Elevation of ipsilateral • Usually seen only in lower lobe collapse
hemidiaphragm • Juxtaphrenic peak signQ:
Mediastinal shift • Upper lobe collapse – Ipsilateral tracheal deviation
• Lower lobe collapse – Heart may be displaced
• Least mediastinal shift is seen in RML collapseQ
• Maximum mediastinal shift is seen in right/left lower lobe collapseQ
Contd... 93
Hilar displacement • Upper lobe collapse – Hilum is elevated
(Direct sign according to • Lower lobe collapse – Hilum is depressed. More accurate description is that the hilum appears smaller than
Grainger) normal in lower lobe collapse.
CONCEPTUAL REVIEW OF RADIOLOGY

Compensatory • Normal lung may appear hypertranslucent (black) with widely-spaced vessels
hyperinflation • One complete lung if collapsed may cause over-inflation of the contralateral lung and herniation across
midline
• Shifting granuloma sign: Hyperexpansion may result in a change in position of lung lesions previously
documented
• Luftsichel sign (meaning air crescent)Q– seen in LUL collapse
Crowding of Ribs • Seen due to volume loss of involved lung

Fallen Lung Sign •• The aortic knuckle is lined normally by the


apicoposterior segment of LUL – and hence it may not
ƒƒ Appearance of collapsed lung occurring with a fractured
be seen as a sharp margin in cases of LUL collapse.
bronchus
•• But in severe LUL collapse, the normal LUL may undergo
ƒƒ The lung falls way from the bronchus.
compensatory hyperinflation. When hyperinflated,
Lobar Collapse Patterns the superior segment of LUL now comes and lines
the aortic knuckle (normally it lines the descending
thoracic aorta laterally).
RUL Collapse
•• This results in a sharp margin being visible of the aortic
ƒƒ RUL Collapses – UPWARDS knuckle and an “air-crescent” seen lateral to the aortic
ƒƒ Forms increased density at the apex of the hemithorax knuckle medial to the collapsed lung – LUFTSICHEL
adjacent to the right side of the mediastinum. SIGN.
ƒƒ Horizontal fissure is elevated and forms a concave inferior ƒƒ Juxtaphrenic peak signQ:
border depending on the degree of collapse.
ƒƒ Golden S Sign: Named after eminent radiologist Ross RML Collapse
Golden Refer to Image 4 in “Imaging Signs in Radiology” ƒƒ Very subtle findings on a CXR PA view – a Lordotic AP view
•• Seen in RUL collapse due to a central mass lesion is better for depiction.
•• Centrally located mass lesion either infiltrates into the ƒƒ Collapsed lobe lies immediately adjacent to the right heart
RUL bronchus or occludes it – hence the RUL collapses border – this border is obscured – Silhouette sign
•• RUL collapses – UPWARD – thus pulling the horizontal ƒƒ Lateral CXR – relatively easy to identify as horizontal fissure
fissure upward. and lower portion of oblique fissure are pulled toward each
•• The medial half of the horizontal fissure is prevented other with an intervening opacity of the collapsed lobe.
from retracting upward by the central mass lesion –
ƒƒ RML syndrome: Chronic RML collapse + Bronchiectasis
Lobar Collapse and Airway Disorders

hence it wraps around the mass lesion.


•• Thus, the lateral half of the horizontal fissure is Concave
inferiorly (due to RUL collapse) and its medial half is   Clinical Pearls
Convex inferiorly (due to the mass lesion) – this Concavo- Lady Windermere syndrome
Convex appearance is termed as Golden S sign.
Named after Lady Windermere, a character in Oscar Wilde’s 1892
ƒƒ Juxtaphrenic peak signQ: A small triangular density seen
play “Lady Windermere’s fan”. Refers to a pattern of pulmonary
along the diaphragm surface in a left/right upper lobe Mycobacterium avium complex (MAC) infection seen typically
collapse due to reorientation of an inferior accessory fissure in elderly white women who chronically suppress the normal
cough reflex. Bronchiectasis, centrilobular nodules and eventual
LUL Collapse scarring and volume loss are common features. It predominantly
ƒƒ LUL collapses – FORWARDS affects the middle lobe and lingula.
ƒƒ Direction of collapse is anteriorly and medially (rather than
superiorly as in RUL collapse) and the entire oblique fissure
is displaced in that direction parallel to the chest wall on the Right and Left Lower Lobe Collapse
lateral view ƒƒ Similar collapse pattern seen on right and left sides.
ƒƒ On a CXR PA view there is a Veil-like/Curtain-like - ƒƒ On a lateral CXR the posterior half of ipsilateral
increased density of the whole of the affected hemithorax. hemidiaphragm may not be seen as a sharp margin. Also,
The increased density maximum at the hilum and gradually the vertebral column may appear progressively denser from
fades out laterally, superiorly, and inferiorly. superior to inferior – this is opposite to the normal finding.
ƒƒ Loss of sharp margins of Aortic arch, left upper mediastinum, ƒƒ On CXR PA view, collapsed lower lobes form a triangular
left heart border – may be seen – Silhouette sign. density behind the heart on the affected side. The medial
ƒƒ Luftsichel sign: portion of the hemidiaphragm may be obscured. The lower
•• Literally means an “air-crescent” lobe pulmonary artery is not seen.
94
ƒƒ Superior triangle sign: Refers to a triangular density to the
right of the mediastinum seen in right lower lobe collapse   Clinical Pearls
due to displacement of anterior junctional structures in Round Atelectasis/Folded Lung/Blesovsky’s Syndrome
superior mediastinum • Unusual form of collapse related to asbestos exposure with

RESPIRATORY SYSTEM
ƒƒ Flat waist sign: It is seen in extensive collapse of the left infolding of redundant pleura – often misdiagnosed as a mass
lower lobe and seen as flattening of the contours of the lesion.
aortic knuckle and main pulmonary artery due to cardiac • CT is Best investigation for diagnosis
rotation and displacement to the left • On CXR appears as a homogenous mass, up to 5 cm diameter,
with ill-defined edges. It is always pleural based and is
Complete Lung Collapse associated with pleural thickening.
• Comet tail sign and Crow feet sign: Vascular shadows radiating
ƒƒ Results in Opaque hemithorax/White-out lung – discussed from the opacity
previously

LUL collapse –
RML Collapse –
Veil/Curtain like
Triangular right
opacity in the
paracardiac
left hemithorax
opacity lined by the
with a sharp
Horizontal fissure
aortic knuckle
(Yellow line) and
(Luftsichel sign)
the lower part of
Oblique fissure
(Blue line)

Lobar Collapse and Airway Disorders


Left lower Complete right lung
lobe collapse – collapse – Right
Triangular wedge- opaque hemithorax
shaped opacity with trachea pulled
overlapping the toward the right side
heart shadow (Black arrow)

HRCT - Extensive
Multiple cystic lucencies right lung
in bilateral paracardiac Bronchiectasis –
areas and lower zones – Bunch/Custer of
Bronchiectasis Grapes appearance

95
AIRWAY DISORDERS •• Alteration of lung vessels:
 Arterial depletion - absence or displacement of
Bronchiectasis vessels caused by bullae
 Eventually cor pulmonale may occur
CONCEPTUAL REVIEW OF RADIOLOGY

ƒƒ It is chronic local, irreversible dilatation of bronchi,


•• Bullae are emphysematous spaces with a diameter
associated with inflammation.
of >1 cm in distended state with their walls formed
ƒƒ Pathologically may be of 3 types: Cylindrical – Varicose –
by compressed surrounding lung/pleura. May lead to
Saccular/Cystic
Pneumothorax
ƒƒ Traction bronchiectasis: Bronchiectasis that results from
ƒƒ Important types and corresponding CT findings:
the traction of fibrotic lung surrounding an airway
ƒƒ CXR is usually the initial investigation. Centrilobular/ • Mainly involves the proximal respiratory
ƒƒ HRCT/Volumetric multidetector Chest CT – is now the Centriacinar bronchioles and alveoli in the central part
Investigation of choice emphysema of the acinus. Usually develops in upper
ƒƒ CXR findings: portions of lungs
•• May be normal in early cases • On CT - Presence of multiple, small round
•• Tram-track sign: Thickened bronchial walls seen areas of abnormally low attenuation,
either as single thin lines or as parallel line opacities. distributed throughout the lungs but mainly
•• Ring shadows: When seen end-on, airways appear as in upper lobes
poorly defined ring or curvilinear opacities. Paraseptal • Areas of low attenuation are seen in the
•• Dilated bronchi filled with mucous or pus result in emphysema subpleural areas, along the peripheral or
tubular or ovoid opacities of variable size. mediastinal pleura, mainly in the upper lobe
ƒƒ CT findings: and along the fissures.
•• Bronchial dilatation: Cardinal sign of bronchiectasis. Panacinar/ • Nonselective process characterized by
Seen as lack of tapering of bronchial lumen as they Panlobular destruction of all the lung distal to the
travel distally. emphysema terminal bronchiole–almost always most
•• Increased bronchoarterial ratio: severe in lower lobe
 Diameter of a bronchus should measure • Panlobular emphysema, secondary to
approximately 0.65-1.0 times that of the adjacent α1-antitrypsin deficiency, is commonly
pulmonary artery branch associated with bronchiectasis
 Between 1 and 1.5 may be seen in normal
individuals, especially those living at high altitude
Paracicatricial • Distension and destruction of terminal
emphysema airspaces adjacent to fibrotic lung lesions –
 Greater than 1.5 indicates bronchiectasis
most commonly seen in TB
•• Signet ring sign: Dilated bronchus and accompanying
pulmonary artery branch are seen in cross-section. The
bronchus and artery should be of the same size, whereas Croup/Laryngotracheobronchitis(IBQ NEET 2020
Pattern) (Refer to Q. No 1 in Image-based Questions)
in bronchiectasis, the bronchus is markedly dilated.
•• String of pearl sign: In Varicose bronchiectasis there
Lobar Collapse and Airway Disorders

ƒƒ Viral infection of the upper airway by parainfluenza virus or


are intermittent narrowed and dilated segment to the respiratory syncytial virus (RSV).
affected bronchus when viewed in long axis and can give ƒƒ Steeple sign/Wine bottle sign: Uniform narrowing of
a string of pearls sign or irregularly beaded appearance.
subglottic airway seen on an AP radiograph.
•• Cluster of grapes sign: the dilated bronchi in
approximation appear as a bunch of grapes – seen in
cystic bronchiectasis
Epiglottitis
•• Lobulated gloved finger, V- or Y-shaped densities – ƒƒ Life-threatening condition caused by inflammation of the
seen due to mucus-filled bronchi. Often seen in ABPA. epiglottis and aryepiglottic folds.
ƒƒ Various signs may be seen on a lateral neck radiograph.
Emphysema ƒƒ Thumb sign: Edematous and enlarged epiglottis which
is seen on lateral soft-tissue radiograph of the neck, and it
ƒƒ Permanent, abnormal enlargement of airspaces distal to the
suggests a diagnosis of acute infectious epiglottitis.
terminal bronchioles, accompanied by the destruction of
ƒƒ Omega sign: Seen on laryngoscopy - refers to the thickened
their walls without obvious fibrosis
aryepiglottic folds and epiglottis.
ƒƒ CXR is initial investigation. HRCT is Investigation of choice.
ƒƒ CXR findings: Over-inflation and alteration of lung vessels
are most important findings.
Swyer-James/MacLeod’s Syndrome
•• Signs of over inflation include: ƒƒ Constrictive obliterative bronchiolitis–occurs as a result
 Lung height >29.9 cm, location of the right of childhood viral infection causing bronchiolitis and
hemidiaphragm at or below the anterior aspect of obliteration of small airways. This results in long-standing
the seventh rib air trapping and subsequent panacinar emphysema
 Flattening of the hemidiaphragm ƒƒ CXR shows Hypertransradiant hemithorax due to air
 Narrowing of the transverse cardiac diameter – trapping.
called as Tubular appearing heart
96
Paraseptal Panacinar Emphysema
Emphysema - - Widespread areas
Areas of low of abnormally low

RESPIRATORY SYSTEM
attenuation attenuation (White
are seen in the arrows), with fewer
subpleural areas, blood vessels mainly in
along the peripheral the lower lobes
or mediastinal
pleura, mainly in
the upper lobe and
along the fissures.

Centrilobular
Emphysema:
Presence of
multiple,
small areas of
emphysema
scattered
throughout the
lung is diagnostic
of centrilobular
emphysema.

4
CLINICAL QUIZ - SOLUTION
The CXR here shows a very peculiar combination of abnormalities, this syndrome. One should always keep in mind the possibilities
namely of Kartagener's syndrome in those patients presenting with
• Dextrocardia – heart is on the right side – look at the cardiac recurrent upper and lower respiratory tract infections, sinusitis
apex or bronchiectasis. Although there is no specific treatment for
• Bronchiectasis – See the lucencies in the left lower zone close this condition, failure to recognize the condition may subject
to the hilum. the patient to unnecessary and repeated hospital admissions,
These imaging findings along with the clinical history of sinusitis – investigations and inappropriate treatment. 
suggests a diagnosis of Kartagener syndrome (Ans is C). The IOC of Bronchiectasis is HRCT (Thus answer is C.). CXR may be
Kartagener's syndrome is an autosomal recessive disorder used as the initial investigation but HRCT can outline the extent
characterized by the classic triad of dextrocardia, bronchiectasis and severity of bronchiectasis is much better.
and sinusitis. Abnormal ciliary motility is the basic pathology in

Mediastinum
5
Mediastinum

CLINICAL QUIZ
8. A patient presents with a mediastinal mass lesion – that on imaging 9. CE-CT is considered as the investigation of choice for
is localized to the posterior mediastinum. On further evaluation this almost all Mediastinal mass lesions. However there
patient is also found to have a vertebral segmentation anomaly, namely is an exception where in MRI is the IOC for which of
hemivertebra. The most likely lesion that this patient has is: the following:
A. Schwannoma A. Anterior mediastinal mass
B. Neuroblastoma B. Middle mediastinal mass
C. Neurofibroma C. Thyroid mass
D. Neurenteric cysts D. Posterior mediastinal mass 97
MEDIASTINAL COMPARTMENTS
Felson
Anatomical Division classification of
Mediastinum
CONCEPTUAL REVIEW OF RADIOLOGY

ƒƒ Anterior mediastinum: Lies anterior to anterior


pericardium and trachea
ƒƒ Middle mediastinum: Within the pericardial cavity
including trachea
ƒƒ Posterior mediastinum: Behind the posterior mediastinum
and trachea

Radiologic Division (Roentgen Compartments/


Felson Classification)Q
2 lines are drawn on a lateral CXR:
ƒƒ 1st line: Drawn along the back of the heart and front of
trachea
ƒƒ 2nd line: Drawn connecting a point on each thoracic vertebra
around 1 cm behind its anterior margin

Mediastinum is divided as:


Anterior Mediastinal Masses
ƒƒ Anterior mediastinum: In front of 1st line
ƒƒ The anterior mediastinum mainly contains - thymus, lymph
ƒƒ Middle mediastinum: Between the 2 lines
nodes, ascending aorta, pulmonary artery, phrenic nerves
ƒƒ Posterior mediastinum: Behind the 2nd line
and thyroid.
Imaging Modalities ƒƒ The most common mediastinal lesion overall – ThymomaQ
ƒƒ The most common anterior mediastinal lesion –
CXR • Usually 1st investigation ThymomaQ.
• May help in localizing lesions based on ƒƒ MOST COMMON Anterior mediastinal massesQ: (Mnemonic
Silhouette sign, Cervicothoracic sign, - 4T’s)
Thoracoabdominal sign, Displacement •• Thymus - Thymoma
of junction lines. •• Teratoma (germ cell)
•• Thyroid masses
CE-CT • IOC for most mediastinal lesionsQ
– except posterior mediastinal/
•• (Terrible) Lymphoma
ƒƒ Important CXR findings:
Neurogenic tumors
• CT-Guided biopsies may be done •• Displaced anterior junction line
•• Obliteration of cardiophrenic angle
CE-MRI • IOC for Posterior mediastinal/ •• Obliteration of retrosternal lucency (on lateral film)
Neurogenic tumorsQ as it can best •• Hilum overlay signQ: Opacity (depicting a mass lesion)
demonstrate neural/neural foraminal with the hilar vessels seen through it.
extension •• Obscuration of ascending aorta margin
• Useful for Solid vs Vascular vs Cystic
lesion differentiation Important Features of Anterior Mediastinal
• Poor for detection of calcification
Masses
USG/ • Solid vs Cystic lesions
Thyroid • Most thyroid masses represent downward
Echocardiography/ • Cardiac vs Paracardiac masses
masses extensions of multinodular colloid goiter/
Endoscopic USG • USG-guided biopsies
adenoma/carcinoma.
PET/PET-CT • LN involvement in Lymphoma and • Rounded or irregular, well-defined areas of
Lung Ca calcification - Benign lesions
• Specific radionuclide scans useful • Amorphous cloud-like - Carcinomas.
Mediastinum

in Thyroid masses, Neuroendocrine • Trachea is displaced and narrowed.


lesions, Pheochromocytomas • Radionuclide imaging with 123I or 131I
demonstrates the presence of thyroid tissue
within the mediastinum.
Contd...

98
Parathyroid • Mediastinal parathyroid tumors – Best
masses method for detection are Radionuclide
scansQ:
� 99mTc-sestamibi imagingQ or

RESPIRATORY SYSTEM
� 99mTc-thallium subtraction scanQ Well-defined Right
Cardiophrenic
Thymoma • Most common tumor of the thymus in
angle lesion –
adultsQ.
Pleuropericardial cyst
• Most common mediastinal mass lesion
(Confirmed on CT)
overallQ
• Most common primary tumor of the
anterior mediastinum in adultsQ.
• Most common association with thymoma -
Myasthenia gravisQ (50%)
• CT with contrast – is BEST modalityQ.
• Homogeneous density and uniform
enhancement after contrast medium and   Clinical Pearls
may occasionally be cystic.
• Large, heterogeneous masses, containing Cardiophrenic Angle Mass Lesions
areas of necrosis and calcification—Invasion This was a question asked in you DNB pattern examination in
of adjacent structures – may indicate thymic Dec 2018. Very few specific lesions are seen at the cardiophrenic
carcinoma angles:
• Large thymic masses with fat content – may • Pleuropericardial cyst
indicate Thymolipoma • Epicardial fat pad
Germ cell • Mediastinum is the most common • Morgagni’s hernia
Tumors (GCTs) extragonadal site for GCTsQ • Lymphadenopathy
• Mature teratoma – is Most common • RML mass
mediastinal germ-cell tumorQ • Pleural mass
• On CT - presence of fat, either as focal
collections or fluid fat or calcifications, is a very
helpful diagnostic feature favoring mature
(benign) cystic teratoma
• Seminoma – is Most common Malignant
GCT in mediastinum. Grows rapidly with
hemorrhage/necrosis and metastasis
Pleuropericar- • Thin wall cyst lined by mesothelial cells
dial cyst filled with clear fluid and attached parietal
pericardium
• Mostly occurs in Right anterior cardio-
phrenic angle

Soft tissue density Middle Mediastinal Masses


lesion with a ƒƒ The middle mediastinum contains: lymph nodes, trachea,
sharp right lateral
esophagus, azygos vein, vena cavae, posterior heart and the
margin, with broad
aortic arch.
base toward the
mediastinum – ƒƒ The majority of middle mediastinal masses will be foregut
mostly an Anterior duplication cysts (e.g. esophageal duplication or
mediastinal mass bronchogenic cysts)- in childrenQ and lymphadenopathy
– in adultsQ. Aortic arch anomalies can also present as
Mediastinum

middle mediastinal masses.


ƒƒ Mediastinal Lymphadenopathy:
•• Egg shell calcification in lymph nodesQ:
 Silicosis
 Treated lymphoma: (post-irradiation Hodgkin
disease)

99
 Coal workers pneumoconiosis Important Features of Posterior
 TB
Mediastinal Masses
 Sarcoidosis
 Scleroderma Neurenteric Discussed below
CONCEPTUAL REVIEW OF RADIOLOGY

 Amyloidosis: Rare cysts


 Blastomycosis Neurogenic • Most common tumors to arise in the
 Histoplasmosis tumors posterior mediastinumQ
• MRI - INVESTIGATION OF CHOICEQ
Important Features of Middle Mediastinal Neurogenic • Originate in an intercostal nerve in the
Masses tumors – paravertebral region.
Peripheral • Neurofibromas and Schwannomas present
Bronchogenic • Located adjacent to the trachea or main nerve tumors as well-defined round or oval posterior
cysts bronchi and can grow very large. mediastinal masses.
• CXR: Spherical or oval masses with smooth • Pressure deformity on adjacent bones (ribs,
outlines projecting from either side of the vertebrae) results in scalloping with cortical
mediastinum. thickening - diagnostic of a neurogenic lesionQ.
• CT is the BEST InvestigationQ. • Widening of intercostal spaces and
Esophageal • Imaging features are identical to those of intervertebral foramina
duplication bronchogenic cysts except that in these the • Show heterogeneous enhancement on post
cysts wall of the lesion may be thicker, the mass contrast CT/MRI
may assume a more tubular shape, and it • Target signQ - characteristic high signal intensity
may be in more intimate contact with the peripherally and low signal intensity centrally on
esophagus. T2-weighted image
• Barium swallow will show the features of • Dumb-bell shaped massesQ: Occurs when
extrinsic or intramural compression these extend into the spinal canal with widening
of the affected neural foramen – seen in
Neurofibromatosis type IQ
Posterior Mediastinal Masses Lateral thoracic • Protrusions of the spinal meninges through
ƒƒ The posterior mediastinum contains: Sympathetic meningoceles an intervertebral foramen - associated with
ganglia, nerve roots, lymph nodes, parasympathetic chain, neurofibromatosis
thoracic duct, descending thoracic aorta, small vessels and • Cystic lesions with pressure effects –
the vertebrae. scallopingQ – on adjacent bones
ƒƒ Neurogenic masses – MOST COMMONQ. These can arise Extramedullary • Rare phenomenon caused by compensatory
from the sympathetic ganglia (e.g. neuroblastoma) or from hematopoiesis expansion of bone marrow in various anemias,
the nerve roots (e.g. schwannoma or neurofibroma). particularly congenital hemolytic anemias

Typical left-sided CT Axial, Coronal and


Posterior mediastinal Sagittal reformatted
mass images – showing a
left-sided posterior
mediastinal mass
(Yellow arrows)
Mediastinum

Lateral CXR – Well-defined


opacity in the retrocardiac
region, overlying the spine,
close to the diaphragm Pregnant lady sign –
– suggests a Posterior in case of Left-sided
mediastinal mass (Confirmed Pleural based mass
on CT) lesion (Yellow arrow)

100
  Clinical Pearls Continuous left Free air present between the pericardium
hemidiaphragm and diaphragm, causing the entire left
Most common in Mediastinal lesions signQ hemidiaphragm to become apparent as a
• Overall most common mediastinal mass – ThymomaQ sharp margin on a lateral CXR – Silhouette

RESPIRATORY SYSTEM
• Most common anterior mediastinal mass – ThymomaQ sign
• Most common middle mediastinal mass – Lymph nodal massQ
• Most common middle mediastinal mass in children – Pneumopericar- Air anterior to the heart seen on a lateral
Duplication cystsQ diumQ CXR
• Most common posterior mediastinal mass – Neurogenic Naclerio’s V signQ Free air outlines the lateral margin
tumorsQ of descending aorta and laterally
between the parietal pleura and
medial left hemidiaphragm. Seen in
Miscellaneous Mediastinal Abnormalities pneumomediastinum secondary to
esophageal rupture – not specific for this
Pneumomediastinum cause
ƒƒ Important causes include Spontaneous (in asthma, V signQ Air outline the confluence of
coughing), esophageal perforation, Boerhaave’s syndrome, brachiocephalic veins superiorly
trauma.
Extrapleural air Round lucent gas bubble between the
ƒƒ CXR – initial investigationQ. CT – Best InvestigationQ
signQ parietal pleura and diaphragm in an infant
ƒƒ On CXR:
•• Radiolucent streaks representing free air may be
observed tracking along the margins of the heart, within   Clinical Pearls
the retrosternal space, or surrounding the trachea. Differentiating between Pneumomediastinum and Pneumo-
•• Since there is no anatomic partition between the pericardium
mediastinum and neck, free air in the mediastinum on an
Pneumomediastinum Pneumopericardium
erect CXR will ascend to the neck and will be invariably
seen as streaky lucencies at the base of neck soft tissuesQ Abnormality Free air in the Free air in the
•• Various named signs seen on a CXR: mediastinum pericardial cavity
Configuration Multiple lucent, thin Broad band-like
Ring around the A radiolucent area is observed surrounding of gas streaks lucency, Halo sign
arteryQ: the right pulmonary artery when viewed on
Distribution Lines the mediastinal Enclosed in the
a lateral chest radiograph.
structures – Aorta, pericardium, outlines
Tubular artery Air around the major aortic branches bronchi, trachea the ascending aorta
signQ esophagus. and main pulmonary
Commonly extends to artery, but does not
Double bronchial Clear depiction of bronchial walls due to air the neck extend along aortic
wall signQ both on inside and outside – Silhouette sign arch or into neck
Changes None Yes
Thymic sail signQ/ In infants with pneumomediastinum, the with patient
Spinnaker sail thymic lobes are shifted upward resembling position
signQ/Angel wing a full sail
signQ: Etiology May be spontaneous Usually postsurgical
or post traumatic when the pericardium
Continuous Free air is present between the pericardium was breached –
diaphragm signQ: and diaphragm, causing the central parts Cardiac surgery,
of the diaphragm to become apparent as a iatrogenic
sharp margin – Silhouette sign
Contd...
5
CLINICAL QUIZ - SOLUTION
The question here is asking you to identify the lesion which is commonly associated with vertebral segmentation anomalies like
Mediastinum

Hemivertebra. Remember that posterior mediastinal lesions like Neurenteric cysts are associated with such vertebral defects and thus
this combination is an important diagnostic clue. Hence the answer is D.
Neurenteric cysts result from incomplete separation of the foregut from the notochord in early embryonic life. It is seen as well-defined,
round, oval or lobulated mass in the posterior mediastinum between the esophagus (which is usually displaced) and the spine. MRI -
INVESTIGATION OF CHOICEQ for demonstrating the extent of intraspinal involvement
Another useful concept to remember is that CE-CT is the IOC of choice for all mediastinal mass lesions except Posterior mediastinal
masses. This is because most of the posterior mediastinal masses are neurogenic in origin and hence may have an intraspinal
component. For better delineation of this neurological extension/intraspinal component – MRI is IOC (as CT fails to image this extent
well). Hence the answer is D.
101
Lung Tumors
CONCEPTUAL REVIEW OF RADIOLOGY

6
CLINICAL QUIZ
10. A 70-year-old man, a known chronic smoker presents with cough and
hemoptysis. His CXR shown here reveals a focal mass lesion in the left
lung apex. He recently has started complaining of left upper limb pain
and weakness. On examination he has left-sided ptosis. Most likely
diagnosis is:
A. Bronchoalveolar carcinoma
B. Pancoast tumor
C. Lung metastases
D. Invasive aspergillosis
11. All lung carcinomas for diagnosis requires CECT thorax but MRI is useful
in which of the following: (AIIMS May 2019)
A. Small cell lung carcinoma
B. Adenocarcinoma
C. Carcinoid
D. Pancoast tumor

LUNG CANCER Non-small Cell Carcinoma


Adeno- • Most common type - overallQ
Role of Imaging Modalities carcinoma • Most common in womenQ
• Most common cell type in non-smokersQ (but
CXR still most patients are smokers)
ƒƒ Usually the 1st investigationQ • Usually peripheral in locationQ (Remember A
ƒƒ May suggest the presence of tumor and reveal associated in adenocarcinoma stands for Away)
findings (collapse, consolidation, effusion) • May arise in a fibrotic lung and rarely cavitates
– also known as Scar carcinoma
ƒƒ May suggest mediastinal extension (lymphadenopathy,
phrenic nerve palsy causing elevation of diaphragm) Alveolar cell • Subtype of Adenocarcinoma with certain
carcinoma/ special features
CT with Contrast Bronchiolo- • Arises more peripherally in Type II
alveolar pneumocytesQ
ƒƒ Most common and Best modality for assessment of • NOT associated with smokingQ
carcinoma
primary tumorQ (BAC) Arise within alveoli and hence produce
ƒƒ Reveals mediastinal involvement/chest wall involvement CONSOLIDATIONQ. AIR BRONCHOGRAMSQ are
ƒƒ Best for detection of calcificationQ also seen
• Extremely slow growing tumor
MRI with Contrast • The most common radiographic finding is a
ƒƒ MRI has proved to be better than CT in demonstrating chest solitary lobulated or spiculated pulmonary
wall and diaphragmatic invasion, mediastinal vessels/ mass indistinguishable from other types of
pericardial involvement. carcinoma.
ƒƒ MRI is BEST modality for demonstrating the extent of • Bubble-like lucenciesQ corresponding to patent
superior sulcus tumors/Pancoast’s tumorQ, best sequence small bronchi, air containing cystic lucencies,
or air bronchograms and cavitation may be
being coronal T1W images.
seen
PET/PET-CT • Endobronchial spread of tumor – specific for
BACQ
Lung Tumors

ƒƒ Used for staging lung carcinomaQ - greater accuracy


Squamous • Strongly associated with smokingQ
compared to CT and MRI in the detection of nodal diseaseQ. cell • Most common type associated with Pancoast
ƒƒ Fused PET–CT imaging provides registration of FDG carcinoma tumorQ
metabolic activity with the anatomical detail of CT - more • Arises centrally and grows slowlyQ
accurate than PET or CT alone in staging patients with non- • Most common carcinoma to show cavitationQ
small cell lung cancer • Corona RadiataQ: Multiple fine strands
ƒƒ Best for detection of distant metastasisQ. Most common radiating into the lung from a central mass -
endocrinological site involved is Adrenal glandQ. is highly suggestive of bronchial carcinoma.
In order to avoid a descriptive approach, we will study the Refer to Image 12 in “Imaging Sings in
102 imaging findings in a concise and tabular format. Radiology”
Contd...
• Collapse/consolidation of the lung beyond the • Usually Central in location. A mass in, or adjacent
tumor is the most frequent feature seen with to, the hilum is a particular suggestive of small cell
squamous cell carcinoma, in keeping with its carcinomaQ
central origin. • Associated with mediastinal and hilar

RESPIRATORY SYSTEM
• Poor prognosis lymphadenopathy
Large cell • Peripherally located • Worst prognosisQ
carcinoma • Presents as very large masses (in keeping with
its name), usually more than 4 cm   Clinical Pearls
Specific Types of Lung Metastases
Lymphangitis carcinomatosisQ:
Rounded large
soft tissue opacity • Denotes permeation of pulmonary lymphatics and/or their
in the left lung adjacent interstitial tissue by neoplastic cells.
field with mildly • The most common causes - carcinomas of the bronchus,
indistinct margins, breast, stomach and prostate.
in an elderly male • ON CXR: Fine reticulonodular shadowingQ and/or thickened
smoker – suggests septal linesQ
Lung Cancer. Needs • Subpleural edemaQ - results from lymphatic obstruction by
CE-CT for better tumor cells, seen as thickening of the fissures. Pleural effusion
characterization is common.
• CT: More sensitive than CXR. Non-uniform, often nodular,
thickening of the interlobular septa and irregular thickening
of the bronchovascular bundles in the central portions of the
Small Cell/Oat Cell Carcinoma lungs.

Small • Almost always in smokers Cannon-ball metastasisQ:


Cell Ca: • Considered a Neuroendocrine lung tumorQ • Multiple well-defined lesions in both lungs are called
Imaging • Fastest rate of growthQ - metastasizes early, dissemi- cannonball metastases due to their large, round appearance.
features nation usually evident at time of presentation The rich vascular bed of the lungs is hospitable to tumor
• Almost always treated medically emboli.
• Most common primary lung malignancy to cause • Renal cell carcinoma is by far the commonest causeQ. Other
necrosis, paraneoplastic syndromes and SVC causes include Choriocarcinoma, Endometrial carcinoma,
obstructionQ Germ cell tumors, Synovial sarcoma and Prostate carcinoma.
Contd..

Multiple discrete CXR – Left apical


nodules scattered in soft tissue density
both lung fields (White mass lesion
arrows) in a Post- with gross rib
Mastectomy patient destruction –
(Right breast shadow Pancoast tumor
is not seen – black
arrow) – suggests Lung
metastasis in a case of
Carcinoma Breast

Lung Tumors

Typical bilateral
Cannon-ball lung
metastasis HRCT - Known case of Ca breast, showing smooth as well as nodular
peribronchovascular interstitial, fissural and interlobular septal 103
thickening on right side
Bronchial Carcinoid Tumors SOLITARY PULMONARY NODULE
Q Q
ƒƒ Neuroendocrine tumors derived from bronchial APUD ƒƒ Defined as a solitary circumscribed pulmonary opacity
(Amino Precursor Uptake Decarboxylation) cells. with no associated pulmonary, pleural, or mediastinal
CONCEPTUAL REVIEW OF RADIOLOGY

ƒƒ Typical carcinoids: abnormality, measuring less than 3 cm in diameterQ (>3 cm


•• Account for 90% carcinoid tumors is called a mass)
•• Benign behavior and show slow growthQ. ƒƒ Important causes of SPN:
•• Classically cold on PET scanQ like bronchoalveolar
carcinomaQ. Infective/Inflammatory lesions
ƒƒ Atypical carcinoids: • Granuloma • Hydatid cyst
•• Features in between benign lesions and small cell lung • Lung abscess • Round pneumonia
carcinoma and commonly metastasize • Fungal ball • Rheumatoid nodule
ƒƒ Most arise in main/lobar/segmental bronchi – hence cause Neoplastic lesions: Benign
features of bronchial obstruction and lung collapse • Hamartoma • Chondroma
ƒƒ Incomplete bronchial occlusion may result in recurrent
segmental pneumonia, abscess formation, bronchiectasis, Neoplastic lesions: Malignant
air trapping • Bronchial carcinoma • Carcinoids: Central bronchial
ƒƒ Iceberg lesionsQ: Bronchial carcinoids with a large bulk of • Solitary pulmonary or Peripheral
mass outside the bronchus than within it. metastasis • Lymphoma
ƒƒ Produce multiple hormones – serotonin, kallikrein, hista- Congenital
mine, insulin ACTH – may lead to Carcinoid syndrome.
• Lung cysts/Bronchogenic cyst • Bronchial atresia with mucus
• AV malformations impaction
  Clinical Pearls • Pulmonary sequestration
Carney’s Triad
Miscellaneous
This is a specific triad seen in cases of Pulmonary hamartomas/
chondromas. It consists of: • Intrapulmonary lymph node • Amyloidosis
• Multiple pulmonary chondromas • Pulmonary infarct • Progressive massive fibrosis
• Gastric epithelioid leiomyosarcoma/GISTs • Mucoid impaction/ in coal workers
• Functioning extra-adrenal paragangliomas Bronchocele
Pulmonary hamartomas/chondromas are usually peripheral
ƒƒ Usually detected incidentally on CXR/CT. CXR is initial
lesions and present as a well-defined round/oval solitary
investigationQ. CE-CT is overall the Best investigation todayQ
pulmonary nodule with internal fat and “Popcorn calcification”Q
ƒƒ PET/PET-CT is an emerging modality – may soon be the
preferred modality.

ƒƒ Role of imaging is to try and differentiate benign from possible malignant lesions.

CXR PA View –
Left mid zone
SPN
Lung Tumors

104
  Clinical Pearls
Differentiating between Benign and Malignant SPNs

RESPIRATORY SYSTEM
Feature Benign lesions Malignant lesions
Patient age Q
<35 years >35 years
Lesion size – not very <4 mm are almost never malignant >20 mm have 75% chance of being
reliable malignant
Shape • Polygonal lesionsQ Three-dimensional ratio <1.78Q
• Three-dimensional ratio of >1.78Q – suggests benignity
(3D ratio = ratio of maximum transverse dimension to the
maximum vertical dimension)
Rate of growth of Doubling timeQ of • Doubling time of 1 – 18 months
lesion – PRIMARY • <1 month or (Median 3 months)
CRITERIAQ • >18 months • Exception is Bronchioloalveolar
• Summarily a lesion that is unchanged over 2 years is carcinoma – are slow growing with
considered benign much longer doubling times
Attenuation and Calcification: Calcification:
Enhancement – • Dense central nidus/Laminated/Diffuse calcificationQ – • Amorphous/Granular patternQ
PRIMARY CRITERIAQ suggest granulomatoid cause like TB, Histoplasmosis AttenuationQ: Ground glass attenuationQ
• Popcorn calcification - Hamartoma (with or without solid component)
Fat – suggests Hamartoma increases possibility of malignancy
Enhancement: Absent/minimal enhancement (<15 HU)
Attenuation: Purely solid lesions are usually benign
Margins Well defined margins with Ill-defined margins with
• Smooth • Corona Radiata appearanceQ
• Pencil sharp margins • Irregular
(Metastasis – is exception – can have such margins) • Spiculated
• Lobulated
• Umbilicated
• Notching
Air Bronchogram sign Less commonly seen in benign nodules May suggest malignancies like bronchiole-
alveolar carcinoma, Lymphoma

6
CLINICAL QUIZ - SOLUTION
This is a History clincher:
• An elderly male, chronic smoker – predisposition for malignancy
• Cough with hemoptysis and mass lesion on CXR – suggests malignancy
• Ipsilateral upper limb pain, paresthesia and weakness and Ptosis – suggests neural extension into brachial plexus nerve roots and
cervical sympathetic ganglia
Thus, this history suggests a neoplastic mass lesion in left lung apex with neural extension – Pancoast tumor – hence answer is B.
The 2nd quiz question is just like what we discussed in the previous topic of Mediastinal tumors. IOC for all mediastinal tumors is CE-CT,
except posterior mediastinal lesions (IOC is MRI) – to look for neurological extension.
Similarly, IOC for all lung tumors is CE-CT, except for Pancoast tumor – it is MRI. And the reason is the same – to look for neurological
extension. Pancoast tumor is known to infiltrate into the Brachial plexus nerve roots/Cervical sympathetic ganglia – and hence MRI is
Lung Tumors

IOC (Answer is D.)

105
HRCT Thorax
CONCEPTUAL REVIEW OF RADIOLOGY

7
CLINICAL QUIZ
12. A 35-year-old patient presents with persistent cough. CXR reveals a
peculiar abnormality. A Gallium scan was done and revealed “Lambda
sign”. What is the most likely diagnosis?
A. TB
B. Chronic bronchitis
C. Sarcoidosis
D. Pleural effusion

HRCT THORAX: BASIC CONCEPT ƒƒ It is supplied by terminal bronchiole and centrilobular


artery.
ƒƒ It stands for High Resolution Computed Tomography. ƒƒ Branches of pulmonary veins and lymphatics run in the
ƒƒ It is a technique in which, axial (cross sectional) images of adjacent connective tissue septae.
lung are obtained using very thin slices.
ƒƒ Thorax and Temporal bones are the two body structures
which are imaged using HRCT.Q

Physics behind HRCT


ƒƒ A bone algorithm that uses high spatial resolution to
increase the contrast between 2 widely differing densities is
used, like air and vessels in thorax.
ƒƒ Typically slice thickness of 0.625–1.25 mm are used.Q
ƒƒ Good patient breath-hold is necessary for it.
ƒƒ Images are viewed on the lung window setting (window
width of 1000 – 1500 HU and level of -600 – 700).

Concept of Secondary Lobule


ƒƒ It is the fundamental unit of HRCT imaging.Q
ƒƒ It is the smallest unit of lung that is surrounded by connective
tissue.
ƒƒ It consists of 5-15 pulmonary acini and measures 1-2 cm. Diagrammatic representation of secondary Lobule

CXR PA view – HRCT – Bilateral


Bilateral coarse Ground glass
Reticulonodular opacities (vascular
opacities – in a known structures seen
HRCT Thorax

case of Interstitial within it)


Lung Disease (ILD)

106
Immunocompromised
HRCT Thorax patient with
CO symmetric ground
NC E P T

RESPIRATORY SYSTEM
glass opacities (White
HRCT Thorax is an extensive topic and one can write an entire book arrows) and confluent
dedicated to it. Till now we have studied what is HRCT, the lung consolidations in both
anatomy behind it, the basic physics related to image creation. The lower lobes (Black
thing to note is that any disease will have combination of more arrows) – suggests
than one of these patterns. Now we will study commonly occurring Pneumocystis
and clinically significant diseases with their imaging findings with pneumonia
respect to NEET examination. This section will contain high yield
facts, important signs and other information that will help you not
only answer factual MCQ but also common image-based questions.

ƒƒ Pulmonary alveolar proteinosis:


•• On HRCT bizarre shape (generally bilobed or clover
•• It is a rare diffuse lung disease of unknown etiology
shaped) multiple cystsQ are seen.
characterized by alveolar and interstitial accumulation
•• Spontaneous pneumothoraxQ is common.
of a periodic acid-Schiff (PAS) stain-positive
ƒƒ Lymphangioleiomyomatosis:
phospholipoprotein derived from surfactant.
•• It is a rare disease characterized by progressive
•• On imaging it gives crazy paving appearanceQ which
proliferation of spindle cells, resembling smooth muscle.
consists of both septal thickening and ground glass
•• It occurs only in women, usually of child-bearing age,
opacity in a patchy distribution.
between 17 and 50 years.
ƒƒ Usual interstitial pneumonia (UIP): Refer to Image 11 in
•• On imaging uniform-shaped cysts are seen scattered in
“Imaging Signs in Radiology”
lungsQ.
•• It is most common ILD and honey-combing is its
ƒƒ Sarcoidosis: Discussed below
hallmarkQ.
ƒƒ Silicosis and Coal worker’s pneumoconiosis:
•• Honey combing represents stacking of thin cysts
•• In early stage there are nodules with upper lobe
one over other. The distribution of UIP is basal and
distributionQ and in late stages there is sudden significant
subpleuralQ and is again a peculiar finding.
fibrosis which is known as progressive massive fibrosisQ.
ƒƒ Langerhans cell histiocytosis (LCH):
•• Eggshell calcificationQ is seen in hilar regions.
•• It affects young adults with history of smokingQ
•• Patients affected with silicosis are more liable for
tuberculosis and concomitant infection is labeled as
SilicotuberculosisQ.
Nonspecific
Interstitial
•• Caplan syndrome/Rheumatoid pneumoconiosisQ:
Pneumonia (NSIP) It is a combination of Seropositive rheumatoid arthritis
– HRCT showing and a characteristic pattern of lung fibrosis. Commonly
bilateral posterior associated with coal workers pneumoconiosis. Imaging
sub-pleural ground findings include lung nodules – that may calcify/cavitate,
glass opacities – no background changes of pneumoconiosis, pleural
honeycombing effusions.
ƒƒ Asbestosis:
•• Fibrosis similar to UIP is seen with additional calcified
pleural plaques.
•• The fibrosis shows basal predominanceQ. (Remember
Enlarged bilateral this as As-base-tosis)
hilar and right •• Focal in folding of pleura giving rise to round
paratracheal atelectasisQ or comet tail signQ can be seen.
lymph nodes – •• It is a premalignant condition and can cause pleural
Sarcoidosis mesotheliomaQ.
HRCT Thorax

Honeycombing/Honey-comb Lung is seen in


ƒƒ Usual interstitial pneumonia (UIP)
ƒƒ Rheumatoid arthritis
ƒƒ Scleroderma
Lymphangioleiomyo- ƒƒ TB
matosis: ƒƒ Pneumoconiosis
uniform-shaped cysts ƒƒ Histiocytosis X
are seen scattered in ƒƒ Sarcoidosis
lungs 107
7
CLINICAL QUIZ - SOLUTION
• The CXR reveals bilateral hilar lymphadenopathy. A middle-aged female patient with bilateral hilar lymphadenopathy and few clinical
CONCEPTUAL REVIEW OF RADIOLOGY

complaints like cough, dyspnea – most likely suggests Sarcoidosis. (Ans C.)
• Sarcoidosis is a systemic disorder of unknown origin and characterized by non-caseating granulomas in multiple organs, that may
resolve spontaneously or progress to fibrosis.
• On CT scan Lambda signQ or 1-2-3 signQ (bilateral hilar and right paratracheal lymphadenopathy) is seen.
• Panda signQ: Seen on a gallium-67 citrate scan. It is due to bilateral involvement of parotid and lacrimal glands in sarcoidosis,
superimposed on the normal uptake in the nasopharyngeal mucosa.
• In lung perilymphatic and subpleural nodules are seen.
• In later stages coalescent nodules surrounded by multiple satellite nodules - Galaxy signQ can be seen.
• Most common radiologic finding at presentation:
ƒƒ In Indian patients: Bilateral adenopathy with parenchymal infiltrates
ƒƒ In Western patients: Bilateral adenopathy

Miscellaneous Topics
8
CLINICAL QUIZ
13. A 35-year-old male with history of 4 weeks of immobilization for
fracture femur developed sudden onset of chest pain and hemoptysis.
ECG shows S1 Q3 T3 pattern. Diagnosis is: (NEET 2018 Pattern)
A. Acute myocardial infarct
B. COPD
C. Pulmonary embolism
D. Cor pulmonale
14. The Gold standard investigation for Pulmonary embolism is:
A. CXR 
B. CT Pulmonary angiography
C. V/Q scan 
D. Cather pulmonar angiography

DIAPHRAGM Diaphragmatic Elevation on an Erect CXR


Anatomically consists of central tendon, and right and left Causes of Unilateral Elevated Causes of Bilateral Elevated
hemidiaphragms HemidiaphragmQ HemidiaphragmQ
Miscellaneous Topics

ƒƒ On CXR PA View: • Pulmonary collapse/ • Supine position


•• Each hemidiaphragm is normally seen on the PA hypoplasia • Poor inspiration
radiograph by a smooth, curved line which is convex • Phrenic nerve palsy • Obesity
upward. • Eventration of ipsilateral • Pregnancy
•• Lateral CP angle: The lateral attachment of the dome of diaphragm • Abdominal distension –
diaphragm to the ribs is represented by the lateral • Subphrenic mass/abscess Ascites, obstruction, mass
costo­phrenic recess, a sharply-defined acute angle. • Overdistended stomach/ • Bilateral diaphragmatic palsy
Most common cause of a blunted CP angle is pleural colon • Painful conditions – post
effusionQ. • Dorsal scoliosis surgery
•• Cardiophrenic angle: Medially, the diaphragm meets • Rib fracture or other • Diffuse pulmonary fibrosis
the heart at the cardiophrenic angle. This is higher painful conditions • Lymphangitis carcinomatosa
than the costophrenic angle and is ill-defined owing • Lateral decubitus film – • Bilateral basal pulmonary
to the presence of fat. Common lesions seen at the dependent side dome will emboli
cardiophrenic angles have been discussed previously be elevated
108 in the Mediastinal abnormalities chapter.
Diaphragmatic Eventration
ƒƒ Thinning of a hemidiaphragmQ on account of normal CXR PA view – Broad
diaphragmatic muscle being replaced by a thin layer of based bulge/hump
seen along the right

RESPIRATORY SYSTEM
connective tissue and a few scattered muscle fibers.
hemidiaphragm –
ƒƒ Total eventrationQ—more common on left-sideQ. Presents
represents a focal
as elevation of entire hemidiaphragm
eventration of
ƒƒ Focal eventrationQ—more common on right sideQ (antero-
diaphragm
medial aspect). Presents as a focal bulge/hump in the
hemidiaphragm

Dromedary DiaphragmQ
ƒƒ More severe form of diaphragmatic hump presenting as a
double contour on a CXR PA view.

Diaphragmatic Hernias
Hiatus Hernias
Discussed in GI tract radiology

Morgagni Hernia
ƒƒ Foramen of Morgagni is a persistent developmental CXR PA view –
defect in the diaphragm anteriorly, between the septum Significantly elevated
transversum and the right and left costal origins of the left hemidiaphragm
diaphragmQ. in an adult patent
ƒƒ Seen in adults mostly at the right cardiophrenic angleQ – due – Diaphragmatic
to protective effect of pericardium on the left. eventration
ƒƒ Present as an anterior mediastinal massQ
ƒƒ Morgagni hernias containing gut can be diagnosed using
barium but the diagnosis is more simply established by
means of CT or MRI. Abdomen Radiograph
– showing air-filled
Bochdalek Hernia lucent structure
superimposed on the
ƒƒ Foramen of Bochdalek is a persistent developmental heart shadow – Highly
defect in the diaphragm posteriorly, due to failure of pleura- suggestive of a Sliding
peritoneal canal membrane to fuse with dorsal esophageal Hiatus hernia
mesentery medially and body wall laterallyQ.
ƒƒ Seen on left sideQ – due to protective effect of liver on right
side.
ƒƒ A well-defined, dome-shaped, soft tissue opacity is seen
midway between the spine and lateral chest wall on the
frontal view and above the posterior costophrenic recess on
the lateral view.
ƒƒ This is the usual type of congenital diaphragmatic hernia.
ƒƒ A large congenital hernia may contain the stomach,
small bowel and colon and appears as multiple gas-filled Miscellaneous Topics
lucencies/ring shadows with mediastinal shift toward right
side.
ƒƒ In congenital diaphragmatic hernia the prognosis correlates
with the degree of underlying pulmonary hypoplasiaQ
ƒƒ 13 pairs of ribsQ may occur in association with a Bochdalek’s
hernia. CXR PA view –
Air-filled bowel
Diaphragmatic Trauma/Rupture segment herniated in
ƒƒ Left hemidiaphragm is commonly involved than rightQ due the left paracardiac
to protective effect of liver on the right. Most commonly area – most likely
herniated viscera is stomach and colon on left sideQ and to be a Rolling type
hiatus hernia
liver on right sideQ.
109
ƒƒ In acute phase –
•• CXR may be normal Left-sided
•• Collar signQ: Intrathoracic herniation of a hollow Congenital
viscus (stomach, colon, small bowel) with or without diaphragmatic
CONCEPTUAL REVIEW OF RADIOLOGY

focal constriction of the viscus at the site of the tear hernia (CDH). Note
ƒƒ Multidetector CT – is the Best investigation. the air-filled bowel
•• Discontinuity of the diaphragm with direct visualization loops in the left
of the diaphragmatic injury hemithorax with
•• Collar/Hourglass signQ: A waist-like constriction marked rightward
of the herniating hollow viscus from the abdominal mediastinal shift.
into the chest at the site of the diaphragmatic tear - is
classical of diaphragmatic rupture
•• Dependent viscera signQ: When a patient with a
ruptured diaphragm lies supine at CT examination, the
herniated viscera (bowel or solid organs) are no longer
supported posteriorly by the injured diaphragm and fall
to a dependent position against the posterior ribs
ƒƒ Other signs seen in Diaphragmatic rupture:
•• Thick crus signQ
•• Dangling Diaphragm signQ
•• The absent diaphragm signQ Coronal reformatted image CT
•• The hump and band signsQ Thorax – Showing herniated
•• The sinus cut-off signQ bowel and mesentery into
•• Diaphragmatic or Peridiaphragmatic Contrast the left hemithorax through
Medium ExtravasationQ a diaphragmatic defect –
Bochdalek’s hernia

FOREIGN BODY ASPIRATION ƒƒ In all suspected cases of foreign body obstruction who
Foreign body aspiration is most commonly seen in children have evidence on imaging or show no improvement or
below 4 years of ageQ deteriorate clinically should undergo bronchoscopy for
It is sometimes seen in adults and known as Cafe coronary confirmation and removal.
syndrome.Q

Imaging Modalities PULMONARY THROMBOEMBOLISM


Chest Radiograph Dicussed below.

ƒƒ It is the first investigation to be usedQ and should be


combined with an abdominal radiographQ also as many time SCIMITAR SYNDROME
the child may cough up the foreign body and immediately
ƒƒ Synonymous with hypogenetic lung syndrome
swallow it.
ƒƒ It consists of a hypoplastic lung or lung segment that
ƒƒ Most common foreign body to be aspirated is PeanutsQ
drains into the systemic circulation instead of pulmonary
ƒƒ Hence in a suspected foreign body obstruction expiratory
circulation due to an anomalous vein - is a type of partial
radiographs should be obtainedQ
anomalous pulmonary venous return.
ƒƒ Findings on chest radiograph include:
Miscellaneous Topics

ƒƒ Hyperinflated (dark) lung due to air trapping.


•• Radio opaque foreign body maybe directly visualized. CXR AP view -
In such cases a PA and lateral radiograph will be helpful Bilateral Bat-wing/
perihilar opacities – a
for exact localization.
case of Pulmonary
•• The orientation of the structures in body also helps when
edema
single view is available. Like for a coin lodged in neck, if
it is esophagus then it will be seen enface on frontal (AP)
view that is broad in traverse dimension while if it is in
trachea then it will appear en-profile that is it will slit like
and better appreciated on lateral view.

Bronchoscopy
ƒƒ Bronchoscopy is considered the gold standardQ in the
diagnosis and management of tracheobronchial tree foreign
110 bodies
ƒƒ It always affects the right side. The right lung is hypoplasticQ.
ƒƒ The anomalous vein drains into either inferior vena cava
CT Pulmonary
(most common), right atrium or portal vein. Angiography – Large
ƒƒ On CXR PA view the anomalous draining veins can be seen hypodense thrombus

RESPIRATORY SYSTEM
directed inferiorly giving appearance of scimitar signQ seen in the right main
(Turkish sword). pulmonary artery (White
ƒƒ CE – CT is the modality of choiceQ arrow)

Cervical Rib or a
CO Hypoplastic 1st RIB
NC E P T
• Though most cervical ribs are easily detected, sometime they
pose a problem.
• This may happen when the 1st rib is hypoplastic and appears
thin and linear like a cervical rib and may also be asymmetric.
This is when we look at few other findings. CXR – PA view – the
• Whenever in confusion just examine the vertebra with which above patient shows a
the rib in question is articulating, bearing in mind the following triangular wedge-shaped
points: area in mid right lung
ƒƒ Transverse process of Thoracic vertebra: Directed upward with its apex directed
and laterally toward the hilum –
ƒƒ Transverse process of Cervical vertebra: Directed down- Hampton hump
wards and laterally
• So, by identifying the vertebra you can easily call a rib as cervical
or the 1st thoracic rib definitively.

8
CLINICAL QUIZ - SOLUTION
This is also a history clincher. A history of immobilization • Right ventricular hypokinesia (McConnell signQ), right ventricular
(predisposes to venous stasis and DVT), acute presentation with dilatation and pulmonary arterial hypertension can be identified.
breathlessness and hemoptysis and typical S1Q3T3 pattern on ECG Nuclear/Radionuclide Scans: Second Line Test for PTE
– suggests pulmonary embolism. Thus answer for 1st Quiz is C. • Also known as Ventilation – Perfusion (V/Q) scan.
Initial screening test – D-dimer assayQ – has high negative predictive • Mismatched defectsQ on V/Q scan – suggestive of Pulmonary
value embolism
Role of Imaging Modalities ƒƒ Present ventilation – Absent perfusion – in the same segment
Chest Radiograph of lung
It is not sensitive or specific rather it is used to rule out other clinical ƒƒ In early cases segmental/lobar defects are seen, in later cases
mimics like pneumothorax or pneumonia. But there are some signs more peripheral defects become apparent.
which act as indirect evidence for PTE. ƒƒ Usually the defects are bilateral
• Fleischner signQ: Enlarged main pulmonary artery - can be caused For a more detailed discussion of V/Q scan – kindly jump over to
either by pulmonary hypertension that develops or by distension of radionuclide imaging in this book!
the vessel by a large pulmonary embolus. Invasive Pulmonary Angiography - Gold Standard/Most Accurate
• Hampton humpQ: Peripheral wedge-shaped opacity with apex Method (Answer for 2nd Quiz is D.)
pointed towards hilum. Denotes underlying lung infarction. • Invasive – hence CT Angiography is preferred for diagnosis
• Westermark signQ: Regional oligemia (increase in darkness of film). • Reserved for patients with technically unsatisfactory CT.
• Palla’s signQ: Enlarged right descending pulmonary artery. • Can be therapeutic as well. Miscellaneous Topics
• Chang sign/knuckle signQ: Dilated right descending artery Remember this as an easy rule, whenever the examiner asks you
showing abrupt cutoff. about the Gold Standard Investigation for any vascular abnormality,
• Felson’s signQ: Pleural effusion Left > Right the answer is Invasive angiography. Few examples include:
• Melting ice cube sign: Appearance of a resolving pulmonary infarct
on a CXR – looks like an ice-cube melting peripherally to centrally. Condition Gold Standard Investigation
CT Pulmonary Angiography: Investigation of ChoiceQ Berry aneurysm Cerebral digital subtraction
• Acute emboli: Show Polo mint signQ: Central non-enhancing filling angiography (DSA)
defect due to thrombus with the contrast flowing peripherally. Brain AV malformation Cerebral digital subtraction
• Chronic emboli: The thrombus forms obtuse angle (c/f acute angle angiography (DSA)
in acute emboli) with vessel walls and may show calcification in it. Renal artery stenosis Invasive catheter angiography
Sometimes additional bands, webs or collateral vessel formation Varicose veins/DVT Invasive catheter venography
maybe seen. Aortic dissection Invasive catheter venography
Ultrasound/Echocardiography
• Helps to identify deep vein thrombosis which is major contributor
111
to PTE.
Multiple Choice Questions
CONCEPTUAL REVIEW OF RADIOLOGY

Normal Findings on a Chest Radiograph 11. Silhouetting of left border of heart on a CXR PA view
1. Normal hilar shadows on a CXR are formed by all except: indicates involvement of: (CET Nov 2014)
(CET Nov 2014) A. Left lower lobe B. Lingular segment
A. Pulmonary arteries B. Upper lobe veins C. Right upper lobe D. Left hilum
C. Main bronchi D. Lower lobe veins 12. Consolidation of which portion of lung is likely to obliterate
2. On a CXR PA view the right heart border is formed by all the Aortic knuckle on a CXR PA view: (AI 2011)
except: (NEET Dec 2012) A. Lingular segment
A. SVC B. Right atrium B. Right upper lobe
C. IVC D. Left atrium C. Apex of left lower lobe
D. Left upper lobe (apicoposterior segment)
3. Which of the following about chest X-ray is true? (PGI 1997)
A. Right hilum is lower than left hilum 13. A triangular opacity with clear borders, base toward the
B. Right dome of diaphragm is lower than left midline and obliterating the right heart border on a CXR PA
C. Pulmonary arteries normally extend up to periphery of lung view suggests involvement of: (MH CET 2011)
fields A. Apical segment of right lower lobe
D. Heart appears abnormally small in an A-P film than a P-A film B. Medial segment of right lower lobe
C. Right middle lobe
4. Which of the following is not a part of the left mediastinal
D. Anterior segment of right upper lobe
border on a P-A chest radiograph? (CET July 2016)
A. Aortic arch B. Pulmonary trunk 14. Air Bronchogram sign is seen in: (MH CET 2003)
C. Left ventricle D. SVC A. Consolidation B. Alveolar cell carcinoma
C. ARDS D. All of the above
5. The normal carinal angle at the tracheal bifurcation is
around: 15. A positive Hilum Overlay suggests that a lesion is located in
A. 40–50 degrees B. 50–60 degrees the:
C. 60–75 degrees D. 75–90 degrees A. Anterior mediastinum B. Heart
C. Diaphragm D. Posterior mediastinum
6. Which of the following is most commonly seen accessory
fissure seen in lungs?
A. Left horizontal fissure B. Azygos fissure
Lung Infections
C. Superior accessory fissure D. Inferior accessory fissure 16. Bulging fissures are seen in: (PG 1999)
7. Kerley lines are seen in: (AIIMS 1997) A. Klebsiella pneumonia
A. Pulmonary edema B. Congenital heart disease B. Staphylococcal pneumonia
C. Sarcoidosis D. All of the above C. Pulmonary edema
8. False regarding the diaphragm is: D. Pneumococcal pneumonia
A. In most people, the left hemidiaphragm is 1.5–2.5 cm 17. Investigation of choice of Lung abscess is:
higher than the right. (Recent Pattern Dec 2016)
B. The angle of contact with the chest wall is acute and A. CXR B. CE-CT
sharp, but blunting of this angle can be normal in athletes, C. MRI D. Ultrasound
because they can depress their diaphragm to a remarkable 18. Most common causative agent of Atypical pneumonia is:
Multiple Choice Questions

degree on deep inspiration. A. Mycoplasma B. Influenza A


C. The normal excursion of the diaphragm is usually between C. RSV D. Pneumocystis
1.5 and 2.5 cm. 19. Which of the following is not suggestive of a lung abscess?
D. An eventration is composed of a thin membranous sheet A. Round lesion in all projections
replacing what should be muscle. B. Air-fluid level
9. On a chest radiograph the upper limb of a normal hilar C. Obtuse angle with the chest wall
shadow is formed by: D. Thick enhancing walls
A. SVC 20. Mendelson syndrome is:
B. Azygos vein A. Community acquired pneumonia
C. Superior pulmonary artery B. Atypical pneumonia
D. Superior pulmonary vein C. Aspiration pneumonia
D. Lipoid pneumonia
Silhouette Sign and Lung Infections 21. An AIDS patient presents with history of dyspnea and non-
10. Air Bronchogram sign on a CXR denotes: (AI Dec 2013) productive cough. X-ray shows bilateral perihilar opacities
A. Intrapulmonary lesion without obvious pleural effusion or lymphadenopathy. Most
B. Extrapulmonary lesion probably etiology is:
C. Intra-thoracic lesion A. TB B. CMV
D. Extra-thoracic lesion C. Kaposi’s sarcoma D. Pneumocystis carinii
112
22. All of the following show perihilar ground-glass haziness on 35. False about pneumothorax:
chest X-ray, except: A. Most common type is spontaneous pneumothorax
A. ARDS B. Traumatic pneumothorax results only from penetrating

RESPIRATORY SYSTEM
B. Pulmonary edema chest trauma
C. Pneumocystis carinii infection C. May be life threatening
D. Staphylococcal pneumonia D. ICD insertion is treatment
Miscellaneous Lung Infections 36. Spinnaker sail sign is seen in:
A. Pneumothorax B. Pneumomediastinum
23. Most common type of Aspergillosis lung involvement is: C. Pneumoperitoneum D. Pneumopericardium
A. Aspergilloma B. ABPA
C. Semi-invasive aspergillosis D. Invasive aspergillosis Lobar Collapse
24. Monod sign is seen in: 37. Least mediastinal shift is seen in which of the following lobar
A. Aspergilloma B. ABPA collapse:
C. Semi-invasive aspergillosis D. Invasive aspergillosis A. RUL collapse B. RML collapse
25. Finger in glove appearance is seen in: C. RLL collapse D. LUL collapse
A. Aspergilloma 38. Investigation of choice for identifying lung atelectasis is:
B. ABPA A. CT B. MRI
C. Semi-invasive aspergillosis
C. CXR D. USG thorax
D. Invasive aspergillosis
39. Juxtaphrenic peak sign is seen in:
Pleural Effusion A. RUL collapse B. RML collapse
26. Best investigation for detection of minimal pleural effusion C. RLL collapse D. LLL collapse
is: 40. Golden S sign is seen in: (AIIMS May 2015)
A. Lateral decubitus X-ray B. Erect CXR PA view A. RUL collapse B. RML collapse
C. Erect CXR Lateral view D. USG C. RLL collapse D. LUL collapse
27. Earliest finding of pleural effusion on an Erect CXR PA view is: 41. Luftsichel sign is seen in:
A. Meniscus sign B. Blunting of CP angle A. RUL collapse B. RML collapse
C. Opaque hemithorax D. Mediastinal shift C. RLL collapse D. LUL collapse
28. Best Radiograph for detection of minimal pleural effusion is: 42. False statement about Round atelectasis is:
(Recent Pattern Dec 2016) A. Central in location
A. Lateral decubitus X-ray B. Erect CXR PA view B. Related to asbestosis exposure and pleural thickening
C. Erect CXR Lateral view D. Supine CXR AP view C. Comet tail sign is seen
29. Phantom lung tumor is: D. Crow feet sign is seen
A. Lung adenocarcinoma B. Carcinoid tumor
C. Lung metastasis D. Pleural effusion
Airway Disorders
30. Fluid-Color Sign on USG Thorax is suggestive of: 43. Best method for diagnosis of Bronchiectasis is:
A. Pleural thickening B. Pleural effusion  (AIIMS Nov 2014)
C. Pleural vascularity D. Pleural mass lesion A. CXR PA view B. HRCT
31. All are true regarding loculated pleural effusion except: C. MRI chest D. Bronchography
A. Makes an obtuse angle with the chest wall 44. Which of the following signs are seen in Bronchiectasis?
B. Margins are diffuse when viewed end on A. Tram-track sign B. Signet ring appearance
C. Not confined to any bronchopulmonary segment C. Cluster of grapes sign D. All of above

Multiple Choice Questions


D. Air bronchograms are seen within the opacity 45. Vanishing lung syndrome is characterized by:
Pneumothorax A. RML consolidation B. Total lung collapse
C. Giant lung bullae D. Tracheal stenosis
32. Features of Pneumothorax are all except: (MP 2005)
46. Best investigation for Emphysema detection is:
A. Absent vascular markings
A. CXR PA view B. HRCT
B. Hypertranslucency
C. MRI chest D. V-Q scan
C. Shift of mediastinum to same side
D. Collapse of ipsilateral lung 47. Steeple sign is a feature of:
33. 1st step in the management of Tension pneumothorax is: A. Bronchiectasis
(AIIMS May 2016) B. Tracheobronchomegaly
A. ICD insertion B. Needle thoracotomy C. Acute laryngotracheobronchitis
C. IV fluids D. USG Thorax D. Epiglottitis
34. Best for detection of pneumothorax: 48. Thumb sign/Omega sign is seen in:
A. Lateral decubitus X-ray A. Emphysema
B. Erect Inspiratory CXR PA view B. Tracheomalacia
C. Erect Expiratory CXR Lateral view C. Acute laryngotracheobronchitis
D. CT scan D. Acute Epiglottitis
113
49. A child with acute onset breathlessness with features of 62. Best investigation today for an SPN evaluation is: (AI Dec 2014)
hyperinflation on a CXR suggests: A. CXR B. CE-CT
A. Staphylococcal bronchopneumonia C. MRI D. USG thorax
CONCEPTUAL REVIEW OF RADIOLOGY

B. Aspiration pneumonia 63. The best investigation for evaluation of a solitary pulmonary
C. Bronchial asthma nodule is:
D. Foreign body aspiration A. CXR PA view B. CT
50. Dirty chest appearance on CXR is seen in: C. MRI D. Bronchoscopy
A. Chronic bronchitis B. TB
C. Bronchiectasis D. Cystic fibrosis
HRCT Thorax
64. HRCT is investigation of choice in:
Mediastinum A. Interstitial lung disease
51. Egg shell calcification is not seen in:  (AIIMS 1998) B. Lobar pneumonia
A. Sarcoidosis C. Lung carcinoma
B. Silicosis D. Chest trauma
C. Post irradiation lymphoma 65. The primary unit of study in HRCT Thorax is:
D. Bronchogenic Carcinoma A. Primary lobule
52. Which of the following is not a differential diagnosis of an B. Respiratory bronchiole
anterior mediastinal mass?  (AIIMS 2002) C. Secondary lobule
A. Teratoma B. Neurogenic tumor D. Lung lobe
C. Thymoma D. Lymphoma 66. Crazy paving pattern is seen in:
53. Commonest mass in middle mediastinum: (AIIMS Sep 1996) A. Alveolar proteinosis
A. Lipoma B. Aneurysm B. Idiopathic pulmonary fibrosis.
C. Lymph nodal mass D. Congenital cysts C. Pulmonary edema
54. All are true regarding thymus swelling on a CXR except: D. Silicosis
A. Widening of mediastinum on CXR 67. Which is not a component of Caplan syndrome?
B. Sharp border with Sail like appearance A. RA factor positive
C. Steroid administration will result in shrinkage B. Lung nodules
D. Tracheal shift to left side C. Pleural effusion
55. The following is not an anterior mediastinal mass: D. Cardiomegaly
A. Teratoma B. Thymoma 68. The routine slice thickness in HRCT is:
C. Lymphoma D. Neurogenic tumor A. 4-5 mm B. 0.6 to 1.2 mm
Lung Cancer C. 0.1 to 0.2 mm D. 7 to 8 mm
69. Which is not a feature of sarcoidosis? (AI Dec 2013)
56. Best imaging modality for detection of bronchogenic
A. Perilymphatic lung nodules
carcinoma is:  (NEET Pattern 2012)
B. Hilar lymphadenopathy
A. CT B. MRI
C. Paratracheal lymphadenopathy
C. Bronchoscopy D. USG thorax
D. Pleural plaques
57. Investigation of choice for Pancoast tumor/Superior sulcus
70. Bilateral pleural plaques and pulmonary fibrosis involving
tumor:
base of lungs is seen in:
A. CT B. MRI
A. Silicosis
C. Bronchoscopy D. USG thorax
B. Asbestosis
58. Air bronchograms are typically seen in which of the following
Multiple Choice Questions

C. Coal workers pneumoconiosis


lung malignancies:
A. Adenocarcinoma D. Sarcoidosis
B. Squamous cell carcinoma 71. All of the following statements about silicosis are true, except:
C. Bronchioloalveolar carcinoma A. Pleural plaques (Recent Pattern June 2018)
D. Large cell carcinomas B. Predilection for upper lobes
59. Iceberg lesions are a feature of: C. Calcified hilar lymphadenopathy
A. Lung carcinoma B. Bronchial carcinoids D. Associated with tuberculosis
C. Lymphoma D. Lung metastasis 72. Miliary calcified nodules distributed diffusely in pulmonary
60. Popcorn calcifications are seen in: parenchyma bilaterally may be seen in:
A. Lung carcinoma B. Bronchial carcinoids (NEET Pattern Dec 2012)
C. Pulmonary hamartoma D. Lung metastasis A. Varicella pneumonia B. Influenza
C. Herpes D. RSV
Solitary Pulmonary Nodule 73. Black bronchus sign is seen in:
61. SPN is defined as a solitary nodule in the lung that measures A. Pneumocystis pneumonia
less than: B. Staphylococcal pneumonia
A. 3 mm B. 3 cm C. Emphysema
C. 1 cm D. 30 cm D. Pneumothorax
114
Miscellaneous Topics 87. What is the role of ultrasound examination in PTE?
74. The differential diagnosis of a cardiophrenic angle mass A. To look for right ventricular hypokinesia
lesion includes: B. To look for deep vein thrombosis

RESPIRATORY SYSTEM
A. Fat pad B. Pleuropericardial cyst C. Both A and B
C. Morgagni hernia D. All of the above D. None of the above
75. CT signs of diaphragmatic rupture include: 88. What happens to the ventilation perfusion scan in PTE?
A. Collar sign B. Dependent viscera sign A. V/Q = 1 B. V/Q > 1
C. Thick crus sign D. All of above C. V/Q < 1 D. V/Q = 0
76. Which radiograph is least useful in assessment of foreign 89. Which of the following is associated with scimitar syndrome?
body? A. Vertebral anomalies
A. Chest PA B. Chest expiratory B. Horseshoe lung
C. Chest lordotic D. Abdomen supine C. Congenital heart disease
77. False about aspirated foreign body is: D. All of the above
A. Right side more commonly involved 90. What is first imaging modality used in suspected cervical rib?
B. Inorganic foreign body is less dangerous than organic A. Chest radiograph
C. In infants Heimlich maneuver should be tried to relieve it B. CT scan
D. Chest and abdomen radiograph should be obtained C. Chest Ultrasound
78. All of the following investigations are used in suspected D. Radiograph of dorsal spine
foreign body aspiration except: 91. What is not true about cervical rib?
A. Chest radiograph B. Ultrasound A. Usually asymptomatic
C. CT scan D. Virtual Bronchoscopy B. Can be bilateral 
79. Which is not associated with ARDS: C. Always identified on chest radiograph
A. Lungs injury D. Can cause compression of brachial plexus
B. Opacities on chest radiograph 92. Not true regarding Pectus excavatum:
C. Fall in oxygen saturation A. Heart shifted toward left side
D. Non progressive course B. Features may suggest RML consolidation
80. Which imaging modality is least useful in a case of ARDS? C. Anterior ribs are more vertically oriented – 7-shaped ribs
A. Radiograph B. Ultrasound D. Heart shifted toward right side.
C. CT scan D. MRI 93. Gold standard for diagnosis of pulmonary thromboembolism
81. How is diagnosis of ARDS made? is:
A. Clinical B. Sputum examination A. CXR B. CT Angiography
C. Chest ultrasound D. Biopsy
C. V/Q scan D. Pulmonary angiography
82. Which is true regarding flail chest?
94. Patient with history of tachyarrhythmias is on an implantable
A. Discontinuous ribs are involved
cardioverter defibrillator. He develops shock. Best method to
B. Only one end of rib is involved
know the position and integrity of ICD is to do:
C. Always associated with stab injury
(AIIMS May 2015)
D. Paradoxical chest wall movement is seen
A. Contrast CT B. MRI
83. What is best imaging modality used in flail chest?
C. USG D. Plain radiograph
A. Chest radiograph
B. CT scan 95. On abdominal imaging, air in rectum is/are seen in: 
C. Chest Ultrasound  (PGI May 2019)
D. Radiograph of dorsal spine A. Large bowel obstruction B. Small bowel obstruction

Multiple Choice Questions


84. Which of the following is modality of choice for pulmonary C. Hirschsprung disease D. Normal individual
thromboembolism? E. Paralytic ileus
A. CT Pulmonary angiography 96. Deep sulcus sign is/are seen in:  (PGI May 2019)
B. Invasive pulmonary angiography A. Pneumomediastinum B. Pneumothorax
C. Transesophageal echocardiography C. Pleural effusion D. Pulmonary edema
D. Ventilation perfusion scan E. Peritoneal effusion
85. Not a sign of pulmonary thromboembolism: 97. All lung carcinomas for diagnosis requires CECT thorax but
A. Palla sign B. Hampton hump MRI is useful in which of the following?  (AIIMS May 2019)
C. Lambda sign D. Fleischner sign A. Small cell lung carcinoma
86. What pattern may be seen in ECG in PTE? B. Adenocarcinoma
A. S2Q3T3 B. S1Q3T3  C. Carcinoid
C. S4Q2T1 D. S2Q2T2 D. Pancoast tumor

115
Answers
1. D 14. D 27. B 40. A 53. C 66. A 79. D 92. D
CONCEPTUAL REVIEW OF RADIOLOGY

2. D 15. A 28. A 41. D 54. D 67. D 80. D 93. D


3. A 16. A 29. D 42. A 55. D 68. B 81. A 94. D
4. D 17. B 30. B 43. B 56. A 69. D 82. D 95. D,E
5. C 18. A 31. D 44. D 57. B 70. B 83. B 96. B
6. B 19. C 32. C 45. C 58. C 71. A 84. A 97. D
7. D 20. C 33. B 46. B 59. B 72. A 85. C
8. A 21. D 34. D 47. C 60. C 73. A 86. B
9. D 22. D 35. B 48. D 61. B 74. D 87. C
10. A 23. A 36. B 49. D 62. B 75. D 88. B
11. B 24. A 37. B 50. A 63. B 76. C 89. D
12. D 25. B 38. A 51. D 64. A 77. C 90. A
13. C 26. D 39. A 52. B 65. C 78. B 91. C

Explanations to Questions
5. Carinal angle is influenced by indentation from adjacent 79. ARDS results from acute direct or indirect lung injury which
structures/lesions on the tracheal bifurcation. Normally it is manifests as alveolar shadowing with clinical hypoxia and a
60 to 75 degrees. Any mass at carina like duplication cysts or progressive course.
left atrial enlargement results in widening of the angle. 82, 83. Flail chest occurs when three or more contiguous ribs
45. Giant bullous emphysema/Vanishing lung syndrome/Primary are fractured in two or more place. It is observed in a non-
bullous disease of lung: Seen in young men, characterized ventilated patient as a paradoxical movement of the flail
by the presence of large progressive asymmetric upper lobe segment. It can often be clinically occult but may lead
bullae which occupy a significant volume of a hemithorax. to severe ventilation difficulties as well. CT scan is Best
50. Dirty chest appearance-because of recurrent infection with investigation. Traumatic 1st rib fracture is always considered
scarring, the bronchovascular structures have irregular as a marker of severe chest trauma, as the rib is protected
contours. This is the only sign of bronchitis in chest X-ray. by the clavicle and scapula. Costal hook sign elephant trunk-
76. In suspected foreign body Chest PA and abdomen can be shaped ribs owing to rotation of segmental fractures.
used to localize radio opaque foreign body. Expiratory films 92. In pectus excavatum the heart is shifted toward left side. As
may show focal air trapping which is indirect sign of foreign right heart border goes behind the sternum it is obscured and
body. Lordotic view is used to view lung apices like in TB and can thus mimic right middle lobe disease.
for middle lobe pathologies and is the least useful here. 94. The patient has a cardiac device – hence MRI is contra-
77. Right-sided aspiration is more common due to the straight indicated. Patient is in shock – hence a Contrast CT is
in-line orientation of the right main bronchus. Organic foreign impractical. Though a bedside USG (Echocardiography) will
bodies are more dangerous as they may swell and secrete oil/ show the intraventricular leads, it will not be able to trace
salts causing irritation of mucosa. Heimlich maneuver should the entire leads up to the battery pouch. A simple CXR will
be tried in patients >1 year age. In infants back blows and visualize the entire device and help us assess its position and
Explanations to Questions

chest thursts are used. Both CXR and abdomen radiographs integrity – hence this is the answer.
are used to ascertain the position of foreign body.

116
CARDIOVASCULAR
IMAGING
C hapter O utline
• Imaging in Congenital Heart Disease
CONCEPTUAL REVIEW OF RADIOLOGY

• Imaging in Acquired Heart Disease


• Imaging in Pulmonary and Aortic Diseases
• Multiple Choice Questions with Explanations

Imaging in Congenital Heart Disease


1
CLINICAL QUIZ
1. A neonate presents with cyanosis at birth. A CXR is done and is shown here.
What is the most likely diagnosis? (NEET 2020 Pattern)
A. TOF C. TAPVC
B. TGA D. Ebstein’s anomaly

2. Pulmonary plethora is a feature of: (NEET 2020 Pattern)


A. TOF C. Right heart failure
B. Ebstein’s anomaly D. TAPVC

CARDIAC IMAGING MODALITIES ƒƒ Indications for cardiac CTQ:


•• Noninvasive evaluation of coronary artery anomalies
Echocardiography (2D-Echo) and other thoracic vesselsQ
Imaging in Congenital Heart Disease

•• Symptomatic patients with low/moderate probability


ƒƒ Investigation of choice for: of coronary artery disease (CA)Q
•• Antenatal detection of anomalies •• Moderate risk non-acute symptomatic patients without
•• Cardiac chamber enlargement detection known heart disease (may be able to exercise)Q
•• Functional assessment of heart—cardiac output, •• Low risk non-acute symptomatic patients without
stroke volume, pressure gradients. known heart disease (if the patient cannot exercise or
ƒƒ It can be either: undergo stress test)Q
•• TransthoracicQ—More commonly done, non- •• Evaluating the patency of a coronary artery bypass graft
invasive (CABG)Q
•• Transesophageal (TEE)Q—better for visualization •• Preoperative assessment for transcatheter aortic valve
of right atrium, aorta, posterior portions of heart implantation (TAVI/TAVR)
and in the case of prosthetic cardiac valvesQ). ƒƒ Typical acquisitions times for each rotation are around
Requires more expertise, is expensive and not 10 msQ.
easily available. ƒƒ Noncontrast CT scan can be used for estimation of coronary
ƒƒ Stress echocardiographyQ: Induced either by ExerciseQ or calcification giving the—Agatston or CT calcium scores
Pharmacologically (Dobutamine, adenosine infusion)Q ƒƒ Contrast injection is needed for assessment of coronaries
and cardiac chambers.
Cardiac CT ScanQ
ƒƒ ECG Gating is required to minimize motion blur during Cardiac MRI (CMR)
image acquisition. ƒƒ The advantages are noninvasive, nonionizing, multiplanar
ƒƒ Earlier it was only the Electron bean CT (EBCT)/5th capabilities, optional use of intravenous contrast.
generation CT that had this capability, however in recent ƒƒ Delayed contrast enhancementQ—is used to study myocar-
times Multidetector CT scan (MDCT)Q are used. dial viability.
118
CONGENITAL HEART DISEASES

Classification of Congenital Heart Disease

CARDIOVASCULAR IMAGING
They are broadly classified as acyanotic and cyanotic.

Atrial Septal Defect


ƒƒ Second most common congenital heart defectQ and most Black-blood
common to become symptomatic in adulthoodQ cardiac MRI
ƒƒ Associations are: Down’s syndrome, Holt-Oram syndrome, image shows
Lutembacher syndrome, Ellis van creveld syndrome dark signals from
ƒƒ On CXR— there will be signs of pulmonary plethora in the flowing blood in
pulmonary artery
form of dilated pulmonary arteries, prominent pulmonary
(red arrow) and
vessels in periphery of the lung.
aorta (white
ƒƒ Goose neck deformity seen on LV angiography—in Septum
arrow)
primum ASD
ƒƒ Great hilar dance seen on fluoroscopy—vigorous pulmonary
arterial pulsations due to increased blood flow Axial 4 chamber
ƒƒ Atrioventricular septal defect also known as endocardial view of white-blood
cushion defect is seen in Down’s syndrome. cardiac MRI shows

Imaging in Congenital Heart Disease


hyperintense signal
Ventricular Septal Defect from blood in left
(red arrow) and
ƒƒ Most common congenital heart defectQ
right (black arrow)
ƒƒ Associations seen with trisomies 13, 18, 21 and Holt-Oram
ventricles and left
syndrome. atrium (green arrow).
ƒƒ On radiograph there will be left atrial enlargement with Septum (yellow
changes of pulmonary arterial hypertension. arrrow) appears dark
ƒƒ On MRI a multiple muscular defect type of VSD can show -
Swiss cheese appearanceQ
ƒƒ Maladie de roger defectQ—seen in small VSDs
•• Unilateral and right-sided if the coarctation is proximal
Coarctation of AortaQ (NEET 2020 Pattern) to the left subclavian artery.
On CXR: •• Unilateral and left-sided if associated with an anoma-
ƒƒ “Figure of 3 sign”Q–formed by prestenotic dilatation of the lous right subclavian artery distal to the coarctation.
aortic arch and left subclavian artery, indentation at the
coarctation site (also known as the “tuck”), and poststenotic Tetralogy of Fallot
dilatation of the descending aorta. ƒƒ It is the only cyanotic heart disease that may present in
ƒƒ “Reverse figure of 3 sign Q/E - signQ” - seen on barium adulthoodQ.
swallow. ƒƒ It consists of 4 components of:
ƒƒ Inferior Rib notchingQ/Dock signQ is an important finding i. Ventricular septal defect
but it is seen only when significant intercostal collateral ii. Over-riding of aorta
circulation develops (after 10 years of age) and seen sparing iii. Right ventricular hypertrophy
the first 2 ribs. iv. Right ventricular outflow tract obstruction/Pul-
•• Affects 4th–8th ribs bilaterally. monic stenosis. 119
Digital Echocardiography:
subtraction Normal 4 chamber
cardiac view, with Mitral and
CONCEPTUAL REVIEW OF RADIOLOGY

angiogram Tricuspid valves seen

ƒƒ On chest radiograph there is Boot-shaped heartQ/Cor en


Appearance/Sign Seen in
SabotQ—mainly due to RVHQ
ƒƒ The pulmonary bay is concave and there will be pulmonary Pentagon-shaped heart Mitral incompetence
oligemiaQ. Ground glass appearance Obstructive type of TAPVC
ƒƒ Diagnosis is made on echocardiography of lung
ƒƒ Trilogy of Fallot: Consists of ASD, Right ventricular Sitting Duck heart Persistent truncus arteriosus
hypertrophy and Pulmonary stenosis.
Jug handle appearance Primary pulmonary hypertension
ƒƒ Pentalogy of Fallot has additional ASDQ along with other
tetralogy features. Cor-triatriatum Pulmonary vein obstruction
Snowman’s heart/Figure of Supracardiac type of TAPVC
Other Important Signs 8 heart/Cottage loaf sign Head of snowman is made or
superior mediastinal shadow
Appearance/Sign Seen in formed by SVC on right side,
Tear drop heart Chronic emphysema vertical vein on left side and left
brachiocephalic vein superiorly.
Egg in cup heart Constrictive pericarditis
Body of snowman is the heart.
Triangular-shaped heart Constrictive pericarditis
Egg-on-side appearance Transposition of great arteries
Box-shaped heart Ebstein’s anomaly/Tricuspid
Imaging in Congenital Heart Disease

(TGA)
atresia
Scimitar (Turkish sword Partial anomalous pulmonary
Tubular heart Emphysema/Addison’s disease appearance) venous return
Contd…

Chest radiograph Chest


showing ASD radiograph
(red line denotes of VSD shows
enlarged left atrial enlarged left
appendage) with atrium with
Eisenmengerization pulmonary
(yellow lines denote plethora
dilated pulmonary
arteries)

120
Chest radiograph Chest
shows a boot- radiograph
shape heart shows box-

CARDIOVASCULAR IMAGING
suggestive of shaped heart
Tetralogy of Fallot suggestive
of Ebstein’s
anomaly

Antenatal
USG Image –
Antenatal USG showing Over-
image – Showing riding of Aorta
Hypoplastic Left in Tetralogy of
heart syndrome Fallot

1
CLINICAL QUIZ - SOLUTION
The radiograph clearly shows a upturned cardiac apex creating the Boot-shaped heart seen in a cyanotic congenital heart disease –

Imaging in Congenital Heart Disease


Tetralogy of Fallot (Answer is A). This topic of congenital heart disease – its appearances – clinical features – is an absolute favorite of your
examiners and hence extremely high yield for you. An application-based question was also asked in the recent pattern NEET examination
and that is our 2nd quiz question here.
Pulmonary plethora cannot be seen in TOF due to pulmonary stenosis. Also in Ebstein’s anomaly the RV is small dysplastic with a large and
dilated RA – hence there is pulmonary oligemia rather than plethora. Right heart failure will not present with pulmonary plethora as the
RV is not able to pump blood adequately. TAPVC is associated with pulmonary plethora (hence answer is D). Usually a left-to-right shunt
of 2:1 is required for pulmonary plethora to occur. Increased pulmonary perfusion occurs in a number of situations:
• Left-to-right cardiac shunts, e.g., ASD, VSD, PDA
• Partial or total anomalous pulmonary venous return
• Transposition of the great arteries
• Truncus arteriosus
• Vein of Galen malformation
Pulmonary plethora can occur with or without cyanosis as follows:
• Without cyanosis: Left-to-right shunt
• With cyanosis: Transposition of great arteries, TAPVR, truncus arteriosus

121
Imaging in Acquired Heart Disease
2
CONCEPTUAL REVIEW OF RADIOLOGY

CLINICAL QUIZ
3. A young healthy male who has joined the Army presents with sudden onset chest pain during his training sessions, collapses and
dies. What could be the most likely cause?
A. Large VSD
C. Dilated cardiomyopathy
B. HOCM
D. Complete heart block

4. The hallmark finding seen on screening echocardiography in this condition is:


A. Ventricular wall hypokinesia
C. Mitral regurgitation
B. Systolic anterior motion of mitral valve
D. Pericardial thickening

Mitral Stenosis •• In chronic cases due to repeated pulmonary hemor-


rhage there will be pulmonary hemosiderosis.
ƒƒ This the most common valvular heart pathologyQ
ƒƒ Echocardiography, Cardiac CT, Cardiac MRI: All 3
ƒƒ Due to increased pumping pressure, the left atrium espe-
are reliable modalities for measurement of the mitral
cially the left atrial appendage undergoes enlargement and
valve dimensions and flow dynamics across it. On
it generally enlarged in anteroposterior manner.
Echocardiography - short axis view the stenotic valve shows
ƒƒ Chest radiograph:
a Fish mouth appearanceQ. On 2 or 4 chambered views the
•• Left atrial enlargement: thickened calcified anterior mitral leaflet may give a Hockey
 Straightening of left heart borderQ—earliest
stick appearanceQ.
signQ of LA enlargement on CXR PA/AP view
 Posterior displacement of esophagus—earliest
signQ of LA enlargement on CXR Lateral view
 Widening of carinaQ (to >900) with elevated left
CONGESTIVE HEART FAILURE
main bronchusQ ƒƒ It is basically a clinical diagnosis but sometimes in NEET
Imaging in Acquired Heart Disease

 Double density signQ—When the right side of you may get a clinical scenario and you should know the
the left atrium pushes into the adjacent lung, and basic imaging patterns seen.
becomes visible superimposed or even beyond ƒƒ Left ventricle (LV) failure is the most commonQ and results
the normal right heart border in decreased cardiac output and increased pulmonary
 Walking man signQ—seen on lateral view. Results venous pressure
from posterior displacement of the left main ƒƒ LVF presents with bilateral pleural effusionQ. If effusion is
bronchus such that it no longer overlaps the right unilateral it is more common on right sideQ than left
bronchus. The left and right bronchus thus appear ƒƒ Massive pericardial effusion may give the heart—Flask-
as an inverted ‘V’, mimicking the legs of a walking shaped heart appearanceQ
man. ƒƒ EchocardiographyQ is most commonly used modality and
 Posterior esophageal displacement on Barium provides a semi-quantitative assessmentQ of cardiac size
swallow and function.
 Third Mogul signQ—enlarged left atrial appendage ƒƒ On CT Chest—Cardiomegaly, bilateral pleural effusions
projecting along the left mediastinal margin and smooth septal thickening in lungs associated with
•• There will be upper lobe diversion of pulmonary vessels pulmonary edema.
with development of pulmonary venous hypertension
giving the Inverted Moustache signQ or Stag antler
signQ

122
CO Imaging Signs of Cardiac Failure—Conceptual Imaging
NC E P T

CARDIOVASCULAR IMAGING
Stage of CCF What happens? What is seen on Imaging? Why?
1 Redistribution of blood • Cephalization of pulmonary With a failing left heart, pulmonary venous hypertension
flowQ vesselsQ/Prominent upper develops. The blood returning to the LA from the pulmonary
PCWP: 13–18 mm HgQ lobe vessels veins becomes stagnant. These changes are especially more
• Broad vascular pedicleQ prominent in upper lobe veins
• CardiomegalyQ A failing heart starts to dilate or with time undergoes
hypertrophy
2 Interstitial edemaQ Kerley B linesQ The fluid in pulmonary venous system enters the pulmonary
PCWP: 18–25 mm HgQ Peribronchial cuffingQ interstitium due to increased capillary hydrostatic pressures
and reduced pump mechanism
When fluid leaks into the peribronchovascular interstitium it is
seen as thickening of the bronchial walls (peribronchial cuffing)
and as loss of definition of these vessels (perihilar haze).
3 Alveolar edemaQ • Batwing opacitiesQ Fluid fills the alveoli due to excessive Pulmonary Capillary
PCWP: >25 mm HgQ • ConsolidationQ Wedge Pressure
• Cotton wool spotsQ

Chest radiograph
PERICARDIAL DISEASES
showing various
signs associated Pericardial Effusion
with mitral ƒƒ Normally the pericardial cavity contains 30–50 mLQ of
stenosis like fluid. Any more than this is abnormal and is termed as
convex pulmonary effusion
bay (yellow line),
ƒƒ CXR findings:
widening of carina
(red line), double
•• Usually seen with >200 mLQ fluid in pericardial cavity
heart border •• Water bottle heartQ/Flask shaped heartQ/Money bag
(green lines) heartQ: Enlarged cardiac contour with maintained

Imaging in Acquired Heart Disease


cardiac apex position, tapering upwards.
•• Oreo cookie signQ: seen on lateral CXRQ. A vertical
opaque line (pericardial fluid) separating a vertical
lucent line directly behind sternum (pericardial fat)
anteriorly from a similar lucent vertical lucent line
Hypertrophic Obstructive Cardiomyopathy (epicardial fat) posteriorly.
•• Double density signQ: A fainter cardiac border seen
(HOCM)Q
beyond the cardiac margin—seen due to the effusion
ƒƒ It is familial in origin and one of the common causes of ƒƒ EchocardiographyQ:
sudden cardiac death in young individuals, especially after •• Investigation of choiceQ–Remember the concept
strenuous exercise. “Fluid is Friend of Ultrasound”
ƒƒ Echocardiography: •• USG-guided fluid aspiration can be performed
•• The hallmark on echocardiography is abnormal systolic
anterior motion (SAM)Q of the mitral valve causing left Pericardial TamponadeQ
ventricular outflow obstructionQ.
ƒƒ It is an emergent situation that needs immediate drainage.
ƒƒ Cardiac MRI:
ƒƒ On echocardiography there is right atrial inversionQ into the
•• Cardiac MRI is superior to echocardiographyQ in
ventricle on systole with compression of the ventricle on
identifying areas of segmental hypertrophy
diastole.

123
Chest Chest
radiograph radiograph
show Bat-wing showing Antler
CONCEPTUAL REVIEW OF RADIOLOGY

appearance sign of CCF with


of pulmonary pulmonary
edema venous HTN

White blood
Antenatal USG MRI image
shows echogenic shows
masses in pericardial
both ventricles effusion
suggestive of surrounding
bilateral Cardiac the heart (red
Rhabdomyomas arrows)

2
CLINICAL QUIZ - SOLUTION
Sudden cardiac death following strenuous exercise in a previously normal individual mostly points towards hypertrophic obstructive
cardiomyopathy. On echocardiography systolic anterior motion of the mitral leaflet is considered classical. Initial medical management
consists of beta blockers and Verapamil class of CCB with add on Di-isopyridamol. In refractory cases septal myomectomy or ethanol
ablation is used.
Imaging in Pulmonary and Aortic Diseases

Imaging in Pulmonary and Aortic Diseases


3
CLINICAL QUIZ
5. A middle-aged man with uncontrolled hypertension came with c/o
retrosternal chest pain. Cardiac markers were negative. A CE-CT Thorax was
done and is shown here. What could be the most probable diagnosis?
(NEET Jan 2018 Pattern/AIIMS May 2018 Pattern)
A. Aortic dissection
B. Myocardial infarction
C. Nutcracker esophagus
D. GERD

6. The classification systems used in such cases include:


A. Stanford classification C. Dissect score
B. DeBakey classification D. All of the above
7. The imaging signs seen in this condition include:
A. Cobweb sign C. Mercedes-Benz sign
B. Beak sign D. All of the above

124
PULMONARY ARTERIAL Chest
HYPERTENSION (PAH) radiograph
shows

CARDIOVASCULAR IMAGING
ƒƒ It is defined as resting mean pulmonary arterial pressure Pacemaker
(PAP) more than 25 mm HgQ at rest or >30 mm Hg on device (black
exercise measured by right heart catheterization. arrow) with
ƒƒ Normal resting mean PAP is <20 mm HgQ tip of its lead
ƒƒ Dana Point classification systemQ is used lying in left
ventricle (red
Imaging Features arrow)
ƒƒ Chest radiograph is always abnormal in symptomatic cases
and includes:
•• Elevated cardiac apex due to RVH.Q
•• Enlarged right atrium.
•• Enlarged pulmonary arteries (Normally the dimension
of descending branch of right pulmonary artery
Zoomed in
less than 15 mm in females and 16 mm in males is
radiograph of
considered as normal.)Q chest shows
•• Peripheral pruning of pulmonary arteries.Q prosthetic
ƒƒ On echocardiography based on tricuspid regurgitant jet metallic aortic
velocity a presumptive estimate of the PAH can be made.Q valve
ƒƒ CT scan/MRI should be performed with contrast for better (black arrow)
comment. The findings include: with post
•• HRCT will identify changes in lung parenchyma like sternotomy
ILD, connective tissue disorder which may have caused metallic sutures
PAH. (red arrows)
•• On angiography the diameter of main pulmonary
artery just prior to bifurcation >29 mmQ is treated as
abnormal.
•• Sex-specific reference values obtained from the
Framingham Heart study suggest cut-off values Axial CECT shows

Imaging in Pulmonary and Aortic Diseases


27 mmQ for women and 29 mmQ for men (on ECG- descending
gated non-contrast CT). thoracic aortic
dissection with
small true lumen
AORTIC DISEASES (black arrow) and
larger false lumen
(white arrow)
Thoracic Aortic Aneurysm
ƒƒ Diameter of thoracic aorta >5 cmQ.
ƒƒ AtherosclerosisQ is most common cause, other causes
include rheumatoid arthritis, syphilis, Marfan syndrome,
trauma, etc.
Axial CECT at
ƒƒ On chest radiograph there will be a mediastinal widening,
the level of
an opacity with long axis oriented along the aorta and
aortic arch
showing curvilinear wall calcification.
shows saccular
ƒƒ CT AngiographyQ—is the investigation of choice as it can aneurysm
identify the true size (black arrow)
ƒƒ MRA can also show these findings and is non-ionizing,
disadvantages are longer scan times and slightly reduced
resolution.

125
•• Tangential calcium sign
•• Crescent sign
•• Draped aorta sign
•• Periaortic fat stranding
CONCEPTUAL REVIEW OF RADIOLOGY

•• Extravasation of contrast

Peripheral Vascular Disease


A dilated arch ƒƒ USG and Doppler are investigation modality of choice,
of aorta on a especially for deep venous thrombosis, varicose veins,
CXR PA view Atherosclerosis, Burger’s disease.
– suggests
ƒƒ In DVT there is no colour flow on Doppler with loss of
an Aortic
compressibility of deep veins. The thrombus echogenicity
aneurysm
may increase with time.
ƒƒ Interventional radiology can be used to place IVC filter in
ƒƒ DSA is nowadays used mainly for endovascular repair of DVT, treat varicose veins using Radio frequency ablation
aneurysm. or laser diode ablation, embolize AV malformations or
ƒƒ Signs of impending rupture of aneurysm include: aneurysms using gel foam, polyvinyl alcohol.
•• Rapid increase in size
•• Missing calcium sign

Axial CECT Chest radiograph


show shows dilated right
crescentic pulmonary artery
nonenhancing (black arrow)
thrombus in with peripheral
the descending pruning (red arrow)
aorta (white suggestive of
arrow) pulmonary arterial
hypertension
Imaging in Pulmonary and Aortic Diseases

3
CLINICAL QUIZ - SOLUTION
The CT shows a dilated descending aorta with an intimal flap classification (More commonly used)Q. A Dissect score Q is also
separating the lumen into a true and a false lumen – suggests described recently comprising : Duration – Intimal tear – Size of
an Aortic dissecting aneurysm (Answer is A). All of the above dissecting aorta – Segmental extent of involvement – Clinical
conditions can present with acute chest pain and cardiac markers complications – Thrombosis of false lumen. (Ans to 2nd quiz
often remain negative in early stage of myocardial infarction. question is D. All of the above)
• It is most common type of acute aortic syndromes and caused • On CTA we can visualize intimal flap, true and false lumen,
by either penetrating ulcer or traumatic intimal flap with blood Mercedes Benz signQ in triple dissection, Windsock signQ
flowing below the intima. associated hematoma or thrombus. Cobweb signQ – is seen
• HypertensionQ - is the most common underlying cause. along the false lumen due to shreds of media seen along the
CXR: wall. Beak signQ is seen at the terminal end of the dissection.
ƒƒ May show mediastinal widening. (Thus, answer to 3rd quiz question is also D. All of the above)
ƒƒ The only specific sign of aortic dissection on chest radiograph • False lumen is generally bigger oneQ, thrombosed oneQ, less
is the inner displacement of intimal calcification by more than opacified postcontrastQ, may show cobweb signQ
1 cmQ • Invasive angiographyQ—is Gold standard investigation. Also used
ƒƒ The other signs are double aortic contour, irregular aortic therapeutically to put a stent in the true lumen thus obliterating
contour, mediastinal widening. the false lumen.
• Contrast enhanced CT scanQ—is the investigation of choice and
it can classify dissection based on either DeBakeyQ or Stanford
126
Multiple Choice Questions

CARDIOVASCULAR IMAGING
Imaging Modalities in CVS and Imaging in Congenital 13. Which of the following causes rib-notching on the chest
radiography? (AI Dec 2013)
Heart Disease A. Bidirectional Glen shunt B. VSD
1. Most important Doppler mode on echocardiography is: C. IVC occlusion D. Coarctation of aorta
A. Color Doppler
14. Persistent fetal heart rate of less than—beats per minute
B. Continuous wave Doppler
is associated with high incidence of the structural cardiac
C. Pulsed Doppler abnormality:
D. Power Doppler A. 100 B. 120
2. Single rotation acquisition times in cardiac CT using MDCT C. 150 D. 180
are around:
15. Phlebography refers to the contrast examination of which
A. 5 ms B. 7 ms of the following?
C. 10 ms D. 15 ms A. Arteries B. Veins
3. The only cyanotic heart disease that may present in C. Lymphatics D. Pulmonary circulation
adulthood is:
16. Aortic bump seen in the chest radiograph is characteristic
A. TOF B. TAPVC of which of the following CHD?
C. PAPVC D. Ebstein anomaly A. TOF B. PDA
4. What is the 5th component of Pentalogy of Fallot? C. ASD D. VSD
A. Tricuspid regurgitation B. Pulmonary hypertension 17. All of the following are features of Eisenmenger’s syndrome
C. Atrial septal defect D. Cardiac failure except:
5. Snowman appearance is seen in:(Recent Pattern Dec 2016) A. Triangular heart
A. TAPVC B. TGA B. Large main and central pulmonary artery
C. TOF D. DORV C. Large peripheral vessel in the lung parenchyma
6. Box shaped heart maybe seen in: (CET July 2016) D. Pulmonary arterial calcification
A. Ebstein anomaly B. VSD 18. Commonly associated in tetralogy of Fallot is:
C. PAPVR D. Trilogy of Fallot A. Aberrant right subclavian artery
7. Swiss cheese appearance may be seen in which CHD? B. Right sided aortic arch
A. ASD B. VSD C. Aortopulmonary window
C. DORV D. TAPVR D. Coarctation of aorta
8. Turkish sword sign is seen in: 19. Which of the following is best for VSD diagnosis?
A. Scimitar syndrome B. ASD A. 2D-Echo B. CT scan
C. TOF D. Pentalogy of Cantrell C. MRI D. Plain chest X-ray
9. All are causes of inferior rib notching except: 20. ‘Cottage loaf heart’ on chest radiograph is characteristic of
A. Aortic coarctation (AI Dec 2013) which of the following?
B. Osteogenesis imperfecta A. Supracardiac TAPVC B. Cardiac TAPVC
C. Infracardiac TAPVC D. Mixed type TAPVC

Multiple Choice Questions


C. Pulmonary AVM
D. Post Blalock-Taussig shunt 21. Radiological features of Ebstein’s anomaly includes all
except:
10. Egg on string appearance on chest radiograph is seen in:
 (AIIMS May 2008) A. Box shaped heart B. Pulmonary oligemia
A. TGA B. TAPVR C. Tricuspid abnormality D. Tubular heart
C. TOF D. VSD
Imaging in Acquired Heart Disease
11. Egg-on side appearance on X-ray chest is seen in:
 (PGI Nov 2009) 22. Walking man sign is seen in enlargement of:
A. Tetralogy of Fallot B. Uncorrected TGA A. RA B. LA
C. Tricuspid atresia D. Ebstein’s anomaly C. RV D. LV
12. Figure of 8 in chest X-ray is seen in:  (All India Dec 2014) 23. Flask-shaped heart is seen in: (NEET Pattern Dec 2012)
A. TAPVC A. Mitral stenosis
B. Tetralogy of Fallot B. Pericardial effusion
C. Transposition of great vessels C. Pulmonary edema
D. Ebstein’s anomaly D. Tetralogy of Fallot

127
24. Plethoric non-collapsible IVC on echocardiography denotes: 34. Parachute mitral valve is seen with:
A. Budd-Chiari syndrome A. Multiple papillary muscles
B. Hepatic venous malformation B. Single papillary muscle
CONCEPTUAL REVIEW OF RADIOLOGY

C. Right heart failure C. Absent chordae tendineae


D. IVC thrombosis D. Single chordae tendineae
25. Earliest sign of left atrial enlargement on a lateral CXR is: 35. Hoffman-Rigler sign for enlargement of the left ventricle
A. Posterior displacement of esophagus  (AI 2008) is seen on:
B. Carinal angle widening A. PA view of the chest
C. Elevation of left bronchus B. AP view of the chest
D. Double shadow of right border C. Apicogram of the chest
26. Straightening of left heart border is seen in: D. Lateral view of the chest
A. Mitral stenosis B. Concentric LVH
C. Aortic regurgitation D. Mitral valve prolapse
Imaging in Pulmonary and Aortic Diseases
36. Which of the following is not radiographic sign of PAH?
27. Abnormal systolic anterior motion (SAM) is hall mark of:
A. Downward displaced apex (NEET Pattern Dec 2012)
A. Hemochromatosis cardiomyopathy
B. Peripheral pruning of PA
B. HOCM
C. RA enlargement
C. Dilated cardiomyopathy
D. Dilated main pulmonary artery
D. Tubercular cardiomyopathy
37. Peripheral pruning of arteries is seen in:  (AI Dec 2013)
28. The most common neoplasia in heart is:
A. Interstitial lung disease
A. Lymphoma B. Rhabdomyoma
B. Pulmonary arterial hypertension
C. Angiosarcoma D. Metastasis
C. Left ventricular failure
29. Investigation of choice in acute pericardial tamponade is:
D. Bronchial carcinoid
 (AI Dec 2014)
A. Chest radiograph B. Echocardiography 38. Kerley B lines are seen in:
C. CT scan D. Cardiac MRI A. Pulmonary venous HTN B. Pulmonary Arterial HTN
C. Severe systemic HTN D. Renal HTN
30. Oreo cookie sign is seen in:
A. Pericardial effusion B. MS 39. At PCWP more than 20 mm Hg what finding is seen on:
C. Myocardial infarction D. CCF A. Stag Antler sign B. Consolidation in air spaces
C. Kerley A Lines D. Kerley B Lines
31. Calcifications of constrictive pericarditis are best seen on:
A. Penetrated chest radiograph (AI Dec 2013) 40. Cobweb sign is seen in:
B. Cardiac MRI C. Digital radiograph A. Aortic dissection B. Aortic aneurysm
D. CT scan C. Aortic hematoma D. Aortic rupture
32. Acute myocardial infarct scintigraphy is done with: 41. Stanford classification is used for:
A. Thallium A. Aortic dissection B. Aortic aneurysm
B. Gallium C. Aortic hematoma D. Aortic rupture
C. Neodymium 42. Figure of 3 sign is seen in:
D. Tc Stannous pyrophosphate A. Aortic dissection
Multiple Choice Questions

33. Kerley-B lines on the PA view of the chest is seen at which B. Aortic aneurysm
of the following region of the lung? C. Aortic hematoma
A. Upper zones D. Aortic coarctation
B. Near the hilum 43. IOC for DVT is: (Recent Pattern Dec 2016)
C. At the bases near the costophrenic angles A. USG with Doppler B. MRI
D. Near the cardiophrenic angles C. Invasive angiography D. CT

Answers
1. B 5. A 9. B 13. D 17. C 21. D 25. A 29. B 33. C 37. B 41. A
2. C 6. A 10. A 14. A 18. B 22. B 26. A 30. A 34. B 38. A 42. D
3. A 7. B 11. B 15. B 19. A 23. B 27. B 31. D 35. D 39. B 43. A
4. C 8. A 12. A 16. B 20. A 24. C 28. D 32. D 36. A 40. A

128
Explanations to Questions

CARDIOVASCULAR IMAGING
1. The velocities of blood flowing in cardiac chambers is large ƒƒ Atrial septal defect (ASD) or Patent ductus arteriosus
which can be measured by Continuous wave DopplerQ, (PDA) (termed pentalogy of Fallot)
however this is done at expense of depth. ƒƒ Coronary artery anomalies.
14. The normal fetal heart rate ranges from 120 to 160 bpm. ƒƒ Persistent left-sided superior vena cava.
Mild bradycardia is observed transiently in normal second‐ •• Extra-cardiovascular associations: In 15% of cases
trimester fetuses. Fixed bradycardia, especially heart rates ƒƒ Congenital lobar emphysema (CLE)
that remain below 100–110 bpmQ requires timely evaluation ƒƒ VACTERL association
by a fetal cardiac specialist for possible heart block. Repeated ƒƒ Tracheoesophageal fistula
heart rate decelerations during the third trimester can be ƒƒ DiGeorge syndrome
caused by fetal hypoxia. Persistent tachycardia (≥180 bpm), ƒƒ Prune belly syndrome
however, should be evaluated further for possible fetal ƒƒ Fetal rubella syndrome
hypoxia or more serious tachydysrhythmias. 24. In Heart failure increasing IVC diameter is associated with a
15. Phlebography refers to the contrast examination of worse prognosisQ. The IVC diameter is a summary measure
veinsQ. Arteriography refers to contrast examination of of cardiac function as well as a marker of venous congestion.
arteries. Lymphography refers to examination of the Measurements of IVC collapse are commonly reported as
lymphatics. Venography (also called phlebography or the ‘collapsibility index’Q which is calculated as (maximum
ascending phlebography) is a procedure in which an X-ray of IVC diameter on expiration – minimum IVC diameter on
the veins, a venogram, is taken after a special dye is injected inspiration/maximum IVC diameter on expiration). Mean IVC
into the bone marrow or veins. The dye has to be injected diameter correlates with CVPQ.
constantly via a catheter, making it an invasive procedure. 28. Occurrence of tumors in heart is Metastasis > benign
Normally the catheter is inserted by the groin and moved (atrial myxomas) > malignant (Angiosarcoma). Cardiac
to the appropriate site by navigating through the vascular rhabdomyomas are associated with Tuberous sclerosisQ.
system. Contrast venography is the gold standard for judging 31. Remember whenever we have to image calcification
diagnostic imaging methods for deep venous thrombosis; anywhere in body CT scan is the imaging modality of choice.
although, because of its cost, invasiveness, and other 34. Parachute deformity of the mitral valve is characterized by
limitations this test is rarely performed shortened chordae tendineae which converge and insert into
17. Eisenmenger syndromeQ refers to the reversal of shunt direc- a single papillary muscle.
tion in long standing acyanotic heart disease due to develop- 35. The Hoffman-Rigler signQ is a sign of left ventricular
ment of pulmonary arterial hypertension. Features are: enlargementQ inferred from the distance between the inferior
ECG: RAE, RVH, Right axis deviation, pulmonary artery vena cava and left ventricle. On a lateral chest radiographQ,
calcification. if the distance between the left ventricular border and
Chest X-ray: CardiomegalyQ, dilated pulmonary arteriesQ, the posterior border of IVC exceeds 1.8 cm, at a level 2 cm
triangular heartQ, Pulmonary artery calcificationsQ. above the intersection of diaphragm and IVC, left ventricular
18. Associations of Tetralogy of Fallot (TOF) are: enlargement is suggested. Hoffman-Rigler sign should not be
•• Cardiovascular associations confused with Rigler sign, Rigler triad or Rigler notch sign.
ƒƒ Right-sided aortic arch: seen in 25% of cases (Most
commonQ)
ƒƒ Pulmonary hypoplasia with or without atresia.

Explanations to Questions

129
_______________________________________________________________________________________________________________________

_______________________________________________________________________________________________________________________

_______________________________________________________________________________________________________________________

_______________________________________________________________________________________________________________________

Note
_______________________________________________________________________________________________________________________

_______________________________________________________________________________________________________________________

_______________________________________________________________________________________________________________________

_______________________________________________________________________________________________________________________

_______________________________________________________________________________________________________________________

_______________________________________________________________________________________________________________________

_______________________________________________________________________________________________________________________

_______________________________________________________________________________________________________________________

_______________________________________________________________________________________________________________________

_______________________________________________________________________________________________________________________

_______________________________________________________________________________________________________________________

_______________________________________________________________________________________________________________________

_______________________________________________________________________________________________________________________

_______________________________________________________________________________________________________________________

_______________________________________________________________________________________________________________________

_______________________________________________________________________________________________________________________

_______________________________________________________________________________________________________________________

_______________________________________________________________________________________________________________________

_______________________________________________________________________________________________________________________

_______________________________________________________________________________________________________________________

_______________________________________________________________________________________________________________________

_______________________________________________________________________________________________________________________

_______________________________________________________________________________________________________________________

_______________________________________________________________________________________________________________________

_______________________________________________________________________________________________________________________

_______________________________________________________________________________________________________________________

_______________________________________________________________________________________________________________________

_______________________________________________________________________________________________________________________
GASTROINTESTINAL
TRACT IMAGING
C hapter O utline
• Peritoneal Abnormalities • Hepatobiliary and Pancreatic Imaging
CONCEPTUAL REVIEW OF RADIOLOGY

• Imaging in Foregut • Miscellaneous Abdominal Imaging


• Small and Large Bowel Abnormalities • Multiple Choice Questions with Explanations

Peritoneal Abnormalities
1
CLINICAL QUIZ
1. A patient presented to the emergency department with severe
abdominal pain and tachycardia. An erect abdominal radiograph
was done and is shown here. What is the diagnosis?
A. Intestinal obstruction
B. Sigmoid volvulus
C. Perforation
D. Hiatus hernia

2. The best investigation for detection of minimal pneumoperi-


toneum is?
A. X-ray chest erect C. CT abdomen
B. X-ray erect abdomen D. USG fast

PNEUMOPERITONEUM
Peritoneal Abnormalities

ƒƒ Free air in abdomen in adults—is usually a result of perforation—either from a peptic ulcerQ or from intestinal perforationQ.
ƒƒ Most common cause of neonatal pneumoperitoneum is—Necrotizing enterocolitisQ.

  Clinical Pearls
PseudopneumoperitoneumQ: Conditions Simulating a Pneumoperitoneum on Abdominal Radiograph:
• Chilaiditi syndrome • Cysts in pneumatosis intestinalis
• Subphrenic abscess • Pneumomediastinum
• Curvilinear atelectasis of lung • Properitoneal fat stripe
• Subdiaphragmatic fat • Basal lung bulla
• Diaphragmatic irregularity

132
CO X-ray Chest or X-ray Abdomen Erect for PneumoperitoneumQ?
NC E P T

GASTROINTESTINAL TRACT IMAGING


• All our hospitals, all over India routinely do X-ray Erect abdomen for detection of pneumoperitoneum/free air under domes of
diaphragm. Are we all wrong?
• Well, as we have read above a Chest radiograph is better at depiction of pneumoperitoneum. But why?
• Look at the adjoining image.

  Clinical Pearls
Best Investigation for Pneumoperitoneum
• A lot has been said and controversies have been created • USG: Usually 1st investigation done in acute abdomen cases.
regarding the best investigation. I will make an attempt at Studied have proven it to be better than plain radiographs – but
settling doubts once and for all. is operator dependent, subjective.
• CT Abdomen – BEST Investigation overallQ • X-ray abdomen Erect: Commonly done in our hospitals, though
• X-ray Chest Erect – Best radiographic projectionQ it is theoretically wrong and erect CXR is considered better
• X-ray Chest Erect Lateral view is better than X-ray Chest Erect • X-ray Supine Abdomen: Easy to do in a critically ill patient and
PA view is valuable because a lot of named/peculiar signs have been
• X-ray left lateral decubitus abdomen: 2nd Best radiographic described. Less sensitive than other radiographs.
projection—but is of limited use because the patient may be
too ill to be placed in this position

Named signs in Pneumoperitoneum: Long list of signs has been described. Important point to note is that most of these signs are
described on Supine abdomen radiographs—being done quickly in critically ill patients.
Though the list of long, remember that most of these signs are direct applications of the Differential Radiographic Density
Theory (DRDT)—the mother of all CONCEPTS…remember!

Radiographic Signs of Pneumoperitoneum

Peritoneal Abnormalities
Falciform ligament sign/ • Gas (air density) lines the falciform ligament (water density structure) on either sides thus making it clearly
Silver’s signQ visible
Liver edge sign Q
• Gas (air density) lines the inferior edge of liver (water density structure) thus making it clearly visible
Gallbladder sign Q
• Gas (air density) lines the outer surface of GB (water density) thus making it clearly visible
Leaping dolphin signQ/ • During early stages of diaphragmatic contractions, the muscle slips may create curvilinear elevations on
Diaphragmatic muscle the undersurface of diaphragm
slip signQ • Gas (air density) lines these muscle slip curvilinear elevations (water density) thus making them clearly
visible
Lucent liver signQ • In supine position the air rises up and collects anterior to the anterior liver surface
• On supine films the opacity of the liver may appear slightly lucent/black because of this air here
Anterosuperior bubble • Small air bubbles may rise to the non-dependent part and float in the abdominal free fluid on the
signQ anterosuperior surface of liver
Doges cap signQ - Air in • Air trapped in the Morrison’s pouch (Hepatorenal pouch) takes a peculiar shape that resembles a cap worn
Morrison’s pouch by Italian Doges
Contd… 133
Radiographic Signs of Pneumoperitoneum
Rigler’s double wall • Normal patients:
signQ/Gas relief signQ ƒƒ The bowel wall (water density) is lined on the outside by peritoneal fluid (also water density) and hence
CONCEPTUAL REVIEW OF RADIOLOGY

the serosal surface is never seen as a sharp margin


ƒƒ The mucosal surface of bowel (water density) may be lined by intraluminal gas (air density) and hence
may be seen as a sharp margin
ƒƒ This in normal patients or in Intestinal obstruction radiographs—the bowel wall margin you see is always
the inner/mucosal surface
• Pneumoperitoneum:
ƒƒ Gas (air density) lines the outer surface of bowel (water density) thus making it clearly visible
ƒƒ The mucosal surface of bowel (water density) lined by intraluminal gas (air density) is also seen as a sharp
margin
ƒƒ Thus both inner and outer surfaces/walls become visible—Rigler’s Double wall sign
Football signQ • In supine position all the free air rises up and collects in the central abdomen just beneath the anterior
abdominal wall—seen as an oval central lucency—like an American football
• Indicates massive pneumoperitoneum
Cupola signQ/Saddlebag • 1st radiographic sign described in 1915
signQ/Mustache signQ • The undersurface of diaphragm (water density) is lined by gas (air density) hence seen as a sharp margin
Left sided • Free air seen as a lucency under the left dome of diaphragm seen separately from the fundic gastric bubble
pneumoperitoneum signQ
Visible transverse • The mesocolon/mesentery (water density) are lined by gas (air density) and hence is seen as a sharp
mesocolonQ/Small bowel margin
mesentery signQ
Visible urachus signQ • The urachus (water density) is lined by gas (air density) and hence seen as a sharp margin
Inverted V sign Q
• The lateral umbilical ligaments/inferior epigastric vessels (water density) are lined by gas (air density) and
hence are seen as a sharp margin—creating an inverted V shaped configuration. Seen as oblique linear
opacity extending from the umbilicus to the pelvis
PneumoscrotumQ • Free air travels through the patent processus vaginalis in neonates and infants and enters the scrotal sac—
scrotal pneumatocele
Tell-tale triangle signQ • Triangular collection of air in between 3 closely related bowel loops or between 2 loops and the abdominal
wall
Ligamentum teres sign Q
• Seen on CT, free air outlines the ligamentum teres
Peri-portal free gas sign Q
• Seen on CT, suggests upper GI tract perforation
Peritoneal stripe signQ • Seen on USG, as enhancement of the peritoneal stripe with distal reverberation/comet tail artefacts
Anterior-superior oval • A round, oval, or pear-shaped collection of air may be projected over the liver shadow between the ventral
signQ liver surface and the anterior thoracic or abdominal wall

Pneumoperito-
ASCITES IMAGING
Peritoneal Abnormalities

neum – Free air


under domes of ƒƒ Pathologic fluid collection in the peritoneal cavity (usually
diaphragm more than 100 mL)
(yellow arrows) ƒƒ Plain abdomen radiographs:
•• Nonspecific as cannot differentiate the fluid (water
density) from abdominal soft tissues (also water
density)
•• Direct signsQ:
 Hellmer signQ: Lateral edge of liver is displaced
medially from the thoracoabdominal wall
 Mickey mouseQ/Dog’s ear appearance of
bladderQ: Due to fluid accumulating in the recto-
vesical pouch and spilling into the paravesical
fossa

134
•• Indirect signsQ: •• Hypoechoic with internal echoes—seen in exudates,
 Elevated bilateral domes of diaphragm hemoperitoneum, internal debris, infected fluid,
 Reactionary pleural effusions neoplastic ascites
•• Internal septae—may be seen in chronic ascites/

GASTROINTESTINAL TRACT IMAGING


 Diffuse abdominal haziness
infective/inflammatory etiology
 Bulging of flanks
ƒƒ CT Scan:
 Indistinct psoas muscle margins
•• Sensitive for small amounts of fluid
 Central floating gas containing bowel loops
•• CT value/Hounsfield Unit (HU) value of fluid may
ƒƒ USG: Best/Most sensitive modalityQ for detection of free
indicate underlying etiology, but is not specific:
fluid: Remember “Fluid is Friend of ultrasound”. Can detect
 –10 to + 10 HUQ: Transudative fluid
5–10 mL of fluid.
 15 HUQ: Exudative fluid
•• Seen as anechoic/hypoechoic collection in dependent
parts of peritoneal cavity  45 HUQ—Hemoperitoneum
•• Anechoic fluid—is jet black—seen in transudates/clear CT signs to distinguish between Pleural effusion and Ascitis –
fluid discussed in topic “Pleural effusion”:

PSEUDOMYXOMA PERITONEI IMAGING


ƒƒ Intraperitoneal accumulation of gelatinous deposits sec-
ondary to rupture of mucinous adenocarcinoma of appen-
dix.
ƒƒ USG:
•• Ascites with echogenic internal contents/echogenic
focal lesions–mucinous deposits
•• Star-burst appearance of bowel loopsQ—loops
displaced posterior and centrally—instead of floating
freely
ƒƒ CE-CT/CE-MRI:
•• Low attenuation fluid on CT often with loculations,
and ill defined mildly enhancing soft tissue deposits
throughout the peritoneal cavity Hypodense ascites (white arrows) in the peritoneal cavity
•• Scalloping of liver/spleen surfaceQ—is characteristic
•• MRI has greater accuracy than CT

1
CLINICAL QUIZ - SOLUTION
• Pneumoperitoneum/Perforation is the answer to the 1st clinical quiz. See the free air under the right dome of diaphragm in the radiograph.
• Seems like a too simple question isn’t it ?
• Well, it is. But the point I want to make here is that though it is a simple diagnosis, it is a potentially life threatening one if missed. Hence
we must always examine the area below the dome of diaphragm (be it a CXR or an Abdomen radiograph so that we do not miss it).
• As discussed above the overall best investigation for minimal pneumoperitoneum is CT. In fact anywhere in the body you want to detect

Peritoneal Abnormalities
minimal air the IOC will be CT. (Pneumoperitoneum/Pneumothorax/Pneumomediastinum – all of them have CT as the IOC)

135
Imaging in Foregut
CONCEPTUAL REVIEW OF RADIOLOGY

CLINICAL QUIZ
2
3. Patient presented with neck swelling and regurgitation with
gurgling sound when pressed over the neck. A barium swallow
was performed and is revealed here. Most likely diagnosis will
be: (NEET 2020 Pattern)
A. Esophageal stricture
B. Zenker’s diverticulum
C. Esophageal web
D. Dysphagia lusoria
4. Plummer–Vinson syndrome is associated with:
A. Esophageal web
B. Iron deficiency anemia
C. Dysphagia
D. All of the above

Congenital Anomalies of Esophagus, Radiograph: Foreign


Stomach and Duodenum body (coin) in
proximal esophagus
Tracheoesophageal Fistulas which appears
ƒƒ Antenatal USG: Antenatally due to reduced or absent round on AP view
swallowing there is– – as the coin is in
•• PolyhydramniosQ coronal plane. If
•• Non visualization of fundic bubbleQ. in the trachea –
the coin will be in
ƒƒ Postnatal chest radiograph:
•• Coiling of nasogastric tube/Ryle’s tube in upper thoraxQ sagittal plane - will
appear round in
suggesting esophageal atresia.
lateral view
•• Always look for fundic gas, if present then there is
Q
associated tracheoesophageal fistula.
ƒƒ Commonly associated with VACTERL anomaliesQ

Congenital Hypertrophic Pyloric Stenosis Barium meal:


ƒƒ Typically presents as non-bilious vomiting in a few weeks focal collection
of birth up to 3 monthsQ in male babiesQ. Does not present of barium
immediately at birthQ (red arrow),
ƒƒ On abdominal radiographs a distended stomach with suggests
paucity of air in distal bowel loops maybe seen. peptic ulcer
ƒƒ On barium meal we can see a host of signs which include:
•• Caterpillar signQ - due to hyperperistalsis
•• Mushroom signQ - pylorus indenting on duodenal bulb
Imaging in Foregut

•• Shoulder signQ - pylorus indenting on antrum


•• String signQ - contrast in the narrowed elongated pylorus
•• Double track signQ - Mucosal prolapse into the pyloric
canal
•• Beak signQ - beak shaped entrance of the pylorus
•• Diamond signQ
ƒƒ Ultrasound - is imaging modality of choiceQ - as it is non
ionizing and can demonstrate dynamic obstruction of
gastric outlet and measure the length and thickness of the
hypertrophied pylorus. USG signs include Barium meal:
•• Cervix signQ Hiatal hernia –
•• Antral nipple signQ sliding type
•• Target sign/Doughnut signQ
136
ƒƒ Barium examinations: Double Contrast techniqueQ
  Clinical Pearls better demonstrates the imaging signs
Bubble signs on an Infantogram/Babygram •• The ulcer appears as focal collection of barium with
• Single bubble sign—Pyloric stenosisQ - due to dilated stomach radiating thin gastric folds reaching up to the ulcer margin.

GASTROINTESTINAL TRACT IMAGING


• Double bubble sign—Duodenal atresiaQ (or any cause of •• Ring shadowQ—may be seen in a ulcer on the non-
duodenal obstruction) - due to dilated stomach and duodenal dependent gastric wall with the barium lining the rim
cap of the ulcer crater
• Triple bubble sign—Jejunal atresiaQ—due to dilated stomach, •• There is edematous ring surrounding the focal barium
duodenal cap and duodenal loop collection which appears as lucent (black) rim also
• Multiple bubble sign—Ileal atresiaQ known as Ulcer collar
•• Hamptons line Q—Pencil thin lucent line seen crossing
the base of the ulcer—diagnostic of benign ulcer
•• Ulcer commonly extends beyond the lumen of the
MOTILITY DISORDERS stomachQ
•• Commonly present on lesser curvatureQ and in distal
Achalasia Cardia Refer to Image 18 in stomach. Presence of multiple ulcers or post bulbar
“Imaging Signs in Radiology” locations should lead to possibility of Zollinger-Ellison
syndromeQ.
ƒƒ Associated with Chagas diseaseQ and CREST syndrome.Q •• Sometimes 2 opposite ulcer may causing severe
ƒƒ Failure of dilatation of the lower esophageal sphincterQ stricture giving the Kissing ulcer signQ or Hourglass
along with loss of distal esophageal peristalsisQ stomach signQ
ƒƒ Secondary AchalasiaQ/PseudoachalasiaQ—Obstruction of
distal esophagus due to a tumor HIATUS HERNIA
ƒƒ Plain radiograph:
•• Air-fluid level in thorax usually central in the medi- ƒƒ Sliding type hernia: Most common type
astinum/retrocardiac area due to food filled dilated •• Gastroesophageal junction is seen to lie above the
esophagus. diaphragmQ (at least >2 cm above diaphragm)
ƒƒ Fluoroscopy with barium swallow: Radiologic investiga- •• On CXR - gastric fundus maybe seen in lower thorax in
tion of choiceQ midlineQ.
•• Bird beak signQ: Smooth elongated narrowingQ of the •• On barium swallow the position of GE junction above
dilated contrast filled esophagus in distal portion. diaphragm can be demonstrated. 3 or more gastric
foldsQ identified passing from stomach across the hiatus
•• On CT—Wide esophageal hiatus is seen >3 cm in
  Clinical Pearls diameterQ. Also the diaphragmatic crura are separated
Hurst Phenomenon in Achalasia by more than 15 mmQ
With the patient erect the barium builds up in the esophagus, ƒƒ Rolling (Paraesophageal) hernia: Here GE junction in
up to a point that the hydrostatic pressure overcomes the lower infradiaphragmaticQ but gastric fundus protrudes into the
esophageal sphincter and barium passes into the stomach. hiatusQ. It can lead to gastric volvulusQ both of which are
Sudden release of the barium column into stomach on inhalation well demonstrated on barium studies.
of amyl nitrateQ/carbonated drinkQ. IOC:
ƒƒ Traditionally Barium swallow was considered as the IOC for
Hiatus hernia
Diffuse Esophageal Spasm ƒƒ But as per recent edition of Love & Bailey – CT with oral
contrast is the IOC for Hiatus hernia
ƒƒ It is an extremely painful condition caused by incoordinated
movements of esophagus.
  Clinical Pearls
ƒƒ On barium swallow a CorkscrewQ or Rosary bead appearanceQ
Schatzki’s Ring/B ring –
or Curling esophagusQ is seen.
• Mucosal ring seen at proximal end of a hiatal hernia Imaging in Foregut
Hypercontracting Esophagus • Schatzki rings are located at the gastro-esophageal junction,
illustrated by the fact that there is squamous epithelium above
ƒƒ Also known as Nutcracker esophagusQ. and columnar epithelial below the ring.
ƒƒ Barium imaging is normal and diagnosis is based on • Causes intermittent, non-progressive dysphagia
manometry. • Management is usually by dilation using Bougie
A ring –
• Uncommon
PEPTIC ULCERATIONS • Located proximal to B ring
• They represent a physiological contraction of esophageal
The more common sites include stomach, duodenum, gastroe- smooth muscle covered by mucosa
sophageal junction. • Seen at the proximal to squamocolumnar junction
ƒƒ Plain radiographs: Dysphagia lusoria –
•• On plain radiograph findings are seen only if the ulcer • Dysphagia due to compression of the esophagus by a Aberrant
perforates where free gas under diaphragm can be right subclavian artery 137
seen. • Diagnosis is done by CT angiography
•• A specific type of infiltrating gastric malignancy is
Linitis plasticaQ where there is severely reduced
Barium meal:
volume of stomach which can be demonstrated on
Grossly reduced
barium exams.
capacity of stomach,
CONCEPTUAL REVIEW OF RADIOLOGY

suggests Linitis
plastica of stomach
Differences between Benign and
CO
NC E P T Malignant Ulcer on Barium StudiesQ
Findings Benign Malignant
Hampton’s line Present Absent
Extends Yes No
beyond gastric wall
Folds Smooth, even Irregular,
nodular, may
fuse
NEOPLASMS
Associated mass Absent Present
ƒƒ Most of the neoplasms are adenocarcinoma. But rare types
Carman Absent Present
include GIST, lymphoma etc.
meniscus
ƒƒ Barium studies:
•• The ulcer crater is irregularQ, usually does not protrude Ulcer shape Round, oval, linear Irregular
beyond gastric marginsQ, the gastric folds are thick, Healing Heals completely Rarely heals
nodular, club shaped, radiating towards ulcer but
falling short of it.
•• Carman meniscus signQ - is lenticular shape of barium CT Scan: Staging Modality of ChoiceQ
in cases of large and flat gastric ulcers, in which the for Esophageal/Gastric/Duodenal
inner margin is convex toward the lumen. It usually
indicates a malignant ulcerated neoplasmQ. Maligancies
•• Esophageal malignancies can present as Ulcerating ƒƒ Apart from the lesion size, shape, extent it can identify
variety where there is irregular mucosal lining with invasion of adjacent vascular structures and other vital
holdup of barium or the Stricturing variety where the organs, & lymphadenopathy.
mucosa is smooth but there is sudden narrowing of ƒƒ Distant metastases to organs like liver, lungs, ovaries
the barium column giving rat tail appearanceQ with (Krukenberg tumors) are also well demonstrated
shouldering.Q

DIVERTICULI
Esophageal Diverticuli Duodenal Diverticulum
ƒƒ Intraluminal diverticulum: Also known as duodenal web
Zenker's DiverticulumQ - Discussed Below
often seen in Down’s syndrome, annular pancreas etc. On
barium is typically give a Windsock signQ which is gradual
Axial CECT: ballooning of the duodenal diaphragm or Halo signQ due to
enhancing the web projecting caudally into the duodenal lumenQ.
Imaging in Foregut

wall thickening
at pylorus,
suggestive
of Gastric
carcinoma

138
MISCELLANEOUS CONDITIONS
ƒƒ Esophageal WebsQ—Appear as transverse shelf like filling defects in the barium column with a Jet phenomenonQ
ƒƒ Plummer-Vinson syndrome/Paterson—Brown—Kelly syndrome: - described below

GASTROINTESTINAL TRACT IMAGING


Barium swallow: Barium
filling defect, swallow image
mucosal irregularity demonstrating a
(red arrows), horizontal shelf like
suggests of filling defect (yellow
esophageal arrows) – suggests a
malignancy Esophageal web

Radiograph:
coiled RT in
neck suggesting
Radiograph: esophageal
Double bubble atresia, gas in
sign suggestive abdomen suggests
of congenital a lower down
duodenal tracheoesophageal
atresia fistula

2
CLINICAL QUIZ - SOLUTION
• A posterior outpouching from the esophagus as seen here with ƒƒ On barium swallow best seen on lateral view - as barium
a history of neck swelling, regurgitation – suggests Zenker’s filled outpouching (will appear white) in midline posteriorly
diverticulum (Answer is B). It is a false diverticulum arising at the just above cricopharyngeus
Killian’s dehiscence above the Cricopharyngeus muscle. A anterior/ Plummer-Vinson syndrome/Paterson—Brown—Kelly syndrome:
antero-lateral diverticulum that arises below the cricopharyngeus • Predisposes to squamous cell carcinoma of hypo­pharynx Imaging in Foregut
• Seen in women and consists of:
muscle is known as Killian-Jamieson diverticulum. Few important
ƒƒ Hypopharyngeal esophageal webs
points about Zenker’s diverticulum include :
ƒƒ Iron deficiency anemia – Koilonychia (spoon shaped nails)
ƒƒ Also known as Posterior pharyngeal pouch
ƒƒ Dysphagia and
ƒƒ Pulsion hypopharyngeal false diverticulumQ (consisting ƒƒ Weight loss
only mucosa and submucosa) protruding through Killian’s ƒƒ Associated with increased risk of Squamous cell carcinoma of
dehiscenceQ between horizontal and oblique components of esophagus & Hypopharyngeal carcinoma
cricopharyngeus muscleQ. (Thus the answer is D. All of the above)

139
Small and Large Bowel Abnormalities
CONCEPTUAL REVIEW OF RADIOLOGY

3
CLINICAL QUIZ
5. A 65-year-old female patient was on antipsychotics. She presented to
the ER with absolute constipation since 4 days and massive abdominal
distension and pain. An abdominal radiograph revealed the following
finding. What is the most likely diagnosis?
A. Small bowel obstruction
B. Large bowel obstruction
C. Sigmoid volvulus
D. Caecal volvulus

Small Bowel Obstruction (SBO) conniventes in the non-dependent part of a fluid filled
dilated bowel.
ƒƒ AdhesionsQ—most common cause, followed by herniaQ
ƒƒ USG:
ƒƒ X-ray Abdomen erect—Investigation of choice for diagnosis
of obstructionQ
•• Dilated small bowel loops (>3 cm diameter) with
vigorous to-and-fro peristalsis. Bowel may appear
ƒƒ CT Abdomen and Pelvis (plain + contrast)—Investigation of
choice for cause of obstructionQ edematous.
ƒƒ Plain Abdominal radiograph/X-ray Abdomen Erect: ƒƒ CT:
•• Dilated small bowel loops >3 cm in diameterQ •• Dilated small bowel loops
•• “Transition zone/point”Q: point of obstruction
•• Centrally located loopsQ—“Picture of small bowel”Q where there is abrupt transition between dilated loops
•• Step ladder appearanceQ: Multiple abnormal air-fluid proximally and collapsed/empty loops distally—Most
levels—especially >2.5 cm wide, in same loop of bowel reliable CT criteria for diagnosisQ
Small and Large Bowel Abnormalities

but at different heights •• Small bowel faeces signQ: Contents resembling faeces
•• Prominent valvulae conniventes seen—Herring bone (particulate contents with gas bubbles) seen in a small
pattern/Concertina effectQ bowel.
•• Gasless abdomenQ: rare presentation seen in high •• Closed loop obstruction—is when a bowel loops is
(proximal) obstruction obstructed at 2 adjacent points along its course. Such
•• String-of-beads/pearls signQ: Seen on X-ray and CT. a closed loop usually rotates on itself thus forming a
Small linear gas bubbles trapped between the valvulae small bowel volvulus. Seen as

Erect radiograph Axial CT –


abdomen shows dilated small
multiple centrally bowel loops
located dilated bowel filled with solid
loops with air fluid material with
levels– Step-ladder interspersed
appearance of Small air – Small
bowel obstruction bowel feces
(yellow arrows). Inset sign – Small
shows the Valvulae bowel
conniventes markings obstruction
characteristic of small
bowel obstruction (blue
arrows)

140
 U or C—shaped configurationQ of dilated loops
 Beak signQ: Fusiform tapering of the bowel lumen Coronal
at the site of obstruction reconstructed image
 Whirlpool sign/Whirl signQ: The twisting of bowel CE-CT – Prominent

GASTROINTESTINAL TRACT IMAGING


loops at the site of obstruction—as seen on CT vasa recta resulting
•• Strangulation—Closed loop obstruction with bowel in Comb sign of
wall ischemia. Seen as Crohn’s disease
(yellow arrows).
 Halo/Target sign of bowelQ: Thickened bowel
Ascites (blue
wall with increased attenuation on NC-CT arrow) is also seen
due to the bowel
Large Bowel Obstruction inflammation
ƒƒ Colonic cancer is most common causeQ, followed by acute
sigmoid diverticulitis
ƒƒ X-ray findings:
•• Gaseous distention of colon (>5–6 cmQ for most of the
colon, for cecum it has to be >9 cmQ)
•• Peripherally locatedQ bowel loops—“Picture frame of
large bowel” Q
•• Few, long air fluid levels Conventional
•• Cut-off signQ: Colon proximal to the point of obstruction Fistulogram
is dilated (due to obstruction), while that distal to it is showing a
collapsed (as it empties through the rectum) tract extending
•• HaustrationsQ—transverse incomplete wall folds—very from the skin
different from the Valvulae seen in small bowel. up to the
ƒƒ CT: anal canal –
Perianal fistula
•• Similar findings as seen on X-rays with a definite point
of transition
•• Cause of obstruction (mass/diverticulitis) seen at the
point of transition.
  Clinical Pearls
Rigler’s Triad in Gallstone Ileus
• Gall stone ileus occurs due to impaction of a large gall stone,
following its passage into the bowel through a fistula between
CO Small versus Large Bowel Obstruction
NC E P T

Small and Large Bowel Abnormalities


GB and small bowel usually the duodenum
Let us revise: • Most common site of impaction and mechanical obstruction—is
the terminal ileumQ.
Imaging Small bowel Large bowel • Rigler’s triadQ:
Feature obstruction obstruction ƒƒ Pneumobilia
ƒƒ Small bowel obstruction
Dilatation of Diameter usually Diameter >5 cmQ ƒƒ Ectopic gallstone—usually in the RIF
bowel loops 3–5 cmQ
Location of Centrally locatedQ Peripherally locatedQ Inflammatory Bowel Disease: Crohn’s Disease/
dilated loops loops loops
“Picture of small “Picture frame of large
Regional Enteritis
bowel” bowel” ƒƒ Progressive, chronic, transmuralQ bowel wall inflammation
that may involve the entire bowel—Terminal ileum is most
Bowel wall Valvulae HaustrationsQ –
common siteQ
markings conniventesQ – Incomplete wall folds
extend completely ƒƒ Skip lesionsQ + Discrete ulcersQ—diagnostic of Crohn’s
across the lumen disease
ƒƒ Barium Meal Follow Through (BMFT)/Double Contrast
Air fluid levels Multiple, short Few, long fluid levels Barium Enema Imaging findings:
fluid levels •• Aphthoid ulcersQ (Longitudinal or Transverse) -
Gas in large Usually absent Gaseous distension characteristic feature. Seen as focal collections of barium
bowel proximal to site of with a surrounding halo due to mucosal edema. Deeper
obstruction ulcers may lead to formation of sinuses and fistulae
•• Cobble stone appearanceQ: Caused by combination

141
of longitudinal and transverse ulceration, with normal ƒƒ Double Contrast Barium Enema Findings:
islands of intervening mucosa. •• Fine granular appearance of mucosa—earliest findingQ
•• String sign of KantorQ: Narrowing of bowel due to •• Mucosal ulcers—Button-hole appearance of ulcersQ/
spasm/stricture formation Undercut T-shaped ulcersQ/Collar stud ulcersQ
CONCEPTUAL REVIEW OF RADIOLOGY

•• Raspberry thorn ulcersQ •• Pseudo-polyp appearanceQ—Most of the mucosa is


•• Pseudodiverticulum formation lost with few small islands of normal mucosa remaining
ƒƒ CE-CT Findings: •• Lead pipe signQ: Complete loss of haustral wall
•• Bowel wall thickening and enhancement seen with markings resulting in a featureless and pipe like
Creeping FatQ- fibrofatty proliferation of mesentery/Fat cylindrical appearance of colon. Refer to Image 26 in
wrapping. “Imaging Signs in Radiology”
•• Comb signQ: Hypervascular appearance of mesentery ƒƒ CE-CT Appearance:
seen due to fibro-fatty proliferation and perivascular
•• Target appearance of bowel wallQ: Due to mural
inflammatory changes. Linear hyperdensities are seen
stratification created by varying densities of alternating
on the mesenteric side of the bowel—appears like a
wall layers
comb.
ƒƒ Toxic Megacolon: (AIIMS MAY 2017)
•• Fat Halo signQ: Excessive sub-mucosal fat deposition
between the mucosa and muscularis propria. •• Acute fulminant colitis with neuromuscular
•• Bowel strictures/sinuses/fistulae with abscess degeneration and extensive colonic dilatationQ– most
formation commonly involving transverse colonQ.
•• Seen as a complication of inflammatory bowel
Inflammatory Bowel Disease: Ulcerative disease—more commonly seen in Ulcerative colitis
Colitis/Backwash Ileitis •• Dilatation of transverse colon >6 cmQ associated with
loss of haustral markings, thumb-printing, pseudo-
ƒƒ Predominantly involves the colonQ.
polyps extending into lumen.

  Clinical Pearls
Crohn’s Disease versus Ulcerative Colitis—A Clinico-patho-radiological Comparison
Feature Crohn’s disease Ulcerative colitis
Gender predilection None Male predilectionQ
Gross blood/mucus in stools Rarely Commonly seen
Bowel involvement Small bowel – Terminal ileum - most
Q
Colonic involvement – RectumQ – always
Small and Large Bowel Abnormalities

commonly involved involved


Distribution Skip lesionsQ common Continuous involvement
Depth of inflammation Transmural involvementQ Mucosa and sub-mucosa involvement
Granulomas Non-caseating granulomas seen None
ASCA/ANCA ASCA positive Q
p-ANCA positiveQ
Mesenteric Creeping fat Commonly seenQ Not seen
Abscess formation Commonly seen Q
Not seen
Perianal involvement Commonly seen Rarely seen
Extraintestinal complications Pancreatitis Primary sclerosing cholangitis
Gall stones
Liver abscess
Toxic Megacolon Seen but rare Commonly seenQ
Stricture formation CommonQ Less common
Cobblestone appearance CommonQ Less common
Smoking Smoking predicts a worse outcome, Has a protective effectQ
requiring frequent surgery
Risk of Malignancy Less More

142
Acute Mesenteric Ischemia •• Thrombus may be demonstrated in the SMA/IMA/
Mesenteric veins
ƒƒ Presents with severe abdominal pain, bloody diarrhoea
ƒƒ CE-CT/CT Angiography: Investigation of choiceQ Familial Adenomatous Polyposis (FAP)

GASTROINTESTINAL TRACT IMAGING


•• Thrombus is seen as intraluminal filling defect within
an artery/vein ƒƒ Most common of all polyposis syndromesQ. Colonoscopy is
•• Affected bowel may show wall thickening with IOCQ.
diminished/delayed/absent enhancement ƒƒ Double Contrast Barium Enema (DCBE) appearance of
•• PneumatosisQ—free intramural air in bowel wall—is polyps:
late finding and suggests irreversible disease •• Filling defect: A sessile polyp usually forms a filling
ƒƒ Invasive catheter angiography—Gold standard investiga- defect in the contrast column
tionQ •• Meniscus appearanceQ: Thin layer of contrast covering
the mucosa forms a ring at the base of the polyp
Ischemic Colitis •• Ring ShadowQ: En-face visualization creates a ring
shadow with a sharp inner margin while the outer
ƒƒ Plain radiograph:
margin fades into the surrounding normal mucosal
•• Dilated bowel with “Thumb-printing”Q—seen due to
surface
mucosal edema/hemorrhage
•• Mexican Hat signQ: When viewed obliquely the
•• Pneumatosis coli—Intramural air followed by meniscus sign and ring shadow seen together
Pneumoperitoneum—if perforation has occurred
resemble a hat (Refer to Image 25 in "Imaging Signs
•• Stack of coin appearanceQ—may be seen in chronic in Radiology")
cases
•• Stalk signQ: Seen in pedunculated polyp. The stalk of
ƒƒ CE-CT:
the polyp is outlined by 2 parallel lines of barium, wider
•• Bowel wall thickening seen with Target signQ: Seen at its base as the stalk merges with the normal mucosa
due to low attenuation edematous submucosa
•• Target signQ: A peduculated polyp seen en-face the
•• Enhancement variable: Absent/Hyper-enhancement head of the polyp and the stalk will superimpose and
may be seen in “Shock bowel” due to slow circulation
form a target appearance
•• Pneumoperitoneum/Pneumatosis coli/Portal venous
gas

Barium enema Barium enema


shows polyp with showing a polyp
Mexican hat
appearance

Small and Large Bowel Abnormalities


Barium enema: CECT:
apple core Asymmetric
appearance of enhancing
carcinoma wall thickening
of rectum
(white arrow)
suggestive of
malignancy

143
Diverticulitis
ƒƒ Sigmoid colonQ—most common site
Coronal CECT: ƒƒ RectumQ—least common site
CONCEPTUAL REVIEW OF RADIOLOGY

air filled ƒƒ DCBE:


outpouchings •• Seen as single/multiple barium lined outpouchings
(diverticuli) •• Bowler hat appearanceQ is seen when viewed en-face
with adjacent •• Saw tooth appearanceQ – JIPMER MAY 2017 pattern
inflammatory is seen in diverticulitis Refer to Image 22 in “Imaging
changes Signs in Radiology”
suggestive of •• Champagne glass sign
diverticulitis ƒƒ CE-CT: IOC for Diverticular disease
•• Diverticular perforation (extravasation of air/
intraluminal contrast) and surroundings abscess seen.
•• Arrowhead signQ: Seen on CE-CT with rectal
Colorectal Cancer contrast—Arrowhead shaped collection of contrast
ƒƒ DCBE findings: Depend upon the luminal surface between thickened folds of colon (except caecum).
characteristics of lesion
•• ShoulderingQ: Seen typically in annular/semi-annular Sigmoid Volvulus- Discussed below
lesions that cause sudden, severe constriction of the Intussusception
lumen ƒƒ Common in <2 years age—peak incidence between 5 and 9
•• Apple core signQ/Napkin ring signQ: Seen in an monthsQ—correlates to the start of weaning
annular lesion with irregular overhanging edges and ƒƒ Ileo-colic intussusceptionQ—Most common type in
shouldered margins. Most commonly seen in the children. Colo-colicQ – Most common type in adults
Sigmoid colon. Not usually seen in the cecum. Refer to ƒƒ Dance’s signQ: RUQ mass (intussusception) with empty
Image 21 in “Imaging Signs in Radiology” RLQ (cecum moving out of this area)
ƒƒ CE-CT/CT Colonography : IOC for Colorectal cancer ƒƒ Plain radiograph:
•• Demonstrates luminal characteristics + Mural wall •• May be normal
thickening + Extramural extension/findings •• Target signQ—Fat density alternating with soft tissue
•• Mass lesion along colonic wall with usually density
homogenous enhancement with extramural spread/ ƒƒ USG:
local invasion into adjacent organs. •• Investigation of choice in pediatric age groupQ
•• Shows lymph node enlargement (>10 mmQ— •• Sandwich appearanceQ—in longitudinal orientation
considered abnormal) •• Target signQ/Doughnut signQ/Bulls eye signQ—in
•• Shows metastatic deposits (liverQ– most common site), transverse orientation
Small and Large Bowel Abnormalities

ascites, omental caking, etc. •• Pseudo-kidney signQ—The central mesentery and


ƒƒ Ultrasound: vessels resemble the renal medulla whereas the
•• May be used to screen for liver metastasis edematous bowel wall resembles the renal cortex
•• Endorectal UltrasoundQ: Provides better visualization •• Crescent-in-doughnut appearanceQ—apex of the
of wall layers, hence may be used to asses depth of intussusceptions in transverse plane, with the trailing
invasion assessment and selecting T1N0 cases for local mesentery forming the crescent
excisionQ ƒƒ CT:
ƒƒ PET-CT : Overall best for staging of Colorectal Ca. •• Investigation of choice in adultsQ- can identify the
lead point responsible for intussusception
•• Sausage signQ—longitudinal orientation
  Clinical Pearls •• Target sign/Bowel-within-BowelQ —transverse
Right Sided versus Left Sided Colorectal Cancer orientation
• Right side colorectal cancer ƒƒ Reduction of intussusception:
ƒƒ Ulcerative growths that bleed •• Pneumatic reduction using air under fluoroscopy
ƒƒ Iron deficiency anemia associated guidance—preferred methodQ
ƒƒ Mass + •• Hydrostatic reduction using barium/water soluble
ƒƒ Liquid stools – obstruction presents late contrast/water under fluoroscopy or USG is also used
• Left side colorectal cancer •• Coiled spring appearanceQ(NEET 2020 pattern)/Claw
ƒƒ Annular/Constricting/Napkin ring growths
signQ—seen on Barium/water soluble contrast enema
ƒƒ Solid stools – Obstruction is an early feature
Refer to Image 24 in “Imaging Signs in Radiology”
ƒƒ Altered bowel habits
•• Absolute contraindications for reduction—Perforation
ƒƒ Apple core deformity seen
with pneumoperitoneumQ and SepticemiaQ

144
Axial CECT: BMFT: barium
bowel within filled normal
bowel sign appendix

GASTROINTESTINAL TRACT IMAGING


of ileocolic (black arrow)
intussusception

CECT:
inflamed
appendix
USG: blind (red arrow)
ending inflamed with adjacent
structure in RIF abscess
suggestive of formation
acute appendicitis (black arrow)

Acute Appendicitis •• The radionuclide is concentrated in the gastric


mucosa—that in the stomach as well as that in the
ƒƒ History clincher:
Meckel’s thus localizing it
•• Child with fever
•• Pre-procedure Ranitidine/Cimetidine, administration
•• Pain in abdomen, vomiting
—increases sensitivity
•• Pain at tenderness in RIF at McBurney’s point
ƒƒ USG:
•• Investigation of choice—in childrenQ. Initial/1st   Clinical Pearls
investigation—in adults and in pregnant patientsQ
Meckel’s Diverticulum Secifics
•• Gradual graded compression techniqueQ is used, with

Small and Large Bowel Abnormalities


Rule of 2Q:
a high frequency linear transducer
• 2 inches long
•• Seen as a—Tubular, blind ended, aperistaltic, non- • 2 feet from Ileocecal junction
compressible structure measuring >6–7 mm in • 2% population affected
diameterQ. May be filled with fluid/pus with or without • 20% have ectopic mucosa (gastric mucosa > pancreatic)
an appendicolith • GI bleedingQ – MC presentation in children
•• Surrounding free fluid—may suggest perforation • Bowel obstructionQ – MC presentation in adults
•• Adjacent mesentery may appear echogenic/inflamed • Littre’s herniaQ – Meckel’s diverticulum in the hernia sac
with enlarged lymph nodes
ƒƒ CT:
•• Investigation of choice in adultsQ Perianal Fistulas/Fistula-in-Ano – MRI
•• Cecal bar signQ: Inflammatory soft tissue at the base of Fistulography is IOC. Imperforate Anus
appendix, separating the appendix and the cecum
ƒƒ Invertogram:
•• Arrowhead signQ: The positive contrast material in
•• A coin is placed at the site of expected anal opening.
the caecal lumen assumes the shape of an arrowhead
The baby is inverted and held so for around 3–5 minutes
pointing towards the appendix
and a radiograph is obtained. The distance between
the gas lucency and the coin may help in estimating the
Meckel’s Diverticulum type:
ƒƒ USG—may show a blind ended peristaltic loop connected  Distance <2 cm—Low type
to the ileum  Distance >2 cm—High type
ƒƒ CT—limited in uncomplicated cases. ƒƒ Transperineal USG:
� 99m
Tc pertechnetate scan: •• Can also help in determining type by measuring distance
•• Most sensitive for diagnosisQ between the skin and the point of air in the rectum.
145
•• <15 mm—Low type
•• >15 mm—High type   Clinical Pearls
Gasless AbdomenQ (Recent Pattern Jan 2019)
Few Peculiar Bowel Related Signs
CONCEPTUAL REVIEW OF RADIOLOGY

GASLESS ABDOMEN ON AN ABDOMINAL RADIOGRAPH


ƒƒ Moulage signQ:
•• Seen in Coeliac diseaseQ/SprueQ Etiology Cause/pathophysiology
•• Due to a markedly dilated jejunal loop with loss of wall Pancreatitis • Vomiting–main mechanism
markings—appears as if wax is poured in a tube • Diarrhea
ƒƒ Hide bound appearanceQ: Ascites • Central displacement of bowel loops
•• Seen in SclerodermaQ • Collapsed bowel in tense ascites
•• Narrow separation between valvulae conniventes in a Small bowel • Proximal high obstruction–at pylorus,
dilated bowel loop obstruction and duodenum, jejunum
ƒƒ Stacked coin appearanceQ: ischemia • Fluid filled distended bowel loops
•• Another term for Hide bound appearance seen in (instead of air filled loops)
Scleroderma
Acute • Vomiting and diarrhoea
ƒƒ Picket fence appearanceQ: gastroenteritis
•• Seen in Whipple’s diseaseQ and AmyloidosisQ
•• Due to markedly thickened bowel wall and nodular Large abdominal • Displacement and compression of
mass bowel loops
deposits
ƒƒ Aneurysmal dilatation of bowelQ: Total colectomy • Exact mechanism unknown. Possibly
•• Seen in LymphomaQ rapid transit time and absence of
ƒƒ Accordion signQ : colonic gas
•• Pseudomembranous colitis/Clostridium difficile Clinical pearls related to “GASLESS ABDOMEN”
associated disease - Seen on CT as a result of gross wall • Most serious condition causing gasless abdomen: Small
thickening with trapped hyperdense oral contrast in the bowel obstruction with/without ischemia
lumen/mucosal hyperenhancement and hypodense • IOC/next best investigation for gasless abdomen: CE-CT
submucosal edema. Characteristic but not specific for • MC cause of gasless abdomen in adult = Ascites
this disease.

Invertogram: Absence Radiograph:


of gas in rectum, showing foreign
suggests imperforate body in stomach
anus which may get
lodged at IC
Small and Large Bowel Abnormalities

junction causing
obstruction.

Axial CECT:
twisting of
mesenteric vessels
giving Whirlpool
sign, suggestive of
midgut volvulus

146
3
CLINICAL QUIZ - SOLUTION

GASTROINTESTINAL TRACT IMAGING


Sigmoid Volvulus—Most Common Site of Volvulus in GIT Q
ƒƒ Frimann-Dahl signQ: 3 dense lines (sigmoid walls) are seen
• The examiner has given us a history clincher here - converging at the site of twist
ƒƒ Elderly female ƒƒ Northern exposure signQ: A dilated sigmoid colon rises to
ƒƒ Known case of mental handicap/institutionalization the level of transverse colon
ƒƒ Massive gaseous distension of abdomen ƒƒ Air: Fluid ratio in the loop is usually more than 2:1
• Plain radiograph: Usually diagnostic— ƒƒ Contrast enema—“Bird of prey sign”Q—smooth curved
ƒƒ Massively dilated gas filled colonic loop tapering of lumen at the site of torsion
ƒƒ Loss of haustrationsQ (In Caecal volvulus – Haustrations • CT Findings:
are maintainedQ) ƒƒ The above findings are confirmed.
ƒƒ Inverted U shaped loopQ/Coffee bean signQ/Kidney bean ƒƒ Whirl signQ—suggests twisting of the mesentery and
signQ/Bent inner tube signQ/Omega signQ Refer to Image mesenteric vessels
27 in “Imaging Signs in Radiology” ƒƒ Birds beak signQ—rectal contrast shows tapering of bowel
ƒƒ Apex is above the T10 level, usually under the left hemidi- loops
aphragm ƒƒ X-marks-the-spot-signQ—crossing of bowel loops at the site
ƒƒ Liver overlap signQ/Left flank overlap signQ/Pelvis overlap of torsion
signQ ƒƒ Split wall signQ—Folding of sigmoid wall by partial twisting

Hepatobiliary and Pancreatic Imaging


4
CLINICAL QUIZ
6. A 35-year-old female patient presented with fever, severe pain in the
epigastrium and right hypochondriac region. A HIDA scan was done with
non-visualization of GB as shown here. Most likely diagnosis is:
(NEET 2017 Pattern)
A. Acute pancreatitis C. Biliary stricture

Hepatobiliary and Pancreatic Imaging


B. Acute cholecystitis D. Acute gastritis
7. On the HIDA scan in the above patient, increased uptake of the isotope
was seen in the liver parenchyma adjacent to the GB fossa (yellow arrow).
This sign is known as:
A. Rim sign C. Reactive uptake sign
B. Halo sign D. Para GB sign

CO Multiphasic CT/MR Imaging/Triple Phase Hepatic Imaging


NC E P T
• CT/MR with contrast is done to bring out differential enhancement properties of lesions so that they appear different from the normal
liver parenchyma and hence can be easily seen.
• This based on the understanding the Dual blood supply to the liver parenchyma

Hepatic • Very minimal supply to the normal liver parenchyma • Selective contrast within the Hepatic artery will result
artery occurs from the hepatic artery in enhancement of HCC/Hemangioma
• Supplies a Hepatocellular carcinoma/Hemangioma, etc.
Portal • Major blood supply to normal liver parenchyma comes • So when the contrast is predominantly within the
Vein from the portal vein Portal vein – the normal liver parenchyma will enhance

Contd…
147
• So we make use of this understanding of Hepatic DUAL BLOOD SUPPLY and after contrast injection intravenously, acquire images at
particular time intervals as follows

Multiphasic Hepatic Imaging


CONCEPTUAL REVIEW OF RADIOLOGY

Time from Phase Contrast is in Imaging properties


Contrast
injection
30–40 secs Arterial Hepatic artery Normal liver parenchyma will not enhance
phase HCC/Hemangioma will enhance – Arterial phase enhancement

60–70 secs Portal Portal vein Normal liver parenchyma will enhance
venous HCC–Contrast will be washed out – will appear hypodense compared to liver
phase Hemangioma–Progressive filling of lesion with contrast due to blood pooling
5–10 min Delayed Hepatic veins/IVC/Filtered Hepatic venous pathology better seen
phase into renal pelvicalyceal Ureteric/Bladder lesions seen creating a CT Urography like lesion
system & ureter
• Please note that the timings mentioned here are approximate values and in fact every institute will have its own specific protocol and
timings may vary. I have used average values here.

FOCAL LIVER LESIONS   Clinical Pearls


Pneumobilia versus Portal Venous Gas
Air in Liver Both of these structures being tubular branching structures may
ƒƒ It can be either be Portal venous airQ or Pneumobilia (air be identical. The following points will help distinguish between
in biliary tree)Q. It is very subtle finding on radiographs and them:
ultrasound but CT scan is imaging modality of choice.Q Pneumobilia Portal venous gas
ƒƒ PneumobiliaQ: Causes include -
• Large confluence of air in • Peripheral distribution of
•• Spontaneous biliary-enteric fistulaQ—Most common central portion of liver air specks
non-iatrogenic cause
•• Recently passed out biliary calculus • Remember that Bile flows • Remember that blood
from the liver parenchyma flows from the porta to
•• Endoscopic retrograde cholangiopancreatography
towards the porta so the liver periphery, so
(ERCP) guided Sphincterotomy (at ampulla of Vater)
Hepatobiliary and Pancreatic Imaging

the air will be restricted it carries the air specks


•• Post Roux-en-Y surgery
towards the porta distally with it. So most air
ƒƒ Portal vein gasQ: Causes include -
specks are usually seen
•• Mesenteric/Bowel ischemiaQ—Most common cause within 2 cm from the liver
•• Necrotizing enterocolitis surface/capsule
•• Crohn’s disease
•• Volvulus • CBD air - Central • Portal vein air-Peripheral

USG: small shrunken USG image shows


liver, echogenic a well-defined
parenchyma (red anechoic (jet
arrow), nodular black) lesion with
margins (yellow thin imperceptible
arrow), dilated walls in the liver –
portal vein (white suggests a Simple
arrow) suggests hepatic cyst
cirrhosis

148
USG Image showing a Radiograph:
hypoechoic lesion in Air in bile
liver with thick internal ducts

GASTROINTESTINAL TRACT IMAGING


echoes and septae – (green
with a history of fever arrow) and
– is suggestive of a liver gallbladder
abscess. Compare and (red arrow)
differentiate between suggests
the image of Simple pneumobilia
cyst (Anechoic – Jet
black) and Abscess
(Hypoechoic with
thick internal echoes/
contents)

Liver Abscess •• May show air within abscess cavity


ƒƒ May be Pyogenic or Amoebic. •• Double target signQ—Seen on contrast CT images
ƒƒ USG—1st investigationQ and also the Imaging modality of  Central hypodense area—liquefied pus
choice for Follow-up to assess liquefactionQ once diagnosis  High attenuation inner rim—abscess membrane—
has been established and for USG guided aspirationQ enhances early and persists
ƒƒ CE - CT—Best investigationQ  High attenuation outer rim—edematous liver
ƒƒ USG— parenchyma—enhances only on delayed phase
•• Appears as relatively well defined hypoechoic lesion •• Cluster signQ - Multiple smaller hypodense lesions
with thick internal echoesQ coalesce together to form a larger abscess cavity. Seen
•• Appears iso—mildly hypoechoic in non-liquefied stage. in pyogenic liver abscessesQ
•• As it liquefies—appears progressively hypoechoic ƒƒ MRI - Hypointense on T1, hyperintense on T2 with
ƒƒ CE - CT— enhancing walls and central pus may show diffusion
•• Investigation of choiceQ restriction (DWI)Q.
•• Well defined hypodense lesion with thick enhancing
walls

Hepatobiliary and Pancreatic Imaging


Contrast CT Axial CECT:
(look at contrast peripherally
in aorta) shows enhancing thick
liver lesion walled hypodense
(black arrows) lesion (black arrow),
with peripheral suggests liver
puddling of abscess
contrast,
suggests
Hemangioma

USG showing
echogenic
hemangioma
(Black arrow) 149
Hepatocellular Carcinoma (HCC)/Hepatoma  Neuroendocrine tumors (Carcinoid, Islet cell
tumors)
ƒƒ Most common malignant liver lesionQ
 Choriocarcinoma
ƒƒ Any focal lesion seen in a cirrhotic liver should be considered  Melanoma
CONCEPTUAL REVIEW OF RADIOLOGY

malignant unless proven otherwise


ƒƒ On USG they appear as ill-defined hypoechoic area, Other Important Liver Focal Lesions
however if there is baseline liver parenchymal disease then
identification becomes difficult. Liver • USG—Well-defined hyperechoic (white)
ƒƒ CT- Hemangioma lesionsQ relative to liver parenchyma.
•• Hypodense irregular lesion on NC-CT • CT scan—
•• Contrast imaging ƒƒ Show discontinuous, peripheral, nodular
 Arterial phase—Enhancement due hepatic arterial enhancement on arterial phase with
supplyQ progressive centripetal (from periphery
 Portal venous phase—Iso/hypodense to liver due towards centre) on portal and venous
to wash-out of contrast phases.
 Delayed phase—Capsule/fibrous components ƒƒ Bright dot signQ—Bright dot seen within a
may enhance hemangioma seen on Arterial and Portal
venous phases of CT
 Portal vein thrombus—Arterial enhancement
• MRI-
within the portal vein thrombusQ with vein
ƒƒ Light bulb signQ - On increasing T2
diameter >23 mmQ
weighting of the images it appears
 Enucleation signQ—Seen in an intraperitoneally
progressively hyperintense (bright)
ruptured HCC • 99mTc—RBC SPECT scanQ - There is progres-
ƒƒ MRI findings are similar to CT scan additionally there is sive pooling of RBCsQ seen within the lesion
some role for MRI to differentiate between regenerative,
siderotic and dysplastic nodulesQ. Focal Nodular • Benign lesion containing a central scarQ and
Hyperplasia Kupffer cellsQ—reticuloendothelial cells
ƒƒ SPIO enhanced MRIQ—Imaging gold standard for diagnosis
(FNH)Q ƒƒ Comet Tail appearance—seen on Colour
of HCC
Doppler as diffuse vascularity throughout
ƒƒ On Angiography a Streaks and threads like appearanceQ
the lesion.
is seen.
• Superparamagnetic Iron Oxide (SPIO) MRIQ-
ƒƒ LI-RADSQ: Liver Imaging—Reporting and Data system is ƒƒ SPIO is a unique MRI contrast medium
used. that undergoes selective phagocytosis by
reticuloendothelial system cellsQ (Kupffer
Metastases cells)Q
ƒƒ Most commonly organ involved by metastasesQ ƒƒ Uptake causes hypointense appearance on
Hepatobiliary and Pancreatic Imaging

ƒƒ Most common primary resulting in Liver metastasis—Colon T2W imagesQ


carcinomaQ ƒƒ Specific for FNH—shows around 60–70%
ƒƒ USG— signal loss on T2W imagesQ. Hepatic
•• Most commonly multiple hypoechoic lesionsQ seen, adenomas show only 15–20% signal loss.
but may be variable • 99mTc—Sulphur colloid scanQ—Shows
Sulphur colloid uptake due to presence of
•• Bull’s eyeQ/Target appearanceQ may be seen
Kupffer cells
•• Calcified liver metastasisQ seen in—
 Mucinous metastasis from ovary/colon Fibrolamellar • Solitary, lobulated, well-defined/
ƒƒ CT— CarcinomaQ encapsulatedQ tumor containing
•• Most metastasis are hypovascularQ hence appear ƒƒ Central fibrous scarQ &
hypodense to normal liver on portal venous phase ƒƒ Central stellate calcificationQ
•• Hypervascular liver metastasisQ occur from— • Normal Serum Alfa fetoprotein (AFP)
 RCC • Scar of Fibrolamellar carcinoma: is
 Thyroid cancer hypointense on T2W imagesQ as compared
to FNH in which scar is hyperintense on
T2W imagesQ
CECT: focal
heterogeneously
enhancing hypodense   Clinical Pearls
lesion (red arrows)
Hepatic Focal Lesions with a Central Scar
with central calcified
• Focal nodular hyperplasia (FNH)
scar (white arrow).
• Fibrolamellar hepatocellular carcinoma
FLC on Histopathology
• Cholangiocarcinoma
• Hemangioma
• HCC
150 • Hepatic adenoma-rarely
PATHOLOGIES OF GALLBLADDER ducts are seen giving beaded appearanceQ. Eventually
cirrhosisQ develops
Inflammatory Conditions ƒƒ Left lobe is also atrophicQ in PSC, differentiating it from
cirrhosis from other causes in which left lobe is usually

GASTROINTESTINAL TRACT IMAGING


Gallbladder Calculi/Cholelithiasis hypertrophied.
Mixed stonesQ—Most common type followed by Cholesterol
ƒƒ
Choledochal Cysts
and Pigment stones
ƒƒ X-ray/CT — Most of them (nearly 90%) are radio lucent ƒƒ Todani classification of Choledochal cystsQ:
(mostly cholesterol or pigment stones). When radio opaque •• Type I - Fusiform cystic dilatation of extrahepatic CBD
(mostly calcium) they show faceted appearanceQ and are •• Type II—Eccentric fluid filled cyst/diverticulum
laminatedQ/lamellatedQ •• Type III—Localized cystic dilatation of distal intramural
ƒƒ Mercedes Benz signQ—Star shaped pattern of air fissuring segment of CBD—CholedochoceleQ
within a gall stone, seen on a X-ray •• Type IVA—Multiple intrahepatic & extrahepatic bile
ƒƒ USGQ—Investigation of choice/Gold standard. Appear as duct cysts
echogenic mobile foci showing post acoustic shadowing. •• Type IVB—Multiple extrahepatic bile duct cysts
ƒƒ Rolling stone signQ—Movement of stones, upon change in •• Type V—Multifocal saccular dilatations of IHBR—
patient position Caroli’s diseaseQ – shows Central dot signQ on MRI.
ƒƒ MRI/MRCP-Stones appear as filling defects (black spots) ƒƒ MRCP/ERCP is best investigationQ
against background of hyperintense fluid intensity of bile.

Acute Cholecystitis – Discussed below


Porcelain GallbladderQ
ƒƒ Uncommon condition of mural calcification on GB wall -
due to chronic cholecystitis.
ƒƒ On radiograph there is calcification which follows the
contour of the gallbladder wall.
ƒƒ High association between Porcelain GB and GB Adenocar-
cinoma—Cholecystectomy is usual treatmentQ

Pathologies of Bile Ducts


Primary Sclerosing CholangitisQ
ƒƒ Associated with - Ulcerative colitisQ.

Hepatobiliary and Pancreatic Imaging


ƒƒ On MRI/MRCP multiple alternating strictures and biliary
dilatation seen involving both intra and extra hepatic bile

CECT: Axial CECT:


heterogeneously multiple variable
enhancing sized enhancing
hepatic lesion hypodense
(red arrows) lesion scattered
with non in liver (black
enhancing arrows), suggests
necrotic areas metastases
within it,
suggests HCC

USG: Echoreflective
GB calculus (red
arrow) with CECT:
posterior acoustic heterogeneously
shadowing enhancing mass
(white arrow) and (black arrow)
thickened walls within gallbladder
(yellow arrow) (red arrows),
favouring suggestive of
cholecystitis carcinoma
151
MRCP – Main
pancreatic duct
directly draining
CONCEPTUAL REVIEW OF RADIOLOGY

into minor papilla


(blue arrow), while
CBD and ventral
duct drain into
ampulla – suggests
Pancretic divisum

CE-CT Abdomen image-


Bulky heterogeneously
enhancing Pancreas
(yellow arrows) with
Coronal MRI: Focal dilatation of extrahepatic CBD with multiple fuzzy margins & fluid
hypointense calculi in it, suggests Type I Choledochal cyst density collections along
its anterior surface (blue
arrows) – suggests Acute
Cholangiocarcinoma pancreatitis
ƒƒ Presents as obstructive jaundice
ƒƒ Classified according to location:
•• Intrahepatic/Peripheral Cholangiocarcinoma –
 Delayed enhancing lesion in liver, causing liver
capsule retraction
•• Hilar Cholangiocarcinoma/Klatskin tumor— Radiograph of the
 Bismuth classificationQ is used for grading abdomen showing
•• Extrahepatic cholangiocarcinoma—Most common type chunky calcification
 Biliary dilatation with abrupt cut-off, without a in epigastrium
which is crossing
discrete mass
the midline,
ƒƒ Periampullary carcinomaQ—Tumors arising from/1 cm
Hepatobiliary and Pancreatic Imaging

suggests chronic
within the papilla of Vater. Double duct signQ on MRCP –
pancreatitis
due to dilatation of both common bile duct and pancreatic
duct. Refer to Image 20 in “Imaging Signs in Radiology”

CONGENITAL ANOMALIES OF Acute Pancreatitis


PANCREAS ƒƒ
ƒƒ
Most common causes: Gall stonesQ and AlcoholismQ
Abdominal radiograph—Non-specific
Pancreatic DivisumQ •• Sentinel loop signQ - dilated duodenal loop due to
local ileusQ
ƒƒ Results from a failure of ventral and dorsal bud fusion. •• Colon cutoff signQ - abrupt end of the gas column in
ƒƒ The ventral (Wirsung) ductQ drains only the ventral transverse colon.Q
pancreatic anlage, whereas the majority of the gland empties ƒƒ USG—Limited assessment of pancreas. May reveal
into the minor papilla through the dorsal (Santorini) ductQ an edematous hypoechoic appearing pancreas with
which can lead to recurrent pancreatitisQ. peripancreatic fluid and ascites
ƒƒ MRCP is investigation of choiceQ and ERCP is nowadays ƒƒ CE—CTQ—Investigation of choiceQ
only used for confirmation or intervention. •• Diffusely enlarged pancreasQ with fuzzy marginsQ with
peripancreatic fat stranding/fluid collections
Annular PancreasQ •• Uniform enhancement of pancreatic parenchyma seen
ƒƒ Incomplete rotation of the ventral anlage leads to a segment in early/mild cases
of the pancreas encircling the second part of duodenum •• Pancreatic necrosisQ is suggested by –
causing intestinal obstruction.  Non-enhancing areas in pancreasQ
ƒƒ Abdominal radiographs reveal the classic Double-bubble  Loss of normal glandular texture
signQ  Fragmentation/liquefaction of pancreas
152
•• Pseudocyst formationQ—develop >4 weeks after an •• Reverse 3 sign of Frostberg/Frostberg inverted
acute attack 3 signQ—distortion and effacement of mucosal
 Fluid collection (usually pancreatic secretions) markings on medial wall of 2nd part of duodenum
lined by a wall of fibrous/granulation tissue ƒƒ USG - can identify hypoechoic lesion.

GASTROINTESTINAL TRACT IMAGING


 Seen as well-defined hypodense fluid collections ƒƒ CE- CTQ - is Imaging modality of choice.
around pancreas without any enhancement •• Well defined lesion that enhances lessQ than the
ƒƒ Scoring of severity of pancreatitis: normal pancreas parenchyma, most commonly in
•• Balthazar Grading systemQ—has prognostic implica- head of pancreasQ
tions •• Teardrop signQ - it is an alteration of the superior
•• CT Severity Index (CTSI)Q—Based on combination of mesenteric vein from its normal round shape to a
Balthazar grading and Pancreatic necrosis. Correlates teardrop shape on axial images secondary to tumor
better with morbidity and mortality infiltration or peritumoral fibrosis

Chronic Calcific Pancreatitis Cystic Pancreatic Neoplasms


ƒƒ Abdominal radiographs & CT—Pancreatic calcification—
represent intraductal calculi & parenchymal calcifications Serous Cystadenoma
ƒƒ USG— ƒƒ Also known as Grandmother lesionQ of pancreas as it affects
•• Atrophic pancreas with multiple calcifications later in the 6th decade.Q
•• Pancreatic duct dilatation with intraductal calculiQ– ƒƒ Appearance wise on imaging it is microcystic or honey-
Most specific combed cyst with central stellate calcified scarQ
ƒƒ MRCP/ERCP—Best for evaluation of pancreatic duct •• Cluster of grapes appearanceQ
abnormalities. ERCP is better but is invasive. •• Swiss cheese appearanceQ
•• Chain of lakes appearanceQ •• Honey-comb appearanceQ
•• Duct penetrating signQ—non-obstructed main pancre- ƒƒ The septations are enhancing however there is no
atic duct seen penetrating an inflammatory pancreatic communication with pancreatic duct.
mass (it is compressed/obstructed in cases of neoplasm) ƒƒ MRIQ - imaging modality of choice as it can easily identify
the small cysts and also rule out duct communication.
Pancreatic Solid Neoplasms
Mucinous Cystadenoma
Carcinoma Pancreas
ƒƒ Also known as Mother lesionQ (4th to 5th decade)
ƒƒ AdenocarcinomaQ–Most common pancreatic neoplasm. ƒƒ MacrocysticQ—large multiloculated cystic lesionQ with thick
ƒƒ Radiograph/Barium meal follow-through (BMFT): wall septations and peripheral calcificationsQ
•• Widening of C loop of duodenumQ

Hepatobiliary and Pancreatic Imaging


CE-CT abdomen - CECT: enhancing
Well defined fluid pancreatic
density lesion neoplasm (red
in the region arrow) with
of pancreas – central non
following an enhancing
attack of acute necrosis. HPR-
pancreatitis Neuroendocrine
– Pancreatic tumor
pseudocyst

T tube
cholangiogram MRI:
(red arrows) Heterogeneous
shows free enhancing
passage of pancreatic
contrast into neoplasm
duodenum (green (red arrow).
arrow) HPR-carcinoma

153
4
CLINICAL QUIZ - SOLUTION
• Non-visualization of GB on a HIDA scan is suggestive of Acute • Non-visualization of GB on HIDA scanQ - NEET 2017 pattern.
CONCEPTUAL REVIEW OF RADIOLOGY

cholecystitis (Answer is B) • This is because there is a calculus impacted at the GB neck/cystic


• Imaging in Acute Cholecystitis: duct, that prevents HIDA from entering the GB–hence it is not
• USGQ—Investigation of choiceQ seen.
ƒƒ Distended/Over-distended GB with thickened/edematous • Rim signQ - Increased uptake of the 99mTc-HIDA agent in the liver
walls and pericholecystic collection. adjacent to the gallbladder fossa due to reactive hyperemia, is a
ƒƒ Impacted stone/sludge may be seen at GB neck/cystic duct useful ancillary finding seen in patient’s with acute cholecystitis
ƒƒ Sonographic Murphy’s signQ - Focal probe tenderness in right (Thus answer to Quiz question 2 is A. Rim sign)
hypochondriac region • CT—
• HIDA scan— was investigation of choice in older days. ƒƒ May show above imaging features with GB wall enhancement,
• HIDAQ—stands for Hepatic useQ of Imino-Di-Acetic acid (–IDA) peri-GB fat stranding/enhancement.
compounds. ƒƒ Emphysematous cholecystitisQ - seen in elderly diabetic males
• Most common clinical indication for performing Cholescintigra- and diagnosis can be made on X-ray/CT scan where intramural
phyQ air outlining the gallbladder can be demonstrated.

Miscellaneous Abdominal Imaging


5
CLINICAL QUIZ
8. A patient with abdominal trauma was brought to the Emergency Room.
IV fluid were started and the patient was found to be hemodynamically
stable. The next step in the management of this patient would be:
(NEET 2020 Pattern)
A. E-FAST C. Laparotomy
B. CE-CT D. CXR PA view
9. Following image of FAST in a trauma patient suggests:
(AIIMS May 2019 Pattern)
A. Positive FAST C. Equivocal FAST
B. Negative FAST D. None of the above
Miscellaneous Abdominal Imaging

FAST
Q
It stands for Focused Assessment with Sonography in Trauma
ƒƒ Aim – Bedside procedure to identify presence of free fluid (hemoperitoneum) which is an important factor that decides further
surgical verses non surgical management.

CO FAST – Is It Really That Fast?


NC E P T
• Remember we have studied a concept before highlighting how
• “Fluid is Friend of Ultrasound and Air is Enemy of Ultrasound”
• USG is the BEST modality for detection of minimal fluid anywhere in the body.
• FAST – is done to identify free fluid in the abdomen of an abdominal trauma patient. Can be done within seconds.
• If free fluid is seen then it is presumed to be Hemoperitoneum, most likely secondary to a solid organ injury (Most common being
Spleen).
• Then depending upon patients clinical and hemodynamic status further decisions are made
Contd…

154
GASTROINTESTINAL TRACT IMAGING
• Thus the role of Imaging in abdominal trauma is:
ƒƒ FAST: Screening for free fluid/hemoperitoneumQ
ƒƒ CE-CT Abdomen is Investigation of choiceQ

Standard FAST ProtocolQ Imaging Features


ƒƒ Abdominal radiography: A curvilinear calcification maybe
seen in prevertebral regions.
ƒƒ Ultrasound:
•• Usually the 1st investigation doneQ – aneurysms
detected incidentally
ƒƒ CT scan:
•• CE-CT (arterial phases) is investigation of choiceQ
•• Diameter less than 5.5 cmQ which is not enlarging
rapidly and is asymptomatic requires - Regular follow-
up on ultrasound
ƒƒ Signs of impending rupture on CT scan include:
•• Draped aorta signQ—seen in Contained ruptureQ. The
posterior wall of the aorta either is not identifiable as

ƒƒ Subxiphoid transverse viewQ: To assess for pericardial Sagittal CECT:


Saccular abdominal

Miscellaneous Abdominal Imaging


effusion and left lobe liver injuries
ƒƒ Longitudinal view of the right upper quadrantQ: To assess aortic aneurysm
for right lobe of liver injuries, right kidney injury, and fluid (red arrow)
in Morrison’s pouch
ƒƒ Longitudinal view of the left upper quadrantQ: To assess for
splenic injury and left kidney injury and fluid in spleno-
renal pouch
ƒƒ Transverse and longitudinal views of the suprapubic regionQ:
To assess the bladder, pelvic organs and pouch of Douglas.

Extended FAST (eFAST) ProtocolQ


ƒƒ Includes additional scanning of the right and left thoracic
wings/intercostal regions to look for hemo/pneumothorax.

Axial CECT:
bowel (red
ABDOMINAL AORTIC arrow) and
ANEURYSM (AAA) fat/omentum
(white arrow)
ƒƒ Focal dilatations of the abdominal aorta that are 50% greater containing
than the proximal normal segmentQ or >3 cmQ in maximum inguinal hernia
diameter.

155
distinct from adjacent structures or when it closely •• Vessel/Intestine embedded signQ
follows the contour of adjacent vertebral bodies. •• Floating aorta signQ—Aorta displaced anteriorly away
•• High attenuation crescent signQ - Represents an acute from the vertebral column by the lymph nodal mass
hematoma within either the mural thrombus or the •• Sandwich/Hamburger signQ—Fat/vascular structures
CONCEPTUAL REVIEW OF RADIOLOGY

aneurysmal wall. This sign is strongly associated with embedded in the large lymph nodal masses
aneurysm rupture. •• Cobblestone signQ—Diffuse, round, non-confluent,
•• Focal discontinuity in the curvilinear wall calcificationQ enlarged lymph nodes with homogeneous density and
•• Tangential calcium signQ - The intimal calcification clear margins
points away from the aneurysm ƒƒ Gastrointestinal involvement is typical and on barium
ƒƒ Signs of frank rupture on CT scanQ: studies can show nodular thickening of the walls or there
•• Extravasation of contrastQ can be aneurysmal focal dilatation of the affected bowelQ
•• Retroperitoneal hematomaQ if the lymphoma invades the GI neural plexus.
•• Perilesional fat strandingQ
ƒƒ Invasive angiography – is Gold standard investigationQ: Midgut Volvulus
Endovascular coiling of aneurysm can be done. ƒƒ Occurs as a complication if intestinal malrotation,
presenting typically in 1st month of life
ƒƒ X-ray—Rarely complete obstruction may give rise to a
ABDOMINAL LYMPHOMA Double bubble sign
ƒƒ CE-CT scan is imaging modality of choiceQ. If available PET- ƒƒ Contrast fluoroscopy:
CT is considered better than CE-CTQ •• Corkscrew signQ—Corkscrew configuration of
ƒƒ On CT scan the nodes appear as enlarged, oval to round duodenum and jejunum twisted on a shortened small
structures showing mild homogenous post contrast bowel mesentery
enhancement. ƒƒ USG:
ƒƒ There is associated encasement of the tubular structures •• Whirlpool signQ—Clockwise whirling of mesentery
giving rise to numerous named signs. and SMV around the SMA
•• Inverted SMA-SMV relationshipQ

CECT: Non CECT:


enhancing pseudoaneurysm
splenic (red arrow)
hypodensity arising from
extending splenic artery
to surface (blue arrow)
(red arrows)
suggests splenic
Miscellaneous Abdominal Imaging

laceration

Well-defined mildly homogeneously enhancing lymph nodal mass encasing the aorta
and mesenteric vessels—forming the Sandwich sign/Hamburger sign of Lymphoma
156
5
CLINICAL QUIZ - SOLUTION
• So we have discussed the Management protocol in an abdominal trauma patient. The patient in the Quiz no 1 is a case of abdominal

GASTROINTESTINAL TRACT IMAGING


trauma and has been stabilized hemodynamically. So the 1st step in the management of this patient would be to do a E-FAST to
check whether there is any fluid in the peritoneal cavity. Hence the Answer is A. E-FAST.
• As you can see the USG image a thin rim of fluid (appearing hypoechoic – Black) is seen in the hepatorenal pouch – suggests free fluid
in the peritoneal cavity hence – Positive FAST (Answer is A.). In this patient further management will depend upon hemodynamic
status. If the patient is stable – we may do a CE-CT abdomen. If hemodynamically unstable – Emergency laparotomy is indicated.

Multiple Choice Questions


Peritoneal Abnormalities 8. Cupola sign is: (Jharkhand 2006)
A. Radiological finding in supine posture for pneumoperi-
1. Best radiographic projection, in case of suspected
perforation is:  (Jharkhand 2003) toneum
A. Erect abdomen B. Radiological finding in supine posture for pneumothorax
B. Erect chest C. Air in Morrison’s pouch
C. Lateral decubitus abdomen D. Both walls of bowel is seen
D. Supine abdomen
2. The best view for visualization of minimal pneumo­
Imaging in Foregut
peritoneum is: (AI 2012) 9. Bird beak appearance is seen in: (CET July 2016)
A. X-ray chest erect A. Achalasia cardia B. Carcinoma esophagus
B. Left lateral decubitus abdomen
C. Esophageal varices D. Esophagus foreign body
C. Right lateral decubitus abdomen
D. X-ray erect abdomen 10. Corkscrew esophagus is seen in:  (AI Dec 2013)
A. Chagas disease B. Esophageal trauma
3. Most sensitive investigation for detection of pneumo­
C. Diffuse esophageal spasm D. Esophageal diverticulum
peritoneum is: (AI 2015)
A. USG B. CT 11. Windsock appearance on barium is seen in:
C. X-ray chest lateral D. X-ray chest PA view A. Duodenal perforation
4. Minimal ascites can be best detected by: (AIIMS 1998) B. Duodenal ulcer
A. USG B. X-ray abdomen erect C. Brunner gland hyperplasia
C. CT D. MRI D. Internal duodenal diverticulum
5. Which of the following statement(s) is/are true regarding 12. Hampton’s line on barium is seen in:
X-ray abdomen? (PGI Nov 2017) A. Gastric lymphoma B. Gastric ulcer
A. Gas under diaphragm—seen in perforated gastric ulcer C. Gastric carcinoma D. Antral gastritis
B. Rigler sign-seen in acute peritonitis with pneumoperito- 13. Radiating folds falling short of the barium collection is seen
neum in:

Multiple Choice Questions


C. Differentiation of small and large bowel obstruction can A. Gastric carcinoma B. Gastric lymphoma
be done by location of distended bowel loop C. Gastric ulcer D. Linitis plastica
D. Large intestine- shows taenia coli and haustration 14. Diagnosis of Barrett’s esophagus is best done with:
E. Valvulae conniventes are circular folds reach around the A. Radionuclide study B. Barium study
whole circumference of the large intestine C. Endoscopy and biopsy D. MRI
6. Dog’s ear appearance of bladder is seen in: 15. All are true about gastrointestinal stromal tumors (GISTs)
A. Pneumoperitoneum except:
B. Ascites A. The benign and malignant counterparts are indistin-
C. Pseudomyxoma peritonei guishable from each other
D. UB rupture B. Size >5 cm favor malignant lesion
7. Which of the following feature is characteristic of C. In stomach, the commonest is leiomyosarcoma
pseudomyxoma peritonei? D. Ulceration is uncommonly seen
A. Gross ascites 16. A neonate presented on day 1 of life with bilious vomiting.
B. Internal septae What investigation will you first: (AIIMS Nov 2015)
C. Scalloping of liver/spleen A. Ultrasound B. X-ray Abdomen
D. Intense enhancement C. Chest radiograph D. CT scan

157
17. Carman’s meniscus sign is diagnostic of:  (AI Dec 2014) 30. Saw tooth appearance on barium enema is seen in:
A. Peptic ulcer B. Cholecystitis  (AI 2014/JIPMER 2017)
C. Meconium ileus D. Carcinoma of stomach A. Diverticulosis B. Sigmoid volvulus
CONCEPTUAL REVIEW OF RADIOLOGY

18. Trifoliate appearance is seen in: (CET Nov 2014) C. Carcinoma colon D. Ulcerative colitis
A. Peptic ulcer 31. Hat sign on DCBE is seen in:  (MH 2009)
B. Pyloric stenosis A. Gastric ulcer B. Polyp
C. Carcinoma head of pancreas C. Carcinoma D. Diverticulitis
D. Periampullary carcinoma 32. Investigation of choice for small bowel tumor is:
19. X-ray feature of pyloric stenosis is: (AIIMS 1997) A. Barium meal follow through (JIPMER 1998)
A. Single bubble appearance B. USG
B. Double bubble appearance C. X-ray erect abdomen
C. Triple bubble appearance D. CE-CT
D. Multiple air fluid levels 33. Contrast used in Barium enema is:  (JIPMER 2003)
20. String sign is seen in: A. Barium oxide B. Barium sulfate
A. Crohn’s disease C. Barium chloride D. Barium sulfide
B. TB of the ileocecal region 34. Best investigation of acute appendicitis in children is:
C. Idiopathic hypertrophic pyloric stenosis  AIIMS May 2015)
D. All of the above A. X-ray abdomen B. USG
21. Following are common features of malignant gastric ulcer C. CT scan D. Barium enema
on barium meal, except: 35. Investigation of choice for diagnosis of intussusception in
A. Location on the greater curvature children is: (AIIMS Nov 2015)
B. Carman’s meniscus sign A. Barium enema B. USG
C. Radiating folds which do not reach the edge of the ulcer C. CE-CT D. MRI
D. Lesser curvature ulcer with a nodular rim 36. Gardner’s syndrome is characterized by:
A. Intestinal polyposis B. Bony osteomas
Small and Large Bowel Abnormalities C. Desmoid tumors D. All of the above
22. Barium study for small bowel is known as: (AI 2013) 37. An 85-year-old woman has a 48-hour history of generalized
A. Barium swallow abdominal pain and vomiting. On examination, she is
B. Barium enema dehydrated multiple dilated small bowel loops measuring
C. Barium meal follow-through up to 4 cm in diameter. A linear gas-filled structure is
D. Barium meal present in the right upper quadrant with short branches
extending from it. What is the most likely diagnosis?
23. Earliest sign of Ulcerative colitis on DCBE:  (AI 2014)
A. Acute mesenteric ischemia
A. Mucosal granularity B. Loss of haustrations
B. Emphysematous cholecystitis
C. Lead pipe colon D. Collar button ulcer
C. Gallstone ileus
24. Barium meal follow-through study can diagnose a:
D. Obstructed right inguinal hernia
A. Colonic stricture B. Ileal stricture (AI 2015)
C. Rectal stricture D. Esophageal stricture 38. Midgut volvulus is characterized by all except:
A. SMA located to left of SMV
25. Lead pipe appearance on DCBE is seen in: (AI 2014)
B. Corkscrew pattern of duodenum and jejunum
A. Crohn’ s disease B. Ulcerative colitis
C. Whirlpool sign is seen on US
Multiple Choice Questions

C. Schistosomiasis D. Carcinoma colon


D. DJ junction is seen to the right side of the pedicle of the
26. String sign may be seen in:  (CET Nov 2014)
vertebrae
A. Crohn’s disease
39. Apple core appearance is seen in:
B. Hypertrophic pyloric stenosis
C. Iliocecal TB A. Carcinoma esophagus B. Carcinoma colon
D. All of the above C. Sigmoid volvulous D. Diverticulosis
27. String of Beads appearance on an abdominal radiograph is 40. Claw sign on x-ray is seen in: (PGI May 2018)
seen in:  (AI 2013) A. Ischemic colitis B. Intussusception
A. Small bowel obstruction B. Large bowel obstruction C. Sigmoid volvulus D. Crohn’s disease
C. Carcinoma stomach D. Gastric outlet obstruction 41. Feathery appearance of jejunum is due to presence of:
28. Coffee Bean sign is seen in:  (AI 2014)  (PGI 1999)
A. Cecal volvulus B. Sigmoid volvulus A. Valvulae conniventes B. Haustrations
C. Intussusception D. CHPS C. Luminal gas D. Vascular pattern
29. Radiological feature of Acute Ischemic colitis is: 42. ‘Hat sign’ on double contrast barium enema is seen in:
 (CET July 2016)  (MH 2009)
A. Saw tooth appearance B. Thumb-printing A. Ulcer B. Polyp
158 C. Cobble stone appearance D. Stacked coin appearance C. Carcinoma D. Diverticulum
43. The following are radiological features of sigmoid volvulus 57. Non-visualization of GB on HIDA scan is suggestive of:
except:  (Recent Pattern 2017)
A. Inverted U-shaped bowel loop A. Acute cholecystitis B. Adenomyomatosis

GASTROINTESTINAL TRACT IMAGING


B. Liver overlap sign C. Carcinoma GB D. Cholelithiasis
C. Bird of prey 58. Best investigation for a diagnosis of Hydatid cyst in liver:
D. Cupola sign A. USG B. CE-CT (CET July 2016)
44. Lead pipe appearance is seen in: (AI Dec 14) C. MRI D.v MRCP
A. Crohn’s disease B. Ulcerative colitis 59. ‘Starry sky appearance’ of liver on USG is seen in:
C. Schistosomiasis D. Carcinoma colon A. Steatosis B. Acute viral hepatitis
45. Capsule endoscopy is used for: (NEET Pattern Dec 2012) C. Passive hepatic congestion D. Lymphoma
A. GI Bleeding B. Motility disorders 60. Which of the following is false regarding Primary Sclerosing
C. GERD D. None of the above cholangitis?
46. Which among the following is false regarding small bowel A. Show beaded appearance of biliary tree
appearance on abdominal radiograph? B. CBD is almost always involved
A. Valvulae conniventes are present C. More common in females
B. Peripheral distribution D. Caudate lobe goes for hypertrophy
C. Radius of curvature is small 61. Investigation of choice in GB calculi: (AI Dec 2013)
D. Solid faces are absent A. CT B. PET scan
47. Soap bubble appearance in X-ray is seen in: (AI Dec 2015) C. USG D. HIDA scan
A. Multiple cystic kidney B. Neuroblastoma 62. Investigation of choice for gallstone:
C. Cystic lymphangiectasis D. Meconium ileus A. X-ray B. USG
C. Cholecystography D. CT scan
Imaging in Hepatobiliary System 63. Investigation of choice in obstructive jaundice is:
48. Investigation of choice for acute calculous cholecystitis is: A. ERCP B. USG
A. Ultrasound B. Erect abdomen X-ray C. Cholecystography D. Laparoscopy
C. CT scan D. MRI 64. Focal and diffuse thickening of gallbladder wall with high
49. Peripheral puddling of contrast followed by centripetal amplitude reflections and ‘comet tail’ artifacts on USG
enhancement is hallmark of: suggest the diagnosis of: (AI 2009)
A. Hemangioma B. HCC A. Xanthogranulomatous cholecystitis
C. Metastases D. Carcinoid B. Carcinoma of gallbladder
50. Budd-chiari syndrome is seen in: C. Adenomyomatosis
A. Inferior vena cava web B. Portal vein thrombosis D. Cholesterolosis
C. Liver cirrhosis D. NCPF 65. Solitary an-echoic lesion of the liver without septate or
51. Which imaging sign is seen in gall stones: (AI Dec 2015) debris is most likely to be:
A. Honey comb sign B. Mercedes Benz sign A. Hydatid cyst B. Caroli’s disease
C. Signet ring sign D. Scotty dog sign C. Liver abscess D. Simple cyst

52. Predominant delayed enhancement is a feature of: 66. A 22-year-old man presents with a solitary 2 cm space-
occupying lesion of mixed echogenicity in the right lobe
A. HCC B. FNH
of liver on ultrasound examination. The rest of the liver is

Multiple Choice Questions


C. Fibrolamellar HCC D. Cholangiocarcinoma
normal. Which of the following tests should be done next?
53. Triangular cord sign in a neonate is seen in: A. Ultrasound-guided biopsy of the lesion
 (NEET Pattern Dec 2012) B. Hepatic scintigraphy
A. Galactosemia B. Hepatitis C. Hepatic angiography
C. Biliary atresia D. Hemangioblastoma D. Contrast-enhanced CT scan of the liver
54. Which is not sign of liver cirrhosis: 67. Which one of the following hepatic lesions can be
A. Nodular liver B. Hepatomegaly diagnosed with high accuracy by using nuclear imaging?
C. Caudate lobe hypertrophy D. Dilatation of portal vein A. Hepatocellular carcinoma
55. Central dot sign is seen in: (CET July 2015) B. Hepatic adenoma
A. Caroli disease B. Hepatic adenoma C. Focal nodular hyperplasia
C. Hepatic hydatid cyst D. Hepatic abscess D. Cholangiocarcinoma
56. Which is not radiographic feature of liver abscess:
A. Raised right hemidiaphragm
Pancreatic Imaging
B. Thick walled lesion 68. Imaging modality of choice for pancreatic divisum is:
C. Peripheral enhancement A. Endoscopic USG B. MRCP
D. Hyperintense on T1 C. ERCP D. 3 D CT scan
159
69. Cluster of grapes appearance is seen in: 83. CT findings of acute pancreatitis are all except:
A. Mucinous cystadenoma pancreas A. Dilation of pancreatic duct
B. Serous cystadenoma pancreas B. Fuzzy outline of pancreas
CONCEPTUAL REVIEW OF RADIOLOGY

C. Carcinoma pancreas C. Peripancreatic fluid collection


D. IPMT
D. Edematous pancreas
70. Which is not imaging criteria for pancreatitis?
A. Balthazar score Miscellaneous Abdominal Imaging
B. APACHE Score 84. Extended FAST technique includes:
C. CT severity index A. Thoracic wings B. Iliac fossae
D. Modified CT severity index
C. Perineum D. Pericardium
71. Which is not feature of chronic pancreatitis?
85. On imaging the cut-off diameter for diagnosis of abdominal
A. Dilated pancreatic duct
aortic aneurysm is:
B. Pancreatic calcification
C. Peripancreatic fat stranding A. 3 cm B. 3.5 cm
D. Pancreatic atrophy C. 4 cm D. 4.5 cm
72. Reverse 3 sign is seen in: (MH 2001) 86. Draped aorta sign suggests:
A. Carcinoma pancreas B. Pancreatitis A. Contained rupture B. Frank rupture
C. Pancreatic pseudocyst D. Pancreatic duct injury C. Aortic dissection D. Intramural thrombus
73. Sentinel loop sign is a feature of: 87. Floating aorta sign is seen in:
A. Acute pancreatitis B. Chronic pancreatitis A. Aortic rupture B. Aortic dissection
C. Carcinoma pancreas D. Pancreatic trauma C. Retroperitoneal fibrosis D. Abdominal lymphoma
74. Double duct sign is seen in: 88. Hernia containing Meckel’s diverticulum is:
A. Pancreatitis B. Biliary calculi A. Hernia of Winslow B. Littre hernia
C. Periampullary carcinoma D. Insulinoma
C. Maydl's hernia D. Pantaloon hernia
75. Which radionucleotide is used in pancreatic neuroendocrine
89. Signs of impending aortic aneurysm rupture:
tumor imaging?
A. 110In octreotide B. 111In octreotide A. Draping aorta sign
C. 111In somatostatin D. 110In somatostatin B. Hyperattenuating crescent sign
76. Gasless abdomen is seen in: (Recent Pattern 2019) C. Tangential calcification sign
A. Intussuception B. Pancreatitis D. Any of the above
C. Ulcerative colitis D. Polyps 90. Aortic diameter less than—cm at the level of the renal
77. On CT, Balthazar grading is for: arteries should raise the suspicion of hypovolemic shock.
A. Acute pancreatitis B. Cholecystitis A. 12 mm B. 15 mm
C. Pancreatic carcinoma D. Chronic pancreatitis C. 20 mm D. 25 mm
78. Widening of the C loop in X-ray is diagnostic of: 91. Majority of the abdominal aortic aneurysm are– in location.
A. Chronic pancreatitis A. Suprarenal B. Infrarenal
B. Carcinoma head of pancreas C. Renal D. Iliac
C. Periampullary carcinoma
92. Which of the following neoplasm is inhomogeneous and
D. Calculi in the ampulla of Vater
irregular with some fat density on CT?
79. ‘Spongy appearance’ with central sunburst calcification is
A. Lipoma B. Lymphangioma
seen in: (AIIMS May 2007)
Multiple Choice Questions

C. Liposarcoma D. Neurofibroma
A. Pancreatic adenocarcinoma
B. Mucinous cyst adenocarcinomas 93. Best diagnostic imaging tool for retroperitoneal lymphoma:
C. Somatostatinoma A. IVU B. MRI
D. Serous cystadenoma C. PET-CT D. Ultrasound
80. Most sensitive investigation for pancreatic carcinoma is: 94. Dependent viscera sign is a feature of:
A. Angiography B. ERCP A. Diaphragmatic palsy
C. Ultrasound D. CT scan B. Eventration of diaphragm
81. A patient complains of epigastric pain, radiating to back off C. Diaphragmatic rupture
and on. The investigation of choice is: (AIIMS May 1999) D. Diaphragmatic hump
A. MRI B. CT scan 95. ‘Sandwich sign’ is positive in:
C. USG D. Radionuclide scan A. Carcinoid
82. Chain-of-lakes appearance ERCP is seen in: (CET Nov 2014) B. Pancreatic pseudo cyst
A. Acute pancreatitis B. Chronic pancreatitis C. Lymphoma
C. Carcinoma pancreas D. Ductal adenoma D. Mesothelioma

160
Answers
1. B 13. A 25. B 37. C 49. A 61. C 73. A 85. A
2. A 14. C 26. D 38. A 50. A 62. B 74. C 86. A

GASTROINTESTINAL TRACT IMAGING


3. B 15. D 27. A 39. B 51. B 63. A 75. B 87. D
4. A 16. B 28. B 40. B 52. D 64. C 76. B 88. B
5. A,B,C,D 17. D 29. B 41. A 53. C 65. D 77. A 89. D
6. B 18. A 30. A 42. B 54. B 66. D 78. B 90. A
7. C 19. A 31. B 43. D 55. A 67. C 79. D 91. B
8. A 20. D 32. D 44. B 56. D 68. B 80. D 92. C
9. A 21. D 33. B 45. A 57. A 69. B 81. B 93. C
10. C 22. C 34. B 46. B 58. B 70. B 82. B 94. C
11. D 23. A 35. B 47. D 59. B 71. C 83. A 95. C
12. B 24. B 36. D 48. A 60. C 72. A 84. A

Explanations to Questions
11. Windsock appearanceQ is intraduodenal barium contrast-
filled sac that is surrounded by a narrow lucent line seen in 60. PSC-young to middle-aged male are most frequently affected.
intraduodenal diverticulumQ. Cholangiographic findings in PSC depend on the stage of the
14. Barrett’s esophagus is squamocolumnar metaplasia and disease process. Early in the course of the disease, randomly
hence diagnosed on biopsy only. distributed, short (1–2 mm), annular intrahepatic strictures
16. Bilious vomiting on Day 1 of life – cannot be hypertrophic alternating with normal or slightly dilated segments produce a
pyloric stenosis – as it presents with non-bilious vomiting beaded appearance. Strictures usually occur at the bifurcation
and presents after a few weeks after birth. The implicated of ducts and are out of proportion to upstream ductal
diagnosis in this patient is likely Duodenal atresia – which dilatation. As the fibrotic process worsens, strictures increase
may be achieved by doing a X-ray abdomen – showing a and the ducts become obliterated, and the peripheral ducts
Double bubble appearance. USG will not show the atretic cannot be visualized to the periphery of the liver, producing
segment and CT would lead to a very large radiation exposure. a “pruned tree” appearance. The key features of PSC are
Confirmation would be intra-operative. randomly distributed annular strictures out of proportion to
37. Small bowel obstruction in an elderly patient with evidence upstream dilatation. Other findings include webs, diverticula,
of pneumobilia is suggestive of Gallstone ileusQ. Imaging and stones. End result is cirrhosis characterized by a markedly
features of gall stone ileus are: Pneumobilia which appears distorted biliary tract with atrophy of the entire liver with
as central branching air lucencies in the liver, small bowel the exception of the caudate lobe which is hypertrophied in
obstruction and radiopaque density in the abdomen (most almost all cases. Atrophy involving the left lobe is a feature
common site of impaction is the terminal ileum). which somewhat distinguishes it from cirrhosis from other
40. Claw signQ is seen in Intussusception. It is the appearance of causes, in which the left lobe is usually hypertrophied
dilated loop of bowel which appears like a claw of a bird. It is 64. Features of adenomyomatosis are:

Explanations to Questions
described in both plain abdominal films as well as on barium Ultrasound: Mural thickening (diffuse, focal, annular),
studies segmental/annular form, comet-tail artefact: echogenic
45. Common indications for capsule endoscopy are: Obscure intramural foci from which emanate V-shaped comet
gastrointestinal bleeding, Crohn’s disease, polyps, recurrent tail reverberation artefacts are highly specific for
abdominal pain, celiac disease. adenomyomatosis, -are due to cholesterol crystals within the
53. The triangular cord signQ is a triangular or tubular lumen of Rokitansky-Aschoff sinuses
echogenic cord of fibrous tissue seen in the porta hepatis at CT: Abnormal gallbladder wall thickening. CT rosary sign
ultrasonography and is relatively specific for the diagnosis of has been described, formed by enhancing epithelium
biliary atresiaQ. It is useful in the evaluation of infants with within intramural diverticula surrounded by the relatively
cholestatic jaundice, helps in differentiating from neonatal unenhanced hypertrophied gallbladder muscularis.
hepatitis. MRCP/MRI: Mural thickening, focal sessile mass, fluid-
59. It refers to a USG appearance of bright echogenic dots filled intramural diverticula (Pearl necklace sign). Hourglass
throughout a background of decreased liver parenchymal configuration in annular types
echogenicity. This sign has been found to have poor sensitivity 66. The investigation of choice for any incidentally detected
and specificity. hepatic lesion is triple phase CT scan

161
75. Radionuclides used in pancreatic neuroendocrine tumor the appendix is found inflamed in an inguinal hernia.
imaging are: DeGarengeot Hernia is similar, in that, the appendix is
A. In-111 pentetreotide. found in a hernia. Rene Jacques Croissant de Garengeot
CONCEPTUAL REVIEW OF RADIOLOGY

B. In-111 octreotide. was an 18th century Parisian surgeon. He was the first to
C. I-131 metaiodobenzylguanidine (MIBG). describe the appendix in a femoral hernia.
D. I-123 MIBG. •• Hernia of Littre: The hernia of Littre is the presence of a
E. 2-[fluorine-18] fluoro-2-deoxy-d-glucose (FDG). Meckel’s diverticulum in a hernia. The common sites are
81. Clinically the patient has pancreatitis and hence should be inguinal (50%), umbilical (30%) and femoral (20%).
evaluated by CE-CT scan 90. The CT criteria for shock bowel are bowel-wall thickening
88. Some unusual hernias: greater than 3 mm and mucosal enhancement greater than
•• Spigelian hernia is a hernia through the Spigelian the psoas muscle. The finding of a collapsed IVC was defined
fascia, which is the aponeurotic layer between the as an anteroposterior diameter of less than 9 mm on at least
rectus abdominis muscle medially, and the semilunar three contiguous sections. A collapsed aorta was defined as
line laterally. These are generally interparietal hernias, an anteroposterior diameter less than 1.3 cm at the levels of
meaning that they do not lie below the subcutaneous 2 cm above and 2 cm below the renal arteries. CT findings
fat but penetrate between the muscles of the abdominal of shock pancreas were peripancreatic fluid and variable
wall; therefore, there is often no notable swelling pancreatic enhancement. Hypoenhancement of the spleen
•• Richter's Hernia: It can happen at multiple locations & and liver was assessed subjectively. Hypotension was defined
it involves only a portion of the bowel wall. The portion as a systolic blood pressure less than 90 mm Hg or a diastolic
that is stuck may become strangulated which can result in blood pressure less than 60 mm Hg.
perforation of the bowel at that site and contamination 92. Fat density of lipoma is homogeneous, lymphangioma has
of the abdomen. Seen more commonly these days at the fluid density and neurofibroma has soft tissue density.
sites of laparoscopic port insertions.
•• Amyand’s Hernia and DeGarengeot’s Hernia: This hernia is
named after the English surgeon who performed the first
successful appendectomy in 1735. In an Amyand's hernia,
Explanations to Questions

162
GENITOURINARY
TRACT IMAGING
C hapter O utline
• Urolithiasis Imaging • Prostate Imaging
• Renal Parenchymal Abnormalities Including Infections • Miscellaneous GUT
CONCEPTUAL REVIEW OF RADIOLOGY

• Renal Masses • Multiple Choice Questions with Explanations


• Congenital Anomalies of Genitourinary System

Urolithiasis Imaging
1
CLINICAL QUIZ
1. A young male presented with recurrent UTI and pain in abdomen. An X-ray
KUB was done and is revealed here. What is the most likely diagnosis?
(JIPMER May 2016 Pattern)
A. Cholelithiasis
B. Pyelonephritis
C. Staghorn calculus
D. Emphysematous pyelonephritis

2. The most likely composition of this calculus will be:


(AIIMS Nov 2015 Pattern)
A. Calcium oxalate calculus
B. Uric acid calculus
C. Triple phosphate calculus
D. Indinavir calculus

  Clinical Pearls

Type of calculus Peculiar feature


• Most common typeQ
• RadiopaqueQ
Calcium oxalate • May be associated with underlying metabolic disorders like – HypercalciuriaQ, HyperoxaluriaQ
• Dumb-bell shaped appearance on microscopyQ
• Jackstone calculusQ in UB – is a calcium oxalate calculus
• 2nd most common typeQ
Struvite/Triple • Seen in alkaline/infected urine
Urolithiasis Imaging

phosphate • These are magnesium ammonium phosphate (MAP) stonesQ


• Associated with UTI – Proteus – MC associated infectionQ
• Seen in association with recurrent UTIs occurring • MC type of a primary bladder calculus
due to Urease producing bacteriaQ – like Proteus, • Associated with
Klebsiella, Pseudomonas (Except E. coli) ƒƒ GoutQ (hyperuricemia, hyperuricosuria)
Uric acid • Most common cause of Staghorn calculiQ – these ƒƒ Acidic urine pH (<5.5)
fill up the cast of renal pelvis and calyces and thus ƒƒ Lesch Nyhan syndrome
resemble the horns of a stag ƒƒ Tumor lysis syndrome
• May be entirely radiolucentQ • Rasburicase is used for treatment in resistant casesQ
Cystine • Hardest calculiQ – hence cannot be treated with extracorporeal shock wave lithotripsy (ESWL)Q
• Associated with Xanthinuria
Xanthine
• Brick red colour stonesQ
164
X-ray – KUB - Ultrasound
Large discrete image showing
radiodensity in echoreflective

GENITOURINARY TRACT IMAGING


the right renal calculus in lower
fossa suggests a pole calyx of left
renal calculus kidney with a post
acoustic shadow

A delayed IVU image


shows a obstructive
Imaging Modalities left distal ureteric
calculus (yellow
Noncontrast Abdomen-Pelvis Helical CT scan - Investigation
arrow) causing
of choiceQ moderate left
ƒƒ Imaging Findings: hydroureter (blue
•• Hyperdense appearing calculus seen anywhere along arrow). The right
the urinary tract with/without hydronephrosis/ renal pelvicalyceal
hydroureter system and ureter
•• Stones not detected on NC-CTQ: Pure matrix calculiQ has completely
and Indinavir calculiQ drained its contrast
•• Soft tissue Rim signQ: A ureteric calculus appears as and hence is not well
a hyperdensity surrounded by a soft tissue rim – the seen in this image
edematous ureteric wall. Used to distinguish a ureteric
calculus from a phlebolithQ.
•• Dual Energy CT (DE-CT): Can determine of calculus
composition, Double J (DJ) Stents
USG - 1st investigationQ/Screening investigation in a renal colic
patient ƒƒ Placed in a retrograde manner during a cystoscopy done for
ƒƒ Considered investigation of choice for detection of management of a calculus/following an ESWL
Hydronephrosis/PyonephrosisQ (Fluid is Friend of ƒƒ Best method for stent position confirmation is—Plain
Ultrasound) radiographQ.
ƒƒ Imaging findings:
•• Calculi seen as hyperechoic foci with posterior acoustic SteinstrasseQ Refer to Image 35 in “Imaging
shadowing
Signs in Radiology”
•• Dilated pelvicalyceal system appears as black/
hypoechoic – as its fluid (urine) filled 1
Plain abdominal radiograph (Flat plate or X-ray KUB): CLINICAL QUIZ - SOLUTION
ƒƒ Screening investigation for renal colic - has poor sensitivity.
Look at the large radiodensity seen in the left renal fossa. It is
ƒƒ Imaging findings:
•• Radiodense foci seen in the renal fossa region, along evident now that this patient has renal calculi. Any calculus
the ureteric course or in pelvis. which first involves the pelvis and then also involves 2 or more Urolithiasis Imaging
IVP/IVU (Intravenous pyelography/urography): calyces is known as staghorn calculus/coral calculus. These are
ƒƒ Calculi seen as hyperdense foci prior to contrast injection most commonly composed of Struvite/magnesium ammonium
ƒƒ Following contrast injection—the pelvicalyceal system phosphate and are seen in the setting of recurrent urinary tract
proximal to the point of obstruction is filled with radio- infection with urease-producing bacteria (e.g., Proteus, Klebsiella,
opaque contrast – the degree of dilatation can be assessed Pseudomonas and Enterobacter). These are slow-growing stones
which later on turn painless and patient often presents with renal
failure.
  Clinical Pearls
So answer to both the questions is C. Staghorn calculus and
• MC cause of Hydronephrosis in adults – Renal/ureteric calculiQ
C. Triple phosphate calculus
• MC cause of Hydronephrosis in children – Congenital PUJ
obstructionQ

165
Renal Parenchymal Abnormalities
Including Infections
CONCEPTUAL REVIEW OF RADIOLOGY

2
CLINICAL QUIZ
3. A 59-year-old poorly controlled diabetic patient presented with pain in
abdomen and fever. X-ray was KUB done. What is your most probable
diagnosis?
A. Acute appendicitis
B. UTI
C. Emphysematous pyelonephritis
D. Acute cholecystitis
4. The IOC for diagnosis of this condition is:
A. X-ray KUB
B. USG
C. CE-CT
D. MRI
Renal Parenchymal Abnormalities Including Infections

Acute Pyelonephritis ƒƒ “Great Imitator”Q – as it often mimics and is misdiagnosed


as a tumor
ƒƒ IVU:
•• IVU - Calculus with non-filling of pelvicalyceal system
•• Dense, persistent or striated nephrogramQ
as a result of non-functioning parenchyma. USG/
ƒƒ USG:
CT may show round hypoechoic/hypodense areas
•• Enlarged kidney with diffusely edematous hypoechoic – suggest debris/xanthoma cavities surrounded by
cortex
thickened parenchyma. Perinephric inflammatory
ƒƒ CE-CT:
changes seen.
•• Best method for detection of parenchymal changesQ •• Bear’s Paw signQ: Seen on CE-CT.
and abscessesQ
•• Patchy enhancement, with band- and wedge-shaped Renal Parenchymal Disease/Medical
areas of decreased enhancement extending from the
papillae to the renal margin are seen. Perinephric fat Renal Disease
stranding, fluid rim may be seen. ƒƒ The patient presents clinically with elevated serum
creatinine/blood urea levels and is sent for imaging.
Emphysematous Pyelonephritis ƒƒ USG:
ƒƒ Renal infection with gas formation either within/around the •• Increased echogenicity of renal cortex with or
kidney seen in poorly controlled diabetics, Immunocom- without loss of corticomedullary differentiationQ (in
promised. Most common organism – E. coliQ comparison to adjacent liver echogenicity) – is the
ƒƒ Plain X-ray KUB/USG: May show streaky air in the renal most common findingQ.
fossa region or Crescentic air collection within Gerota’s •• Kidney sweat signQ
fascia with reverberation artifacts on USG.
ƒƒ CT: Renal Transplantation Imaging
•• Best diagnostic modalityQ ƒƒ Transplant kidney placement:
•• Enlarged kidneys with streaky gas/gas-fluid levels/free •• Placed extraperitoneallyQ in recipients Iliac fossaQ.
retroperitoneal air/fat stranding/collections •• Renal artery is anastomosed most commonly to External
iliac arteryQ (sometimes the internal iliac artery).
Xanthogranulomatous Pyelonephritis •• Renal vein – anastomosed to external iliac veinQ
ƒƒ Calculus disease (mostly staghorn calculus) with •• Ureter is implanted into UBQ
superimposed Proteus mirabilis/E. coli infection causing •• Baseline USG is done 2–3 days after transplant. Renal
chronic parenchymal inflammation. artery shows normal – low resistance flow with con-
tinuous forward flow in diastole – ‘Ski slope pattern’.Q
166
IVU film showing dense KUB radiograph
persistent nephrogram in showing mottled
left kidney (white arrow) air lucencies in right

GENITOURINARY TRACT IMAGING


due to obstructive mid- renal region (black
ureteric calculus (black arrows) suggestive of
arrow) right emphysematous
pyelonephritis

Ultrasound
image of

Renal Parenchymal Abnormalities Including Infections


kidney showing
anechoic simple
cyst (white
arrow)
Axial CECT scan shows obstructive right renal calculus
(black arrow) with resultant dense peripherally enhancing
collection in kidney (white arrow) suggesting pyonephrosis

Nephrocalcinosis/Albright’s Calcinosis ƒƒ Immediate dense persistent nephrogramQ:


•• Most commonly seen in Acute tubular necrosis
ƒƒ Calcium deposition in the renal parenchyma, outside the
•• Severe inflammatory renal disease
pelvicalyceal system ƒƒ Increasingly dense nephrogramQ:
ƒƒ X-ray KUB, USG, CT all may be used but NC-CT is Best •• Acute renal obstruction (calculus, urinary retention)
investigation. •• Hypotension
ƒƒ Cortical Nephrocalcinosis: •• Renal ischemia
•• Most common cause – Acute cortical necrosis (due to •• Acute renal vein thrombosis
any reason) •• Acute glomerular disease
•• “Single pencil-line”Q or “Double Tram-line”Q •• Renal intra-tubular obstruction
calcifications ƒƒ Striated nephrogramQ:
ƒƒ Medullary nephrocalcinosis: Most common type •• Variant of obstructive nephrogram
•• Due to elevated serum calcium – Hyperparathyroidism, •• Also seen in pyelonephritis, Trauma, Medullary cystic
Medullary sponge kidney (Paint brush calcificationQ) kidney disease, Polyarteritis nodosa
ƒƒ Rim nephrogram/Rim signQ:
Typical Nephrogram Patterns •• HydronephrosisQ
•• Acute complete renal artery occlusion
ƒƒ Immediate faint persistent nephrogramQ:
•• Renal vein thrombosis
•• Seen in chronic glomerular disease
•• Acute tubular necrosis

167
2
CLINICAL QUIZ - SOLUTION
A large streaky lucency is seen on this X-ray KUB in the right lumbar region – with a reniform shape – suggests gross collection of air in
CONCEPTUAL REVIEW OF RADIOLOGY

the renal parenchyma. Air in the kidney/calyces – is Emphysematous pyelonephritis. This diagnosis is aided by the history given – elderly
patient, with poorly controlled diabetes and fever. Thus the answer is C. Emphysematous pyelonephritis.
Remember that “Fluid is friend of Ultrasound – Air is enemy of ultrasound”. So the air here will obscure visualization on USG. Hence
CE-CT is the IOC (Answer is C. CE-CT) as the contrast images will outline the renal parenchymal enhancement, any abscess formation and
perinephric inflammatory changes.

Renal Masses
3
CLINICAL QUIZ
5. A 49-year-old man presented with gross painless hematuria with a
complex mass lesion in right kidney on USG. A CE-CT was done, is shown
here. Most likely diagnosis is: (AIIMS May 2017 Pattern)
A. Renal cyst
B. RCC
C. Oncocytoma
D. Metastasis
6. The IOC for renal vein/IVC invasion in this case will be:
A. USG + Doppler
B. CE-CT
C. MRI
D. Catheter angiography

Renal Cell Carcinoma/HypernephromaQ/ ƒƒ Spoke wheel pattern of vascularityQ & Lucent rim signQ
(peritumoral halo) - is seen on angiography
Grawitz TumorQ
ƒƒ Associated with von Hippel-Lindau disease (vHL)Q Renal Angiomyolipomas (AMLs)
ƒƒ Gross painless hematuriaQ – is most common presentation
ƒƒ Benign lesions comprising abnormal blood vessels – Smooth
ƒƒ Ultrasound: Variable echogenicity mass solid/cystic with
muscles – Fat. May lead to retroperitoneal hemorrhage.
internal vascularity on Doppler
ƒƒ Associated with
ƒƒ CE-CT: IOC for renal masses (Refer to Q. No. 46 in Image-
•• Tuberous sclerosisQ: Multiple AMLs at young age
Based Questions)
•• Neurofibromatosis type IQ, von Hippel-Lindau
•• On plain images – seen as soft tissue attenuation
syndromeQ
masses with/without calcification.
•• Autosomal Dominant polycystic kidney diseaseQ
•• Postcontrast images: Variable enhancement – usually
ƒƒ Imaging diagnosis is based on demonstration of
less than the adjacent normal renal cortex. Renal vein
macroscopic fat within a renal lesionQ
tumor thrombus maybe seen.
Renal Masses

ƒƒ Angiography: Well-defined hypervascular mass with Sun-


ƒƒ MRI: IOC for renal vein/IVC involvement
burst appearance of vesselsQ/WhorledQ and Onion-peel
•• Best for distinguishing between a Bland thrombus
appearanceQ of vessels.
versus Tumor thrombusQ

Renal Oncocytomas Urothelial Tumors: Transitional Cell


Carcinoma (TCC)
ƒƒ Associated with Tuberous sclerosisQ and Birt-Hogg-Dubé
syndromeQ ƒƒ Most common urothelial tumor is—Transitional cell
ƒƒ Large enhancing mass with a central non-enhancing carcinomaQ.
stellate scarQ – is a characteristic feature.
168
ƒƒ Bladder - is most common site of TCC. Imaging findings:
•• IVU/CT Urography:   Clinical Pearls
 Irregular filling defectsQ along the UB wall/rest of
the urinary tract MC renal malignancy Renal Cell CarcinomaQ

GENITOURINARY TRACT IMAGING


 Stipple signQ: Seen in a papillary tumor as the MC benign solid renal tumor Renal oncocytomaQ
contrast enters the lesion interstices MC pediatric renal tumor Wilms' tumorQ
•• CE- CT/MRI
MC urothelial malignancy Transitional Cell carcinomaQ
 Asymmetric mural wall thickening or
 Mass lesion projecting into bladder lumen or MC site for TCC Urinary bladderQ
outside it Renal mass associated with AngiomyolipomaQ
 May show mild enhancement Tuberous sclerosis
ƒƒ Renal pelvic/Ureteric TCC: CT classification system used Bosniak ClassificationQ
•• USG: for Complex renal cysts
 May be seen as hypoechoic masses along the renal
Wunderlich syndrome – Renal AngiomyolipomaQ
pelvis/ureteric wall or within a calyx
mass with retroperitoneal
 OncocalyxQ: Dilated tumor filled renal calyx
hemorrhage is associated with
•• On IVU/CT Urography:
 Phantom calyxQ: A calyx that does not fill with 1st group of lymph nodes Para-aortic LNsQ
contrast while rest of the pelvicalyceal system is involved in RCC
well filled with contrast. Most important prognostic Stage of tumorQ
 Goblet signQ/Champagne glass sign of UreterQ: factor for RCC
Appearance of ureteral dilatation below the site of
an intraluminal ureteral filling defect, best seen at
retrograde ureteropyelography.

Ultrasound shows USG pelvis image


a well-defined shows urine filled
cortical echogenic anechoic bladder with
renal lesion (white a polypoidal mass
arrow) suggestive of (white arrow) arising
Angiomyolipoma from the lateral wall
of UB - suggestive of
Carcinoma

Coronal Coronal reconstructed


reconstructed CECT CE-CT image showing a
image–Exophytic heterogeneously (yellow
heterogeneously arrow) enhancing lesion at
enhancing mass upper pole of right kidney
lesion at lower with a central non-enhancing
pole of right scar (blue arrow)–suggests
kidney–suggests Oncocytoma
RCC Renal Masses

Axial and Sagittal


reconstructed CECT image
show diffuse enhancing wall
thickening the urinary bladder
suggest Transitional cell
carcinoma

169
Wilms Tumor/NephroblastomaQ
ƒƒ USG – 1st Investigation. Shows large iso-hypoechoic mass arising from kidney with internal vascularity/cystic change with or
without renal vein/IVC involvement.
CONCEPTUAL REVIEW OF RADIOLOGY

ƒƒ CT – Heterogeneous soft-tissue density masses with rare areas of calcification and patchy enhancement
ƒƒ MRI – Investigation of choice for StagingQ. Also Best investigation for assessment of renal vein & IVC invasionQ
ƒƒ Metastasis most commonly occur to Lungs (most common)Q, liver and lymph nodes.

3
CLINICAL QUIZ - SOLUTION
A renal mass in an elderly patient with gross painless hematuria is RCC unless proven otherwise ! Another important clue here is that the
heterogeneously enhancing mass is extending into the IVC—another peculiar feature of RCC. (Thus answer is B. RCC)
Though CE-CT/PET-CT may be used for imaging/staging of RCC, MRI is the IOC for depiction of renal vein/IVC involvement (Answer is C.
MRI). Remember that renal vein/IVC involvement by RCC does not make it a contraindication for surgery !

Congenital Anomalies of
Genitourinary System
4
CLINICAL QUIZ
Congenital Anomalies of Genitourinary System

7. A 2-year-old female child presents with recurrent episodes of UTI.


Following an ultrasound an IVU study was done and is shown here. Most
likely diagnosis is: (NEET 2019 Pattern)
A. Ureteric cyst
B. VUR
C. Fish-hook ureter
D. Ureterocele
8. The appearance shown in the IVU image above is known as:
A. Cobra head appearance
B. Adder head appearance
C. Spring onion appearance
D. All of the above

Anomalies Related to Position


Ectopic Kidney • The most common sites for ectopic kidneys include pelvis, presacral space and iliac fossa. Rarely it
may lie in the thorax.Q
Crossed Renal Ectopia • Both kidneys lie on same side of abdomen.
• The ureter of the ectopic kidney always crosses midline and inserts on the urinary bladder on original
side.
• USG identifies it and confirmation is obtained by IVU that demonstrates ureters.
Anomalies Related to Form
Horseshoe Kidney • Most common type of renal fusion anomalyQ - results due to fusion of the lower poles of both
kidneys in midline. Refer to Q. No. 47 in Image-Based Questions
• On USG there is medial deviation of the lower poles.
• Investigations like IVU or CT scan are used for confirmation.
• A kidney fused at both the upper and lower poles is known as - Sigmoid kidneyQ
• Fusion of both poles in pelvis results in pancake or discoid kidney.
170
Contd…
Anomalies of Number
Duplex Collecting System • The least severe type is bifid pelvicalyceal systemQ followed by partial duplex (double) ureter where
to begin with there are 2 ureters arising from one kidney which fuse somewhere along the ureteric

GENITOURINARY TRACT IMAGING


course to unite and insert into the urinary bladder as single ureter.
• The severest is the complete duplication of uretersQ where the upper and lower pole moiety of kidney
is drained by separate ureters of which the lower ureter inserts in normal position of vesicoureteric
junction while the upper ureter has ectopic insertion.
• IVU or CE-CT - IOCQ.
• Drooping Lily signQ – Seen on IVU of duplex collecting system with obstructed upper pole moiety.
Refers to the inferolateral displacement of the opacified lower pole moiety due to an obstructed (and
unopacified) upper pole moiety.
Miscellaneous
Ureterocele • Congenital dilatation of distal end of ureter. Dilated portion of ureter may herniate into the UB at the
ureterovesical junction.
• On USG is appears as cystic anechoic (black) lesion at the vesicoureteric junction (VUJ), while on IVP
- Cobra/Adder head/Spring onion appearance.Q
Vesicoureteric Reflux • Recurrent UTI in childrenQ
• USG – is the 1st/Screening modality - may show some pelvicalyceal system or ureteric fullness/
dilatation.
• Micturating cystourethrogram (MCU)Q is investigation of choice as it shows anatomical detail and
helps in grading.
• Radionuclide MCUQ is preferred for follow-up – due to less radiation exposure than MCU.
• Best investigation for detection of Renal scars secondary to VUR – DMSA scanQ
Pelviureteric Junction • MCC of hydronephrosis in children –
Obstruction (PUJ) • Seen as dilated collecting system and dilated renal pelvis, however ureter is normal in caliber with a
transition point seen at the PUJ. Can be diagnosed on antenatal USG.
• IVU Will show a dilated contrast filled renal pelvis with a normal ureter and a transition point at the

Congenital Anomalies of Genitourinary System


PUJ.
Retrocaval/Circumcaval Ureter • Developmental variant seen only on right side.
• The proximal ureter courses posterior to the IVC, then emerges to the right of the aorta, coming to
lie anterior to the right iliac vessels.
• IVP shows a “J shaped ureterQ/Fish-hook ureterQ/S shaped ureterQ ” – with mild dilatation of the
proximal third of ureter.
Ectopia vesicae/Bladder • Associated with bladder directly opening through a defect in anterior abdominal wall.
exstrophy • Manta ray signQ: Failure of the pubic bones to meet in the midline result in a widened pubic symphysis
on AP radiograph
Prune belly syndromeQ/Eagle • Seen exclusively in males
Barrett syndromeQ/Triad • Triad of:
syndromeQ: ƒƒ Lax anterior abdominal wallQ
ƒƒ Gross ureteric dilatationQ
ƒƒ Bilateral undescended testisQ.

Posterior Urethral Valve • It is exclusively disease of malesQ


• Key hole/Spinning top appearanceQ: On antenatal USG the overdistended bladder and dilated
prostatic urethra give rise to a peculiar appearance
• Postnatally MCU will demonstrate the slit like valve in posterior urethra with/without associated
vesicoureteric reflux.
Refer to Image 33 in “Imaging Signs in Radiology”

171
IVU image shows an ectopic IVU image shows
pelvic right kidney (black crossed fused ectopia
arrow)
CONCEPTUAL REVIEW OF RADIOLOGY

IVU image shows left partial Pelviureteric junction (PUJ) KUB radiograph shows calcifications in bilateral renal cortex and
obstruction (black arrow). Faintly opacified ureter is seen medulla (black arrows) suggestive of Nephrocalcinosis
Congenital Anomalies of Genitourinary System

(White arrow)

Right duplex MCU image


collecting (black arrow) shows bilateral
system – vesicoureteric reflux (VUR)
Yellow and (white arrows)
Blue arrows
point to 2
separate
ureters.

4
CLINICAL QUIZ - SOLUTION
Conventional procedures – GIT/GUT system – are absolute favorites of your examiners. In each and every exam conducted over past
2–3 years, at least 1–2 questions on IVU/MCU/RGU/Barium swallow/Barium enema have been asked. It is a high yield topic – to be
revised in the last 7 days of your exam countdown for a maximum strike-rate!
So this is one such recently asked question. The appearance is classical for Ureterocele (Option D). It is more common in females, is usually
asymptomatic however may occasionally present with UTI. This bulbous dilatation of the ureter has been described as Cobra head/Adder
head/Spring onion appearance (hence answer is D. All of the above).
172
Prostate Imaging
5

GENITOURINARY TRACT IMAGING


CLINICAL QUIZ
9. A 65-year-old man presented with severe bone pain. A radiograph and
CT study was done and is shown here. Blood tests revealed an elevated
serum PSA levels. What is the most likely diagnosis in this case?
A. Hyperparathyroidism
B. BEP
C. Paget’s disease
D. Prostate cancer
10. The best screening technique for this condition is:
A. MRI
B. TRUS
C. CE-CT
D. DRE + Sr. PSA

Transrectal Ultrasound (TRUS)


  Clinical Pearls
ƒƒ Diagnostic indications for TRUSQ: Prostate Imaging
•• Suspected Carcinoma of Prostate along with TRUS
guided prostate biopsy MC site of BPH in prostate Transitional zoneQ
•• Benign prostatic hypertrophy MC site of Ca prostate in the gland Peripheral zoneQ
•• Prostatitis/Prostate abscess Most common cancer in Men Ca ProstateQ
ƒƒ Therapeutic indications for TRUSQ: MC site of Prostate cancer Peripheral zoneQ
•• Prostate cancer Brachytherapy MC site of metastases from Bone – Vertebral column
•• Prostate cancer cryotherapy Prostate cancer (Lumbar region)Q
ƒƒ TRUS guided prostate biopsy: MC primary tumor in sclerotic Ca ProstateQ
•• TRUS is used to localize the prostate, not suspected metastases in men
lesions within it. However, areas of the prostate in which Earliest lymph node to be involved Obturator LNQ
tumors are most frequently found can be sampled in in Prostate cancer
a systematic fashion on the basis of zonal anatomy at
Score used for Grading of Gleason scoreQ
transrectal US–guided biopsy. Prostate Ca
•• Sextant Protocol and Modified Sextant protocols are
commonly usedQ:
ƒƒ Screening for Prostate cancerQ: Benign Prostatic Hyperplasia (BPH)
•• Digital Rectal Examination (DRE) + Serum PSA ƒƒ Is nodular enlargement of prostate that occurs with age –
(Prostate Specific Antigen): Best screening toolQ involves the Transitional zoneQ
•• TRUS: not useful – due to variable appearance of can- ƒƒ Lower Urinary Tract Symptoms (LUTS)Q is most common
cer lesions on USG, operator dependability presentation
ƒƒ IVU/Micturating Cystourethrography:
Prostate Imaging

Multiparametric MRI (MP-MRI) of Prostate Q •• Posterior urethra may appear elongated/narrowed


with a smooth, rounded filling defect in bladder floor
ƒƒ Endorectal MRI coilQ: provides high resolution images. •• Fish-hook/J shaped ureters/Hockey stick ureterQ:
ƒƒ T2W images: Best for demonstration of zonal anatomyQ Distal ureters from a J shaped curve as the enlarged
ƒƒ Diffusion Weighted Imaging (DWI): Most effective MRI prostate elevates the bladder floor
sequence for detecting prostate cancerQ. •• Bladder wall trabeculations, Diverticuli, calculi seen
due to long standing bladder outlet obstruction
ƒƒ Dynamic Contrast Enhanced MRI (DCE-MRI): Used for
ƒƒ USG: TRUS is better than transabdominal USG. Pre- and
detecting residual/recurrent tumor following therapyQ.
post-void volumes can be measured

173
USG Image Benign Prostatic Hyperplasia or
demonstrates
CO Hypertrophy?
the Median NC E P T
CONCEPTUAL REVIEW OF RADIOLOGY

lobe indenting • Both these terms have been used interchangeably in literature
bladder base over a long time.
(White arrow) • But they have different meanings and hence only one of them
is correct.
• With age there is an increase in the number of glandular and
stromal cells in the periurethral transitional zone. This results
in the increase in the gland size centered on the periurethral
region and thus producing the Lower Urinary Tract Symptoms
(LUTS)
• Because it is actually an increase in the number of cells and not
MRI coronal, sagittal enlargement of cells the CORRECT TERM IS – HYPERPLASIAQ.
and axial images
of the prostate
(white arrow)
PROSTATE CANCER
ƒƒ USG: TRUS is the best ultrasound technique Lesions most
commonly appear – hypoechoicQ
ƒƒ MRI: Multiparametric MRI is Best investigation for local
staging/Pre-operative staging of diseaseQ
•• Lesion usually appears hypointense to the adjacent
normal peripheral zone
•• Most important features of extracapsular extension:
Obliteration of the rectoprostatic angle and asymmetry
of the neurovascular bundles
ƒƒ Bone Scan:
•• Used for assessment of skeletal metastasis.

PI-RADS: Prostate Imaging Reporting and


Data System (V2 – Second Revision)
ƒƒ Structured reporting scheme for prostate findings – to be
used in a pre-therapy patient
ƒƒ Score assessment is done on Multiparametric Prostate MRIQ
– includes T2WQ, DWIQ and DCE-MRIQ

5
CLINICAL QUIZ - SOLUTION
So the CT image shows multiple sclerotic areas involving multiple vertebral segments – in an elderly male these findings most likely suggest
Bone metastases. Another clue the question gives us is the elevated Sr. PSA – which points to a diagnosis of Ca Prostate. Remember that
MC cause of blastic/sclerotic bone metastasis in an adult is Ca Prostate (Answer is D). Hence the diagnosis is obvious – needs to be
confirmed by imaging (MP-MRI/TRUS guided biopsy)
Prostate Imaging

The best screening strategy for Ca prostate is a combination of Digital rectal examination (DRE) + Sr. PSA values (Answer is D)

174
Miscellaneous Gut

GENITOURINARY TRACT IMAGING


6
CLINICAL QUIZ
11. A 29-year-old female presented with systemic hypertension 12. The 1st step in evaluating this patient will be:
– refractory to medical treatment. Most probable clinical A. CT angiography
diagnosis in this patient will be: B. Renal artery Doppler
A. Hyperthyroidism C. Invasive catheter angiography
B. Renovascular hypertension D. Captopril DTPA scan
C. Adrenal tumor
D. Aortic aneurysm

RENAL PAPILLARY NECROSIS ƒƒ CT MRI: Used as a problem solving tool. Can distinguish
between urinary obstruction and a parapelvic cyst,
Causes: evaluation of retroperitoneal cyst rupture, hemorrhage
within cysts etc.
High Yield Mnemonic
  Clinical Pearls
POSTCARDS Spectral of Abnormalities in ADPCKD
• P : pyelonephritis • Renal cysts
• O : obstruction • Cerebral berry aneurysms:
• S : sickle cell disease • Intracranial dolichoectasia (dilated and elongated vessels)
• T : tuberculosis • Colonic diverticulosis
• C : cirrhosis • Bicuspid aortic valve, mitral valve prolapse:
• A : analgesic abuse • Aortic dissection:
• R : renal vein thrombosis • Multiple biliary hamartomas
• D : diabetes mellitus • Cysts in other organs: liver (most common)Q, ovaries, spleen,
• S : systemic vasculitis seminal vesicles, prostate, pancreas.

Imaging Features
ƒƒ IVU/CT Urography is the imaging modality of choiceQ.
IMAGING IN URINARY TRACT
ƒƒ The IVU signs seen are as follows TRAUMA
•• Golf Ball on tee signQ -- Seen as a contrast filled cavity
ƒƒ USG: First modality done, to look for hemoperitoneum -
(Golf ball) on a blunted calyx (the tee)
FAST
•• Lobster claw signQ – Due to excavation around the ƒƒ IVU: Single shot IVUQ is used in patients of renal injury who
edge of the papilla. The contrast material fills this
are hemodynamically unstable and taken for emergency
excavation – taking the shape of lobster claws
surgery.
•• Signet ring signQ – The necrotic tip of papilla may ƒƒ Retrograde pyelogram or cystogram: It can help to
remain in the excavated calyx resulting in this sign identify the site or urinary leak. In bladder injuries it
•• Sloughed papilla with clubbed calyx signQ can demonstrates whether leak is intraperitoneal or Miscellaneous Gut
extraperitoneal
ƒƒ CE-CT: It is imaging modality of choiceQ and should follow
AUTOSOMAL DOMINANT the CT IVU protocol. Renal hematomas and lacerations
POLYCYSTIC KIDNEY DISEASE ƒƒ
appear as nonenhancing hypodense lesions.
Page kidneyQ is a large non resolving subcapsular
(ADPCKD) hematoma that causes compression of underlying normal
renal parenchyma.
It is an autosomal dominant inherited multisystemic disease.
ƒƒ Inverted pear sign/Inverted tear drop shape of UB:
Imaging modalities:
Seen in bladder contusion as the accumulation of blood
ƒƒ IVU: Swiss cheese appearanceQ on Nephrogram phase
compresses upon the extraperitoneal part of bladder
– multiple smoothly marginated filling defects seen
throughout cortex/medulla – due to cysts
ƒƒ USG: Initial examination of choice/Screening modality
175
Renovascular Hypertension
USG showing
enlarged   Clinical Pearls
left kidney
CONCEPTUAL REVIEW OF RADIOLOGY

with multiple Pathophysiology of Renovascular Hypertension


cysts in it
Renal artery stenosis
suggestive of
ADPKD
Decreased glomerular perfusion detected by baroreceptors

Renin secretion

Angiotensin II release

CECT shows Vasoconstriction and resultant hypertension


irregular non
enhancing
area in post- ƒƒ It is the most common cause of secondary hypertension.Q
traumatic
ƒƒ Atherosclerotic narrowingQ - Most common cause of renal
kidney
artery stenosis. Involves mainly the origin or proximal one-
suggestive
third of renal artery
of renal
laceration ƒƒ Fibromuscular dysplasia (FMD)Q:
(black arrow) •• Most common non-atherosclerotic cause of renal
artery stenosisQ
•• Affects young femalesQ
•• “String-of-Beads” appearanceQ – on angiography as a
result of multiple short stenosis
  Clinical Pearls
Urinary Bladder Rupture – Intra- versus Extra-peritoneal
Imaging Features
Rupture ƒƒ Renal Ultrasound & Renal Artery Doppler:
•• Affected kidney may be small
Distinguishing Intraperitoneal Extraperitoneal •• Main renal artery findings:
Features Bladder Rupture Bladder Rupture  Color Doppler: Turbulent flow at and just distal to
Incidence Uncommon – Most common typeQ the stenotic segment with aliasing artefact
10 – 20% – 80 – 90%  Spectral Doppler: Peak systolic velocity more
than 100 cm/secQ
Etiology Blunt trauma Associated with  Renal Artery to Aorta Velocity ratio/RARQ: Ratio
abdomenQ with penetrating injuriesQ/ of PSVs in Renal artery to that in Aorta - >3.5 – is
distended UB pelvic fracturesQ diagnostic
Site of Bladder domeQ – Bladder base •• Intrarenal arteries:
rupture urine enters the anterolaterallyQ – urine  “Parvus-et-Tardus” waveform/effectQ: Low
peritoneal cavity enters the prevesical velocity within intrarenal arteries with a very slow
soft tissues, scrotum, upstroke/acceleration of velocity in systole
perineum, thigh and  Acceleration timeQ – Time from start of systole to
abdominal wall peak of systole - >0.07 seconds is abnormal
Miscellaneous Gut

Diagnosis Conventional/CT cystography ƒƒ CT Angiography:


•• Non-invasive technique, but requires iodinated contrast
Imaging Contrast seen Molar tooth signQ – injection and involves radiation exposure
features entering the ill-defined contrast •• More sensitive than Doppler ultrasound
peritoneal cavityQ, extravasated in the ƒƒ Conventional renal angiography/Digital substraction
filling the paracolic prevesical space angiography: Gold standard for diagnosis
gutters and in ƒƒ Captopril Renal Scintigraphy:
between the bowel
•• Captopril enhanced - 99mTc-DTPA scan is done
loops
•• Positive resultQ:
Management Immediate surgeryQ Usually conservativeQ  Decrease in uptake and delayed clearance of by
the affected kidney on the Post – Captopril study
compared to the Baseline study
176
 Exaggeration of baseline asymmetry between the •• Color Doppler ultrasound – Best investigationQ
normal and abnormal kidney in patients with •• On Doppler the involved testis shows absent/reduced
unilateral renal artery stenosis. colorflow
•• Twisting along the length of spermatic cord – Whirlpool

GENITOURINARY TRACT IMAGING


Captopril Renal Scintigraphy signQ of spermatic cord
(99mTc-DTPA Scan) in Renal Artery ƒƒ Radionuclide 99mTc Pertechnetate scan – was done in olden
N C E P Stenosis – Why?
CO T days – not done now.
Principle: In patients with significant renal artery stenosis, the
•• Nubbin signQ/Rim signQ/Halo signQ/Bull’s eye signQ/
efferent arterioles of glomeruli are constricted by the high levels Doughnut signQ seen.
of Angiotensin II to maintain the filtration pressure. Administration
of Captopril (ACE inhibitor) decreases this constriction of efferent Varicocele
arterioles with resultant reduction in GFR and urine flow.
ƒƒ Dilated tortuous veins of the pampiniform plexus and
99m
Tc-DTPA is excreted mainly by glomerular filtration and hence
internal spermatic vein
the captopril induced fall in GFR is seen as decreased renal uptake
and excretion of DTPA too. ƒƒ USG: USG with color Doppler – is Investigation of choiceQ
•• Multiple dilated tortuous tubular venous channels
>2–3 mm diameter
  Clinical Pearls •• Better seen in standing position and on coughing/
Summary of Imaging Modalities for Renal Artery Stenosis/ Valsalva maneuverQ
Renovascular Hypertension
Epididymitis – Orchitis – Funiculitis
1st Investigation/ USG with Renal artery DopplerQ
Screening investigation ƒƒ USG with Doppler – is investigation of choice
ƒƒ The involved structure appears edematous enlarged
Investigation of choice CT Angiography /MR Q

AngiographyQ hypoechoic with increased vascularity on Doppler

Gold standard Invasive/Conventional


investigation angiography/Digital subtraction Primary Testicular Malignancy
angiographyQ ƒƒ Seminoma – most common testicular tumorQ
Functional significance Captopril 99m
Tc-DTPA scanQ
ƒƒ USG has nearly 100% Negative predictive valueQ for
of stenosis testicular tumors
ƒƒ Staging of Testicular malignancy – CT is Best investigationQ
– as it can easily cover the thorax and abdomen for lymph
Testicular Torsion node/hematogenous metastasis. Metastasis – may be to the
ƒƒ Presents as acute scrotal pain +/- fever or vomiting para-aortic lymph nodes, mediastinal nodes, lungs.
ƒƒ USG Findings: ƒƒ Testicular epidermoid cyst/keratocyst - is most common
•• Testis may appear normal in early cases. If infarction benign germ cell tumor and shows Laminated/onion-skin/
occurs – may appear heterogeneous with time. whorled appearance on USG.

3D reconstructed 3D reformats of
normal CT CT urography
Angiogram show a bladder
diverticulum
Miscellaneous Gut
(white arrow)
connected by
thin stalk

177
Cryptorchidism
ƒƒ USG – initial examination of choiceQ.
ƒƒ MRI – Overall Best imaging modalityQ
CONCEPTUAL REVIEW OF RADIOLOGY

ƒƒ Laparoscopy – is most definitiveQ

Anechoic fluid is RGU image shows


noted in tunica narrowing at the
albuginea (white Bulbomembranous
arrow), surrounding junction (white arrow)
the testis suggestive suggestive of partial
of Hydrocele urethral stricture (NEET
Pattern 2020 IBQ)

Doppler image of
spermatic cord
shows dilated venous
channels with
increase and reversal
of color flow on
Valsalva suggestive
of varicocele

6
CLINICAL QUIZ - SOLUTION
Incidentally detected hypertension in a young adult without any associated clinical features points to a diagnosis of renal artery stenosis/
renovascular hypertension. Moreover such hypertension may be refractory to routine treatment. Hence the answer is B. Renovascular
hypertension
As discussed above the 1st step in evaluation such patients is to do a Renal artery Doppler.
Miscellaneous Gut

178
Multiple Choice Questions

GENITOURINARY TRACT IMAGING


Urolithiasis Imaging 14. An enlarged polycystic kidney seen antenatally suggests a
1. Investigation of choice for detection of Hydronephrosis/ diagnosis of:
Pyonephrosis:  (Recent Pattern Dec 2012/Dec 2016) A. Polycystic kidney disease
A. IVU B. USG B. PUJ obstruction
C. X-ray KUB D. CT C. VUR
2. Best investigation for detection of ureteric calculi is: D. Multicystic dysplastic kidney
A. IVU B. X-ray KUB (JIPMER 2017) 15. Bear paw sign is seen in:
C. USG D. NC-CT A. Renal TB
3. Most common renal calculus is: B. Acute pyelonephritis
A. Calcium oxalate B. Struvite C. Renal parenchymal disease
C. Uric acid D. Indinavir D. Xanthogranulomatous pyelonephritis
4. All of the following calculi are seen on an X-ray KUB except: 16. A swimmer from tropics complains of fever, malaise, and
A. Struvite calculi B. Calcium oxalate calculi dysuria. US reveals bladder wall thickening and polypoid
C. Uric acid calculi D. Cystine calculi lesion protruding into ureter. CT scan shows linear eggs shell
calcification in submucosa of bladder. Bladder capacity and
5. Staghorn calculus is:
contractility are well preserved. Findings are suggestive of:
A. Calcium oxalate B. Struvite
A. Carcinoma bladder
C. Uric acid D. Indinavir
B. Schistosomiasis
6. Soft tissue rim sign is a feature seen in:
C. Tuberculosis
A. Renal calculus B. Ureteric calculus
D. Dracunculiasis
C. UB calculus D. Phlebolith
17. Risk of contrast induced nephropathy is increased in:
7. Twinkling/Color comet tail is seen on an ultrasound in:
A. Diabetes mellitus B. Multiple myeloma
A. Portal vein aneurysm
C. Hyperuricemia D. All of above
B. Aortic aneurysm
C. Mesenteric pseudoaneuyrsm 18. Which of the following is not evaluated using 99mmTc DMSA
D. Ureteric calculus scintigraphy:
A. Renal anomalies
8. Steinstrasse is a complication seen after:
B. Renal pseudomass
A. Renal transplant B. IVU
C. Renal scarring
C. PCNL D. ESWL
D. Renal artery stenosis
9. The earliest sign of hydronephrosis on IVU is:
19. The commonest cause of bilateral smooth enlarged kidneys
A. Loss of calyceal cupping
is:
B. Blunting of the forniceal angle
A. Interstitial nephritis
C. Ballooning of the pelvis
B. Diabetes mellitus
D. None of the above
C. Acute glomerulonephritis
Renal Parenchymal Abnormalities Including Infections D. Acute nephropathy
10. Rim sign on IVP is seen in:  (NEET Pattern 2012) 20. Ball in cup appearance on IVP is seen in which of the
A. Polycystic kidney B. Hydronephrosis following:

Multiple Choice Questions


C. Hypernephroma D. Chronic pyelonephritis A. Calyceal diverticulum
11. USG findings of ARPCKD are all except: B. Calyceal calculus
A. Cysts >2 cm diameter C. Transitional cell carcinoma
B. Loss of corticomedullary differentiation D. Papillary necrosis
C. Enlarged kidney 21. Most useful imaging modality in medical renal disease is:
D. Oligohydramnios A. IVP B. US
12. Cortical Rim sign on CT is seen in: C. CT D. MR
A. Renal infarct 22. Best investigation for detection of renal scars in a known
B. Acute pyelonephritis case of vesicoureteral reflux is: (JIPMER May 2017)
C. Chronic pyelonephritis A. X-ray KUB B. USG
D. Putty kidney C. CT D. DMSA scan
13. ‘Great Imitator’ disease of the kidney is another term used 23. Of the following imaging modality which is most sensitive
for: investigation of choice to detect early renal tuberculosis:
A. Renal TB A. Intravenous urography (AIIMS 2003/AI 2005)
B. Acute pyelonephritis B. Ultrasound
C. Renal parenchymal disease C. Computed tomography
D. Xanthogranulomatous pyelonephritis D. Magnetic resonance imaging
179
Renal Masses 38. All are true about ureterocele except:
24. Stipple sign of transitional cell carcinoma of renal collecting A. It is submucosal dilatation of the intramural portion of the
system is best demonstrated by:  (COMED 2009) distal ureter
CONCEPTUAL REVIEW OF RADIOLOGY

B. Most are associated with the upper moiety of the duplex


A. IVU B. Radionuclide scan
collecting system
C. Retrograde pyelography D. USG
C. More common in males
25. Investigation of choice for evaluation of a renal mass is:
D. Cobra head appearance on the IVP is characteristic
A. IVU B. CE-CT (CET July 2016)
C. MRI D. USG
Prostate Imaging
26. Whorled onion peel appearance is seen in:
39. TRUS in carcinoma prostate is most useful for:  (AI 2008)
A. Renal angiomyolipoma B. Oncocytoma A. TRUS guided biopsy
C. RCC D. Simple cyst B. Seminal vesicle involvement assessment
27. Spoke wheel pattern of vascularity on Angiography is seen in: C. Prostatic volume measurement
A. RCC B. Angiomyolipoma D. Detect hypoechoic foci
C. Oncocytoma D. Complex renal cyst 40. The commonest zone of origin of a Prostatic carcinoma is:
28. Faceless Kidney is seen in: A. Peripheral zone B. Transition zone
A. Transitional cell carcinoma C. Central zone D. Anterior zone
B. Pelvicalyceal system duplication 41. Best MRI Sequence for depiction of zonal anatomy of prostate
C. Renal lymphoma is:
D. All of above A. T1W B. T2W
29. A child aged 4 years presented with renal mass. On US, a C. DWI D. MR Spectroscopy
well-defined, solid, iso to hypoechoic mass lesion was seen in 42. Fish-hook ureters are seen in:
renal area. Diagnosis is:
A. TB B. BPH
A. Wilms’ tumor B. RCC
C. Carcinoma prostate D. Carcinoma bladder
C. Neuroblastoma D. Pheochromocytoma
43. Best imaging modality for preoperative staging of Prostate
30. Presence of which tissue type on CT/MR is virtually diagnostic carcinoma is:
of an angiomyolipoma: A. TRUS B. CE-CT
A. Vessels B. Fat C. Multiparametric MRI D. PET
C. Hemorrhage D. Muscle 44. Transrectal ultrasound in Carcinoma prostate is most useful
31. Abdominal ultrasonography in a 3-year-old boy shows a solid for:
circumscribed hypoechoic renal mass. Most likely diagnosis A. Guided prostatic biopsy
is: B. Seminal vesicle involvement
A. Wilms’ tumor B. Renal cell carcinoma
C. Prostate volume measurement
C. Mesoblastic nephroma D. Oncocytoma
D. Hypoechoic mass detection
Congenital Anomalies Miscellaneous GUT
32. Which is not a renal pseudotumor?
45. Lobster claw sign on IVU is seen in:
A. Fetal lobulations
A. Renal papillary necrosis
B. Hypertrophied column of Bertin
B. Renal cell carcinoma
C. Dromedary hump C. Xanthogranulomatous pyelonephritis
D. Persistent calyceal impression D. Renal tuberculosis
33. Cobra head/Adder head appearance on IVU is seen in:
Multiple Choice Questions

46. String of Beads appearance of renal arteries on angiography


A. Ureterocele B. Hutch diverticulum is seen in:
C. Vesicoureteric reflux D. Renal papillary necrosis A. Atherosclerosis
34. J shaped ureter is seen in: B. Fibromuscular dysplasia
A. Retrocaval ureter B. Midureteric stricture C. Takayasu Arteritis
C. Ureteral transection D. Down syndrome D. Renal artery dissection
35. Keyhole sign is seen in: 47. What are antenatal imaging features in ARPKD?
A. Retrocaval ureter B. Posterior urethral valve A. Echogenic kidney B. Normal kidney
C. Patent urachus D. Hutch diverticulum C. Small renal cysts D. Shrunken kidney
36. A pelvic kidney is at increased risk of all of the following 48. Which of the following is not an appropriate investigation for
except: anterior urethral stricture? (AIIMS 2003)
A. Trauma B. VUR A. Magnetic resonance imaging
C. Obstruction D. Tumor B. Retrograde urethrogram
37. Drooping lily sign is seen in which of the following conditions: C. Micturating cystourethrogram
A. Hydronephrosis D. High-frequency ultrasound
B. Crossed fused ectopia 49. Tear-drop bladder is seen in:
C. Duplex kidney with hydronephrotic upper moiety A. Pelvic abscess B. Pelvic lipomatosis
D. Ureterocele C. Bladder rupture D. All of the above
180
50. Bell clapper deformity is associated with: 54. Most common location of an undescended testis is:
A. Testicular cancer B. Torsion testis A. Retroperitoneum
C. Hydrocele D. Undescended testis B. Inguinal canal
51. Best investigation for diagnosis of testicular torsion is: C. Superficial inguinal ring

GENITOURINARY TRACT IMAGING


A. B mode USG B. USG + Color Doppler D. Femoral canal
C. CT D. MRI 55. Most definitive for undescended testis is:
52. False about Varicocele is: A. USG B. CT
A. Dilated pampiniform plexus veins C. MRI D. Laparoscopy
B. More common on right side 56. Which of the following is associated with von Hippel-Lindau
C. Doppler USG is best investigation syndrome?
D. Abnormality increases on Valsalva A. Testicular seminoma
53. An intratesticular Whorled/Onion-skin appearance is sug- B. Testicular epidermoid cyst
gestive of: C. Papillary cystadenoma of epididymis
A. Seminoma B. Metastasis D. Hydrocele
C. Lymphoma D. Epidermoid cyst

Answers
1. B 8. D 15. D 22. D 29. A 36. D 43. C 50. B
2. D 9. B 16. B 23. A 30. B 37. C 44. A 51. B
3. A 10. B 17. D 24. A 31. A 38. C 45. A 52. B
4. C 11. A 18. D 25. B 32. D 39. A 46. B 53. D
5. B 12. A 19. B 26. A 33. A 40. A 47. A 54. B
6. B 13. D 20. D 27. C 34. A 41. B 48. A 55. D
7. D 14. D 21. B 28. D 35. B 42. B 49. D 56. C

Explanations to Questions
9. Grades of hydronephrosis on USG are: Grade 1: Minimal 32. A renal pseudotumor is a mass that will simulate a tumor on
blunting of forniceal angle. Grade 2: Blunting of calyces with imaging but is composed of non-neoplastic tissue. Common
intact papillary markings. Grade 3: Loss of papillary markings such lesions are:
Grade 4: Ballooning of calyces. •• Prominent columns (septa) of Bertin
12. The cortical rim sign is useful in distinguishing acute •• Persistent fetal lobulations
pyelonephritis from renal infarct and is seen on contrast •• Dromedary hump
enhanced CT or MRI. It is preservation of thin enhancing •• Splenorenal fusion

Explanations to Questions
cortical rim in infarct. This occurs because the blood supply •• Cross-fused renal ectopia
to the outer aspect of the cortex is derived from perforating 52. Varicocele: The left testicle is affected much more commonly
branches of the renal capsular artery and hence is preserved (≈85%) than the right. This may be due to the shorter course
when rest of the parenchyma is infarcted. of the right testicular vein and its oblique insertion into the
18. DMSA scan is used to study renal morphology thus out IVC which creates less backpressure. Isolated right varicoceles
of given options only renal artery stenosis is not a renal are rare and should prompt evaluation for a secondary
morphological defect. Renal artery stenosis is studied by varicocele.
Captopril-DTPA scan

181
_______________________________________________________________________________________________________________________

_______________________________________________________________________________________________________________________

_______________________________________________________________________________________________________________________

_______________________________________________________________________________________________________________________

Note
_______________________________________________________________________________________________________________________

_______________________________________________________________________________________________________________________

_______________________________________________________________________________________________________________________

_______________________________________________________________________________________________________________________

_______________________________________________________________________________________________________________________

_______________________________________________________________________________________________________________________

_______________________________________________________________________________________________________________________

_______________________________________________________________________________________________________________________

_______________________________________________________________________________________________________________________

_______________________________________________________________________________________________________________________

_______________________________________________________________________________________________________________________

_______________________________________________________________________________________________________________________

_______________________________________________________________________________________________________________________

_______________________________________________________________________________________________________________________

_______________________________________________________________________________________________________________________

_______________________________________________________________________________________________________________________

_______________________________________________________________________________________________________________________

_______________________________________________________________________________________________________________________

_______________________________________________________________________________________________________________________

_______________________________________________________________________________________________________________________

_______________________________________________________________________________________________________________________

_______________________________________________________________________________________________________________________

_______________________________________________________________________________________________________________________

_______________________________________________________________________________________________________________________

_______________________________________________________________________________________________________________________

_______________________________________________________________________________________________________________________

_______________________________________________________________________________________________________________________

_______________________________________________________________________________________________________________________
MUSCULOSKELETAL
IMAGING
C hapter O utline
• Bone and Joint Infection Bone Tumors • Skeletal Dysplasias and Miscellaneous Musculoskeletal
• Arthritis Imaging
CONCEPTUAL REVIEW OF RADIOLOGY

• Systemic Bone Disorders • Multiple Choice Questions with Explanations

Bone and Joint Infection Bone Tumors


1
CLINICAL QUIZ
1. A 35-year-old man presented with multiple swellings around the knee
and elbow regions. A radiograph is shown here. Identify the incorrect
statement regarding this condition:
A. The lesions are covered with a hyaline cartilage cap
B. May undergo malignant degeneration
C. Autosomal dominant inheritance
D. Risk of malignancy is 10%

This topic is a long one but also important as one can Age: Specific Tumors Occur at Specific
have high yielding questions and most of them will be based
on imaging signs. We will have a concise yet comprehensive Age Groups
Bone and Joint Infection Bone Tumors

review of this important topic.


ƒƒ 1st imaging modality is–Plain radiographsQ which combined Let us Consider a Simplified Age Distribution at
with age of patient narrows down to few diagnoses. Your Level
ƒƒ This should be followed by MRI with contrast whose primary
role is to identify extension into bone marrow, adjacent soft Age group Commonly seen tumors
tissues especially neurovascular bundles.Q <20 years • Benign tumors – Eosinophilic granuloma,
ƒƒ In some tumors like osteosarcoma/Ewings sarcoma CT Simple bone cyst, Aneurysmal bone cyst,
Thorax and Bone scan is also done for staging. Fibrous dysplasia, Osteoid osteoma,
•• Biopsy should always be performed along with local Osteoblastoma, Chondroblastoma
bone oncosurgeon as breach of fascial planes will • Malignant tumors – Ewing’s sarcoma,
upstage the tumor. The final diagnosis is always made Osteosarcoma
on histopathology.Q
20–40 years • Benign tumors - Enchondroma, osteoma,
Osteoblastoma
• Malignant tumors - Giant cell tumor,
HOW TO APPROACH BONE LESIONS? Parosteal osteosarcoma
ALTA Protocol : AGE – LOCATION – TRANSITION ZONE – >40 years • Chondrosarcoma, Metastases, Multiple
APPEARANCE myeloma

184
Two cases of Multiple
a Cortical out- Exostoses
growth, direct- (yellow arrows)

MUSCULOSKELETAL IMAGING
ed away from – Diaphyseal
the adjacent aclasis
joint – Osteo-
chondroma/
Exostosis
(yellow
arrows)

Location
This is an important differentiating point. We will discuss this as predilection for specific bone in body and for specific part of the
bone.
Distribution of Neoplasms Based on Site of Bone Involved
Epiphysis Metaphysis Diaphysis
Chondroblastoma NOF–Non-ossifying fibroma Ewing’s sarcoma
Infection SBC Enchondroma
Giant cell tumor (in adults) CMF–Chondromyxoid fibroma Fibrous dysplasia
Geode/Subchondral cyst Osteosarcoma SBC
Chondrosarcoma ABC
Enchondroma Osteoblastoma
Infections

Transition Zone/Margins
To classify osteolytic lesions as well-defined or ill-defined, we need to examine the zone of transition between the lesion and the
adjacent normal bone. This characterization of zone of transition only applies to osteolytic lesions since sclerotic lesions usually have
a narrow transition zone. The zone of transition is the most reliable indicator in determining whether an osteolytic lesion is benign
or malignant. 

Narrow zone of transition – Well-defined margins Wide zone of transition – Ill-defined margins

Bone and Joint Infection Bone Tumors

Usually indicates a Benign lesionQ. A simple bone cyst Usually indicates a Malignant lesionQ–Osteosarcoma
Exceptions: Exceptions:
• In patients aged >40 years – Metastases and Multiple myeloma • Infection and Eosinophilic granuloma – are benign lesions that
lesions are malignancies that show a narrow zone of transition may show wide zone of transition

Appearance
Before we jump to individual appearances of bone tumors let us consider few other important points regarding the appearance. 185
Periosteal Reaction
It is nonspecific reaction that occurs due to irritation of periosteum from underlying inflammatory or neoplastic process. This type of
reactions follows a gradient from being suggestive of benign to a more malignant behavior. Rather than just mugging them up – let us
CONCEPTUAL REVIEW OF RADIOLOGY

look at actual images and try and form a visual impression.

Matrix Mineralization
Chondroid matrix Osteoid matrix
Rings-and-arcs, popcorn, focal stippled or flocculent type of Tubular ossification pattern in benign lesions and amorphous
matrixQ cloud like increase in density seen in malignant lesionsQ
Seen in Enchondroma, Chondrosarcoma Seen in Osteoid osteoma, Osteosarcoma

Benign Bone Tumors


Osteoid osteoma • Cortical lesion with a radiolucent nidusQ (<1 cm in diameter) surrounded by dense sclerosis
• FemurQ—Most common site
• In spine—Lumbar spineQ is commonest site, usually presenting as an Ivory pedicle
• Nidus shows intense vascular blushQ on Angiography
Bone and Joint Infection Bone Tumors

Osteochondroma/ • Most common benign bone tumorQ


exostosis • Bony outgrowth, continuous with adjacent cortex, covered with a cap of Hyaline cartilageQ
• Malignant degenerationQ -
ƒƒ Occurs in 1%
ƒƒ Cartilage cap >3 cm thick
• Coat hanger exostosisQ–Large exostosis projecting away from the joint
Skeletal hemangioma • Most common benign bone tumor of spineQ
• Striated vertebra/Corduroy cloth appearanceQ–coarse vertical striations in vertebral body
• Sunburst/Spokewheel appearanceQ–Hemangioma of skull bone
• Polka dot sign: Axial CT Scan of vertebra
Osteoma • Most common benign tumor of nose and paranasal sinusesQ
• Round/Oval well defined radio-opaque lesion
Enchondroma • Most common benign bone tumor of handQ
• Geographic lucent lesion in phalanges, with mild expansion
• Stippled, punctuate calcification may be seen
Ollier’s disease • Multiple enchondromatosisQ
• Higher rate of malignant degeneration–10%Q
Maffucci’s syndrome • Enchondromatosis + Soft tissue cavernous hemangiomasQ
• Higher risk of malignant transformation–25%Q
Contd…
186
Chondroblastoma/ • Epiphyseal lesionQ occurring before physeal closure
Codman tumorQ • Proximal femurQ–Most common site
• Chicken wire type calcification microscopically
• Fluffy cotton woolQ like calcification on radiographs

MUSCULOSKELETAL IMAGING
Simple bone cyst/ • Proximal humeral metaphysisQ—Most common site
unicameral bone cyst • Geographic lucent defectQ
• Fallen fragment signQ—after a pathologic fracture, the fragment falls to the dependent portion
• Hinged fragment signQ—Incomplete cortical fracture
Aneurysmal bone cyst • 5–20 years
• Eccentric, metaphyseal lucent lesion–that may cross the physeal plate
• Blown-out appearanceQ/Finger-in-the-balloon signQ–Cortical ballooning seen in ABC
• Fluid-fluid levelsQ–on MRI
Intraosseous lipoma • Osteolytic lesion with well-defined/sclerotic border
• Cockade signQ/TargetQ/Doughnut shaped sequestrumQ–central radiopacity within a calcaneal lipoma

  Clinical Pearls
Bone Tumor Associated Syndromes Ollier’s diseaseQ:
Gardner’s syndromeQ: • Multiple enchondromatosisQ
• GI polyposis • Higher rate of malignant degeneration–10%Q
• Multiple osteomasQ, Epidermal cystsQ, Desmoid tumorsQ and Maffucci syndromeQ:
Fibromatosis • Enchondromatosis + Soft tissue cavernous hemangiomasQ
• Extra-intestinal manifestations precede the colonic polyps • Higher risk of malignant transformation–25%Q

Typical cortical Vertebral body


sclerotic lesion hemangioma –
with a central giving rise to typical
radiolucent Striated. Corduroy
nidus – Osteoid cloth appearance
osteoma and the Polka Dot
sign

Bone and Joint Infection Bone Tumors

Calcaneal lipoma –
Lucent lesion with
a central dense
calcification – Cockade/
Target/Doughnut
sequestrum sign

187
Eccentric, Mildly expansile,
metaphyseal lytic lesion in
lucent lesion in phalanges with
CONCEPTUAL REVIEW OF RADIOLOGY

an Immature stippled internal


skeleton with calcification –
fluid-fluid Enchondroma
levels on MRI
– Aneurysmal
Bone cyst

Malignant Bone Tumors

Metastasis • Blow out lesionsQ—highly expansile bone lesions. Seen in primaries form RCC/Thyroid/Lung malignancy
• Cookie bite lesionQ—cortical metastatic deposit with scalloping of cortex. Seen in Lung–Breast–Kidney
cancer
• Lytic lesionsQ—permeative/Moth eaten destructive lesion
• Snowball appearanceQ—multiple blastic/radiodense lesions
• One-eyed pedicleQ/Winking owl signQ JIPMER May 2018 Pattern–Destruction of single pedicle, pedicle
not seen on AP radiograph
• Blind vertebraQ—bilateral pedicle destruction
• Ivory vertebraQ—densely sclerotic vertebra
• Extrapleural signQ—obtuse angled opacity due to deposit in chest wall/ribs
• Sunburst lesionQ/Spiculated lesion of skull vaultQ–seen in Neuroblastoma metastasis
Multiple myeloma • Most common primary malignant bone tumorQ
• SpineQ—most common site
• 99mTc–MDP Bone scanQ—appear as cold spotsQ
• Diffuse osteopenia with multiple punched out lytic lesions
• Raindrop skullQ NEET JAN 2018 PATTERN —multiple uniformly sized punched-out lytic lesions in skull
Bone and Joint Infection Bone Tumors

• Vertebra plana/Wrinkled vertebraQ—vertebral body collapse


• Pedicle sign of myelomaQ—as pedicles are poor in red marrow, hence are spared
• Refer to Image 38 in “Imaging sings in Radiology”
• Solitary plasmacytomaQ:
ƒƒ Localized plasma cell proliferation
ƒƒ Geographic lucent expansile lesion—soap bubble internal architectureQ

Osteosarcoma • 2nd most common primary malignant bone tumorQ


• 10–25 years age group—Metaphyseal locationQ
• FemurQ—most common site
• Permeative/Ivory medullary lesion in metaphysisQ with cortical disruptionQ and soft tissue extensionQ
• Sunray/Sun-burst periosteal reactionQ seen
• Codman’s triangleQ—elevated periosteum–not specific for osteosarcoma
• Cumulus cloud appearanceQ—eccentric lobulated soft tissue mass adjacent to bone
• Cannonball ossifying lung metastasisQ—may be seen

188
Multiple diffusely Osteosarcoma –
scattered dense Irregular humeral
sclerotic bone lesions metaphysic lesion

MUSCULOSKELETAL IMAGING
– Sclerotic bone with a wide zone
metastases in a known of transition,
case of Prostatic Sunray/Spiculated
carcinoma periosteal reaction
(yellow arrows) and
Codman’s triangles
(blue arrows)

Parosteal/ • Cleavage plane sign/String signQ–


Juxtacortical radiolucent cleft between the mass and   Clinical Pearls
osteosarcoma the underlying cortex Bone Metastases Specifics
Secondary • Seen in cases of pre-existing bone disease • Most common malignant bone tumorQ
osteosarcoma like: • Breast cancerQ–most common primary in females
ƒƒ Paget’s diseaseQ • Prostate cancerQ—most common primary in males overall.
ƒƒ Polyostotic fibrous dysplasiaQ Also most common cause of Blastic metastasesQ
ƒƒ Hereditary multiple exostosisQ/ • Lung cancerQ–most common cause of lytic bone metastasis
Diaphyseal aclasiaQ • 99mTc–MDP Bone scanQ—most sensitive/Best technique for
ƒƒ EnchondromatosisQ/Ollier’s diseaseQ detection–appear as Hot spots
• Central/Axial skeletonQ—most commonly involved
Chondrosarcoma • Most common primary malignant bone
tumor of hand and chestQ
• Occurs after 40 years of ageQ
• Pelvis and proximal femur–most common
site
• Large radiolucent ill-defined lesion with
internal popcornQ/fluffyQ/stippledQ/
cotton-woolQ/rings and broken arcsQ
calcification
Ewing’s sarcoma • Diaphyseal lesion presenting at 10–25
years
• FemurQ–most common site

Bone and Joint Infection Bone Tumors


• Permeative lesion in diaphysisQ with
Laminated/Onion peel periosteal
reactionQ
• Cortical saucerizationQ seen
• Groomed/Trimmed whiskers effectQ-
spiculated periosteal reaction
• Skip lesionsQ may be seen
• Refer to Q. No. 52 in Image-Based
Questions
Giant cell tumor • Quasi-malignant tumorQ–most are benign,
some are malignant
• Distal femurQ–most common site A large mass lesion at the left pelvic bone with calcific and
• Seen at 20–40 years soft tissue component – with typical rings and broken arcs
• Lesion begins at metaphysis and extends type calcification – suggests Chondrosarcoma.
to the end of bone–thus is sub-articular in
locationQ
• Eccentric metaphyseal/subarticular,
multilobed, expansile lucent lesionQ
• Soap bubble appearanceQ seen
• Refer to Image 53 in “Imaging Sings in
Radiology”
189
Large Expansile well defined
heterogeneously Soap-bubble lesion
enhancing lesion involving the Tibia –
CONCEPTUAL REVIEW OF RADIOLOGY

at the spheno- Adamantinoma.


occipital region –
Chordoma. This is
2nd most common
site of Chordoma
after sacro-
coccygeal region

1
CLINICAL QUIZ - SOLUTION
The radiograph reveals multiple exostosis/osteochondromas around the knee joint. When >10 in number the condition is termed as
Hereditary Multiple Exostoses (HME)/Diaphyseal aclasis. It is characterized by:
• Autosomal dominant inheritance
• Multiple exostosis ~ 10 in numberQ
• Malignancy risk–20%Q (Hence Answer is D – which is the wrong statement)
• Bayonet hand deformityQ–Seen at wrist due to growth retardation
• Malignant transformation – presents as sudden increase in size, pain. MRI is best investigation to detect malignant change.

Arthritis
2
CLINICAL QUIZ
2. A middle-aged woman with history of backache underwent a Schober test
and came as positive. She had hyperpigmented nose and ears. Looking at
this radiograph, most likely diagnosis is: (AIIMS May 2017 Pattern)
A. Ankylosing spondylitis
B. Degenerative disc disease
C. Ochronosis
D. Fluorosis
Arthritis

190
CLASSIFICATION OF ARTHRITIS
Early case of
Rheumatoid
arthritis –

MUSCULOSKELETAL IMAGING
reveals Juxta-
articular
osteoporosis
(black
appearance)

Wrist AP
radiograph –
RHEUMATOID ARTHRITIS (RA) Uniform loss of
joint space with
  Clinical Pearls juxta-articular
osteoporosis and
RA specifics early erosions
• Selectively targets synovial tissue, particularly in peripheral – Rheumatoid
joints of hands and feet.Q arthritis
• Seropositive arthritis Q—as the patient may have positive RA
factor (70% cases).
• Common in femalesQ in4th to 5th decade of life.
• Felty syndromeQ—RA, Leukopenia and Splenomegaly
• Caplan syndromeQ—Pneumoconiosis with RAQ
• Predilection for PIPQ and MCPQ joints, ulnar styloidQ and
triquetrumQ. Distal interphalangeal joint–is not involvedQ
• Norgaard projection of X-ray: Ball catcher’s position of hand -
done specifically to look for metacarpal head erosions

Radiographic Changes
Basic radiologic features of Rheumatoid Arthritis: Seen at all
affected joints. The specific features at a particular joint may vary
and are discussed later.
• Bilateral symmetric involvement—due to generalized systemic
etiopathology Wrist
• Uniform loss of joint space—differentiating factor from degen- radiograph –
erative joint disease (shows asymmetric joint involvement) Rheumatoid
• Periarticular soft tissue swelling—due to joint effusions and arthritis with
tenosynovitis Swan neck
• Marginal erosions/Rat-bite erosionsQ—due to pannus deformity of
formation fingers
• Juxta-articular osteoporosis—due to increased vascularity/
inflammation
• Juxta-articular periostitis—due to increased vascularity/
inflammation ƒƒ Swan neck deformityQ: Flexion at DIP and extension at PIP
• Large pseudo-cysts—due to intra-osseous extension of pannus/ joints
synovial fluid ƒƒ Hitch hiker thumbQ: Boutonniere’s deformity affecting
• Joint deformities—seen as an end result of disease process thumb-hyperextension at IP and MCP joints
ƒƒ Dot: Dash appearanceQ—intermittent absence of articular
Hand and Wrist Involvement cortex due to bone erosions and subarticular resorption
Arthritis

ƒƒ Marginal erosions: Earliest articular finding


ƒƒ Boutonniere deformityQ: Flexion of proximal interphalangeal ƒƒ Ivory phalanxQ: Homogenous sclerosis of distal phalanx
joint, extension of Distal IP joint, due to rupture of central slip ƒƒ Few more peculiar appearances are also seen–enlisted in
of Extensor digitorum communis at proximal IP joint the table below

191
Other Named appearances/entities in Rheumatoid arthritis
Arthritis mutilansQ • Severe polyarticular joint deformity with marked joint destruction
Boutonniere’s deformity Q
• Flexion of proximal interphalangeal joint, extension of Distal IP joint, due to rupture of central slip of
CONCEPTUAL REVIEW OF RADIOLOGY

Extensor digitorum communis at proximal IP joint


Caput ulnae syndrome • Diastasis of distal radioulnar joint may displace the ulna dorsally and may precipitate extensor tendon
rupture
Haygarth’s nodes Q
• Soft tissue periarticular swellings at dorsal aspects of Metacarpophalangeal joints
Hitchhiker’s thumbQ • Boutonniere’s deformity affecting thumb
Lanois deformityQ • Fibular deviation of digits and dorsal subluxation of metatarsophalangeal joints
Mallet finger Q • Persistent flexion of distal phalanx due to rupture of Extensor digitorum communis
Pencil sharpened spinous • Tapered spinous processes
processesQ • Most commonly in cervical spine
Spindle digitQ • Fusiform appearing digit due to PIP joint swelling
Spotty carpal sign • Characteristic appearance produced by multiple marginal erosions in the carpal bones
Summation effectQ • Decrease in neck length due to upward migration of odontoid, loss of C1-C2 disc height and osteolysis
Swan-neck deformityQ • Flexion of DIP, extension of PIP joint
Ulnar driftQ • Fingers of hand deviated towards the ulna at the MCP joint
Zigzag deformityQ • Combination of ulnar deviation of digits and radial rotation of carpal bones

  Clinical Pearls
Ivory Phalanx Coronal STIR
Homogenous sclerosis of the distal phalanx. Apart from RA, it MRI image of SI
is also seen in: joints – shows
• Hyperparathyroidism hyperintensities
• Psoriasis bilaterally
• Scleroderma – suggests
• SLE Sacroiliitis in
Ankylosing
• Idiopathic
spondylitis

ANKYLOSING SPONDYLITIS
ƒƒ Also known as Bechterew diseaseQ and Marie Strümpell
diseaseQ
ƒƒ Prototype of Seronegative arthritisQ - RA factor negativeQ
and commonly associated with HLA B27Q
ƒƒ Spine: Classical signs very commonly asked in your exams
ƒƒ EnthesitisQ (inflammation of enthesis–the attachment of
ligament/tendon to bone) - is the imaging hallmark and has •• Romanus lesionsQ—Vertebral body corner erosions
propensity for axial skeleton involvementQ •• Shiny corner signQ—Vertebral body corner sclerosis
•• Squaring of vertebral bodyQ/Barrel vertebraQ
Radiographic Features •• Anderson’s lesionQ—Ankylosed previously fractured
ƒƒ Sacroiliac joint involvement: spinal segment resembling a noninfectious spondylo-
•• Sacroiliitis is the earliest clinical manifestationQ discitis
•• MRI—Imaging modality of choice for early sacroiliitisQ •• Bamboo spineQ/Poker spineQ—Diffuse marginal
•• Early sacroiliitis: Rosary bead appearance of SI jointQ syndesmophytes ankylosis
•• Late sacroiliitis:
•• Refer to Image No 40 in Imaging Sings in Radiology
 Bony ankylosisQ
•• Dagger signQ—Interspinous ligament ossification on
Arthritis

 Generalized osteoporosis
 Ghost joint marginQ frontal radiographs
 Star signQ–increased bone density at junction of •• Trolley track signQ—On frontal radiograph of spine
fibrosis and synovial portion of joint there will be ossification of the interspinous ligament

192
in middle and bilateral paraspinous ligaments/facet
joints laterally. Meniscal
•• Carrot stick fractures of the spineQ calcification –
ƒƒ Enthesopathy: The point of insertion of tendon on bone is Chondrocalcinosis

MUSCULOSKELETAL IMAGING
known as enthesis.
•• WhiskeringQ—Seen at the enthesis as coarse spicules
of bone extending away from bone–due to periostitis

OTHER SERONEGATIVE
ARTHRITIS
The cardinal features are same as that of AS but with varying
incidence and intensity. There are few other signs that are more
commonly found in these conditions which will be enumerated
below.

Psoriatic • Presence of nail changesQ—seen in 80% of


Mild to moderate
arthritis arthritis patients
Osteoarthritis:
• Spindle digitQ/Sausage digitQ—soft tissue Asymmetric joint space
swelling of entire digit/toe loss (medial more than
• Ivory phalanxQ—sclerotic phalanx may be seen lateral), mild sclerosis
• Fluffy juxta-articular periostitisQ of articular surfaces
• Mouse ears signQ—fluffy periosteal new bone and early osteophytes
formation adjacent to erosions
• Pencil in cupQ/Pestle and mortarQ/Mushroom
and stemQ/Cup and saucerQ/Balancing pagoda
deformity—Tapered end of a phalanx erodes
into distal articular surface
• Opera glass handQ—Overall shortening of digit Severe
occurs Osteoarthritis –
• Arthritis mutilansQ—end result Loss of medial joint
Reiter’s • Triad of: space, articular
syndrome ƒƒ UrethritisQ margin sclerosis,
ƒƒ ConjunctivitisQ medial joint
subluxation (yellow
ƒƒ PolyarthritisQ
arrows) and loose
• Arthritis mainly affects lower limb joints bodies (blue arrow)
• Lover’s heelsQ—Pain at Achilles’ tendon
insertion on calcaneus involvement
• Non-marginal vertebral osteophytes
ƒƒ ChondrocalcinosisQ—Cartilage calcification
ƒƒ Pseudotumor of GoutQ—Tophi deposition in prepatellar
GOUT ARTHROPATHY region causing a soft tissue swelling
ƒƒ Rising sun signQ—Tophi deposition in olecranon bursa
ƒƒ Disorder of Purine metabolismQ, that leads to hyper-
seen on lateral elbow radiograph
uricemiaQ and deposition of sodium monourate crystalsQ in
the joints. Strong inflammatory reaction occurs as a result of
this deposition–resulting in arthropathy
ƒƒ 1st metatarsophalangeal jointQ—most commonly involved.
PSEUDO GOUTQ/CPPDQ
ƒƒ Overhanging margin signQ—eccentric erosions occur (CALCIUM PYROPHOSPHATE
resulting in protruding bone extending away from bone into
soft tissue
DEPOSITION DISEASE)
Arthritis

ƒƒ TophiQ–pathognomonic–seen as lytic lesions within bones, ƒƒ ChondrocalcinosisQ–most common radiologic abnormality


also occur in soft tissue. ƒƒ KneeQ–Most commonly involved joint

193
Peculiar Named Appearances/Entities
  Clinical Pearls
Associated with Degenerative Joint Disease
ChondrocalcinosisQ–Cartilage CalcificationQ
Maigne’s • Degenerative joint disease of lower
CONCEPTUAL REVIEW OF RADIOLOGY

Remember the causes as 3 C’s: syndromeQ thoracic apophyseal joints—referred


pain to lower lumbar region
Cation diseases : Q

• Fe++ – HemochromatosisQ Robert’s • Degenerative joint disease of costo-


• Ca++ – HyperparathyroidismQ syndromeQ transverse joints—simulates upper GI
• Cu++ – Wilson’s diseaseQ disease pain
Crystal deposition diseaseQ: Vacuum sign of • Accumulation of nitrogen gas (lucency)
• CPPDQ/Pseudo-goutQ KnutssonQ in the intervertebral disc space
• GoutQ Kissing spine • Excessive spinal lordosis—resulting in
• OchronosisQ–Homogentisic acid disorder syndromeQ/ interspinous osteoarthritis
Cartilage degeneration diseasesQ: Baastrup
• Degenerative joint diseaseQ syndromeQ
• AcromegalyQ Movie signQ • Seen in chondromalacia patellae—
• Neurotrophic joint diseaseQ increased pain on sitting in a confined
space
Bouchard’s nodesQ • Osteophytes/enlarged bone at PIP joint
DEGENERATIVE JOINT DISEASE/ Heberden’s nodes Q
• Osteophytes/enlarged bone at DIP joint
Erosive • Interphalangeal joints of handsQ—most
OSTEOARTHRITIS osteoarthritis commonly involved
ƒƒ Most common type of Arthritis overall Q • Gull wing signQ—due to central
ƒƒ Knee and hip are the two most common joints affected. articular erosions

  Clinical Pearls
Inflammatory versus Degenerative Joint Disease

Feature Degenerative joint disease Inflammatory joint disease


Osteoarthritis

Age Elderly Varied–seen even in young patients


Distribution Usually asymmetric Usually symmetric
Systemic clinical features (clinic-pathological) Absent Present
Joint space loss Non-uniform joint space loss Uniform joint space loss
Sub/Periarticular changes Sub-articular sclerosis Periarticular osteopenia
Intra-articular loose bodies Present Absent
Sub-articular cysts/Geodes Present Absent
Bone erosions Absent Present
Periosteal reaction/Periostitis Absent Present
Arthritis

194
ƒƒ Licked candy stick appearanceQ—Pencil-like tapering of
bone
ƒƒ Bag of bonesQ—Clinically palpable signs of advanced disease
ƒƒ Jigsaw vertebraQ—Fragmented vertebra due to multiple

MUSCULOSKELETAL IMAGING
fractures
ƒƒ Tumbling building block spineQ—Multisegmental sublux-
ated vertebral bodies simulating falling building blocks
ƒƒ Refer to Image No 43 in “Imaging sings in Radiology”
Typical central
erosion leading
to classical Gull MISCELLANEOUS ARTHRITIS
wing appearance
– Erosive Diffuse idiopathic • Characterized by bony proliferation
osteoarthritis skeletal hyperostosis at sites of tendinous and ligamentous
(DISH)Q insertion of the spine
• Elderly in 6th to 7th decade of life is
affected
NEUROPATHIC JOINTS/ • Florid, flowing ossification is noted
along the anterior or right anterolateral
NEUROTROPHIC ARTHROPATHY/ aspects of at least four contiguous
vertebrae - Dripping candle wax
CHARCOT’S JOINTS appearanceQ/Flame shaped
ƒƒ Progressive degenerative/destructive joint disorder in osteophytesQ
patients with abnormal pain sensation and proprioceptionQ. Ossified posterior • Also known as Japanese diseaseQ
ƒƒ Diabetes mellitusQ is the most common cause while other longitudinal • Ossification of the posterior
causes include syrinx, amyloid neuropathy, SACD, tabes ligament (OPLL)Q longitudinal ligament, seen as a
dorsalis, leprosy, spinal bifida, etc. continuous band extending over 3–4
ƒƒ Ankle is one of the most commonly affected joint vertebral segments and traversing
ƒƒ Imaging wise it has features based on 6D’sQ: Disorganization, across the disc spaces
destruction, density (osteosclerosis), dislocation, debris • Cervical spineQ–most common
and distension. location
• Associated with DISHQ and Ankylosing
spondylitisQ
• Refer to Q. No. 65 in “Image-Based
Flowing ossification along Questions”
the anterior vertebral Ochronosisq • Kindly refer to the Clinical Quiz box
margins - DISH AIIMS May 2017 Pattern
for the image and the Solution below
(homogentisic acid for the discussion.
is deposited in
tissues)

2
CLINICAL QUIZ - SOLUTION
We have a middle-aged lady coming with back pain. Schober’s
test is done for assessment of flexibility of the spine and is
positive most commonly in Ankylosing spondylitis. Although it
is not specific for AS and may be positive in any condition that
affects the flexibility of the spine. There are 2 important clues in
this patient :
• Intervertebral disc calcification – if you look at the radiograph
carefully you will realize that discs are appearing denser than
bones because they are calcified.
• Hyperpigmentation of nose & ear cartilage – This blue-
black discoloration of  connective tissue  (including bone,
Arthritis

Synovial osteo- cartilage, and skin) caused by deposits of yellow or ochre-


chondromatosis colored pigment and is seen in Ochronosis/Alkaptonuria
– secondary to joint It is characterized by deficiency of Homogentisic acid oxidase and
degeneration ChondrocalcinosisQ/IV-disc calcificationQ. Hence answer is C.
195
Bone Infections and Systemic Bone Disorders
CONCEPTUAL REVIEW OF RADIOLOGY

3
CLINICAL QUIZ
3. A middle-aged female presents with severe bone pain at multiple sites.
Plain radiograph reveals multiple lytic lesions in pelvis, ribs and femur,
fracture of clavicles and subperiosteal resorption of metacarpals over the
radial aspect (as shown in this hand radiograph). Most likely diagnosis
is: (AIIMS Nov 2014 Pattern)
A. Hyperthyroidism
B. Hyperparathyroidism
C. Renal osteodystrophy
D. Osteomalacia

Acute Osteomyelitis •• Penumbra signQ: Seen in sub-acute osteomyelitis.


On MRI Hyperintense rim seen on T1W images due to
ƒƒ FemurQ – Most common bone involved
vascularised granulation tissueQ at infection site
ƒƒ MetaphysisQ – Most common site of involvement (especially
ƒƒ Thus it is delayed phase that helps differentiate between
in children)
themQ
ƒƒ Bone scansQ – Earliest to diagnose osteomyelitis
ƒƒ Radionuclide studies:
ƒƒ MRIQ – Investigation of choice/Best investigation
• 99mTc-MDP bone scanQ/In 111-WBC ScanQ/Gallium
ƒƒ Radiographic latent period – 10 days - for extremities,
scanQ is used
around 21 days - for spine.
ƒƒ X-ray Findings:
•• Soft tissue changesQ – Swelling, displaced obscured fat
Bone Infections and Systemic Bone Disorders

planes – only findings seen in the 1st week


•• Osteopenia in the bone Postcontrast MRI of Knee
•• Periosteal reactionQ – Variable may be Solid/laminated/ region- Shows a irregular
Codman’s triangle formation peripherally enhancing
ƒƒ MRI: lesion in the femoral
•• Best investigationQ metaphysis – suggests
•• Bone marrow edema (T1 – hypointense, T2 - hyperin- Acute osteomyelitis
tense) with post contrast enhancement

CO Osteomyelitis versus CellulitisQ


NC E P T
• This distinction is achieved by performing a 99mTc–MDP Triphasic
bone scan:
99m
Tc–MDP Triphasic bone scan
Phase Time of image Osteomyelitis Cellulitis
acquisition
Chronic osteomyelitis:
Angiographic/ 1 minute Increased Increased Distal femoral
Flow phase uptake uptake involvement with sclerotic
Blood pool 5 minutes Increased Increased bone and multiple
phase uptake uptake lucent defects in this
bone – suggests cloacae
Delayed 2 – 4 hours Increased No uptake formation (yellow arrows)
phaseQ uptake
196
Chronic Osteomyelitis
ƒƒ Mixed pattern of bone destruction, followed by healing and
periosteal reaction

MUSCULOSKELETAL IMAGING
ƒƒ Sinus tractsQ may open into the skin draining the infected/
necrotic material
ƒƒ SequestrumQ:
•• Dead bone as a result of cortical and medullary infarcts
•• Is avascular and acts as a reservoir for infection
•• Needs excision for definitive treatment of infection
•• Button sequestrumQ: Small sequestrum surrounded MRI STIR image of the same
by a lucent rim. Seen in Osteomyelitis and Eosinophilic patient shows a hyperintense
granulomaQ lesion in proximal tibial
ƒƒ InvolucrumQ: metaphysis with a
•• Thick layer of periosteal new bone formation around hypointense sclerotic rim and
the sequestrum adjacent marrow edema –
ƒƒ CloacaQ: Brodie’s abscess
•• Defect developing in an involucrum
•• Also known as Empyema necessitansQ
•• Saber shin appearanceQ – Osteolytic lesions in tibia
Special Types of Osteomyelitis (gummata) with anterior bowing
ƒƒ Clutton’s jointsQ: Bilateral painless joint swelling at knees
Brodie’s Abscess Q due to synovitis
ƒƒ Hutchinson’s teethQ: Peg shaped, hypoplastic and notched
ƒƒ Localized, aborted form of sub-acute osteomyelitis tooth
ƒƒ Abscess surrounded by sclerosisQ
ƒƒ TibiaQ – Most common site MaduromycosisQ/Madura FootQ
ƒƒ X-ray findings:
•• Oval, elliptical radiolucency >10 mmQ- abscess cavity ƒƒ Chronic granulomatous fungal disease - Tarsometatarsal
regionQ – most commonly involved
•• Halo/Doughnut rim of surrounding reactive sclerosisQ
ƒƒ Dot-in-a-circle signQ: Seen on T2W MRI. Seen as round
•• Also Refer to Q. No. 50 in Image-Based Questions
hyperintensity (of granulation tissue) with a central
hypointense dot (due to fungal products).
  Clinical Pearls
Brodie’s Abscess versus Osteoid OsteomaQ

Bone Infections and Systemic Bone Disorders


  Clinical Pearls
Distinct feature Brodie’s abscess Osteoid osteoma Suppurative/Pyogenic Osteomyelitis versus TB OsteomyelitisQ
Central >10 mmQ in size Usually <10 mmQ Distinct Suppurative/Pyogenic TB Osteomyelitis
radiolucency/nidus in size feature Osteomyelitis
None–as it is Central vascular Clinical Fever, acute onset Insidious onsetQ, very
Central vascularity liquefied abscess blushQ seen on history pain and swelling slow progression,
angiography nonspecific signs &
symptoms
Etiology Staphylococcus Mycobacterium
Congenital Syphilis aureusQ – Most tuberculosis
ƒƒ Also known as Master masqueraderQ – Due to wide common
diversity of signs and symptoms Sequestrum CommonQ Not seen
ƒƒ Phase 1 – MetaphysitisQ: formation
•• Radiolucent metaphyseal bandsQ seen Sinus CommonQ in chronic Not seen
•• Saw-toothed appearanceQ – due to irregularity, formation cases
fragmentation of metaphysis
Psoas Not seen CommonQ
•• Epiphyseal separation may occur
abscess
•• Wimberger’s signQ – Symmetrical erosions on medial
metaphyseal surfaces of proximal tibia Most Femur Disco-vertebral
ƒƒ Phase 2 – PeriostitisQ: common site region of spine
•• Solid/laminated periosteal reaction in a diffuse Osteoporosis Moderate Extensive
symmetrical distribution Periostitis Common Not seen
ƒƒ Phase 3 – OsteitisQ:
End stage/ Bony ankylosis Q
Fibrous ankylosisQ
•• Lytic lesions in diaphysis surrounded by reactive Healing 197
sclerosis and periosteal thickening
Knee joint Postcontrast
radiographs – MRI of a patient
Showing marked with fever,
CONCEPTUAL REVIEW OF RADIOLOGY

joint swelling joint swelling &


(white arrow) pain. Thickened
with irregular peripherally
bone destruction enhancing
(yellow arrows) synovium (yellow
– suggests Sep- arrows) with
tic arthritis. As moderate joint
the metaphyseal effusion suggests
end is involved, – Septic arthritis
it may be
termed as Tom
Smith’s arthritis

SICKLE CELL ANEMIA


Irregular contour of the
ƒƒ Salmonella osteomyelitis in Sickle cellQ (remember Sickle
femoral head articular
Anemia = SAlmonella): surface, with sclerosis
•• Bilateral symmetric diaphyseal involvement and decreased joint space
•• Permeative and moth-eaten destruction with – Avascular necrosis with
Involucrum formation secondary osteoarthritis
ƒƒ Fish vertebraQ/H-shaped vertebraQ/Step-down signQ/
Raynaud’s phenomenonQ:

Coronal T1W MRI


image of Hip joint –
Linear hypointensities
in femoral heads
bilaterally suggests –
Bone Infections and Systemic Bone Disorders

AVN

Kienbock’s disease: Severely


sclerotic appearing Lunate
(Dense on X-ray and
Hypointense on MRI). Compare
appearance of lunate on
MRI (yellow arrows) with the
normal intensity of other bones
like Scaphoid (blue arrow)

THALLASEMIA
ƒƒ Marrow hyperplasia:
•• Coarse trabecular pattern
•• Erlenmeyer flask deformityQ—undertubulation due Osgood-schlatter
to lack of remodeling within long bones disease – Irregular
•• Hair-on-end appearance of SkullQ—vertical radiating appearing
Tibial tuberosity
spicules of new bone formation–most severe and earliest
with bone
seen in Frontal bones, with sparing of occipital bones
fragmentation
ƒƒ Rodent faciesQ—malocclusion due to forwardly displaced
incisors, laterally displaced orbits

198
HEMOPHILIA
A deformed,
ƒƒ If ever you think that the answer to a particular question flattened left sided
is Hemophilia, just check whether the patient is male/ femoral head

MUSCULOSKELETAL IMAGING
female—Hemophilia is seen ONLY in males! epiphysis in a young
ƒƒ Hemophilic pseudotumorsQ: child - Perthes’
•• Expansile bone lesions due to hemorrhage within bone disease
•• Most commonly occur in femurQ followed by pelvis
and tibia
ƒƒ Hemophilic arthropathy
•• Knee jointQ–Most commonly affected
•• Widening of intercondylar notchQ
•• Squared configuration of patellaQ
•• Tibiotalar slant deformityQ

Widening, Cupping and


AVASCULAR NECROSIS OF Fraying of metaphysis
at wrist – Rickets
FEMORAL HEAD
ƒƒ Trauma (resulting a fracture neck of femur) - Most common
cause
ƒƒ Ficat and Arlet classificationQ is used.
ƒƒ X-rayQ—is initial examination–may be normal in early
cases
ƒƒ MRIQ—Best investigation
ƒƒ Radiologic features:
•• Anterosuperior weight bearing portionQ of femoral
head is involved
•• Snow cap signQ—homogenously increased density in
this region
•• Bite signQ—snow cap sign with a curvilinear inferior
margin
•• Crescent/Rim signQ—subchondral fracture just
beneath the reticular surface

Bone Infections and Systemic Bone Disorders


•• MRI Crescent signQ—similar curvilinear high signal Appearance of band
intensity zone in subchondral bone–seen earlier than of metaphyseal
on X-ray calcification – 1st sign
•• Asterisk signQ—on CT, normal stellate density of the of Healing Rickets
bony trabeculae becomes fragmented
•• Joint degeneration
•• Double line signQ—on MRI, 2 opposing lines of high
and low signal intensity are seen at interface of viable
and necrotic bone •• Gage’s signQ—lucent defect in lateral epiphysis and
adjacent metaphysic
•• Sagging rope signQ—enlarged femoral head casts a
LEGG-CALVE-PERTHES DISEASE linear shadow over the metaphysic

ƒƒ Avascular necrosis of femoral capital epiphysis before


closure of epiphyseal plateQ OSTEOCHONDRITIS/
ƒƒ Occurs in the vulnerable vascular period of 4–7 yearsQ, Male
> Female OSTEOCHONDROSIS/EPIPHYSEAL
ƒƒ
ƒƒ
MRIQ–Best investigation
Radiologic findings:
DISORDERS
•• Waldenstrom’s signQ— There is a long list of named osteochondritis, a detailed
•• Crescent/Rim signQ— discussion is beyond the scope of this book. What you need
•• Sclerosis of femoral head—head-within-head to know for your exams are the names of the most important
appearanceQ or Snow-cap appearanceQ conditions and peculiar imaging features of a select few!

199
Disease Involves the Disease Involves the
Legg-calve-perthes disease Femoral head in children Van neck’s disease Ischiopubic synchondrosis
Chandler’s disease Femoral head in adults Osteochondritis dissecans/ Femoral condyles in a child
CONCEPTUAL REVIEW OF RADIOLOGY

Osgood-schlatter disease Tibial tubercle Koenig’s disease

Blount’s disease Medial tibial condyle Kohler’s disease of patella Primary centre of Patella

Sever’s disease Calcaneal apophysis Sinding-Larsen-Johansson Secondary centre of Patella


disease
Kohler’s disease Navicular–in children
Iselin’s disease 5th metatarsal base
Müller-Weiss disease Navicular–in adults
Diaz’s disease Talus
Freiberg disease/ Metatarsal head (most
Kohler’s number 2 disease commonly 2nd metatarsal) Hass’s disease Humeral head

Kienbock’s disease Lunate Panner’s disease Capitellum of humerus

Scheuermann’s disease Vertebral endplate ring Kummel’s disease Vertebral body in adult
apophysitis–Limbus bones seen Ellman’s disease Radial head
Contd…

RICKETS
ƒƒ Vitamin D deficiency - Most common cause
ƒƒ 6–12 monthsQ–Most common age group affected
ƒƒ Radiologic features: You will find the list of appearances in rickets in all your books and you must have already memorized it
during your pediatric clinics. But we will study the development of these appearances in correlation to the pathophysiology of
Rickets.

CO RICKETS: Pathophysiology and Corresponding Imaging Appearances


NC E P T

Physiology Normal Rickets


STEP 1: Chondroblasts located at the • Normal lucent appearing physeal plate • This step is unaffected and proceeds as
physeal growth plate secrete Chondroid of cartilage is created normal
Bone Infections and Systemic Bone Disorders

matrix
• Metaphyseal margin becomes well • Due to Vit. D deficiency this step is
↓ defined/sharp arrested.
STEP 2: Vitamin D induces calcification • This calcification giver the metaphysis • Calcification does not occur
of Chondroid matrix towards the its structural strength • Ill-defined metaphyseal margin–
metaphyseal margin Fraying/Paint-brush MetaphysisQ
↓ • Structurally weak metaphysis subjected
STEP 3: Calcification of chondroid to stress–WideningQ
matrix prevents diffusion of nutrients • Structurally weak metaphysis subjected
through the matrix. The nutritionally to stress–CuppingQ
deprived Chondroblasts are converted • This chondroid cell apoptosis prevents • Excess cartilage deposition–Widening
to Chondrocytes and with further dense excess cartilage deposition and ensures of physeal plate of cartilageQ
calcification eventually die through that the physeal plate remains normal • Excess cartilage deposition–
apoptosis in appearance and thickness Enlargement of costochondral junction
↓ of ribs–Rickety RosaryQ
STEP 4: Osteoblasts and Osteoclasts • Thus you will realize that starting from • Generalized poor bone calcification and
appear at this site – ensure remodeling of Cartilage at physeal plate in Step 1, we softening of bones–Bowing of weight
calcified chondroid matrix and convert it have arrived at adult osteoid matrix– bearing bonesQ
into Osteoid thus completing bone growth at the • Generalized poor bone calcification–
metaphyseal end radiolucent bones with coarsened
trabecular patternsQ

Contd…

200
Thus by understanding the Concept of pathophysiology of Rickets, Not only this we can also automatically predict the Radiographic
we can now conceptually explain each and every manifestation of sign of Healing Rickets:
Rickets: • So in Rickets, due to Vitamin D deficiency, STEP 2 is arrested.

MUSCULOSKELETAL IMAGING
• WideningQ • Most important treatment of Rickets is to administer Vitamin
• CuppingQ of Metaphysis D in therapeutic doses
• FrayingQ • As soon as Vitamin D is available STEP 2–will resume–so the
• Widening of physeal plate of cartilageQ metaphyseal margin will get calcified
• Rachitic RosaryQ • Thus: Re-appearance of Opaque/calcified metaphyseal
• Bowing of weight bearing bonesQ lineQ–is the 1st sign of healing Rickets.
• Radiolucent bones with coarsened trabecular patternsQ This is an apt example of “ Radiology being a CONCEPTUAL
Re-appearance of Opaque/calcified metaphyseal lineQ–is the Subject” and not merely about named signs that needs to be
1st sign of healing Rickets. mugged up!. Its magical, isn’t it?

OSTEOMALACIA HYPERPARATHYROIDISM (HPT)


ƒƒ Adult rickets–Lack of osteoid mineralization leading to
generalized bone softening
Round/oval,
ƒƒ Radiologic features:
solitary,
•• Generalized osteopenia–decreased bone density with geographic
coarse trabecular pattern lytic lesion
•• Pseudo-fractures Q /Looser’s zones Q /Milkman’s with endosteal
syndromeQ/Increment fracturesQ/Umbau zoneQ: scalloping –
 Bilateral, symmetrical linear radiolucencies Eosinophilic
 Femur neck–Most common site granuloma
 Pubic and Ischial rami/Ribs/Axillary margins of
scapulae–common sites

SCURVY
ƒƒ Radiologic findings:
•• Generalized osteopenia
•• Wimberger signQ/Ring epiphysisQ: radiolucent

Bone Infections and Systemic Bone Disorders


central area surrounded by a dense margin
•• White line of FrankelQ: Dense zone of provisional
calcification at metaphyseal margin
•• Trummerfeld zone of rarefactionQ/Scorbutic zoneQ:
ACROMEGALY
Lucent band just below the metaphyseal zone of ƒƒ Imaging features:
provisional calcification •• Heel pad thickness >20 mmQ: Seen on a lateral foot
•• Pelken spursQ: Lateral metaphyseal spurs radiograph
•• Corner signQ/Angle signQ •• Skull changes:
•• Subperiosteal hemorrhagesQ  Ballooning/enlargement of sella turcicaQ–Best
seen on a lateral skull radiograph
 Enlargement/Hyperpneumatization of parana-
Radiologic Signs of Scurvy– sal sinusesQ
CO
NC E P T Biochemistry Connection  Occipital protuberance overgrowth and calvarial
thickening
• Scurvy is due to deficiency of Vitamin C
• Vitamin C/Ascorbate–acts as a co-enzyme for cross-linking of  Widened mandibular angle–Prognathic jawQ
collagen fibers. •• Hand changes:
• Cross-linking of Collagen: Involves hydroxylation of LysineQ and  Spade shaped phalanxQ
ProlineQ residues of collagen •• Pituitary imaging:
• Vitamin C/Ascorbate is also involved in biosynthetic pathway of  Macro-adenoma–reveals a typical sellar lesion with
Carnitine. a Snowman’s appearance/Figure of 8 appear-
• Impaired synthesis of Collagen and Carnitine results in weak anceQ
collagen thus affecting Blood vessels, Bones and Joints.
 Microadenoma–Dynamic contrast enhance MR
• All manifestations of Scurvy are reversible with Vitamin C
imaging is required
therapy–except Frankel’s line–that may remain in the metaphysis
as a single growth arrest line. 201
ƒƒ Spine:
EOSINOPHILIC GRANULOMA •• Abnormal vertebral body shape:
ƒƒ Disorder of immune regulation - characterized by abnormal  Vertebra planaQ/Pancake vertebraQ: Compres-
sion deformity with loss of both anterior and
CONCEPTUAL REVIEW OF RADIOLOGY

proliferation of reticuloendothelial cells–predominantly the


Langerhans cell/Histiocyte posterior vertical heights–not very common with
ƒƒ SkullQ—most commonly involved osteoporosis
•• Imaging findings:  Wedged vertebraQ: Loss of anterior vertical height
with preservation of posterior height.
 Seen as round/oval, solitary, geographic lytic
 Biconcave endplate deformity – (Recent Pattern
lesionsQ with endosteal scalloping
Jan 2020): Fish vertebraQ/Cod-fishQ/Fish-
 Hole-within-hole appearanceQ/Bevelled edge
mouthQ/Bi-concaveQ/Hourglass vertebraQ–due
signQ
to pressure of the central nucleus pulposus on the
 Button sequestrumQ—central focus of bone may weakened bone
remain unaffected within a lytic lesion ƒƒ Insufficiency fractures:
 Floating teeth signQ—Lytic lesion around a tooth •• Fractures occurring in weakened bone that is unable to
 Ghostly appearance of vertebra–involvement handle normal physiological stress
of neural arch, destroying the internal matrix but •• Spine, hip and distal radius–most common sites
sparing the cortical outline •• Bone scans–Most sensitive modality for Insufficiency
 Vertebra planaQ/Silver dollar vertebraQ/Coin fractures
on edge vertebraQ: Pathologic fractures of the •• Sacral insufficiency fracture:
vertebral body  Honda signQ/H patternQ/Butterfly signQ:
Bilateral vertical fractures through sacral ala are
connected through a transverse fracture
OSTEOPOROSIS/OSTEOPENIA
ƒƒ Reduction in bone quantityQ, with the bone quality   Clinical Pearls
remaining normalQ.
ƒƒ Most accurate method to diagnose osteoporosis is by Osteoporosis versus Osteomalacia
measuring Bone Mass/Bone mineral densityQ Distinct feature Osteoporosis Osteomalacia
ƒƒ Dual energy X-ray AbsorptiometryQ—most common and Quantity– Decreased quantity Decreased quality
Most reliable modality used. Quantitative CT (Q-CT may quality but normal quality but normal quantity
also be used) of bone of bone
ƒƒ Imaging findings: General:
Bone mineral: Normal Decreased
•• Osteopenia with cortical thinning: termed as matrix ratio
“Pencil thin cortex” Q.
Bone Infections and Systemic Bone Disorders

•• Altered trabecular patterns–“washed-out” Q appear- Etiology Senile/Post- Vitamin D deficiency–


ance of bones menopausal–most most common cause.
common cause
•• Fractures/Deformities

3
CLINICAL QUIZ - SOLUTION
A finding of “subperiosteal resorption” especially involving the ƒƒ Decreased bone density:
radial aspects of hand bones – is a clincher of Hyperparathyroidism.  Salt and pepper skullQ/Pepper pot skullQ–granular
Hence answer is B. appearance of skull due to resorption
• Excess of parathormone results in excess of osteoclastic activity– ƒƒ Brown tumorsQ:
leading to peculiar skeletal findings  Not actually tumors, but result from excess focal osteoclastic
• Associated with Multiple endocrine neoplasia (MEN) type I and activity seen as geographic lucent lesion located centrally,
type IIa with few internal septae within mandible, pelvis, ribs,
ƒƒ Imaging features: femur
ƒƒ Subperiosteal bone resorptionQ: Refer to Image 48 om ƒƒ Rugger-Jersey spineQ:
“Imaging Signs in Radiology”  Sub-endplate sclerosis interspersed with lucency–gives a
 Hallmark/Single most definitive diagnostic sign of HPTQ very peculiar appearance
 Resorption occurs at the outer cortex at insertional points ƒƒ Floating teethQ: Resorption of Lamina dura (cortical bone)
of ligaments and tendons around the tooth socket
 Most common sites–Radial margins of middle and ƒƒ Acro-osteolysisQ may be seen
proximal phalanges of 2nd digit.Q ƒƒ Rib notching–usually superior margin

202
Skeletal Dysplasias and Miscellaneous
Musculoskeletal Imaging

MUSCULOSKELETAL IMAGING
4
CLINICAL QUIZ
4. A lady came for routine ultrasound anomaly scan at 19 weeks of
pregnancy. The fetus showed sever long bone shortening. On follow-
up scans the shortening worsened with bone fractures and deformities
becoming evident on antenatal USG scans. A postnatal Infantogram is
shown here. Most likely diagnosis is:
A. Thanatophoric dysplasia
B. Achondroplasia
C. Marfan syndrome
D. Osteogenesis imperfecta

Skeletal Dysplasias and Miscellaneous Musculoskeletal Imaging


This topic is indeed a treasure of named signs and •• Absent/Hypoplastic claviclesQ
appearances! •• Midline defect at pubic symphysis
•• Chef’s hat femoral headQ

ACHONDROPLASIA
  Clinical Pearls
ƒƒ Most common congenital dwarfismQ
ƒƒ Imaging features: (Remember Tyrion Lannister in Game of Wormian Bones
thrones!!) These are also termed as Intra-sutural bonesQ–bones seen within
•• Foramen magnum stenosis the cranial sutures most commonly the lambdoid sutures. Seen
•• BrachycephalyQ—large cranium with a short AP in: (Remember as PORK-CHOPS):
dimension • Pyknodysostosis
•• Rhizomelic dwarfismQ—symmetric shortening of all • Osteogenesis imperfecta
• Rickets–healing phase
long bones resulting in short stature
• Kinky hair syndrome
•• Trident handsQ • CCD
•• Refer to Image 44 in “Imaging signs in Radiology” • Hypothyroidism, hypophosphatasia
•• Tombstone pelvisQ/Champagne glass pelvisQ • Otopalatodigital syndrome
•• Mickey mouse ear iliac wingsQ • Primary acro-osteolysis, pachydermo periostitis
•• Bullet-nosed vertebral bodiesQ with gibbus forma- • Syndrome–Down’s syndrome
tion

CLEIDOCRANIAL DYSPLASIA/ MARFAN’S SYNDROMEQ


DYSOSTOSIS (CCD) ƒƒ Imaging features:
ƒƒ Characterized by:
•• ArachnodactylyQ/Spider like fingersQ:
•• Skull abnormalities  Elongated fingers without an increase in width
•• Clavicle abnormalities  Metacarpal IndexQ: ratio of length to width of
•• Midline defects metacarpals–is increased
ƒƒ Imaging findings: •• Spine:
•• Hot-cross-bun skullQ–due to widening of sagittal and  Tall vertebral bodies
coronal sutures  Scoliosis/Kyphoscoliosis
•• Persistent metopic suture  Atlantoaxial subluxation
•• Multiple Wormian bonesQ •• Pectus excavatum 203
•• Shepherd’s crook deformity of proximal femurQ
•• Zebra stripe signQ–Cyclical treatment with
Achondroplasia with
bisphosphonates produces band like sclerotic growth
Tomb-stone pelvis, Mickey
arrest lines in long bones
CONCEPTUAL REVIEW OF RADIOLOGY

mouse iliac wings and


horizontally oriented
acetabuli
  Clinical Pearls
Clinical Pearls - OGI versus Child AbuseQ
Whenever you seen pediatric fractures (usually multiple), you
must always think of a possibility of child abuse.
• It would however be dangerous to label a diseased child
(suffering from OGI) as that of Child abuse, and similarly vice
versa.
OGI versus Child abuse
OGI Child Abuse
Blue sclerae Present (Except in Absent
type IV)
Wormian bones Present Absent
Metaphyseal corner Uncommon. Very common/
fractures Fractures are Specific for
mainly diaphyseal child abuse
Absent clavicles – Antenatal fractures Seen None
Cleidocranial
dysplasia (CCD) Urinary Increased Q
NormalQ
Skeletal Dysplasias and Miscellaneous Musculoskeletal Imaging

hydroxyproline levels
Bone density DecreasedQ NormalQ
(by DEXA/
•• Cardiac: Quantitative CT)
 Aortic dissectionQ
Dentinogenesis Present Absent
 Aneurysmal dilatation of aortic rootQ
imperfecta
 Mitral regurgitation
 ASD

NAIL-PATELLA SYNDROMEQ MELORHEOSTOSIS


ƒƒ Also known as Flowing hyperostosisQ/Leri’s diseaseQ
ƒƒ Imaging findings:
ƒƒ Imaging features:
•• Iliac hornsQ: Pair of central posterior iliac horns, formed
•• Flowing candle waxQ/MoltenQ/Dripping candle
from separate ossification centers–is pathognomonic.
wax appearanceQ–Wavy/streaked periosteal cortical
Also known as Fong’s prongsQ
thickening along one side of a long bone
•• Small/absent patellaQ

OSTEOGENESIS IMPERFECTA (OGI) OSTEOPETROSIS


ƒƒ Also known as Albers–Schonberg diseaseQ/Marble bone
ƒƒ Due to quantitative and qualitative defects in synthesis of
diseaseQ/Chalk bone diseaseQ
collagen type 1Q
ƒƒ Imaging features:
ƒƒ Imaging findings:
•• Generalized osteosclerosisQ–increased density
•• Thin and Gracile bonesQ—Diffuse osteopenia with
•• Bone-within-bone appearanceQ/EndobonesQ
pencil thin cortices. Short, thick bones and sometimes
•• Skull base sclerosis, Dental caries–predisposing to
cystic bones may also be seen
mandibular osteomyelitis
•• Multiple fracturesQ–Transverse fractures especially
•• Sandwich vertebraQ–sclerotic vertebral end plates
involving lower limb bones–is characteristic
Now that we have covered most of the Major/Important
•• Antenatal detection of fractures–is also pathognomonic
dysplasias, let us have fleeting review of the rest of them.
•• Persistent Wormian bones
Remember the most peculiar features of these entities; this will
help you solve specific MCQs.

204
MUSCULOSKELETAL IMAGING
Osteogenesis imperfecta Osteopetrosis – Bone within Bone appearance Mucopolysaccharidosis – Proximally
– Thin, gracile, deformed pointed metacarpals
bones

ƒƒ Picture frameQ/Squared-off vertebraQ—enlarged vertebral


Craniosynostosis • Isolated sagittal suture closure—most
common body with a rim of thickened cortex
ƒƒ Ivory vertebraQ—excessively radio-opaque vertebra
Mucopolysacchari- • Enlarged J shaped sella turcicaQ
ƒƒ Rim sign/Brim signQ—thickened cortex along pelvic brim,
doses–MPS 1: • Inferiorly beaked vertebraQ
obliterating the Kohler’s teardrop

Skeletal Dysplasias and Miscellaneous Musculoskeletal Imaging


Hurler syndrome • Paddle/spatulated ribsQ–broad/wide
ribs ƒƒ Blade of grass appearanceQ/Candle flameQ/V–shaped
• Proximally pointed metacarpalsQ appearanceQ—long lucent defect beginning in sub-
• Thoracolumbar kyphosis articular region and extending down along the long bone.
• Humerus–varus deformity ƒƒ Banana fractures—pathological fractures
Mucopolysaccha- • Platyspondyly –flat vertebral bodies
Q

ridoses–MPS IV: at all levels Great imitators of Bone DiseaseQ


Morquio syndrome • Centrally beaked vertebraQ ƒƒ Paget’s disease and
• Pectus carinatumQ–due to a horizontal ƒƒ Fibrous dysplasia
sternum
• Simian pelvisQ Fibrous DysplasiaQ
• Odontoid agenesis
• Poorly formed acetabuli and femoral ƒƒ Bone undergoes resorption and is replaced with abnormal
heads fibrous tissue
ƒƒ Presents at 8–14 yearsQ
ƒƒ Basic radiologic features:
Paget Disease/Osteitis DeformansQ •• Diametaphyseal elongated lesionsQ, sparing the sub-
ƒƒ Basic radiologic features of Paget’s disease– articular region
•• Thickened cortex with coarsened trabeculaeQ •• Geographic lucent lesionsQ with ground glass
•• Bone expansionQ appearanceQ/smoky appearanceQ
•• Increased/decreased densityQ •• Thick sclerotic margin around lesionQ
•• Long lesions in boneQ
•• May show soap-bubble appearanceQ
•• Bone expansion with cortical thinning and scalloping
•• Bone deformitiesQ
•• Pathological fractures
ƒƒ Osteoporosis circumscriptaQ—geographic bone resorption
ƒƒ Shepherd crook’s deformityQ—reduction in femoral angle
involving the skull
to almost 900, thus creating a coxa vara deformity
ƒƒ Cotton wool appearance of skullQ—reparative sclerosis ƒƒ Increment fracturesQ—repeated fractures occurring along
occurs, with irregular radiopacities the length of bone at regular intervals

205
  Clinical Pearls
Refer to Image 13 in “Imaging Signs in Radiology”
CONCEPTUAL REVIEW OF RADIOLOGY

Inferior Rib NotchingQ Superior Rib NotchingQ


• Arterial causes • Connective tissue diseases
ƒƒ Coarctation of the aortaQ ƒƒ Rheumatoid arthritis
 Affects 4th -8th ribs bilaterally. ƒƒ Systemic lupus erythematosus
 Unilateral and right-sided if the coarctation is proximal to the left subclavian ƒƒ Scleroderma
artery. ƒƒ Sjögren’s syndrome.
 Unilateral and left-sided if associated with an anomalous right subclavian • Metabolic
artery distal to the coarctation. ƒƒ Hyperparathyroidism
ƒƒ Aortic thrombosis • Miscellaneous
ƒƒ Subclavian obstruction ƒƒ Neurofibromatosis
• Venous causes ƒƒ Restrictive lung Disease.
ƒƒ Superior vena caval obstruction. ƒƒ Poliomyelitis.
• Arteriovenous causes ƒƒ Marfan’s syndrome
ƒƒ Pulmonary arteriovenous malformation ƒƒ Osteogenesis imperfecta
ƒƒ Chest wall arteriovenous malformation. ƒƒ Progeria.
• Neurofibromatosis – Ribbon ribsQ also seen
NeurofibromatosisQ–causes both superior and inferior rib notchingQ

4
Skeletal Dysplasias and Miscellaneous Musculoskeletal Imaging

Fibrous dysplasia
– Long lesion in a CLINICAL QUIZ - SOLUTION
long bone, with
ground glass Severe bone shortening and detection of deformities and
haziness fractures in-utero is pathognomonic of Osteogenesis inperfecta.
If you observe the bones carefully you will see multiple fractures
and deformities making this a spotter !

206
Multiple Choice Questions

MUSCULOSKELETAL IMAGING
Bone and Joint Infection 16. Cumulus cloud appearance is seen in:
1. X-ray finding of osteomyelitis in the 1st 7 days is: A. Multiple myeloma B. Osteosarcoma
A. Cystic swelling B. Soft tissue swelling C. Ewing’s sarcoma D. Adamantinoma
C. New bone formation D. Sequestrum formation 17. Coat hanger is the term used to describe:
2. Earliest modality to diagnose Osteomyelitis is: A. Osteosarcoma B. Parosteal osteosarcoma
A. X-ray B. CT scan C. Enchondroma D. Osteochondroma
C. MRI D. Bone scan 18. Tumors showing Soap bubble appearance includes all except:
3. Best imaging modality for diagnosis of Osteomyelitis is: A. Chondromyxoid fibroma
A. X-ray B. CT scan B. Giant cell tumor
C. MRI D. Bone scan C. Aneurysmal bone cyst
4. Penumbra sign is a feature of: D. Desmoplastic fibroma
A. Acute osteomyelitis B. Sub-acute osteomyelitis 19. A young male complaining of severe night pain with marked
C. Septic arthritis D. Neuropathic joint sclerosis seen in the tibia with central lucency seen in cortical
5. On a 99mTc-MDP Triphasic bone scan which of the following region, the most likely diagnosis is:
phase helps differentiate between Osteomyelitis and A. Osteoid osteoma B. Osteosarcoma
Cellulitis: C. Brodie’s abscess D. Osteomyelitis
A. Flow phase B. Blood pool phase 20. Double density sign on skeletal scintigraphy is characteristic
C. Delayed phase D. All of above sign of which of the following tumor:
6. Dot-in-a-circle sign is seen in: A. Osteochondroma B. Osteoid osteoma
A. Actinomycosis B. Madura foot C. Osteosarcoma D. Brodie’s abscess
C. TB osteitis D. Tom smith arthritis 21. Falling fragment sign is pathognomonic of which of the
7. Spina Ventosa is: following:
A. Ventral spinal defect B. Dorsal spinal defect A. Simple bone cyst B. Aneurysmal cyst
C. TB of short tubular bones C. Giant cell tumor D. Fibrous cortical defect
D. Pyogenic osteomyelitis of spine 22. Polka dot appearance on axial CT images is seen in which of
8. Dead bone on an X-ray appears: the following vertebral tumor:
A. More radiopaque B. Radiolucent A. Hemangioma B. Osteoblastoma
C. Less audio opaque D. Is not seen at all C. Metastases D. Multiple myeloma
9. Radiologically earliest sign of osteomyelitis is:  23. Modality of choice for screening of bone metastases:
A. Loss of muscle and fat planes  (CET Nov 2014) A. Plain radiograph B. CT
B. Periosteal reaction C. MRI D. Bone scan
C. Callus formation 24. All are true regarding Codman’s triangle except:
D. Presence of sequestrum A. It is a specific feature of osteosarcoma
B. Suggests extension of the tumor process beyond cortex
Bone Tumors C. It demarcates the proximal extent of the extraosseous tumor
10. Which is the most common primary bone tumor? D. May be seen with Ewing’s sarcoma
A. Osteosarcoma B. Multiple myeloma 25. A 14-year-old boy presents with fever, leukocytosis and pain

Multiple Choice Questions


C. Chondroblastoma D. Ewing’s sarcoma in arm. X-ray of arm shows a moth-eaten appearance of the
11. Fallen fragment sign is seen in: diaphysis with cortical destruction, onion peel periosteal
A. ABC B. GCT reaction and cortical saucerization with a large soft tissue
C. SBC D. Osteosarcoma mass. Most likely diagnosis is:
12. Investigation of choice to identify bone marrow involvement A. Osteosarcoma B. Osteomyelitis
is: C. Ewing’s sarcoma D. Langerhans cell histiocytosis
A. CT scan B. MRI 26. Osteoid osteoma is characterized by:
C. Radio isotope studies D. MR spectroscopy A. Night pain relieved by aspirin
13. Lamellar periosteal reaction is seen in: B. Diaphysis of long bone is site of predilection
A. Ewing Sarcoma. B. Osteomyelitis. C. Radiolucent are with sclerotic margin, containing nidus
C. Osteosarcoma D. All of the above D. All of the above
14. Most common bone tumor is: 27. All of the following are true about exostosis except:
A. Metastasis B. Osteosarcoma A. Osseous outgrowth arising from bony cortex
C. Multiple myeloma D. Hemangioma B. When multiple known as diaphyseal aclasis
15. Cockade sign is seen in: C. Malignant risk is 10%
A. Osteoid osteoma B. NOF D. Thickness of cartilaginous cap >1 cm should be viewed
C. Intraosseous lipoma D. Chondrosarcoma with suspicious of malignancy
207
28. Osteoid osteoma of spine affects: 42. Bechterew disease is:
A. Vertebral body B. Neural arch A. Behçet’s Syndrome B. Caisson’s disease
C. Both D. Does not occur in spine C. Ankylosing spondylitis D. Tuberous sclerosis
CONCEPTUAL REVIEW OF RADIOLOGY

29. Winking owl sign is seen in: (JIPMER May 2018) 43. Osteoarthritis is characterized by:
A. Vertebra hemangioma A. Peripheral or central osteophytes
B. Osteoblastoma B. Subarticular cyst
C. Vertebral metastasis C. Joint narrowing
D. Vertebral fracture D. All
30. A 76-year-old man presents with lytic lesion in the vertebrae. 44. Secondary osteoarthritis is seen in:
X-ray skull showed multiple punched out lesions. The A. Aseptic necrosis B. Perthes’ disease
diagnosis is: C. Paget’s disease D. All of the above
A. Metastasis B. Multiple myeloma 45. Osteoarthritis most commonly affects which of the following
C. Osteomalacia D. Hyperparathyroidism joint:
A. Hip B. Shoulder
Arthritis C. Knee D. Spinal
31. The joint most commonly affected in Rheumatoid arthritis is:
A. Shoulder B. Knee Systemic
C. Sacroiliac D. Wrist 46. Lytic lesions of skull vault with beveled edges are seen in:
32. Not a component of Felty syndrome:  (MH 2006 and AIIMS 1997)
A. Splenomegaly B. Neutropenia A. Eosinophilic granuloma B. Metastasis
C. Pneumoconiosis D. Rheumatoid arthritis C. Multiple myeloma D. Hyperparathyroidism
33. Calcification of interspinous ligament seen in Ankylosing 47. Pelkan spur is seen in:  (AI 2014)
spondylitis is known as: A. Rickets B. Scurvy
A. Tram trolley sign B. Romanus lesion C. Hemophilia D. All
C. Dagger sign D. Mickey mouse sign
48. Not a feature of Rickets: (MH 2009)
34. Dot-Dash appearance is a feature of: A. Cupping of metaphysis B. Widening of metaphysis
A. Rheumatoid arthritis B. Psoriatic arthritis C. Fraying of metaphysis D. Subluxation of epiphysis
C. Reiter’s syndrome D. Osteoarthritis
49. Looser’s zone is seen in:  (NEET Pattern 2012)
35. Which is not feature of osteoarthritis? A. Osteoporosis B. Osteopetrosis
A. Osteophytosis B. Joint space reduction
C. Osteomalacia D. Osteochondrosis
C. Enthesitis D. Sclerosis
50. Flaring of anterior ends of ribs is seen in:  (AI 2008)
36. All of the following findings are in favor of an inflammatory
A. Neurofibromatosis B. Rickets
joint pathology except:
C. Scurvy D. Hypothyroidism
A. Juxta-articular osteoporosis
51. Acro-osteolysis is seen in:
B. Non-uniform joint space loss
A. Gout B. Hyperparathyroidism
C. Bone erosions
C. Multiple myeloma D. Amyloidosis
D. Bilateral symmetric involvement
37. Overhanging margin sign is seen in: 52. Hair-on-end appearance is seen in: (AIIMS May 2008)
A. CPPD B. Gout A. Thalassemia B. Sickle cell disease
C. AS D. RA C. Hemophilia D. Megaloblastic anemia
38. Chondrocalcinosis is seen in: 53. The gold standard for the diagnosis of osteoporosis is: 
A. Gout B. CPPD A. Single energy X-ray absorptiometry (AI 2013, AIIMS 2004)
Multiple Choice Questions

C. Wilson’s disease D. All of above B. Dual energy X-ray absorptiometry


39. Licked candy stick appearance of bone is a feature of: C. Ultrasound
A. Degenerative joint disease D. Quantitative CT
B. Rheumatoid arthritis 54. H-shaped vertebra is typically seen in:
C. Neuropathic joint A. Thalassemia B. Sickle cell disease
D. DISH C. Hemochromatosis D. Ankylosing spondylitis
40. Most common site of OPLL is: 55. Kienbock’s disease involves:
A. Cervical spine A. Scaphoid B. Hamate
B. Thoracic spine C. Lunate D. Pisiform
C. Lumbar spine 56. Osteochondritis involving the tibial tuberosity is known as:
D. Sacrum A. Kummel’s disease
41. ‘Cup and Pencil’ appearance of joints (Opera Glass deformity) B. Kienbock’s disease
is seen in: C. Osgood-Schlatter disease
A. Reiter’s syndrome D. Sever’s disease
B. Psoriatic arthritis 57. Single most diagnostic sign of Hyperparathyroidism is:
C. Ankylosing spondylitis A. Subarticular erosion B. Subperiosteal erosion
D. Joints in ulcerative colitis C. Pathological fractures D. Soft tissue calcification
208
58. Heel pad thickness >20 mm is diagnostic of: 74. Marble bone is characteristic of:
A. Gaucher’s disease B. Scurvy A. Osteopetrosis
C. Acromegaly D. Cushing’s disease B. Marfan’s syndrome

MUSCULOSKELETAL IMAGING
59. Spade shaped phalanx is a feature of: C. Osteogenesis imperfecta
A. Acro-osteolysis B. Hyperparathyroidism D. Melorheostosis
C. Acromegaly D. Osteomyelitis 75. “Flowing wax” appearance on anterior and posterior borders
60. Hole-within-hole appearance is seen in: of vertebrae with normal intervertebral disc space occurring
A. Multiple myeloma B. Acromegaly due to ligament calcification is seen in: (CET Nov 2014)
C. Eosinophilic granuloma D. Ewing’s sarcoma A. Ankylosing spondylitis 
61. Pathognomonic feature of hyperparathyroidism: B. Diffuse idiopathic skeletal hypertrophy
A. Osteopenia C. RA
B. Loss of Lamina dura D. Psoriatic arthropathy
C. Brown’s tumor
D. Subperiosteal resorption of phalanges Miscellaneous Musculoskeletal Imaging
62. A 40-year-old male patient on long-term steroid therapy 76. Picture frame vertebra is seen in:
presents with recent onset of severe pain in the right hip.
A. Hyperparathyroidism
Imaging modality of choice for this problem is:
B. Osteopetrosis
A. CT scan B. Bone scan
C. Paget’s disease
C. MRI D. Plain X-ray
D. Metastasis
Skeletal Dysplasia 77. Blade of grass appearance is a feature of:
63. Champagne glass pelvis is seen in: A. Hyperparathyroidism B. Osteopetrosis
A. CDH B. Down’s syndrome C. Paget’s disease D. Metastasis
C. Cretinism D. Achondroplasia 78. Ground glass opacity within bone is seen in:
64. Trident hand is a feature of: A. Hyperparathyroidism B. Osteopetrosis
A. Achondroplasia B. Mucopolysaccharidosis C. Paget’s disease D. Fibrous dysplasia
C. Diaphyseal aclasia D. Cleidocranial dysplasia 79. Great imitators of bone disease are:
65. Bone within bone appearance is a feature of: A. Paget’s disease B. Fibrous dysplasia
A. CML B. Osteoporosis C. Both D. None
C. Osteopetrosis D. Bone infarct 80. Bare orbit sign is seen in:
66. Molten wax appearance is seen in:  (AI Dec 2014) A. Tuberous sclerosis
A. Osteoporosis B. Osteopoikilosis B. Neurofibromatosis
C. Melorheostosis D. Osteogenesis imperfecta C. Sturge Weber syndrome
67. Fong’s prongs are seen in: D. Dandy Walker syndrome
A. Marfan syndrome B. Achondroplasia 81. Investigation of choice to diagnose congenital hip dislocation
C. Nail-Patella syndrome D. Epiphyseal dysplasia in neonates:
68. Antenatal detection of bone fractures on USG suggests: A. Plain radiographs B. US
A. Achondroplasia C. CT D. MRI
B. Marfan syndrome 82. Ossification of posterior longitudinal ligament is seen in:
C. Osteogenesis imperfecta A. As isolated phenomenon in Japanese
D. Chondrodysplasia punctata B. In Ankylosing spondylitis
69. Sandwich vertebra is seen in: C. In Diffuse idiopathic skeletal hyperostosis

Multiple Choice Questions


A. Hyperparathyroidism B. Hypoparathyroidism D. All of the above
C. Osteopetrosis D. Osteoporosis
83. Osteoporosis circumscripta is a feature of:
70. Spool-shaped vertebrae are seen in: A. Perthe’s disease B. Pyknodysostosis
A. Osteopetrosis B. Pyknodysostosis C. Paget’s disease D. Prune belly syndrome
C. MPS D. Pott’s spine 84. Cherubism is term used for:
71. Acro-osteolysis is seen in: A. Osteoclastoma of jaw
A. Pyknodysostosis B. CCD B. Familial form of fibrous dysplasia of jaw
C. Hyperparathyroidism D. All of the above C. Exostosis of midline of palate
72. Inferiorly beaked vertebra is a feature of: D. None
A. Hurler’s syndrome B. Morquio syndrome 85. Madelung’s deformity not seen in:
C. Sickle cell disease D. Gaucher’s disease A. Leri-Weill syndrome
73. Thin osteoporotic fragile bones with dental abnormalities B. Turner syndrome
and lax joints is characteristic of: C. After trauma or infection
A. Osteogenesis imperfecta D. Down syndrome
B. Marfan’s syndrome 86. Wormian bones are seen in all except:
C. Idiopathic juvenile osteoporosis A. Hypothyroidism B. Down syndrome
D. Homocystinuria C. Turner syndrome D. Osteogenesis imperfecta
209
87. Bare orbit is seen in: 89. A person has an injury in the forefinger with glass and it is
A. NF-1 B. Osteomyelitis suspected that he has a retained piece of glass in his finger.
C. VHL D. Tuberous sclerosis What is the first investigation you will do:
CONCEPTUAL REVIEW OF RADIOLOGY

88. Which of the following is not true regarding Ossified posterior A. MRI
longitudinal ligament (OPLL)? B. CT scan
A. Most commonly involves’ thoracic spine C. Plain radiograph
B. Gradient echo MR sequence may overestimate the canal D. Ultrasonography
stenosis
C. MRI is best for diagnosis
D. Low signal intensity on all MR sequences

Answers
1. B 13. D 25. C 37. B 49. C 61. D 73. A 85. D
2. D 14. A 26. D 38. D 50. B 62. C 74. A 86. C
3. C 15. C 27. C 39. C 51. B 63. D 75. B 87. A
4. B 16. B 28. B 40. A 52. A 64. A 76. C 88. A
5. C 17. D 29. C 41. B 53. B 65. C 77. C 89. D
6. B 18. A 30. B 42. C 54. B 66. C 78. D
7. C 19. A 31. D 43. D 55. C 67. C 79. C
8. A 20. B 32. C 44. D 56. C 68. C 80. B
9. B 21. A 33. C 45. C 57. B 69. C 81. B
10. B 22. A 34. A 46. A 58. C 70. B 82. D
11. C 23. D 35. C 47. B 59. C 71. D 83. C
12. B 24. A 36. B 48. D 60. C 72. A 84. B
Multiple Choice Questions

210
Explanations to Questions

MUSCULOSKELETAL IMAGING
8. Dead bone is devoid of vascularity and hence there is no of the skull due to adjacent erosions of the inner and outer
osteoclastic removal of calcium as well as osteoblastic skull vault.
remodeling hence it appears radio opaque on radiographs. 62. Long-term steroid abuse can lead of avascular necrosis of
9. Plain radiography has low sensitivity and specificity for femur which presents as pain in hip. The earliest diagnosis of
detecting acute osteomyelitis. As many as 80% of patients the same using MRI Hip–is the best strategy.
who present in the first two weeks of infection onset will have 68. Ultrasound findings of osteogenesis imperfecta include a
a normal radiograph. The features of acute osteomyelitis marked shortening of the long bones, multiple bone fractures
that may be visible include a periosteal reaction secondary and lack of mineralization of the skull. 
to elevation of the periosteum, a well-circumscribed bony 74. Osteopetrosis, also known as Marble bone disease, or Albers-
lucency representing an intraosseous abscess and soft tissue Schönberg disease is an extremely rare inherited disorder
swelling. whereby the bones harden, becoming denser due to defect
10. The question asked is most common primary bone tumor in osteoclastic bone resorption.
– hence answer is Multiple myeloma. The most common 81. In neonates the femoral head is cartilaginous and not
secondary bone tumor is Metastases and Most common ossified hence it cannot be identified on radiographs but well
osteogenic/osteoblastic tumor is Osteosarcoma. demonstrated on USG. Also the Graf’s classification using
18. It is classically described in Giant cell tumor however can be Alpha and Beta angles are very definitive in assessment and
seen in multiple other tumors. Multiple septae of bone and can be readily calculated on USG.
soft tissue traverse the interior of the tumor and produce the 84. It’s been considered a variant of fibrous dysplasia, but in reality
characteristic “soap bubble” appearance. It is a very peculiar is a distinct entity. It is inherited in an autosomal dominant
appearance seen only in few conditions namely pattern with onset in early childhood. It presents as multiple
•• GCT/Osteoclastoma lytic expansile lesions within the maxilla and mandible, with
•• Aneurysmal bone cyst soap-bubble appearance. Despite the pronounced changes,
•• Telangiectatic osteosarcoma the disease stabilises and often regresses without the need
•• Malignant fibrous histiocytoma for treatment.
•• Desmoplastic fibroma 85. Madelung deformity :
•• Bone angiosarcoma. A deformity which comprises of:
20. It is classically seen in osteoid osteoma and refers to a central •• Short distal radius which shows a dorsal and ulnar curve
focus of intense uptake (the nidus) within a surrounding •• Triangular shape of the distal radial epiphysis
lower uptake. •• Premature fusion of the ulnar side of the distal radial
23. Bone metastases can be seen at any site which maybe even epiphysis
far away from the primary as they spread by hematogenous •• Dorsal subluxation of the distal ulna
route. So for their screening a modality which includes entire •• Enlarged and distorted ulnar head
skeletal survey is employed - hence Bone scan is Best. •• Wedging of the triangular-shaped carpus between the
34. This appearance is classically described in rheumatoid distal radius and ulna.
arthritis. It refers to focal loss of continuity of the subchondral Seen in
bone. It is seen in the radial heads of the second and third •• Dyschondrosteosis (Leri-Weill diseaseQ): Bilateral with
metacarpals in the hand and medial aspect of the heads of mesomelic limb shortening.
the first metatarsal and lateral aspect of the fifth metatarsal •• Diaphyseal aclasis
in the feet. •• Turner’s syndrome
36. Do not forget that asymmetric or non uniform loss of •• Post-traumatic

Explanations to Questions
joint space is characteristic of Degenerative joint disease •• Post-infective
(Osteoarthritis), whereas Symmetric loss of joint space is 89. A non-metallic foreign body like glass cannot be easily
seen in Inflammatory joint disease. identified on radiographs and CT scan. MRI has poor
40. Ossified Posterior Longitudinal Ligament (OPLL) - is most resolution for small foreign bodies. High frequency local USG
commonly seen in mid cervical region where it classically can best identify and localize the suspected glass foreign
causes spinal canal stenosis and cord compression. body.
60. Hole-within-hole appearance also known as beveled edge
appearance is classically seen in the Eosinophilic granuloma

211
_______________________________________________________________________________________________________________________

_______________________________________________________________________________________________________________________

_______________________________________________________________________________________________________________________

_______________________________________________________________________________________________________________________

Note
_______________________________________________________________________________________________________________________

_______________________________________________________________________________________________________________________

_______________________________________________________________________________________________________________________

_______________________________________________________________________________________________________________________

_______________________________________________________________________________________________________________________

_______________________________________________________________________________________________________________________

_______________________________________________________________________________________________________________________

_______________________________________________________________________________________________________________________

_______________________________________________________________________________________________________________________

_______________________________________________________________________________________________________________________

_______________________________________________________________________________________________________________________

_______________________________________________________________________________________________________________________

_______________________________________________________________________________________________________________________

_______________________________________________________________________________________________________________________

_______________________________________________________________________________________________________________________

_______________________________________________________________________________________________________________________

_______________________________________________________________________________________________________________________

_______________________________________________________________________________________________________________________

_______________________________________________________________________________________________________________________

_______________________________________________________________________________________________________________________

_______________________________________________________________________________________________________________________

_______________________________________________________________________________________________________________________

_______________________________________________________________________________________________________________________

_______________________________________________________________________________________________________________________

_______________________________________________________________________________________________________________________

_______________________________________________________________________________________________________________________

_______________________________________________________________________________________________________________________

_______________________________________________________________________________________________________________________
CENTRAL NERVOUS
SYSTEM IMAGING
C hapter O utline
• Stroke Imaging • CNS Neoplasms
• Head Injury • White Matter Diseases, Phakomatoses and Miscellaneous
CONCEPTUAL REVIEW OF RADIOLOGY

• CNS Infections • Multiple Choice Questions with Explanations

Stroke Imaging
1
CLINICAL QUIZ
1. A 42-year-old patient with sudden onset headache, neck rigidity without
any obvious history of trauma. What is the most likely diagnosis based on
this CT image? (NEET 2020 Pattern)
A. Meningitis
B. SAH
C. Intraparenchymal bleed
D. SDH

2. Best investigation for detection of subacute SAH would be:


A. CT
B. MRI
C. CE-MRI
D. PET

  Clinical Pearls

Acute Stroke Imaging and Management Protocol


Stroke Imaging

214
Acute Ischemia
ISCHEMIC STROKE
CT Brain: 5 typical imaging findings are seen. Each of these
Hyperacute Ischemia findings can be understood by considering the underlying

CENTRAL NERVOUS SYSTEM IMAGING


pathophysiology. Let’s try:
CT Brain ƒƒ Wedge-shaped areaQ: Wedge shape corresponds to the
ƒƒ Noncontrast computed tomography (NC-CT) is the primary arterial distribution territory.
imaging modalityQ. ƒƒ Appearing as Diffuse HypodensityQ: This is because of
ƒƒ The role of CT is to rule out hemorrhageQ – because it takes the disruption of blood-brain barrier and fluid efflux the
around 4–6 hours for classical imaging findings of acute affected brain parenchyma has greater fluid content – hence
infarct to appear on a CT. appears hypodense (look at the ventricles, fluid appears
ƒƒ Dense Artery signQ: black/hypodense).
•• Earliest imaging sign can be seen on NC-CT. Varied ƒƒ Involving Gray and White matterQ: Both gray and white
appearances are seen due to hyperattenuating throm­ matter are supplied by the same artery hence both are
bus, which is seen within an artery. affected.
•• Hyperdense MCAQ/Sylvian dot signQ: Hyperdense ƒƒ With loss of gray-white differentiationQ: Since both
thrombus within the Middle cerebral artery - correlates gray and white matter are involved, they are uniformly
with large infarcts distally hypodense and lose differentiation.
ƒƒ Early Parenchymal signs: ƒƒ With evidence of mass effectQ: Affected brain parenchyma
•• Decrease in gray matter densityQ – due to cytotoxic is edematous (cytotoxic edema), hence it causes mass effect
edema – hence it becomes indistinguishable from the compressing the adjacent structures.
white matter.
•• Obscuration of lentiform nucleusQ/Disappearing Subacute Ischemia
Basal ganglia signQ: Due to hypodense appearance – ƒƒ CT Fogging phenomenonQ: In subacute stage tiny cortical
the gray-white matter distinction is lost. petechial hemorrhages occur, causing the density to
•• Insular Ribbon signQ: Loss of distinction from adjacent increase on CT. hence CT can appear misleadingly
white matter due to cytotoxic edema. normal.
MRI Brain
ƒƒ Diffusion Weighted Imaging (DWI): NC-CT – Poor grey-white
•• Earliest imaging modality to detect an infarct/ matter distinction in the
ischemiaQ – as early as 30 minutesQ from onset left basal ganglia (yellow
•• Hyperintense (bright)Q appearing area on DWI arrow) compare with
images and Hypointense (dark)Q appearance in the normal right side (blue
corresponding same area on an ADC (Apparent arrow) – Disappearing
Diffusion Coefficient) map - from approximately basal ganglia signQ
30 minutes to 5 days after the onset of symptoms.

Noncontrast CT: 1St Investigation


CO Done for Acute Stroke
NC E P T
• Empirically, if you are ever asked about the BEST imaging
investigation for any neurological condition, the answer will be
MRI Brain.
• This is because MRI is much superior to CT given its multiplanar
imaging capability, high soft tissue contrast, specific sequences
like DWI/SWI/Perfusion.
• Biggest disadvantage of MRI – is TIME – it takes around 30 – 40
Stroke Imaging

minutes for a dedicated MRI Brain today.


• Biggest advantage of CT – is TIME – it takes hardly a few seconds
for a CT Brain to be done.
• Hence, for most (90 - 95%) of neurological conditions MRI is
IOC. But for those in which Time is a critical factor, the IOC is NC-CT showing the
CT Brain, like “Hyperdense MCA
ƒƒ Acute stroke sign” on left side
ƒƒ Head Injury for detection of EDH/SDH/SAH

215
ƒƒ Requires iodinated contrast material injection for
visualization of arteries in CT scan while MR Angiography
Same patient as on the does not need contrast injection.
previous page images –
CONCEPTUAL REVIEW OF RADIOLOGY

after 6 – 8 hours : NC-CT CT/MRI Perfusion Imaging – Helps Identify


reveals a evolved acute
infarct – Large wedge-
PenumbraQ
shaped hypodensity in the Can be used to measure the following perfusion parameters:
left frontoparietal region ƒƒ Cerebral blood volume (CBV): CBV deficit is the best
involving basal ganglia, predictor of initial infarct sizeQ
with loss of gray-white ƒƒ Cerebral blood flow (CBF)
differentiation and mass ƒƒ Mean transit time (MTT): Most sensitive Perfusion
effect causing midline indicator of strokeQ
shift toward right side ƒƒ Time to peak enhancement (TTP)

Lacunar InfarctsQ – (Recent Pattern 2019)


ƒƒ Lacunar infarctions are small infarcts (<20 mm) resulting
from occlusion of deep, penetrating end arteries.
MRI DWI ƒƒ The most common locations are basal ganglia, internal
and ADC capsule, thalamus, and the corona Radiata.
Images ƒƒ They occur usually due to involvement of lenticulostriate
showing arteries, thalamoperforating, and pontine perforating
an Acute
arteries, recurrent artery of Heubner
right MCA
ƒƒ CT: Appear as small discrete hypodensities.
territory
ƒƒ MRI: T1 hypointense, T2 hyperintense, hypointense with
infarct
– with peripheral hyperintense rim on FLAIR – due to gliosis. Acute
Restricted lacunar infarcts may show diffusion restriction.
diffusion
Global Cerebral Hypoperfusion/Anoxia
ƒƒ May result from severe hypotension/cardiac arrest/
hypoxia/anoxia.
Chronic Ischemia
ƒƒ Reversal signQ:
ƒƒ GliosisQ and Ex-vacuo dilatation of ventricles/sulci: •• The gray and white matter densities on CT are reversed
Hypodense (water density) appearance of affected in a diffused manner throughout the brain.
parenchyma with negative mass effect/dilatation of •• Represents severe anoxic–ischemic brain injury seen in
adjacent ventricles/sulci. global cerebral ischemia, including carbon monoxide
poisoning.
ƒƒ White cerebellum signQ: Refer to Image No. 62 in Imaging
  Clinical Pearls
Signs in Radiology
Alberta Stroke Program Early CT (ASPECTS) Score •• Related to Reversal sign and seen accompanying it in
• Affected middle cerebral artery territory is divided into most cases.
ten segments, namely: internal capsule, caudate nucleus, •• The relative high density of the Cerebellum (and also
lentiform nucleus, insula and six segments for cortical areas the central gray matter nuclei) is maintained and
• One point is deducted for each area that shows early appear brighter relative to the hypodense cerebrum on
ischaemic changes of swelling or reduced attenuation/
CT.
hypodensity.
•• Both these signs are associated with irreversible brain
• A lower total score carries a worse prognosis.
injury & poor prognosis.
• Can be used to predict outcome and risk of post-thrombolysis
haemorrhageQ. Correlates well with DWI findings at
Venous Infarction
Stroke Imaging

presentation
ƒƒ May occur as a result of venous sinus thrombosis or cortical
venous thrombosis.
Other Important Imaging Applications ƒƒ Infarcts are mostly hemorrhagic and multifocal.
CT/Time-of-Flight (TOF) MR Angiography ƒƒ Venous sinus thrombosis: MRI + MR VenographyQ – is
IOC
ƒƒ Useful for evaluating the intracranial and extracranial •• Superior sagittal sinus is most commonly involvedQ –
vessels and guiding appropriate therapy. results in bilateral para-sagittal infarcts
ƒƒ Demonstrates thrombi within intracranial vessels and also •• CT: Involved sinus appears expanded and hyperdense
useful for evaluating the carotid and vertebral arteries in the due to thrombus. On NC-CT expanded triangular
neck
216
CENTRAL NERVOUS SYSTEM IMAGING
Acute left ACA territory infarct (Restricted diffusion) with Left basal ganglia Lacunar infarct – Round hypodensity on CT,
TOF-MR Angiography images. The Left ACA is not seen on MR Restricted diffusion on DWI/ADC
Angiography images – suggests thrombosis.

hyperdense thrombosed sinus is seen and is called – ƒƒ Hyperacute unclotted blood – may appear less dense thus
Delta signQ. On CE-CT the sinus walls enhance but the creating a Blood-Fluid level – commonly seen in bleed
contents do not – Empty or Negative Delta signQ secondary to coagulopathies/patient on anticoagulant
•• Cord signQ: Cordlike hyperattenuation within a dural therapy
venous sinus on NC-CT. Most commonly seen in the ƒƒ Swirl signQ: Internal hypodensity within the bleed –
transverse sinus. may suggest continuous bleed – requires further urgent
management.
Moyamoya DiseaseQ–(AIIMS May 2014) ƒƒ Extension into ventricles: Either forms a blood clot or is
Refer to Image No. 60 in Imaging Signs in Radiology seen as a blood-CSF level in the occipital horns
ƒƒ In Japanese means – Puff of smoke
ƒƒ It is an idiopathic arteriopathy mainly seen in Japan. Subacute Bleed
Characterized by progressive narrowing of supraclinoid ƒƒ As the clot is broken down – its density decreases, hence the
ICA. This gradual narrowing leads to the formation of bleed gradually becomes isodense to the brain
multiple small abnormal net-like vessels/collaterals.
ƒƒ On an invasive Digital subtraction Angiography (DSA) – all Chronic Bleed
these collaterals are opacified at once – this appears as if a ƒƒ Eventually all the products of hemoglobin breakdown
“Puff-of-smoke”Q is rising up – hence the name. MR + MR are engulfed and carried away by macrophages – all that
Angiography can also show these findings. remains at the site is fluid filled cavity – hence in this stage a
bleed area may appear hypodense to the brain.

HEMORRHAGIC STROKE/NON- MRI Appearance


TRAUMATIC CEREBRAL ƒƒ Best sequence for detection of bleed on MRI is Gradient
HEMORRHAGE echo (GRE) and Susceptibility weighted imaging (SWI)
sequencesQ. SWI is much more sensitive than GRE.
ƒƒ Most commonly is due to rupture of a perforating vessel due ƒƒ Appearance on T1W/T2W sequences is variable according
to hypertensionQ to the age of the bleed. As hemoglobin is broken down into
various intermediates and finally into hemosiderin, these
CT Appearance breakdown products along with clot formation, water and
Stroke Imaging

Acute Bleed protein content can alter the signal intensity. The following
sequence is usually followed:
ƒƒ Hyperdense (bleed) with a surrounding rim of hypodensity
(peri-bleed edema)Q – is the most peculiar appearance.

217
“Empty Delta MRI Postcontrast
sign” suggestive of image – showing
Superior sagittal
CONCEPTUAL REVIEW OF RADIOLOGY

filling defect in
sinus thrombosis SSS – suggestive
of thrombosis and
FLAIR images s/o
bilateral multifocal
infarcts

Duration of Bleed Stage T1W T2W Form of Hem Iron


Responsible
<24 hours Hyperacute Iso/Hypo-Intense Slightly hyperintense OxyhemoglobinQ
1–3 days Acute Slightly hypointense Hypointense DeoxyhemoglobinQ
3–7 days Early subacute Hyperintense Hypointense Intracellular
methemoglobinQ
1–4 weeks Late subacute Hyperintense Hyperintense Extracellular
methemoglobinQ
>4 weeks Chronic Iso/Hypo- Intense Markedly hyperintense HemosiderinQ

ƒƒ Charcot-Bouchard aneurysmsQ: Arise from small


  Clinical Pearls penetrating arteries as a result of chronic hypertensionQ
Acute Hemorrhage in the Basal Ganglia– Is Hypertensive Bleed ƒƒ Anterior and posterior communicating artery (ACOM/
Unless Proven Otherwise PCOM) – most commonQ (ACOM > PCOM)
• The most common sites of Hypertensive bleeds in a decreas- ƒƒ Mycotic aneurysmsQ: Arise from infection of arterial
ing order are: walls, associated with septicemia or bacterial endocarditis,
ƒƒ Putamen most commonly caused by Staphylococcus aureusQ and
ƒƒ Thalamus SalmonellaQ.
ƒƒ Pons
• Thus the 1st two most common sites, putamen & thalamus,   Clinical Pearls
are actually components of the Basal ganglia.
• This is a very important diagnostic clue for the next 40–50 years Site of Bleed/Clot may Suggest the Location of Aneurysm/
of your clinical practice ahead. Vessel Involved
Site of Bleed/Clot/ Location of Aneurysm/Vessel
Abnormality Involved
Septum pellucidum/ Anterior communicating artery
ANEURYSMAL BLEED Interhemispheric fissure/ (ACOM) aneurysm – Most
Frontal lobe common site of aneurysm in
Most Common Cause of Nontraumatic supratentorial regionQ
SAHQ
Stroke Imaging

Sylvian fissure/Temporal Middle cerebral artery (MCA)


Types of Aneurysms lobe aneurysm
ƒƒ Saccular: Most commonQ. Also called Berry aneurysms – Sylvian fissure/Isolated 3rd Posterior communicating
are round/lobulated and arise at branching points along Nerve palsy artery (PCOM) aneurysm
circle of Willis. Interpeduncular fossa of Basilar tip aneurysm – Most
ƒƒ Giant aneurysms: >25 mm in diameterQ. Usually has a midbrain/Pons/Thalamus common site of aneurysm in
patent lumen surrounded by variable thickness of laminated posterior fossaQ
thrombus with/without calcification 4th Ventricle/Spinal Posterior inferior cerebellar
subarachnoid bleed artery (PICA) aneurysm
218
Imaging Modalities for Detection of
Aneurysms Acute Basal
ganglia
ƒƒ Multiplanar CT – is best investigationQ bleed on

CENTRAL NERVOUS SYSTEM IMAGING


ƒƒ Multiplanar intra-arterial digital subtraction angiography NC-CT
(DSA) – remains gold standardQ Also used for treatment
of aneurysms – Endovascular coiling of aneurysms with
resultant thrombosis.

SUBARACHNOID HEMORRHAGE
ƒƒ Bleed resulting from an aneurysmal rupture is almost
always SAH. Occasionally along with SAH a parenchymal
clot may be seen.
ƒƒ Another cause of spontaneous SAH is Arteriovenous
malformation (AVM) Vein of Galen MalformationQ (Recent Pattern 2018)
ƒƒ Patient presents with Thunder-clap headacheQ/
ƒƒ Is a misnomer. It actually involves the median prosencephalic
Worst headache of my lifeQ followed by collapse and
vein (MPV) of MarkowskyQ
unconsciousness.
ƒƒ May present in a neonate/infant as hydrocephalus & high
ƒƒ Hunt and Hess grading systemQ is used – predicts outcome
cardiac output failure – most common extracardiac cause of
CHF in neonatesQ
CT Findings ƒƒ Large feeder arteries opening into a large midline venous
CT is Investigation of Choice for Acute SAHQ. pouch in the brain
ƒƒ Hyperdense appearing CSF spaces – most commonly basal
cisterns Venous Angioma
ƒƒ Blood – CSF level may be seen in ventricles. (Developmental Venous Anomaly)
MRI Findings ƒƒ Medusa head appearanceQ at angle of ventricle – dilated
medullary white matter veins converge on a large collector
MRI is more useful in the Sub-acute stage when the bleed vein
becomes isodense to brain on CT while it shines as hyperintense
on T1W MRI. Cavernoma/Cavernous Malformation
ƒƒ GRE/SWI are Best sequences – bleed appears as intensely
hypointense areas/Blooming ƒƒ Benign vascular hamartoma with masses of closely
ƒƒ FLAIR: May show sulcal hyperintensity due to increased opposed immature blood vessels (“caverns”), intralesional
protein content hemorrhages, no intervening neural tissue
ƒƒ On MRI – is seen as Popcorn-likeQ, smooth circumscribed,
Complications of SAH Seen on Imaging well defined complex lesion with bright center and a
hypointense peripheral hemosiderin rim/black halo around
ƒƒ Communicating hydrocephalus: Temporal horns of lateral
the lesion.
ventricles – earliest affected.
ƒƒ Angiography normal – Angiographically occult malforma-
ƒƒ VasospasmQ: Occurs between 4 – 11th day of hemorrhage,
tionQ
and may lead to brain ischemia. Triple H therapy
(Hemodilution-Hypertension-Hypervolemia) – is used. Hypoxic Ischemic Encephalopathy (HIE) –
ƒƒ Superficial siderosisQ: Seen in chronic repeated SAH.
Affecting Preterm Babies
ƒƒ Also known as Periventricular leukomalacia (PVL)Q/
ARTERIOVENOUS Anoxic Ischemic Encephalopathy (AIE)Q
MALFORMATIONS ƒƒ PVL is the HIE-driven periventricular white matter (WM)
Stroke Imaging

necrosis seen in very low birth weight (VLBW) premature


Cerebral A-V Malformation babies
ƒƒ USG through Anterior fontanelle/Neurosonography:
ƒƒ Vascular malformation with arteriovenous shunting and no •• Used as Initial/Screening investigation as it is – Cheap,
intervening capillary bed readily available, can be done at bedside and easily
ƒƒ Bag of black wormsQ - appearance of flow voids of vessels repeated.
on MRI ƒƒ MRI Brain:
ƒƒ Tightly-packed mass looks like honey-comb of flow-voids – •• Is Best investigationQ
represents the nidus of the malformation

219
•• Restricted diffusion on DWI in affected areas – Best
Acute early diagnostic clueQ
Subarachnoid •• Periventricular volume loss, gliosis, ventricular
hemorrhage enlargement – Best late diagnostic clueQ
CONCEPTUAL REVIEW OF RADIOLOGY

(SAH) in the
basal cisterns Posterior Reversible Encephalopathy
of brain Syndrome (PRES)
ƒƒ Also known as Acute hypertensive encephalopathy/
Reversible posterior leukoencephalopathy
ƒƒ A Neurotoxic state seen in the setting of a number of
complex conditions (preeclampsia/eclampsia, allogeneic
bone marrow transplantation, organ transplantation,
autoimmune disease and high dose chemotherapy) with a
unique CT or MR imaging appearance.
ƒƒ On CT/MRI:
•• Focal regions of symmetric hemispheric edema (Bright
on T2/FLAIR) most commonly involving parietal and
occipital lobesQ
•• MR diffusion-weighted imaging (DWI) – does not show
Postcontrast restricted diffusion thus ruling out infarction.
CT images •• All findings are completely reversible
showing a ƒƒ MRI – Best modality
Left Internal
carotid
artery
aneurysm

Cerebral NC and CE-


Digital Sub- CT showing
straction a Vein of
Angiogra- Galen mal-
phy (DSA) formation
Image
showing an
AVM with
nidus well
opacified
with con-
trast
Stroke Imaging

Sagittal
Reconstructed
CE-CT Image
and a CT – VRT
image showing
a Vein of Galen
malformation

220
FLAIR MRI MRI T2W
images and GRE
showing images

CENTRAL NERVOUS SYSTEM IMAGING


hyper­ showing
intensities a left
in Posterior temporal
circulation lobe
areas – Cavernoma
suggestive of
PRES

MRI T1W and FLAIR


images showing
cystic periventricular
leukomalacia (PVL)

1
CLINICAL QUIZ - SOLUTION
So this patient presents with acute onset headache and altered sensorium. Neck stiffness is present – but this is not necessarily specific
for Infection/Meningitis. Also, there is no history of fever to suggest Infection. The CT image shows confluent hyperdense areas in the
basal cisterns – suggestive of Acute SAH. (Answer is B. SAH).
NC-CT is the IOC for acute SAH. But as the bleed becomes old its density changes. Sub-acute bleed is isodense to the brain, whereas
chronic bleed is hypodense. Hence CT is not a good modality to diagnose bleeds in the sub-acute/Chronic stages. MRI is hence the IOC for
sub-acute/Chronic SAH (Answer to Quiz question 2 is B).

Head Injury
2
CLINICAL QUIZ
3. A young male was found unconscious on the roadside after a bike
accident. He regained consciousness and was sent for a CT scan. During
the CT scan he again deteriorated and showed altered sensorium. The CT
image is shown here. Your diagnosis is:
Head Injury

A. Extradural hematoma
B. Subarachnoid hematoma
C. Subdural hematoma
D. Hemorrhagic contusion
4. Talk-and-Die syndrome is associated with:
A. SDH
B. EDH
C. SAH
D. Hemorrhagic contusion 221
Subdural Hemorrhage (SDH)
Acute EDH on ƒƒ Origin is from—Bridging cortical veinsQ
CT Brain – look ƒƒ Bleed in the Subdural space: Large potential spaceQ that can
at the peculiar
CONCEPTUAL REVIEW OF RADIOLOGY

accommodate a large amount of blood before causing mass


BICONVEX
effect on the underlying brain.
shape of the
ƒƒ Spontaneous SDH in the Elderly: May be asymptomatic
bleed
or present with mild headache or transient but repetitive
neurological deficits – termed pseudodementiaQ.
ƒƒ Concavo-Convex shapeQ: Inner margin is concave, outer
margin is convex.
ƒƒ Pseudo empty Delta signQ: The acute SDH lining the
walls of superior sagittal sinus may make the walls look
hyperdense while the contents look relatively hypodense
resembling the Empty Delta sign of sinus thrombosis.
ƒƒ Not limited by cranial sutures but limited by dural folds like
  Clinical Pearls falx cerebri, falx cerebella, tentorium cerebelli
ƒƒ Subacute SDH is isodense to the brain and may be difficult
CT-Brain Diagnostic Protocol in Head Trauma
to detect – hence MRI is more useful.
ƒƒ Chronic SDH/Subdural hygromas: Bilateral hypodense
(CSF density) crescentic collections. Blood-CSF levels may
be seen if fresh hemorrhage occurs.
ƒƒ SDH in neonates and infants should raise suspicion of Non-
accidental injury/Battered baby syndrome

  Clinical Pearls
EDH versus SDHQ
Though it has been discussed above it is pertinent to highlight
the differences between EDH and SDH to consolidate better.
EDH SDH
Origin of Middle meningeal Bridging cortical veins
bleed artery
Side Unilateral Unilateral or Bilateral
Symptoms Lucid interval Gradually increasing
followed by headache, confusion
unconsciousness leading to altered
sensorium
Fracture Commonly Not associated with
Extradural/Epidural associationQ associated with fractures
fractures
Hemorrhage (EDH)Q (AIIMS May 2018 Pattern)
Limited byQ Cranial sutures. Dural folds/venous
ƒƒ Origin is from – Middle meningeal artery (MMA)Q. Usually Not limited by sinuses. Not limited by
associated with a skull fractureQ –the bone fragments cause dural folds/venous cranial sutures
damage to an MMA branch – results in EDH sinuses
ƒƒ It is bleed in the extradural spaceQ – between the outer layer ShapeQ Biconvex Concavo-Convex
of dura (endosteal layer) and the inner table of the skull (Crescentic/banana
vault. shaped)
ƒƒ Clinically associated with Lucid IntervalQ
Surgical Almost always May be assessed
ƒƒ Shape: Typically Bi-convexQ/LentiformQ in shape
drainage required based on neurological
Head Injury

ƒƒ Does not cross the cranial sutures.


status, mass effect &
ƒƒ Not limited by venous sinuses – they can cross and elevate/
midline shift.
displace venous sinuses.
ƒƒ Swirl signQ: Internal hypodensity within the bleed –
may suggest continuous bleed – requires further urgent
Subarachnoid Hemorrhage
management.
ƒƒ MRI is preferred in case of suspected spinal EDHQ. ƒƒ Most common cause of SAH overall is Head injuryQ
ƒƒ It is bleeding within the basal cisterns/sulcal spaces.
222
ƒƒ CT: Appears as linear or serpentine areas/finger-like
extensions of high attenuationQ that conform to the
morphology of the cerebral sulci and cisterns – commonly
seen in the Sylvian fissures and Interpeduncular cisternQ

CENTRAL NERVOUS SYSTEM IMAGING


ƒƒ MRI: Difficult to detect in acute stage than on CT. FLAIR
images may show sulcal hyperintensity. Sub-acute and
chronic SAH is better picked up on MR.

EDH is Biconvex – SDH is


CO Concavo-Convex
NC E P T
• We have already seen the shapes, and all books will tell you the
same. What no one will ever tell you is “Why” the shape is so?
• Consider 2 examples: Crescent-shaped hyperdense bleed along the right cerebral
ƒƒ Local trains: In Mumbai (I love Mumbai!!) hemisphere in the 1st image - suggests Acute SDH. Now look at
 In Rush hours: Jam-packed, over-full, tremendously limited
the 2nd image. The collection is crescent shaped on left side but it
space – so people are hanging out on doors, windows, is hypodense - suggests a Chronic subdural collection/hygroma
even on top of the bogie….
 In Off hours/Holidays: Empty, lots of space (really?) – so   Clinical Pearls
all the people are in the bogie, occupying the seats.
History Clincher of Diffuse Axonal Injury (DAI)
ƒƒ Air:
 In a Balloon: Limited space in the balloon, so when you try
to fill up the balloon it will spread out into a spherical/oval
shape and the balloon is stretched and tense
 Air around us in the atmosphere: Is in an unrestricted
space, so spreads out and uniformly occupies the entire
available space.
Now consider the correlation with EDH and SDH:
EDH: It is bleeding in the limited space between the endosteal
dura and the inner table of skull. Extremely limited space – like the
9.03 CST local & like the Balloon – so whatever blood accumulates
will bulge out in whatever space is available. It cannot bulge
outwards as there is bone but it bulges on the inside – thus
becoming Biconvex.
SDH: Is bleeding in the large potential subdural space – like the
local on holidays or air in the atmosphere – so simply takes up the
entire available space uniformly – Outer margin is convex as that
is the shape of the skull vault, inner margin is concave as it spreads
along the cerebral convexity without any bulge.
Now you know….Mumbai meri jaan…..!
ƒƒ What exactly is DAI:
Parenchymal Contusions •• Shearing stress/force exerted on the Brain, as a result
of rotational acceleration and deceleration forces
ƒƒ May be hemorrhagic/nonhemorrhagic
occurring during a road traffic accident.
ƒƒ Seen in areas of brain close to rough bony surfaces, hence
•• The basic neuronal structure comprises a cell body and
commonly seen in OrbitofrontalQ and temporal regions of
neurofilament attached to it. When such shearing force
brainQ. is applied to a neuron, the neuron simply breaks off at
ƒƒ On CT, seen as ill-defined hyperdensities (blood) with large its weakest point – the junction between the cell body
surrounding hypodensities (marked peri-bleed edema) and neurofilament. This is a permanent break and is
ƒƒ On MRI: Contusions appear as ill-defined areas of variable irreversible.
Head Injury

signal intensity on both T1- and T2-weighted images, •• So this Axonal level injury occurring in a Diffuse
depending on the age of the lesions. manner is called as Diffuse Axonal injury!
ƒƒ CT/MR Findings:
Diffuse Axonal Injury •• Small petechial bleeds seen mainly at 3 sites:
ƒƒ It is a “History Clincher” and follows the following sequence 1. Gray-White matter junctionQ—Most common
on most of the cases - Non-improving altered sensorium in siteQ—corresponds to the site of neuronal break/
a head injury patient, with normal initial CT scanQ discontinuity
2. Corpus callosumQ
3. Dorsolateral brainstemQ
223
Skull Fractures
• Comminuted Caused by vehicular accidents ‘falls from
fracture height, and blow from weapon with a large
CONCEPTUAL REVIEW OF RADIOLOGY

striking surface, e.g. A heavy iron bar. The


bone is broken into two or more pieces
• Pond or Occurs is children due to elasticity of their
Indented skull bone, due to forcible impact against
fracture some protruding effect.
• Gutter When a part of the thickness of skull is
fracture removed, e.g. In glancing bullet wounds.
• Elevated Is result of a blow from a moderately heavy
fracture sharp-edged weapon, e.g. axe, machete,
which elevate one end of the bone above
Extensive Acute surface of the skull while other end may dip
SAH in bilateral down in cranial cavity injuring dura/brain. It
parietal sulcal may indicate relative position of assailant and
spaces victim.
• Growing skull Occurs in children, usually less than 3
•• Nonhemorrhagic foci of DAI; May be seen as focal fractures/ years. Thought to occur secondary to skull
hyperintensities in brain on T2W/FLAIR images. Craniocer- fractures causing dural tears allowing the
ebral erosion leptomeninges and/or cerebral parenchyma
(Leptomenin- to herniate into it. Pulsations from CSF erode
MRI – is IOC in geal cysts)Q the fracture margin, resulting in eventual
CO Diffuse Axonal Injury (DAI) expansion and non-union. CT scan is imaging
NC E P T modality of choice.
We have discussed previously that wherever TIME is crucial CT is
preferred modality chosen – as in case of Acute stroke, Trauma. Pneumocephalus – Intracranial Air (Refer to
Then how is it that in DAI, MRI is preferred? Image No. 63 in Imaging Signs in Radiology)
 Since DAI is a cellular level break it is too small to be seen on a ƒƒ Most common cause: Trauma or SurgeryQ
CT brain and hence the early CT scan is normal in most of the cases. ƒƒ Suggests basal skull fracture with air coming from the
 At most it shows mild generalized cerebral edema. As time
paranasal sinuses.
passes small petechial hemorrhages develop at the site of break.
When a delayed imaging is done (CT/MR) we are able to see such
tiny petechial hemorrhages—thus diagnosing DAI. Since imaging
is done in delayed phase MRI is preferred as it has a better soft Multifocal
tissue and contrast resolution and has specific blood sensitive hyperdensities
sequences (GRE/SWI). at Gray-
White matter
junctions
Skull Fractures – suggests
CT Head is investigation of choice. Diffuse Axonal
Skull Fractures Injury

• Simple Is defined as having one bone fragment; a


fracture compound fracture exists when there are 2
or more bone fragments.
• Depressed Caused by a heavy weapon with a small
fracture or striking surface, e.g. Hammer.
“signature” The fractured bone is driven inward and its
fractureQ shape may indicate the type of weapon with
which it is produced, therefore it is known as
Head Injury

a signature fracture.
A depressed fracture of the frontal or parietal Multiple bilateral
bone can occur before or during labor as a hyperdensities
result of compression of the fetal head by the in basi-frontal
and anterior
maternal pelvis; or it can occur during delivery,
temporal lobes
usually as a result of a forceps operation.
– Hemorrhagic
Physical examination reveals a “derby hat” or
contusions
“ping-pong ball” deformity.
224 Contd...
ƒƒ Tension pneumocephalusQ occurs when subdural air interhemispheric space that mimics a picture of a volcano
causes a mass-effect over the underlying brain parenchyma, just like Mount Fuji in Japan. Treatment is emergent surgical
often from a ball-valve mechanism causing one-way entry decompression.
of air into the subdural space. Mount Fuji signQ - air may ƒƒ CT scan is IOCQ.

CENTRAL NERVOUS SYSTEM IMAGING


separate and compress the frontal lobes, creating a widened

2
CLINICAL QUIZ - SOLUTION
The examiner has given you a typical history of “lucid interval”. Along with this the CT image showing a classical left-sided extra-axial
biconvex hyperdensity – is suggestive of EDH (Answer is A).
Talk-and-Die syndrome – is another clinical peculiarity associated with EDH. The patient may have a seemingly minor fall and is alright
– conscious oriented – after the event. But then after some time suddenly deteriorates and then eventually dies. This is most likely due
to delayed onset of EDH that eventually becomes large in size and compresses the brainstem suppressing the function of vital centres.
Hence asnswer to Quiz question 4 is B. EDH.

CNS Infections
3
CLINICAL QUIZ
5. Young male presented with fever, ear ache and seizures. He has been
having ear discharge since 3 months. MRI brain with contrast shows:
A. Neurocysticercosis
B. Brain abscess
C. Herpes infection
D. Meningioma

Pyogenic Meningitis
  Clinical Pearls
ƒƒ Meningitis is a clinical and laboratory diagnosis. Best
A lot of terms are used to describe various types of CNS investigation for diagnosis – CSF ExaminationQ.
infections. Let us get our concepts right, at the very beginning ƒƒ Role of Neuroimaging is to detect complications of
so that there is no confusion. meningitisQ like Hydrocephalus, Empyema, Abscess, Sinus
• Meningitis: Infection/Inflammation of Meninges. It may be thrombosis, Ventriculitis/Ependymitis, etc.
ƒƒ LeptomeningitisQ – involving the Piamater and Arachnoid ƒƒ CT/MR Findings:
– is more common •• NC-CT is often normal, however sulcal effacement with
ƒƒ PachymeningitisQ - involving Duramater.
slight hyperattenuation of sulci – may be seen.
• CerebritisQ: Focal infection of the brain, without any capsule •• Abnormal leptomeningeal enhancement (pia and
CNS Infections

or pus formation. It is usually bacterial/pyogenic in origin


subarachnoid space)Q is seen – due to inflammatory
• EncephalitisQ: Diffuse, direct infection of brain, usually Viral
breakdown of the blood-brain barrier.
in origin
• Abscess: Focal encapsulated pus filled infective focus Brain Abscess
• EmpyemaQ: A pus collection in a potential space outside the
brain (Subdural, Epidural etc) ƒƒ Usually bacterial/pyogenic.
• GranulomaQ: A chronic focal inflammatory lesion, that is well ƒƒ Site of abscess may point towards the cause:
defined/encapsulated and without a pus formation •• Frontal lobe abscess – may result from frontal sinusitis

225
ƒƒ Glioblastoma multiforme (central necrosis)
Ring enhancing ƒƒ Infections:
lesion in left  Tuberculoma (central caseous necrosis)
temporal lobe. In a
CONCEPTUAL REVIEW OF RADIOLOGY

 Toxoplasmosis (central necrosis)


patient with fever,
 Neurocysticercal granuloma (central cystic contents/fluid)
altered sensorium
and left ear • Sub-acute Infarct (peripheral leucocyte infiltration central
discharge, suggests infarction)
otogenic brain • Cerebral Contusion
abscess • AIDS
• Lymphoma in immunocompromised patient (In immunocom-
petent patient it shows homogenous solid enhancement)
• Demyelination (peripheral inflammation)
• Radiation necrosis (peripheral inflammation, central necrosis)

Herpes Encephalitis
Left frontal
lobe Abscess ƒƒ Encephalitis caused by HSV – 1Q, has very high mortality if
– Shows untreated.
restricted ƒƒ Patient presents with high grade fever, headache, altered
diffusion on sensorium.
DWI, Ring ƒƒ Typical sites of involvement: Bilateral but asymmetrical
enhancement involvement of Limbic system structuresQ –
of Postcontrast •• Temporal lobes, Insular cortex, Cingulate gyrus,
study Subfrontal region
ƒƒ CT: Ill-defined hypodensity involving above mentioned
areas with patchy gyriform late enhancement.
•• Temporal lobe/Cerebellar abscess – from Otitis media/ ƒƒ MRI: Best modality for early diagnosisQ. DWI/FLAIR
Mastoiditis images – earliestQ to show cortical hyperintensity in these
•• Blood borne spread – usually involves Middle cerebral areas with post contrast gyriform enhancement.
artery territory – Frontoparietal lobe
ƒƒ Ring enhancing lesionQ on CT/MR: Abscess is a typical NeurocysticercosisQ – (Recent Pattern June 2018)
Ring enhancing lesion in the brain with surrounding edema ƒƒ Caused by infection due to the pork tapeworm Taenia
and mass effect. soliumQ
ƒƒ Restricted diffusion on DWIQ: Abscess is one of the lesions ƒƒ Cerebral convexity Subarachnoid spaces – Most common
that shows restricted diffusion on DWI. siteQ
ƒƒ MR Spectroscopy: Central area will show abundant lactate, ƒƒ 4th ventricle – Most common intraventricular siteQ
acetate, alanine, succinate, etc. ƒƒ Escobar’s stagingQ AIIMS May 2013: 4 distinct stages have been
described:
Ring-enhancing Lesions ƒƒ Vesicular stageQ:
(On CT/MR) and Doughnut Lesions •• Acute infection with live quiescent parasiteQ seen as an
CO
NC E P T on Radionuclide Studies eccentric dot within the cyst – is the protoscolex of the
parasite.
• I hope you have all had some amazing doughnuts, so you know •• This appearance is called as Target sign/Cyst with a
what they are. If you have not had one (don’t miss it) just
dot sign/Dot in a hole signQ Refer to Image No. 64 in
imagine a POLO mint. That’s how a ring enhancing lesion is –
Imaging Signs in Radiology”
Intense enhancement at the periphery, nothing in the center!
• Ring enhancement/Peripheral uptake of radionuclide – is
because the periphery is metabolically the most active part
of the lesion. This is the site of viable tumor tissue, area of Herpes
CNS Infections

maximum infective/inflammatory activity, hence it shows Encephalitis:


maximum enhancement. Bilateral,
• Central non-enhancing area is so because this is either necrotic, asymmetric
liquefied (as in pus), hemorrhagic or non-viable. So no enhance- involvement of
ment here. Insular cortex,
• Hence now it is relatively easy to remember the list of typical Cingulate
Ring Enhancing Lesions in the BrainQ: (Mnemonic: MAGICAL gyrus,
Temporal lobes
DR)
ƒƒ Metastasis (central necrosis/hemorrhage)
ƒƒ Abscess (central pus)
226 Contd…
NCC – NCC – Granular
Vesicular Nodular stage:
stage : No Left frontal

CENTRAL NERVOUS SYSTEM IMAGING


edema/ granuloma,
Post- decreased
contrast edema and
enhance- enhancement
ment

NCC –
Colloidal
Vesicular
stage: Left
frontal NCC –Nodular
granulo- Calcified
ma, with stage: Dead
marked calcified
edema and granuloma
Ring (see Blooming
enhance- on GRE), No
ment enhancement

Toxoplasmosis
– Gray-white
matter Ring
enhancing
lesion with
an Eccentric
target sign

CNS Infections

HIV Enceph-
alopathy
– Cerebral
atrophy with PML –
Symmetric Bilateral
confluent asymmetrical
white matter white mater
lesions lesions

227
•• Appearance is similar to a cyst with no surrounding
edema, inflammation or post contrast enhancementQ.
ƒƒ Colloidal Vesicular stageQ:
•• Early sub-acute stageQ
CONCEPTUAL REVIEW OF RADIOLOGY

•• As the parasite degenerates, the parasitic antigens


diffuse into the adjacent brain, and being foreign
antigens they induce severe inflammation. Maximum
edema, inflammation and contrast enhancement (Ring
enhancing lesion)Q – is seen
ƒƒ Granular Nodular stageQ:
•• Late sub-acute stageQ
•• The body tries to limit the infection so there is cyst
wall retraction, with a decreasing trend of edema,
inflammation and contrast enhancementQ.
ƒƒ Nodular Calcified stageQ:
•• Chronic stage of infection with dead, densely calcified
parasite. No edema, inflammation or contrast
enhancement
ƒƒ MRIQ. is Best investigationQ.
ƒƒ Multiple basal ganglia and gray-white matter junction
  Clinical Pearls ring enhancing lesions with surrounding edema and
NCC & Japanese Encephalitis mass effectQ – Most common finding.
• NCC apparently predisposes a person to JE infection and is a ƒƒ Eccentric target signQ: A small enhancing nodule along
positive modulator of the encephalitic process. the lesion margin - highly suggestive of a diagnosis of
• Although both diseases have common epidemiologic toxoplasmosis
and sociodemographic factors like pig rearing, poor
socioeconomic status, hygienic conditions, malnutrition, and Progressive Multifocal
so forth, more than a casual association of the 2 conditions Leukoencephalopathy (PML)Q
has been suggested.
• The presence of NCC has been advocated as a prognosticator ƒƒ Central demyelinating disease resulting from the
of poor outcome in JE reactivation of a latent infection of oligodendrocytes by
John Cunningham (JC) polyomavirusQ seen in HIV patients
ƒƒ MRI is Best investigationQ
HIV Encephalopathy/HIV-Associated ƒƒ Multifocal, asymmetric bilateral white matter lesions, most
Dementia/AIDS Dementia complexQ commonly in parieto-occipital regions, seen as high signal
on T2W and low signal on T1W images
ƒƒ Presents clinically as subcortical dementia with slowness,
ƒƒ Scalloped appearance of lesionsQ – due to extension up to
forgetfulness and apathy.
subcortical U fibers
ƒƒ
ƒƒ
MRIQ – is IOC
Cerebral atrophy – Most common imaging findingQ. 3
ƒƒ Symmetric confluent white matter lesions seen in the CLINICAL QUIZ - SOLUTION
periventricular & centrum semiovale white matterQ. These As you can see there is a temporal lobe ring enhancing lesion
appear hypodense on CT, Hyperintense on T2W MRI, and correlating with history of fever and ear ache mostly likely
do not enhance on postcontrast studies. diagnosis is otogenic pyogenic infection complicated by
brain abscess. The risk factors for brain abscess are middle
Toxoplasmosis ear or paranasal sinus infections, congenital heart disease,
ƒƒ Most common CNS opportunistic infection in AIDS patient immunosuppressed state, chronic illness and direct penetrating
worldwide - ToxoplasmosisQ. skull trauma. Thus, auscultation on chest one might hear cardiac
ƒƒ Most common CNS opportunistic infection in AIDS patient murmur like pansystolic murmur of VSD. It has high mortality
in India - TuberculosisQ and treatment consists of obtaining blood cultures followed by
CNS Infections

ƒƒ Toxoplasmosis is also most common cause of a cerebral empirical broad spectrum antibiotics like cephalosporins with
tailoring as per culture sensitivity reports. If abscess is larger than
mass lesion in AIDS patientQ.
2.5 cm then additional stereotactic CT guided aspiration or open
ƒƒ Results from re-activation of latent infection by Toxoplasma
craniotomy is suggested.
gondiiQ

228
CNS Neoplasms

CENTRAL NERVOUS SYSTEM IMAGING


4
CLINICAL QUIZ
6. A 9-year-old female child with history of head ache and visual disturbance.
A CT image revealed a calcified solid-cystic suprasellar lesion as show
here. What could be your possible diagnosis?
(NEET 2020 and 2018 Pattern)
A. Pituitary macroadenoma
B. Rathke’s cleft cyst
C. Craniopharyngioma
D. Hypothalamic hamartoma

In NEET if you get images or any questions on CNS tumors they will anatomical structure). For example, if you find that the signal in
mostly be based on the presence of an intra cranial space occupying a lesion is following that of CSF in the ventricles on all sequences
lesion which is causing mass effect on adjacent structures. So for like T1/T2/FLAIR for MRI mostly likely the mass is a cystic fluid
this chapter instead of studying each tumor separately we will first filled mass or the signal is following that of the subcutaneous fat
study the approach to any CNS neoplasm and then study each then it is mostly a lipoma.
attribute and mention specific high yield points with examples or Whenever confronted with a mass follow this approach:
signs wherever needed. Most of the attributes will be explained in Each and every point in the discussion that follows will give you
tabulated form which will be helpful for your revisions also. a clue to the diagnosis of the tumor:

How to Approach a CNS Neoplasm on Age of Patient


Imaging Under 2 1st decade 2nd to 4th 5th decade
years decade and above
Choroid Medulloblas- Oligodendro- High grade
plexus toma glioma glioma
papilloma Ependymoma Astrocytoma Glioblastoma
Anaplastic Astrocytoma Pituitary multiforme
astrocytoma Craniopharyn- adenoma Meningioma
Teratoma gioma Colloid cyst Schwannoma
Hemangioblas- Metastasis
toma

Common Intra-Axial Tumors Based on


Location
By intra axial we mean that the tumor is originating from the
neuroparenchyma proper and not from the extra-axial structures
like meninges, nerves, blood vessels, etc.
Supratentorial Infratentorial
CNS Neoplasms

Adults Pediatric Adults Pediatrics


Metastases Astrocytoma Metastases Medulloblas-
Astrocytoma Neuroecto- Hemangio- toma
Oligodendro- dermal blastoma Ependymoma
glioma tumors Pilocytic
Ganglioglioma astrocytoma

In neuro imaging you will be mostly provided with either CT scan


Identification of Extra Axial Mass
or MRI images. The trick to remember is that whenever you find
a mass identify the signal intensities within it (i.e. whether it is Most of the extra axial lesions are either meningioma or
black, white, gray, etc. and then match the signal with known schwannomas. However other noteworthy extra axial
229
ƒƒ Colloid cystQ is small cyst that appears hyperdense on non-
Postcontrast MRI contrast CT scan and involves the Foramen of MonroQ often
Image showing a causing hydrocephalus.
heterogeneously ƒƒ Skull base lesionsQ:
CONCEPTUAL REVIEW OF RADIOLOGY

enhancing intra- •• Chordoma


axial lesion, •• Chondrosarcoma
extending across •• Esthesioneuroblastoma (arises from olfactory
the genu of epithelium)
corpus callosum – ƒƒ Common sellar and parasellar (around pituitary gland)
Butterfly Glioma massesQ:
•• Pituitary adenoma
•• Craniopharyngioma
•• Rathke’s cleft cyst
•• Meningioma
•• Germinoma
•• Dermoid
•• Epidermoid
Intensely
•• Schwannoma
enhancing lesion ƒƒ Intraventricular tumorsQ:
with a broad •• Ependymoma
base toward dura •• Subependymoma
(Red arrow) and •• Choroid plexus papilloma (frond like enhancement
extensions along and communicating hydrocephalus)
the dural surface •• Central neurocytoma
– Dural tail sign •• Colloid cyst
(Blue arrows) - •• Sub-ependymal giant cell astrocytoma (associated
Meningioma with tuberous sclerosis)Q

Imaging Characteristics
ƒƒ Calcification containing lesionsQ:
lesions include dermoid, epidermoid, lipoma, meningeal
carcinomatosis, etc. Few of the signs that define an extra axial
•• Oligodendroglioma
lesion include:
•• Meningioma
ƒƒ Broad base toward the periphery
•• Pineoblastoma causes expansion of the already
calcified pineal gland giving exploded calcification
ƒƒ Dural tailQ: Commonly linked with meningioma
appearance.
ƒƒ CSF cleftQ: Commonly linked with meningioma
ƒƒ Buckling of adjacent cortex
•• Craniopharyngioma shows a sellar cyst with wall/
peripheral calcifications
ƒƒ Displacement of adjacent vessels
ƒƒ Sclerosis or scalloping in adjacent calvarial bones,
•• Calcification appears dense (white) on CT scan and
show blooming (extremely dark) on GRE sequence
commonly linked with meningioma
of MRI. Here you need to compare with density of the
Specific Sites calvarial bones.

Can be a very important diagnostic clue


ƒƒ Most of the neoplasms involve the white matter
ƒƒ There are some tumors that may involve the cortex and
can present with seizures (epileptogenic focus arises from
cortex/gray matter.). These include oligodendroglioma,
ganglioglioma, dysembryoplastic neuroepithelial tumor
CNS Neoplasms

(DNET).
Intensely
ƒƒ Cerebellopontine angles lesionsQ
enhancing
•• Acoustic neuroma
lesion in Sellar-
•• Schwannoma suprasellar
•• Epidermoid region with
•• Dermoid a central
•• Arachnoid cyst constriction –
•• Meningioma. Pituitary Macro-
•• The presenting symptoms of masses at this site are - adenoma
vertigo, dizziness, sensorineural hearing loss.

230
ƒƒ Hyperdense lesions on Noncontrast CT BrainQ:
CNS Neoplasm Imaging Findings
•• On plain CT scans some of the lesions which are tightly
packed appears hyperdense (shiny bright but less Epidermoid cyst • Congenital inclusion cysts
white than bones) like meningioma, medulloblastoma, • CSF like mass, insinuates cisterns,

CENTRAL NERVOUS SYSTEM IMAGING


lymphoma. encases nerves/vessels
ƒƒ Common cystic lesionsQ:
• Does not suppress completely on
FLAIRQ
•• Arachnoid cyst (often seen in posterior fossa and
• Shows restricted diffusionQ
follows CSF signals on all sequences)
• Black epidermoidQ – Rich in solid
•• Dermoid cyst crystal cholesterol and keratin
•• Epidermoid cystQ: Similar to arachnoid cysts. DWI can • White epidermoidQ – Rich in
differentiate between them as Epidermoid cysts will triglycerides and unsaturated fatty
show a restricted diffusion on DWI. acids
•• Neurenteric cyst.
Pituitary • <10 mm diameterQ
CNS Neoplasm Imaging Findings Microadenoma • Intrapituitary lesion that enhances less
Glioblastoma • Classified as WHO Grade IV GliomaQ rapidly than normal PituitaryQ
multiforme (GBM) • Most common primary intracranial • Dynamic contrast enhanced MRIQ is
neoplasm in adultsQ used for diagnosis
• Worst prognosisQ Pituitary • >10 mm diameter
• Tumor necrosisQ – hallmark finding of Macroadenoma • Sellar mass without separate
GBM identifiable pituitary gland
• Usually seen as a thick irregular • Snowman” or “Figure-of-eight”
enhancing tumor surrounding a central appearanceQ
necrotic area • Intense enhancement
• May spread across the corpus callosum
CraniopharyngiomaQ • Benign tumor derived from Rathke’s
to involve the contralateral lobe – – (Recent Pattern 2018)
Refer pouch epithelium
Butterfly gliomaQ
to Q. No. 42 in Image- • Solid-cystic, calcified suprasellar lesion
Oligodendroglioma • Most commonly involves Frontal lobe, Based Questions in childQ – is diagnostic
involving the cortexQ and subcortical
Acoustic schwannoma • Most common cerebellopontine angle
white matter
Refer to Q. No. 49 massQ
• CalcificationQ in the tumor – important
in Image-Based • Intensely enhancing lesion at
diagnostic feature – may be ribbon-
Questions cerebellopontine angle
like, central or peripheral
• Ice-cream on cone appearanceQ:
Medulloblastoma • WHO Grade IV tumors Ice-cream is the mass in the CP angle
(MB)/Primary • Most common tumor of posterior cistern, Cone is its extension into the
Neuroectodermal fossa in childrenQ internal auditory meatus
Tumor-MB • Site of origin: Roof of 4th ventricleQ
Meningioma • Most common primary non-glial
(PNET – MB) from superior medullary velum –
Refer to Image No. 58 tumorQ
appears as a solid 4th ventricle mass
in Imaging Signs in • Dural-based intensely enhancing extra-
causing hydrocephalus
Radiology axial mass lesion
• High cellularity – Hyperdense on
• Hyperdense on NC-CTQ
NC-CTQ, Hyperintense on DWI,
• Dural tailsQ – Tapered portion toward
Hypointense on T2
the dural surface – suggest a dural
• Spread along CSF – Entire neuraxis
origin
imaging required
• Mother-in-law sign on angiographyQ:
• ZuckergussQ – cake icing like deposits
Enhancement is like a mother-in-law:
on brain surface due to CSF metastasis
Comes early in life and stays put for a
Colloid cyst/ • Occurs at Foramen of MonroQ – at long time….hope you get it!!
Paraphyseal cyst paraphysis – at junction of lateral • Associated with hyperostosis of skullQ
CNS Neoplasms

(Refer to Q. No. 74 ventricles and 3rd ventricle • Pneumosinus dilatansQ – associated


in Image-Based • Smooth round lesions – Hyperdense with enlargement of paranasal sinuses
Questions) on NC-CTQ – as it is mucin filled • CSF Vascular cleft signQ – Not specific
Arachnoid cyst • Intra-arachnoid CSF filled sac not – seen in all extra-axial tumors
communicating with ventricular system • Sun-burst/Spoke-wheel pattern of
• Well defined CSF filled lesion located vesselsQ on angiography
extra-axiallyQ • Elevated AlanineQ on MR spectroscopy
• Suppressed signal on FLAIRQ (as it is • May be associated with NF2Q: MISME
CSF filled). SyndromeQ: Multiple Inherited
• No diffusion restrictionQ Schwannomas-Meningiomas-
Ependymomas
Contd… 231
CONCEPTUAL REVIEW OF RADIOLOGY

CSF intensity (hyperintense) lesion at right CP angle on T2W image, Hypodense lesion (Fat density) with calcification in the region
with incomplete suppression on FLAIR, restricted diffusion on DWI of corpus callosum – Corpus callosal Lipoma with Bracket
and minimal postcontrast enhancement – Epidermoid cyst calcification

CSF Intensity lesion


(T2 – hyperintense, T1
– hypointense) in right
anterior temporal region –
Arachnoid cyst

  Clinical Pearls
ESTS of Brain Tumors

Most common primary CNS neoplasm Glioblastoma multiforme


2nd Most common primary CNS neoplasm Meningioma
Most common intracranial germ cell tomor Germinoma
Most common mixed glioma Oligoastrocytoma
Most common site of Schwannoma Vestibular division of 8th nerve (Superior vestibular nerve)
Most common intracranial tumor in neonates Teratoma
Most common primary intracranial neoplasm in sellar/ Pituitary adenoma
parasellar region
Most common intra-axial posterior fossa tumor in adults Metastasis from extracranial sites
Most common of all primary intracranial neoplasms Glioblastoma multiforme
CNS Neoplasms

Most common supratentorial neoplasm in adults Glioblastoma multiforme


Most common intracranial tumor to calcify Oligodendroglioma
Most common site of cellular ependymoma in brain 4th ventricle
Most common nonglial primary brain tumor Meningioma
Most common intracranial extra axial tumor Meningioma
Most common spinal extradural neoplasm Metastasis
Most common benign spinal neoplasm Vertebral hemangioma
Most common malignant extradural neoplasm Metastasis
Most common spinal intramedullary tumor in adults Ependymoma (centric, associated with cysts hemorrhage, intensely
enhancing)
232 Most common spinal intramedullary tumor in children Astrocytoma (eccentric ill-defined enhancement, no cyst/hemorrhage)
4
CLINICAL QUIZ - SOLUTION
Remember it’s an imaging clincher. A calcified, solid-cystic lesion in the suprasellar region in a child is Craniopharyngioma unless
proven otherwise (Hence answer is C). Its typical location will result in impingement on the optic chiasma and lead the classical

CENTRAL NERVOUS SYSTEM IMAGING


bitemporal hemianopia.

White Matter Diseases,


Phakomatoses and Miscellaneous
5
CLINICAL QUIZ
7. A 25-year-old man presented with diplopia, urinary incontinence and VI
nerve palsy. An MRI brain revealed multiple hyperintense plaques along
the white matter tracts as shown here. Most likely diagnosis is:
A. Acute infarct
B. Multiple sclerosis
C. Intraparenchymal bleed
D. Alzheimer’s disease

White Matter Diseases, Phakomatoses...


LEUKODYSTROPHIES Leukodystrophy Salient Imaging features
Glutaric aciduria Macrocephaly with cerebral atrophy,
The leukodystrophies are dysmyelinating disorders which non traumatic subdural hematomasQ
typically, although not invariably, affect children. prominent Sylvian fissure
D/D: Battered baby syndromeQ
Leukodystrophy Salient Imaging features
Hallervorden–Spatz Eye of tiger signQ
Metachromatic Most common hereditary syndrome
leukodystrophy leukodystrophyQ autosomal recessive
Tigroid patternQ on MRI
Adrenoleukodystrophy Involvement of deep white matter
which appears hyperintense on T2/
ADEM
FLAIR. ƒƒ It stands for acute disseminated encephalomyelitis.Q
Sparing of the subcortical U fibers. ƒƒ Acute inflammatory demyelination of white matter typically
Edge of demyelination shows garland following a recent viral infection or vaccination.Q
like contrast enhancement. ƒƒ Basal ganglia gray matter and spinal cord is also involved
MRS: decrease in NAA and increase in occasionally.
lactate peak. ƒƒ MRI with contrast - is the imaging modality of choiceQ
Canavan disease Megalencephaly with FLAIR ƒƒ MRI - T2/FLAIR images show hyperintensities with
hyperintensities in subcortical white surrounding edema in subcortical locations with punctate,
matter ring or arc enhancement (open ring sign)Q along the
MRS: Elevated NAA (characteristic)Q leading edge of inflammation.
Alexander disease Macrocephaly, sparing of subcortical
U fibers, cystic leukomalacia and
hydrocephalus in end stage
Contd… 233
Eye of tiger sign MULTIPLE SCLEROSIS
– Hallervorden ƒƒ Acquired chronic relapsing demyelinating disease involving
Spatz syndrome
CONCEPTUAL REVIEW OF RADIOLOGY

CNS.
ƒƒ MRI with contrast is the imaging modality of choiceQ
ƒƒ White matter lesions (WML)Q with involvement of corpus
callosumQ, U-fibers, temporal lobes, brainstem, cerebellum
and spinal cord are the hallmark of MS.
ƒƒ MRI:
•• T1: Black holesQ (iso to hypointense WML)Q. Venus
necklaceQ is formed due to small holes at the
callososeptal interface.Q
•• T2: Regions of high signal, with surrounding edema
•• FLAIR: Dawson fingersQ are the WML propagating
centrifugally along the medullary venules and arranged
Adrenoleuko- perpendicular to the lateral ventricles in a triangular
dystrophy: FLAIR configuration (extending radially outward - best seen
hyperintensity
on parasagittal images),
involving deep
white matter
•• Ependymal Dot-Dash signQ
with rim of •• T1 C+ (Gd): punctate, ring or arc enhancement (open
enhancement on ring sign)Q is often demonstrated along the leading
Postcontrast edge of inflammation.

Multiple
hyperintense
demyelination
plaques oriented
longitudinally
along the white
matter – Dawson’s
fingers in Multiple
sclerosis

Phakomatoses
ƒƒ Optic nerve gliomaQ: Manifest as enlarged optic foramen
NEUROFIBROMATOSIS TYPE 1 (NF1) on skull radiographQ especially Rhese viewQ while on CT/
(VON RECKLINGHAUSEN DISEASE) MRI one can appreciate the central globular enlargement of
the optic nerveQ). It has to be differentiated from optic nerve
Most common phakomatosisQ
meningioma which gives tram-track sign.Q
ƒƒ Sphenoid wing dysplasiaQ which gives the Bare orbit signQ
Imaging Features
Phakomatoses

on Frontal skull radiograph.


Soft Tissues ƒƒ This because of the absence of Innominate lineQ – due to
There are multiple variable sized soft tissue lesions – greater wing of sphenoid dysplasiaQ
neurofibromas – seen as multiple ill-defined soft tissue density
Skeletal
opacities on radiographs.
NF I is also known as peripheral NF and has a host of extra CNS
CNS manifestations.
ƒƒ FASI (focal areas of signal intensity)Q: Areas of T2/FLAIR ƒƒ KyphoscoliosisQ
hyperintensity in deep white matter and basal ganglia or ƒƒ Posterior vertebral scalloping due to mass effect by
corpus callosumQ, with no contrast enhancementQ intraspinal neurofibromas
234
ƒƒ Subependymal hamartomasQ
Thickened optic ƒƒ Subependymal giant cell astrocytomas (SGCA)Q Pathog-
nerve on left nomonic for TSQ.
side with intense ƒƒ Retinal phakomas

CENTRAL NERVOUS SYSTEM IMAGING


heterogeneous
enhancement – Abdomen
Optic nerve ƒƒ Renal angiomyolipoma – Cysts – RCC – Oncocytomas.
glioma in ƒƒ Retroperitoneal lymphangiomyomatosis
Neurofibro­ ƒƒ Gastrointestinal polyps
matosis type 1
Thorax
ƒƒ Lymphangiomyomatosis (LAM): Presents as cysts of
uniform size involving lung parenchyma.
ƒƒ Cardiac rhabdomyomas: Seen in 40-80% of patients with
TS.Q
ƒƒ Ribbon rib deformityQ, rib notchingQ and dysplasia
ƒƒ Dural ectasiaQ
ƒƒ Tibial pseudoarthrosisQ or less commonly ulnar STURGE-WEBER SYNDROME
pseudoarthrosis (ENCEPHALOTRIGEMINAL
Others ANGIOMATOSIS)
ƒƒ Coarctation of aorta, renal artery stenosis which can be It is characterized by facial port wine stains and pial angiomas. It
demonstrated by CT, MR or digital subtraction angiography. is part of a wide spectrum of possible phenotypes included in the
ƒƒ Diffuse idiopathic pulmonary fibrosis which is best seen on Craniofacial Arteriovenous Metameric Syndrome (CAMS).
HRCT Plain Radiograph - Historically used to identify gyriform
calcification however with advent of CT scan it is rarely used.
CT Scan - Detects subcortical calcification - Tram track
NEUROFIBROMATOSIS TYPE II signQ at an earlier age than plain film and also demonstrates
associated parenchymal volume loss
Also known as Central NFQ or MISMEQ (multiple inherited MRI - Intense Leptomeningeal/Gyriform enhancement in
schwannomas, meningiomas, and ependymomas). affected area.

Imaging Features
MRI is imaging modality of choiceQ as most lesions are   Clinical Pearls
intracranial. And in all suspected cases of NF II MRI of whole The Tram-track Signs
spine is also warranted. Tram track sign on Cylindrical bronchiectasis – parallel
ƒƒ Meningioma: No specific location but they can be multiple. HRCT chest non-tapering walls of the bronchus
ƒƒ Schwannomas: ​Commonly affect the inferior vestibular
Tram track sign on a Sturge-Weber syndrome – Cortical
nerveQ (cranial nerve VIII) followed by facial nerve. Other
Skull X-ray/NC-CT calcifications
cranial nerves are rarely affected. Bilateral vestibular
schwannomaQ are often found in NF II.Q. These give rise to Tram track sign on Optic nerve meningioma – parallel
the typical Ice-cream on a Cone appearanceQ Postcontrast MR thickening and enhancement around
ƒƒ Ependymoma: as opposed to nonsyndromic lesions Orbit the optic nerve
where it is intraventricular here location is usually spinal
intramedullary.

VON HIPPEL-LINDAU SYNDROME


TUBEROUS SCLEROSIS
(BOURNEVILLE-PRINGLE DISEASE) Imaging Features
Phakomatoses

CNS
Imaging Features ƒƒ CNS hemangioblastomaQ: Cerebellum is most common site
CNS ƒƒ Choroid plexus papillomaQ: Frond-like enhancing intraven-
tricular lesion often presenting with communicating hydro-
Just start remembering them from the cortex inwards in
cephalus.
sequence – it is easier!
ƒƒ Cortical/subcortical tubersQ: Most common in frontal Head and Neck
lobe. On MRI they appear hyperintense on T2 and
ƒƒ Retinal hemangioblastomasQ: Most common presenting
hypointense on T1 and enhance post contrast.
featureQ and associated with vision loss
ƒƒ White matter abnormalitiesQ
ƒƒ Endolymphatic sac tumors (ELST)Q
235
Cortical tubers ORBIT
(Blue arrows), Orbital Blow-Out FractureQ – (Recent Pattern 2019)
White matter
CONCEPTUAL REVIEW OF RADIOLOGY

hyperintense ƒƒ Fracture involving the infra-orbital plate of maxillary bone.


lesions (Red The inferior orbital rim may be intact
arrow), Sub- ƒƒ Orbital contents are pushed down into the fracture defect
ependymal and get trapped – most commonly Inferior rectus muscleQ
hamartomas ƒƒ Patient complains of pain, swelling and diplopia
(Black arrows) ƒƒ Imaging findings:
– in Tuberous •• Opacification of maxillary sinus with/without a
Sclerosis horizontal air/fluid level
•• Teardrop signQ - Polypoid opacity hanging from the
roof of maxillary sinus
CT Image with •• Black eyebrow signQ - Intraorbital emphysema – free
cortical atrophy
air just above the maxillary sinus in the orbit
and calcifications.
Postcontrast Ocular Pathologies
MRI with intense
gyriform/ Ocular calcifications Adults: Optic drusen, phthisis bulbi
leptomeningeal Pediatric: retinoblastoma unless proven
enhancement – otherwise.Q
Sturge-Weber CT scan useful for both
syndrome
Retinoblastoma CT scan/MRI: Calcification involving
retina with or without detachment.
Heterogeneous enhancement post
contrast
Bilateral in familial cases
Intensely enhancing
Trilateral retinoblastoma: additional
heterogeneous cer-
pineal gland tumorQ
ebellar lesion (Blue
arrows) – Heman- Melanoma Most common intra orbital tumor
gioblastoma, along followed by metastases.Q
with multiple Pan- T1 hyperintense on MRI due to melaninQ
creatic cysts (Red Persistent Martini glass sign Q on MRI
arrow) – von Hippel Hyperplastic Primary
Lindau syndrome vitreous
Choroid detachment Kissing choroid signQ on MRI/B scan
Abdominopelvic Coloboma Association with CHARGE and COACH
ƒƒ Renal lesions: Renal cell carcinomaQ: Usually of the clear syndromes
cell type and multiple. Congenital defect resulting in protrusion
ƒƒ PheochromocytomasQ of eye contents posteriorly.
ƒƒ Numerous pancreatic lesions (may be the earliest CT scan, MRI, B scan can identify it.
manifestation): They include pancreatic cysts, pancreatic
endocrine tumors (usually non-functional), pancreatic Intraconal Pathologies
serous cystadenoma, and rarely pancreatic adenocarcinoma. Optic nerve Associated with NF2Q
ƒƒ Liver cysts/Papillary cystadenomaQ of the epididymis/ meningioma Tram track sign Q on MRIQ
Paraganglioma.
Optic nerve glioma Associated with NF1Q
Radiography: widening of orbital apex
High Yield Mnemonic
Phakomatoses

MRI/CT scan: enhancing central expanded


nerve
Features can be remembered by the mnemonic HIPPEL Thyroid Coke bottle appearance of extra ocular
• Hemangioblastoma ophthalmopathy musclesQ – muscle belly is involved but
• Increased risk of RCC tendon is spared
• Pheochromocytoma Pseudotumor Commonly involves cavernous sinus, MRI
• Pancreatic lesions (cysts, cystadenomas, useful
cystadenocarcinomas)
• Eye dysfunction (retinal hemangioblastomas)
• Liver, renal and pancreatic cysts
236
Extra Conal Pathologies
DANDY-WALKER MALFORMATION
Infections Contrast enhanced MRI – IOC
ƒƒ Rare congenital malformation that involves the cerebellum
Orbital varix Enlarges in size on ValsalvaQ and fourth ventricle.

CENTRAL NERVOUS SYSTEM IMAGING


Caroticocavernous MRI/CT: Dilated superior ophthalmic veinQ ƒƒ Most common posterior fossa malformationQ
fistula DSA is IOCQ ƒƒ Characterized by:
•• Agenesis/hypoplasia of the cerebellar vermisQ
•• Cystic dilatation of the fourth ventricleQ
ARNOLD-CHIARI MALFORMATIONS •• Enlargement of the posterior fossaQ
ƒƒ Commonly associated with obstructive hydrocephalus due
Arnold-Chiari Type I Malformation to aqueductal stenosis
Arnold-Chiari Type I Malformation ƒƒ Dandy-Walker VariantQ:
ƒƒ Also known as Cerebellar tonsil ectopiaQ. •• Less severe malformation
ƒƒ Most common type of Chiari malformationQ •• Usually characterized by partial vermian hypoplasiaQ
ƒƒ Low lying, pointed “Peg-like” cerebellar tonsilsQ – is the •• Key-hole signQ
most striking abnormality seen Other CNS Malformations:
ƒƒ Cervical cord edema/syrinxQ may also be seen
ƒƒ Best seen on MRI – on mid sagittal T1/T2W imagesQ. CNS Malformation Typical Imaging Features
Corpus callosal • Trident-shaped anterior hornsQ of lateral
Arnold-Chiari Type II Malformation dysgenesis ventricles - Viking helmetQ/Moose head
ƒƒ Best seen on MRI – on mid sagittal T1/T2W imagesQ. Refer to Image appearanceQ
ƒƒ Imaging findings: No 71 in Imaging • ColpocephalyQ – dilated posterior horns
Signs in Radiology of lateral ventricles
•• Small sized posterior fossaQ
• Probst bundlesQ – Longitudinally located
•• Cerebellar tonsils wrapped around the medulla
white matter tracts seen
inferiorly and protrude upward through the incisuraQ
• Racing car signQ: Widely spaced lateral
•• Elongated straw-like 4th ventricleQ ventricles due to agenesis of the corpus
•• Severe chronic congenital hydrocephalus callosum with intervening Probst
•• Associated corpus callosal dysgenesisQ bundles.
•• Dilated lateral ventricles, especially posterior horns –
ColpocephalyQ with pointed frontal horns
Corpus callosal • Bracket calcificationQ – on Skull
lipoma radiograph/CT
•• Associated lumbar meningomyelocele–100% associa-
tion Joubert syndrome/ • Molar tooth appearanceQ of brainstem
ƒƒ Can be diagnosed antenatally with the following features: Congenital • Bat-wing/umbrella-shaped 4th ventricle
•• Banana sign of CerebellumQ: Small-sized posterior Vermian
fossa causing the cerebellum to wrap around the hypoplasia
brainstem forming a curved tissue Holoprosen- • Most common Brain + Face
•• Lemon signQ: Downward herniation of posterior fossa cephaly malformationQ
contents leads to decreased intracranial pressure • Variable degree of Frontal lobe fusion
causing depression of frontal bones bilaterally – • Alobar (most severe & associated with
causing the shape resembling a lemon. a monoventricle) – Semilobar – Lobar
•• Fetal hydrocephalusQ (least severe type)
Lissencephaly • Hour-glass configuration of brainQ/
Figure of 8 appearanceQ
Schizencephaly • Clefts in the brain extending from surface
Refer to Q. No. 58 up to ventricles lined by gray matter
Intraocular
in Image-Based • Open-lip or Closed-lip types
mass lesion
Questions
in a child
Phakomatoses

– Retinoblas-
toma Adenoid Hypertrophy
ƒƒ Adenoid tissue is seen on the posterosuperior aspect of the
nasopharynx.
ƒƒ Lateral Head Radiograph – preferable with head tilted
upward and with an open mouth – shows the naso-
pharyngeal soft tissue and its airway compression.

237
Heterogeneously Corpus callosum
enhancing lesion dysgenesis (Blue
involving left arrow), Small
CONCEPTUAL REVIEW OF RADIOLOGY

Optic nerve caus- sized posterior


ing its fusiform fossa with
expansion – Optic cascade of tissue
nerve glioma herniating into
spinal canal (Red
arrow), Lumbar
meningocele
(Black arrow) –
Arnold-Chiari Type
II malformation

Herniation of left
orbital contents Enlarged posterior
through a fracture fossa, with Verm-
in the orbital floor ian hypoplasia and
– Orbital blow-out cystic dilatation
fracture of 4th ventricle
– Dandy-Walker
malformation

Juvenile Nasopharyngeal
MYELOGRAPHYQ (AIIMS Nov 2018/May 2015)
Angiofibroma (JNF)
ƒƒ It is an invasive, indirect visualization modality used in olden
ƒƒ Benign locally aggressive tumor occurring in the
days for assessment of spinal cord/nerve roots/spinal lesions.
nasopharynx of prepubertal and adolescent malesQ.
ƒƒ It was used in olden days when X-rays were the only
ƒƒ Plain radiograph:
modality available in the Radiology department. A lumbar
•• Nasopharyngeal mass/opacity seen puncture was done at L3-L4/L4-L5 levels and radio-opaque
•• Holman-Miller signQ/Antral signQ: Anterior bowing contrast is injected in the spinal Subarachnoid space. The
of posterior wall of maxillary sinus – not specific of JNF
spinal cord and nerve roots as well as spinal lesions if any
ƒƒ CT Scan:
appear as filling defect in this contrast column.
•• Mass lesion centered on the Sphenopalatine foramenQ ƒƒ It is rarely used now because:
– appears lobulated, nonencapsulated and shows
•• Invasive modality
intense bright postcontrast enhancement
•• Indirect visualization
ƒƒ MRI: Salt and pepper appearanceQ due to flow voids of
•• When MRI cannot be done in postoperative patients or
blood vessels
in cardiac pacemaker patient’s CT myelography may be
ƒƒ Digital Subtraction Angiography (DSA): Used for pre-
done now – as it is still much better than conventional
operative embolization – done to reduced operative blood
myelography.
loss.
ƒƒ Lesions seen on Myelography

Extradural Lesions – Most Common Lesions on


RAISED INTRACRANIAL TENSION – Myelography
Phakomatoses

RADIOGRAPHIC SIGNS ƒƒ Located outside the dura


ƒƒ Sutural diastasisQ/Widening of suturesQ: Earliest findingQ ƒƒ Most common extradural lesion is a herniated intervertebral
of raised ICT discQ
ƒƒ Copper beatenQ/Silver beaten appearanceQ of skull/ ƒƒ Other lesions include: Vertebral lesions/tumors, extradural
Increased convolutional markingsQ metastasis
ƒƒ Widening and erosion of sella turcicaQ/Truncation of ƒƒ Imaging findings:
dorsum sella •• Inward pushing of dura – causing compression of
ƒƒ Lacunar skullQ/Membranous transformation of skullQ contrast-filled Subarachnoid space
ƒƒ Pineal gland displacement across midline – suggesting •• Displacement of cord
238 midline shiftQ •• Feathered/ill-defined margin at the cord interface
Intradural – Extramedullary Lesions ƒƒ The child presents with a history of trauma, with neurological
deficits – commonly paraplegia following the trauma.
ƒƒ Located in the dura but outside the spinal cord
ƒƒ On imaging: X-ray and CT of the spine are absolutely
ƒƒ Most common lesions – Meningioma, Nerve sheath tumors
normal, in spite of there being spinal cord injury.

CENTRAL NERVOUS SYSTEM IMAGING


(Neurofibroma, Schwannoma)
ƒƒ Imaging findings:
•• Widening of contrast filled subarachnoid space – as the
lesion is within this space and will expand it. This is in SCIWORA – Another Fancy Term!
CO
absolute contrast to extramedullary lesions that will NC E P T
cause compression of subarachnoid space.
• SCIWORA results from the mismatch in the elasticity properties
•• Meniscus signQ: Sharp interface of the contrast
of the components of the spine:
material as it is in direct contact with the lesion surface
ƒƒ Vertebral column (Musculoskeletal component) – High elasticity
ƒƒ Spinal cord (Neurological component) – Low elasticity
Intramedullary Lesions
• In the pediatric age group the Vertebral column has a very high
ƒƒ Spinal cord tumors – Ependymoma, Astrocytoma elasticity. Thus when the spine is hyperflexed/hyperextended
ƒƒ Imaging findings: during the traumatic event it can easily get deformed without
•• Fusiform widening of the spinal cord – due to the tumor sustaining any fractures/dislocation. Once the event is over –
•• Effacement of contrast filled Subarachnoid space on the spine comes back to its original configuration.
either side of the lesion • Though the vertebral column can tolerate this deformation, the
•• Trouser leg appearanceQ – This appearance of widened spinal cord - being inelastic – cannot.
filling defect of the cord in intramedullary tumors • Hence during acute hyperflexion/hyperextension, there is
neurological damage, in the form of spinal cord contusion,
hematoma, transaction, nerve root tear, nerve root avulsion, etc.
SCIWORA • Thus this child always has neurological deficits.
• In olden days when X-rays/CT was done – because these were
ƒƒ Spinal Cord Injury without Obvious Radiographic the only modalities available – no findings were seen on them,
Abnormality as they are very insensitive for soft tissue abnormalities.
ƒƒ Occurrence of acute traumatic myelopathy despite normal • Hence the term SCIWORA
plain radiographs and normal computed tomography (CT) • But now we have MRI – and thus this term is not relevant in
studies today’s radiology department.
ƒƒ Children – Most commonly affected • MRI is the Investigation of choice for evaluation of spinal cord
ƒƒ Cervical spine – Most common region affected abnormalities.

Molar-tooth Enlarged soft tissue


appearance of opacity in the postero-
brainstem – superior nasopharynx
Joubert syndrome causing moderate
compromise of the
nasopharyngeal
airway – Enlarged
Adenoids

Heterogeneously Dilated lateral


enhancing ventricles (Blue arrows)
lesion at the and 3rd ventricle with
sphenopalatine a normal-sized 4th
foramen region ventricle (Black arrow)
Phakomatoses

in an adolescent – Hydrocephalus
male – Juvenile secondary to
Nasopharyngeal Aqueductal stenosis
Angiofibroma

5
CLINICAL QUIZ - SOLUTION
A 25-year-old man presenting with Diplopia, urinary incontinence and sixth nerve palsy – these are significant neurological deficits for
this age. The typical white matter plaques oriented parallel to the direction of the WM tracts – suggest demyelinating plaques seen in
Multiple sclerosis. These longitudinally-oriented plaques are known as Dawson’s fingers. Thus Answer is B. 239
Multiple Choice Questions
CONCEPTUAL REVIEW OF RADIOLOGY

Stroke Imaging 11. Tests for suspected case of a cerebrovascular syncope


1. A 56-year-old woman presents with sudden onset weakness include the following except:
of left half of her body since 1 hour. Her alert son promptly A. Carotid Doppler B. MRI Brain
brings her to the hospital. The first investigation she should C. EEG D. MRA
undergo in this setting is: 12. Pathognomonic sign of aneurysm rupture is:
A. CT Brain B. MRI Brain contrast study A. Contrast extravasation
C. Transcranial Doppler D. Chest X-ray B. Surrounding clot
2. A 55-year-old man presented with altered sensorium with C. Irregular configuration
pulse 98/min and B.P 190/140 mm Hg. A MRI Brain was D. Swirl sign
performed revealed predominantly iso to hypointense lesion 13. Focal, anterior, interhemispheric bleed usually originates
in the left putamen on T2W images, few hyperintensities from:
on T1W images, with intense blooming on GRE images. A. ACOM aneurysm B. PCOM aneurysm
Diagnosis is: C. ICA aneurysm D. MCA aneurysm
A. Traumatic hemorrhagic contusion. 14. Most common site of lacunar infarct is:
B. Acute infarct A. Basal ganglia B. Cortex
C. Hypertensive bleed C. Brainstem D. Cerebellum
D. Disseminated tuberculosis.
3. “Puff of smoke” appearance on cerebral angiography is seen Head Injury
in: (AIIMS May 2014) 15. A young male was found unconscious on the roadside after
A. ACA Aneurysm a bike accident. He regained consciousness and was sent
B. Cavernous sinus thrombosis for a CT scan. During the CT scan he again deteriorated and
C. Moyamoya disease showed altered sensorium. The CT technician said there was
D. Vein of Galen malformation a biconvex hyperdense lesion on his CT brain. Your diagnosis
4. Which of the following investigation can make an earliest is:
diagnosis of cerebral infarct? (CET Nov 2015) A. Extradural hematoma
A. FLAIR MRI B. DWI MRI B. Subarachnoid hematoma
C. Noncontrast CT D. CE-CT C. Subdural hematoma
5. Investigation of choice for evaluating a case of focal D. Hemorrhagic contusion.
neurologic deficit in an Emergency room is: (AI Dec 2013) 16. A young male was brought unconscious to the emergency
A. CT B. MRI department with external injuries. CT brain showed no
C. Lumbar puncture D. CE-CT midline shift but multiple small hemorrhages were seen with
6. Reversal sign/White cerebellum sign is seen in: effacement of basal cisterns. Most likely diagnosis is:
A. Small vessel ischemic lesions  (AI 2007, AIIMS May 2010)
B. Lacunar infarcts A. Cortical contusions B. Cerebral laceration
C. Global cerebral hypoxia C. EDH D. Diffuse axonal injury
D. Moyamoya disease 17. Classic CT appearance of acute subdural hematoma:
Multiple Choice Questions

7. Best investigation of detection of Acute SAH is: (AI Dec 2014) A. Lentiform hyperdense lesion
A. CT B. MRI B. Crescentic hypodense lesion
C. USG D. PET C. Crescentic hyperdense lesion
8. “Medusa head appearance” is seen in: D. Lentiform hypodense lesion
A. Cavernous angioma B. Venous angioma 18. The origin of an EDH is from:
C. AV fistula D. Berry aneurysm A. Bridging cortical veins
9. PVL is a feature of: B. Middle meningeal artery
A. Head injury C. Middle cerebral artery
B. Hypoxic ischemic encephalopathy D. Posterior cerebral artery
C. Acute stroke 19. Mount Fuji appearance is seen in:
D. Cerebral hemorrhage A. EDH
10. Investigation of choice for detection of an intracranial B. SDH
aneurysm: C. SAH
A. CT D. Tension pneumocephalus
B. MRI 20. Growing skull fracture is also known as:
C. MR Angiography A. Depressed fracture B. Ping-pong fracture
D. Digital subtraction angiography C. Signature fracture D. Leptomeningeal cyst
240
21. In diffuse axonal injury, the typical location of lesions in the 35. The most common intracranial tumor to calcify is: (AI 2005)
brain are: A. Oligodendroglioma
A. Cerebellum B. Ependymoma
B. Spinal cord C. Glioblastoma multiforme

CENTRAL NERVOUS SYSTEM IMAGING


C. Corpus callosum and upper brainstem D. Meningioma
D. Basal ganglia 36. Expansile lytic lesion at skull base typically involving clivus
22. Most common bleed after head injury is: with specks of calcification is suggestive of:
A. EDH B. SDH A. Osteosarcoma B. Secondaries in bone
C. SAH D. Intraventricular bleed C. Chordoma D. Fibrous histiocytoma
23. First and best imaging modality for head trauma is:  37. Most common cause of suprasellar enlargement with calcifi-
A. CT scan without IV contrast  (AIIMS May 2015) cation in brain in children on CT scan is: (AI Dec 2014)
B. CT scan with contrast A. Craniopharyngioma B. Astrocytoma
C. MRI C. Meningioma D. Suprasellar tuberculoma
D. Plain skull radiographs 38. Most common cause of mixed, cystic-solid suprasellar mass
on MRI Brain in a 10-year-old child is:
CNS Infections A. Pituitary adenoma
24. Characteristic features of Tubercular meningitis on MR B. Craniopharyngioma
imaging are: C. Optic chiasm glioma
A. Enhancing exudates predominantly involving basal cisterns D. Germinoma
B. Diffuse leptomeningeal enhancement. 39. Which of the following is used to differentiate between an
C. Hydrocephalus Epidermoid cyst and Arachnoid cyst? (AI Dec 2013)
D. All of the above A. Contrast enhancement B. Smooth margins
C. Restricted diffusion D. T2/FLAIR signal
25. Ring enhancing lesions in brain parenchyma can be seen in:
A. Neurocysticercosis B. Tuberculoma 40. Best investigation for evaluation of a posterior fossa tumor
C. Cerebral abscess D. All of the above is: (MH 2005/SGPGI 2005)
A. CT scan B. MRI
26. In following stage of neurocysticercosis is not associated
C. Myelography D. PET CT
with edema on imaging: (AIIMS 2013)
41. Best investigation for diagnosis of Acoustic schwannoma is: 
A. Vesicular stage B. Colloidal vesicular stage
A. Gadolinium enhanced MRI  (NEET Pattern Dec 2012)
C. Granular nodular stage D. Nodular calcified stage
B. CE-CT
27. Best investigation for diagnosis of Meningitis:
C. PET
A. CE-CT B. CE – MRI
D. SPECT
C. PET D. CSF examination
42. Investigation of choice for meningeal carcinomatosis:
28. “Dot-in-a-hole” sign is seen in: A. NC-CT B. CE-CT
A. TB B. Cryptococcosis C. MRI with Gadolinium D. PET
C. PML D. Neurocysticercosis
43. Ice-cream cone sign on MRI brain is seen in:
A. Pilocytic astrocytoma B. Epidermoid cyst
CNS Neoplasms C. Acoustic neuroma D. Meningioma
29. Which of the most common CNS neoplasm? 44. Most common non-glial primary brain tumor is:
A. Meningioma B. Astrocytoma A. Meningioma B. Astrocytoma
C. Medulloblastoma D. Neuroma C. Medulloblastoma D. Ependymoma
30. Which is not associated with Meningioma? 45. Sugar icing is seen in:

Multiple Choice Questions


A. Female predilection B. Neurofibromatosis type I A. Meningioma B. GBM
C. Mother in law sign D. Dural tail sign C. Medulloblastoma D. Ependymoma
31. What of the following is not skull base tumor? 46. Hyperdense on NC-CT are:
A. Esthesioneuroblastoma B. Chordoma A. Meningioma B. Colloid cyst
C. Myeloma D. Oligodendroglioma C. Lymphoma D. All of above
32. Which imaging modality is least useful in a case of CNS 47. Gyriform enhancement is a feature of:
neoplasm? A. Sturge-Weber syndrome
A. Radiograph B. Ultrasound B. Anaplastic astrocytoma
C. CT scan D. MRI C. Dural sinus thrombosis
33. Investigation of choice for spinal tumors is: D. All of above
A. CT Scan B. MRI 48. Most sensitive imaging modality for evaluation of intracranial
C. Myelography D. X-ray spine metastasis:
34. Ring enhancing lesions on a Brain CT: A. NC-CT B. CE-CT
A. Cerebral abscess C. USG D. MRI
B. Tuberculoma 49. Most common benign spinal tumor is:
C. Glioblastoma multiforme A. Osteoid osteoma B. Hemangioma
D. All of above C. ABC D. GCT
241
Leukodystrophies 63. Tram track calcification is seen in:
50. Tigroid pattern on imaging feature of: A. Sturge Weber syndrome
A. Metachromatic leukodystrophy B. Optic nerve meningioma
CONCEPTUAL REVIEW OF RADIOLOGY

B. Krabbe disease C. Both of the A and B


C. Adrenoleukodystrophy D. None of the above
D. MELAS
51. NAA peak on MRS is seen in:
Miscellaneous Topics
A. Adrenoleukodystrophy B. Kearns Sayre disease 64. Which is cause for orbital calcifications?
C. MELAS D. Canavan disease A. Retinoblastoma B. Phthisis bulbi
52. Obstructive hydrocephalus is a feature of: C. Optic drusen D. All of the above
A. Alexander disease B. Adrenoleukodystrophy 65. Optic nerve tramtrack sign is seen in:
C. MELAS D. Leigh disease A. Meningioma B. Glioma
53. Eye of Tiger sign is seen in: C. Multiple sclerosis D. Trauma
 (Recent Pattern 2015/JIPMER May 2019 Pattern) 66. Which of the following is not a feature of Arnold-Chiari type II
A. Adrenoleukodystrophy malformation?
B. Kearns Sayre disease A. Small sized posterior fossa
C. MELAS B. Hypoplastic falx
D. Hallervorden–Spatz syndrome/Pantothenate kinase- C. Lumbar meningocele
associated neurodegeneration (PKAN) D. Optic nerve glioma
67. Dandy-Walker malformation complex includes:
Multiple Sclerosis A. Cerebellar vermis hypoplasia
54. Which of the following is a type of MS? B. Enlarged 4th ventricle
A. Classic (charcoat) multiple sclerosis C. Large posterior fossa
B. Tumefactive multiple sclerosis D. All of the above
C. Marburg type of multiple sclerosis 68. Bracket calcification is seen in: (AIIMS May 2012)
D. All of the above A. Corpus callosum dysgenesis
55. Which of the site is least involved by MS? B. Corpus callosal rupture
A. Corpus callosum B. Basal ganglia C. Corpus callosum lipoma
C. Spinal cord D. Subcortical U fibers D. Septo-optic dysplasia
56. Which of the most common nerve to be affected in MS? 69. Molar tooth appearance of brainstem is seen in:
A. Olfactory B. Optic A. Arnold-Chiari type I
C. Oculomotor D. Facial B. Dandy-Walker syndrome
57. Dawson’s fingers are seen in: (NEET Pattern Dec 2012) C. Joubert syndrome
A. Cortical vein thrombosis B. Multiple sclerosis D. Rhombencephalosynapsis
C. Alzheimer’s disease D. Lacunar infarcts 70. Empty Thecal sac sign in: (AIIMS Nov 2018)
A. Arachnoiditis b. Tethered Cord syndrome
Phakomatoses C. Vertebral osteomyelitis d. Discitis
58. Which of the following is a nonhereditary phakomatoses? 71. Expansion of the contrast filled space in myelography is seen
A. Neurofibromatosis B. Sturge-Weber syndrome in: (AIIMS May 2015)
C. Tuberous sclerosis D. Ataxia telangiectasia A. Extradural lesion
59. Bare orbit sign is seen in: B. Intradural extramedullary lesion
A. Neurofibromatosis type I C. Intramedullary lesion
Multiple Choice Questions

B. Neurofibromatosis type II D. Spinal dysraphism


C. Tuberous sclerosis 72. Symptomatic spinal injury without any radiological evidence
D. Von Hippel-Lindau syndrome most commonly found in:
60. Radiological features of neurofibromatosis-1 include(s):  (AIIMS Nov 2013)
A. Posterior scalloping of vertebral bodies  (PGI May 2018) A. Children. B. Elderly
B. Sphenoid wing dysplasia C. Teenagers D. Young adults
C. Narrowing of neural foramina 73. Ideal imaging modality to diagnose hydrocephalus in a
D. Scoliosis 2-month-old baby is:
61. Imaging signs of Tuberous sclerosis include all except: A. Myelography.
 (AIIMS Nov 2014) B. USG through anterior fontanel
A. Giant cell astrocytoma B. Ependymoma C. CT scan
C. Subependymal nodules D. White matter lesions D. MRI
62. Pseudoarthrosis is seen in: 74. Characteristic imaging findings in Alzheimer’s disease: 
A. Neurofibromatosis type I. A. Temporal lobe and parietal lobe atrophy  (AIIMS 2013)
B. Neurofibromatosis type II B. Temporal and occipital lobe atrophy
C. VHL C. Frontal and parietal lobe atrophy
D. Tuberous sclerosis D. Parietal and occipital lobe atrophy
242
75. Not a sign of raised ICT: 80. First sign of raised ICT in children is:
A. Erosion of dorsum sella A. Post clinoid erosion B. Large head
B. Sutural diastasis C. Thinned out skull D. Sutural diastasis
81. Hair-on-end appearance on skull X-ray is seen in:

CENTRAL NERVOUS SYSTEM IMAGING


C. Ballooning of sella
D. Copper beaten appearance A. Sickle cell anemia B. Thalassemia
76. On MRI, the differential diagnosis of spinal cord edema is: C. Megaloblastic anemia D. Hemochromatosis
A. Myelodysplasia B. Myelomalacia 82. Face of giant panda sign is seen in:
C. Myeloschisis D. Cord tumors A. Wilson’s disease
B. Japanese encephalitis
77. Basal exudates, infarcts and hydrocephalus on CT are seen in:
C. Rasmussen’s encephalitis
A. TBM B. Viral meningitis
D. Wernicke’s encephalopathy
C. Herpes encephalitis D. Toxoplasmosis
83. Bat wing deformity of cerebral ventricles on CT/MRI is seen in:
78. Which tumor is suggested by “Trouser leg appearance” on an
A. Bilateral SDH
ascending myelogram? B. Bilateral SAH
A. Extradural tumor B. Vertebral tumor C. Corpus callosum agenesis
C. Extramedullary tumor D. Intramedullary tumor D. Cerebral ventricle agenesis
79. Imaging tool of choice for CJD: 84. Hot cross bun sign on MRI is seen in: 
A. DWI MRI with ADC maps A. Unverricht-Lundborg disease  (JIPMER May 2019 Pattern)
B. HMPAO SPECT B. Multisystem atrophy
C. MR Spectroscopy C. Huntington disease
D. PET D. Hereditary spastic paraplegia

Answers
1. A 11. C 21. C 31. D 41. A 51. D 61. B 71. B 81. B
2. C 12. A 22. A 32. B 42. C 52. A 62. A 72. A 82. A
3. C 13. A 23. A 33. B 43. C 53. D 63. A 73. B 83. C
4. B 14. A 24. D 34. D 44. A 54. D 64. D 74. A 84. B
5. A 15. A 25. D 35. A 45. C 55. B 65. A 75. C
6. C 16. D 26. D 36. C 46. D 56. B 66. D 76. B
7. A 17. C 27. D 37. A 47. A 57. B 67. D 77. A
8. B 18. B 28. D 38. B 48. D 58. B 68. C 78. D
9. B 19. D 29. B 39. C 49. B 59. A 69. C 79. A
10. D 20. D 30. B 40. B 50. A 60. A,B,D 70. A 80. D

Multiple Choice Questions

243
Explanations to Questions
CONCEPTUAL REVIEW OF RADIOLOGY

24. In TB meningitis there is inflammation of meninges so 74. The 2 most specific findings of Alzeimer’s disease on
they enhance on post-contrast images. The exudates are neuroimaging are mesial temporal lobe atrophy (especially
predominantly deposited preferentially in basal cisterns hippocampus and Entorhinal cortex) and temporoparietal
where they either block the aqueduct of Silvius or arachnoid cortical atrophy.
villi resulting in hydrocephalus. These exudates also enhance 76. Differentials of cord edema are:
on post contrast enhancement – this basal enhancement is •• Myelopathy due to cord compression as seen in trauma,
peculiar of TB meningitis. degeneration and metastatic disease (Most common) –
32. Here the doubt is between Radiographs and USG. But in with resultant myelomalacia.
CNS tumors radiographs can show many direct and indirect •• Demyelinating diseases like MS (Second most common
signs like calcification, bone erosion, hyperostosis, widening •• Tumor.
of various intracranial foramina, features of raised ICT. But •• Vascular
•• Inflammatory-Vasculitis.
ultrasound simply cannot penetrate the skull and is useful
•• Infection
only in children when the fontanelles are open.
79. Creutzfeldt-Jacob disease is a spongiform encephalopathy
42. Meningeal carcinomatosis is invasion of meninges by
caused by prions. On MRI T2 hyperintensity in basal ganglia,
carcinoma cells. Malignancies of lung, breast and melanoma
thalamus (Hockey stick sign), cortex and white matter with
are commonly involved.
persistent diffusion restriction in considered most sensitive
Contrast enhanced MRI is investigation of choice for sign. On PET scan there can be hypometabolism in affected
evaluation. area however it is less sensitive.
58. Struge-Weber syndrome is the only acquired phakomatosis. 82. Face of giant panda or Midbrain Panda sign refers to the
Rest all major phakomatosis are inherited and most of them typical appearance of midbrain when the red nucleus and
like NF1, NF2, substantia nigra are surrounded by high signal on T2W. Often
Tuberous sclerosis is autosomal dominant while ataxia seen in Wilson’s disease (best answer) but can also be seen in
telangiectasia is autosomal recessive. Japanese encephalitis.
70. Empty thecal sac sign refers to the non visualization of the 83. Also known as the Viking Helmet sign. It refers to the
nerve roots inside the thecal sac on T2 weighted MRI images. appearance of the ventricles on coronal images in cases of
This is considered specific for adhesive arachnoiditis. corpus callosal agenesis.
Explanations to Questions

244
WOMEN’S IMAGING
C hapter O utline
• Breast Imaging
CONCEPTUAL REVIEW OF RADIOLOGY

• Obstetric Imaging
• Gynecologic Imaging
• Multiple Choice Questions with Explanations

Breast Imaging
1
CLINICAL QUIZ
1. A 32-year-old patient comes to the OPD for routine breast examination and screening. Her breast examination is normal. However
there is a strong family history of breast cancer with her mother and elder sister both having breast cancer. In view of the recent
American Society of Breast Surgeons (ASBrS) guidelines at what age should her screening by Mammography begin?
A. Immediately – at 32 years C. At 40 years
B. At 35 years D. At 45 years

IMAGING MODALITIES ƒƒ Not a substitute to screening mammography.


•• It is investigation of choice when there is a malignant
axillary node suspected with occult primary in breastQ
Mammography: (Radio Mammography or ƒƒ Contrast material (Gadolinium) is injected rapidly using
Simply Mammography) a pressure injector.
ƒƒ The X-ray tube has anode made up of MolybdenumQ ƒƒ Rapid image acquisition is done continuously for around
(Characteristic spectrum radiation used predominantly) 5 – 6 minutes and dynamic enhancement curves are
with filter made most commonly of Molybdenum + plotted. The curves consist of 2 phases:
rhodiumQ. ii. Initial upslopeQ – seen during the 1st two minutes.
ƒƒ The standard views include: CC (craniocaudal)Q and ii. Delayed portionQ – seen after 2 minutes.
MLO (mediolateral oblique)Q . It cannot be done in young ƒƒ Types of Curves in Breast MRI:
patients due to dense breasts.
ƒƒ IOC for screening of early breast cancerQ Type I curveQ • Slow initial upslope with
ƒƒ The radiation dose for a standard two-view examination of a continued slow rise in
delayed phase
both breasts is approximately 3.7 - 4.7 mcGyQ
• Very likely to be benign
lesionQ
Ultrasound of Breast:
(Sonomammography)
ƒƒ Used in females less than 35 years of age but has a limited
role for screeningQ
ƒƒ High frequency probe (7 Mhz and above) is used.
ƒƒ USG is the 1st investigation and Investigation of choice in
cases of painful breastQ (Abscess/Mastitis/Galactoceles)
Type II curveQ • Slow or rapid initial
upslope with a plateau in
MRI Breast/Dynamic Contrast-Enhanced
Breast Imaging

the delayed phase


Breast MRI • Indeterminate lesionsQ
ƒƒ Dynamic contrast enhanced MRIQ – Investigation of choice
for detection of Ductal carcinoma in situ (DCIS) breast.
Previously mammography was considered best for DCIS but
now MRI has superseded it.
ƒƒ Sensitivity for detection of breast cancer is 90%, but has low
specificity.
246
Contd…
Type III curve • Rapid initial upslope with
a decline/washout in
delayed phase
• High likelihood of

WOMEN’S IMAGING
malignancyQ

Breast Pathologies Mammogram (CC Views) shows focal asymmetry of left breast

Breast Cysts
ƒƒ Common between 20-50 years of age.
ƒƒ On mammography: Circumscribed, round or oval masses.
ƒƒ On USG: Anechoic thin walled lesionsQ with posterior
acoustic enhancementQ. Echoes or septae or debris maybe
seen in complicated cysts.

Fibroadenoma/Breast Mouse
ƒƒ Most common solid breast massQ
ƒƒ On mammography : Circumscribed, oval masses. Coarse
calcificationsQ particularly in older women – Popcorn MRI Breast shows diffuse inflammatory signals in left breast
calcificationsQ suggestive of mastitis.
ƒƒ On USG : Circumscribed oval hypoechoic masses. They are
typically wider than tallerQ in appearance and may show
posterior acoustic enhancementQ. Most of these masses
show internal vascularity on Doppler.

USG breast shows a well-defined hypoechoic wider than taller


lesion, suggestive of Fibroadenoma
USG Breast showing dilated ducts (yellow arrows) with an
intraductal hypoechoic lesion – suggests an Intraductal papilloma
(white arrow)
Breast Imaging

Mammogram shows focus of popcorn calcification (arrow)


suggesting involuting fibroadenoma

Breast Cancer
ƒƒ Mammography features:
Mammography with intraductal contrast shows branching •• Carcinomas typically appear as irregular masses with 247
pattern with small papilloma (black arrow) indistinct or spiculated marginsQ.
•• Lower-grade cancers tend to be seen as spiculated •• Ill-defined echogenic haloQ around the lesion,
massesQ, due to the presence of an associated desmo- particularly around the lateral margins, and distortion
plastic reaction in the adjacent stroma. of the adjacent breast tissue may be apparent.
•• •• Most of the cancers appear hard on elastographyQ.
CONCEPTUAL REVIEW OF RADIOLOGY

Higher-grade tumors are usually seen as an ill-defined


massQ
•• Lobular carcinomas have tendency to diffusely infiltrate Miscellaneous
fatty tissue – hence difficult to detect on mammography. ƒƒ Breast abscess - USG is modality of choiceQ.
•• Ductal malignancies are often associated with micro- ƒƒ Galactoceles - Mammographic appearance varies
calcificationsQ. depending on the fat and protein content. Ultrasound -
•• Calcifications: is IOC - variable appearances – Cystic/Multicystic with
internal thick echoes/mixed solid cystic/Solid also seen.
ƒƒ Duct ectasia shows broken needle type of calcificationsQ
BI-RADS Morphology for Breast Calcifications
on mammography.
ƒƒ Vascular calcifications have a characteristic ‘tramline’
Typically Benign Suspicious appearanceQ caused by calcification in both walls of the
• Skin • Amorphous vessel.
• Vascular • Coarse heterogeneous ƒƒ Fat necrosis calcifications take on a characteristic ‘lead-
• Coarse of popcorn-like • Fine pleomorphic pipe’ appearanceQ with egg shell calcificationsQ.
ƒƒ Fibrocystic disease of breast shows tea cup appearance of
• Large rod-like • Fine, linear or fine, linear
calcificationQ on mammography.
• Round branching
ƒƒ Breast hamartoma/Fibroadenolipoma – shows a “Breast –
• Rim
within – breast” appearanceQ.
• Dystrophic
• Milk of Calcium
• Suture
BI-RADS (BREAST IMAGING –
ƒƒ Ultrasound features: REPORTING AND DATA SYSTEM)
•• Carcinomas are seen as irregular masses and are ƒƒ Designed to standardize breast imaging reporting and to
markedly hypoechoicQ compared to the surrounding reduce confusion in breast imaging interpretations.
fat. ƒƒ It can be applied to Mammography, Ultrasound and MRI
•• Margins maybe indistinctQ, spiculatedQ or angulatedQ. Breast.
•• Taller than widerQ ƒƒ The latest version classifies lesions into the following six
categories:

BI-RADS Final Assessment Categories

Category Management Likelihood of cancer


0 Need additional imaging or prior Recall for additional imaging and/or await Not Applicable
examinations prior examinations
1 Negative Routine screening 0%
2 Benign Routine screening 0%
3 Probably Benign Short interval-follow-up >0% but <2%
(6 month) or continued
4 Suspicious Tissue diagnosis 4a. Low suspicion for malignancy
(>2% to ≤10%)
4b. Moderate suspicion for malignancy
Breast Imaging

(>10% to ≤50%)
4c. High suspicion for malignancy
(>50% to <95%)
5 Highly suggestive of malignancy Tissue diagnosis ≥ 95%
6 Know biopsy proven Surgical excision when clinical appropriate Not Applicable

248
Mammogram showing different
calcifications, Egg shell type in
fat necrosis (red arrow), vascular

WOMEN’S IMAGING
(yellow arrow), popcorn type of
fibroadenoma (green arrow),
pleomorphic microcalcifications
suggestive of Neoplasm (black
arrow)

Mammogram shows Mammogram


spiculated mass (black shows saline
arrow), clustered breast
microcalcification implant
(red arrow) and nipple (black
retraction (yellow arrow) and
surrounding
arrow) suggestive
normal breast
of infiltrating duct (white arrow)
carcinoma

USG Breast
shows well
defined anechoic
thin walled
simple cyst

1
CLINICAL QUIZ - SOLUTION
Conventionally the starting age for screening by Mammography was 45 years then it was brought down to 40 years and hence many
books and resources you read will create a controversy with different values. Recently the American Society of Breast Surgeons (ASBrS)
Breast Imaging

released a Position Statement that very clearly addresses all doubts in this regard.
AsBrS Breast Cancer Screening Guidelines:
• Women aged >25 should undergo formal risk assessment for breast cancer
• Women with an average risk of breast cancer should initiate yearly screening mammography at the age of 40
• Women with a higher-than-average risk of breast cancer should undergo yearly screening mammography and be offered yearly
supplemental imaging; this screening should be initiated at a risk-based age
• Screening mammography should cease when life expectancy is <10 years

Contd…
249
Summary of ASBrS recommendations for Breast Cancer Screening:

Woman with • Woman with non-dense breasts Annual mammography (3D preferred modality starting at the age
average risk (A and B density) of 40, no need for supplemental imaging)
CONCEPTUAL REVIEW OF RADIOLOGY

• Woman with increased breast density Annual mammography (3D preferred modality starting at the age
(C and D density) of 40, and consider supplemental imaging)
Woman with • Hereditary susceptibility from pathogenic Annual MRI starting at the age of 25 annual mammography
higher-than- mutation carrier status (3D preferred modality) starting at the age of 30
average risk • Prior chest wall radiation at the age of 10–30
• Predicted lifetime risk >20% by any model Annual mammography (3D preferred modality and access to
• Strong family history supplemental imaging (MRI preferred modality) starting at the
age of 35 when recommended by their physician
Woman with prior history of breast cancer at the age of ≥50 Annual mammography (3D preferred modality)
with non-dense breast#
Woman with prior history of breast cancer at the age of <50, Annual mammography (3D preferred modality and access to
or with dense breast# annual supplemental imaging (MRI preferred modality) when
recommended by their physician)
So this patient has a very strong family history of breast cancer and hence as per these guidelines the correct age to start Mammography
screening is 35 years (Thus option B. is the answer)

Obstetric Imaging
2
CLINICAL QUIZ
2. A patient came for the 11 – 13 weeks scan and the following finding was
observed. Based on this finding the fetus would have an increased risk for
which of the following?
A. Downs syndrome
B. Turners syndrome
C. Trisomy 13
D. All of the above

3. The next best step in management of this patient would be:


A. Patient counselling
B. Immediate termination of pregnancy
C. Biochemical screening followed by invasive testing
D. NIPT

Imaging Modalities •• Lower uterine segment assessment in late pregnancy


•• Ectopic pregnancy
Ultrasound •• Pelvic masses
Obstetric Imaging

ƒƒ It is the imaging modality of choice Q •• Emergency cases when bladder is empty


ƒƒ No radiation exposure. •• Follicle monitoring
ƒƒ Transabdominal sonography (TAS)Q images the pelvic •• Oocyte retrieval
organs through the anterior abdominal wall in the supra- •• Endometrial study to assess suitability in IVF-Embryo
pubic region through a distended urinary bladder. Transfer techniques
ƒƒ TVS is indicated whenever a better look at the pelvic •• Cervical canal abnormalities.
structures is required. Common indicationsQ include the ƒƒ Doppler function is useful to study flow dynamics of blood
following: vessels and can help to comment on uteroplacental or
•• Early pregnancy fetoplacental insufficiency.

250
3–5 mm structure, usually eccentrically located within
the gestational sac
•• Double bleb signQ (AIIMS May 2016 Pattern): The
CO Transabdominal vs Transvaginal USG
NC E P T amniotic sac-embryo-yolk sac complex can be seen

WOMEN’S IMAGING
with ultrasonography (US) as two small blebs of almost
Transabdominal Transvaginal USG equal size attached to the wall of the early gestational
USG sac – the double bleb sign.
Full bladder Required Not required ƒƒ >6.0 weeks:
Probe used & Convex probe – 3.5 TVS/Endovaginal probe •• The embryo is first visible at approximately 6 weeks of
its Frequency – 5 Mhz –~7.5 Mhz gestational ageQ as a 1–2-mm structure
•• The length of the embryo is measured from the head
Resolution Mild - moderate High
(crown) to the buttocks (rump), hence the term crown-
Field of view Large Small – limited to pelvis rump length (CRL)Q, which is the most accurate
Interventional Limited Multiple uses measurement of gestational age through the first 12
applications especially in Infertility weeks of pregnancyQ.
management •• Fetal cardiac activity:
Contra- None Virgin patients,  Cardiac pulsation begins at approximately the 6th
indications Vaginismus, Premature week of gestationQ
rupture of membranes  At this stage the heart rate is always >100 beats/
minQ
ƒƒ 7 – 11 weeks:
MRI •• Cephalad and caudal poles can be identified with
ƒƒ It is a newer technique that is gaining wide acceptance. a round hypoechoic structure seen in the fetal
ƒƒ It is used as a problem-solving tool in Obstetric imaging. brain represents a developing embryonic/fetal
ƒƒ Indications can be fetal (in cases of equivocal anomaly USG rhombencephalonQ
scans), placental issues like placental percreta/increta/ •• Limb buds appearQ
accrete. •• Fetal movement can be seen (>9 weeksQ)

Nuchal Translucency (NT) ScanQ


EARLY PREGNANCY (1ST TRIMESTER) ƒƒ According to Fetal Medicine Foundation (FMF-UK)
guidelines NT scan is performed at the end of first trimester,
IMAGING generally from 11 weeks 0 days to 13 weeks 6 daysQ or a CRL
of 45 – 84 mmQ. After 13 weeks 6 daysQ/CRL of more than 84
Important Ultrasound Findings in the mmQ – it is known as Nuchal fold thicknessQ.
1st Trimester ƒƒ Principle: In certain abnormal fetuses the lymphatics
of subcutaneous tissues are defective and this results in
ƒƒ 0–4.5 weeks: No ultrasound findings suggestive of
increased fluid collection and hence abnormal nuchal
pregnancy
translucency thickness.
ƒƒ 4.5 - 5.0 weeks:
ƒƒ A NT more than 2 mm is associated with significant fetal
•• A gestational sac can first be visualized at TVS at 4.5–
anomalies. However this is a crude cut-off and the actual NT
5.0 weeksQ of gestational age as a 2–3-mmQ rounded
varies with gestational age
intrauterine anechoic area.
ƒƒ Associations:
•• Thickened/decidualized endometrium with/without a
corpus luteal cyst
•• AneuploidyQ – Trisomy (most commonly Down’s
syndromeQ), Turner syndromeQ, Trisomy 18Q, Trisomy
•• Intradecidual sac signQ - Eccentrically located
13Q
gestational sac within the echogenic decidua, with
a relatively undisturbed collapsed uterine cavity
•• Non aneuploidyQ - Congenital diaphragmatic hernia,
congenital heart disease, omphalocele, skeletal
visualized as a thin echogenic line
dysplasia, VACTREL association.Q
•• Double decidual sac sign (DDSS)Q - consists of
Obstetric Imaging

ƒƒ The other markers of Aneuploidy that can be evaluated in


two echogenic rings surrounding the hypoechoic
this scan are
gestational sac. The inner ring represents the chorion,
embryonic disc and decidua capsularisQ. The outer •• Nasal boneQ
ring represents the decidua parietalisQ. •• Ductus venosus flowQ
ƒƒ 5 – 6 weeks: •• Tricuspid regurgitationQ etc
•• ​Gestational sac is visible by this timeQ even on trans-
abdominal scans
•• Yolk sac/Secondary yolk sacQ - Earliest structure SECOND TRIMESTER IMAGING
seen inside the gestational sac and can be visualized at ƒƒ Popularly known as Congenital anomaly scan or Targeted
approximately 5.5 weeks of gestational age as a round imaging for fetal anomaly (TIFFA)
251
IMPORTANT FETAL ANOMALIES
Brain and Spine Anomalies
CONCEPTUAL REVIEW OF RADIOLOGY

Neural Tube Defects


ƒƒ Most commonly identified anomalies
ƒƒ Can be varied in severity from Anencephaly, encephalocele
to lumbar meningomyelocele.
ƒƒ Anencephaly is the earliest identified anomalyQ on USG and
can be diagnosed at around 11 weeks.
ƒƒ Banana sign of CerebellumQ:
ƒƒ Lemon signQ:
ƒƒ Fetal hydrocephalusQ
ƒƒ In the spine the vertebral posterior elements are normally
seen converging. Instead in neural tube defects they may
appear as diverging with a spina bifida defect & a protruding
meningocele or meningomyelocele.
ƒƒ Open spinal dysraphism is associated with polyhydramniosQ.

Dandy-Walker Syndrome
Clockwise images showing gestational sac (white arrow) with
yolk sac (black arrow in image 1), double bleb sign (Red arrow in ƒƒ Large cyst may be seen in posterior fossa.
Image 2) and measurement of crown rump length (CRL) in fetus ƒƒ Keyhole sign is seen in Dandy-Walker variantQ

Hydrocephalus
ƒƒ A diameter of either lateral ventricle more than 10 mmQ
ƒƒ Performed in India between 18–20 weeks . Q ƒƒ Aqueductal stenosisQ – is the most common cause. In these
ƒƒ For Biometry assessment (Estimation gestational age cases the lateral ventricles and 3rd ventricle are dilatedQ,
and weight) Hadlock methodQ is used which consists of 4 whereas the 4th ventricle is normal in sizeQ.
parameters, namely
iii. Bi-parietal diameter (BPD)Q
iii. Head circumference (HC)Q Clockwise
iii. Abdominal circumference (AC)Q – Most accurate/ images showing
important for IUGR assessment Q measurements of
iv. Femur length (FL) Q Nuchal translucency
(1), Biparietal
diameter + head
  Clinical Pearls circumference
(2), Abdominal
Antenatal USG – Best parameter for Dating of gestation circumference (3)
and Femur length
Best for Dating of pregnancy Crown-Rump-Length (CRL)Q (4)
in 1st Trimester
Best for Dating of pregnancy Biparietal diameter (BPD) >
in 2nd Trimester Head circumference (HC) Q Grossly
thickened
Best for Dating of pregnancy Femur length (FL) Q Nuchal
in 3rd Trimester translucency
Obstetric Imaging

Overall best time for Dating 1st trimesterQ (around


7.3 mm). Fluid
Overall best parameter for Crown-Rump-Length (CRL) Q collection also
Dating seen around
fetal head
Overall best parameter Abdomen circumference (AC) Q and chest –
for fetal weight (IUGR/ suggests Fetal
Macrosomia) hydrops

252
Holoprosencephaly Corpus Callosal Agenesis
ƒƒ It is due to nonformation of the frontal lobes and has 3 types, ƒƒ Spoke wheel appearance of gyri on sagittal images.
namely Alobar, Semilobar and Lobar. ƒƒ On axial sections there can be Colpocephaly (parallel

WOMEN’S IMAGING
ƒƒ In Alobar type - Fused thalamiQ, Single monoventricleQ orientation of occipital horns of lateral ventricles)Q
seen.
ƒƒ Associated with facial anomalies like - Single eyeQ (Cyclops Other CNS Anomalies
appearanceQ), Proboscis like noseQ
Vein of Galen • It is seen as an elongated supratentorial
malformation midline cystic structure which shows color
flow on Doppler imaging

Absence of Lemon-shaped
cranial vault skull due to frontal
& brain tissue indentation
(white arrow) (red arrows),
with frog-like additionally
prominent seen are dilated
eyes ventricles
(black arrow) (green arrow)

Meningocele (red arrow) arising from the terminal portion of USG image of brain shows monoventricle (black
lumbar spine (white arrow) with surface rendered image arrow) with fusion of thalami (white arrow)
suggests Alobar holoprosencephaly

4-chamber heart Parallel vessels


showing small seen arising
perimembranous from both
VSD (red arrow) ventricles,
suggestive of
Obstetric Imaging

transposition
of great
vessels

253
USG shows VSD USG at
(orange arrow), thorax shows
overriding echogenic lung
CONCEPTUAL REVIEW OF RADIOLOGY

of aorta (red with cysts in


arrow) suggests it (red arrow)
Tetralogy of adjacent to
Fallot. Other the heart
components (black arrow)
are not suggest
demonstrated Congenital
here, but were cystic adenoid
confirmed on malformation
USG

Herniation of Multiple
abdominal cysts in both
contents fetal kidneys,
(Red arrow) suggest
thoroughly Multicystic
defect in anterior dysplastic
abdominal kidneys
wall (black (MCDK)
arrow) suggests
omphalocele

Double bubble
Deformed sign (red arrows)
spine of duodenal
(red arrow) atresia

Hitch hiker Fetal abdomen


thumb with liver (white
Obstetric Imaging

arrow) and fetal


ascites
(red arrows)

254
OTHER IMPORTANT ANOMALIES
Cardiac anomalies:

WOMEN’S IMAGING
The structures of the heart are best seen at 22 to 24 weeksQ of gestation 4 chamber viewQ and 3 vessel viewQ are the 2 most important
views.
The most common conditions diagnosed include ASD, VSD, AVSD, TOF, TGA, TAPAVC, TGA, hypoplastic left heart syndrome, DORV,
truncus arteriosus, etc.
Echogenic intracardiac • Most commonly seen in left ventricleQ.
foci • Considered as a soft marker for aneuploidyQ
Ventricular septal • Membranous/PerimembranousQ - Most common (including the Gerbode defect)
defect (VSD) • Perimembranous VSD can be seen as a septal dropout in the area adjacent to the tricuspid septal leaflet and
below the right border of the aortic annulus.
• Small isolated VSDs can be difficult to detect prenatally.
Other Important anomalies:
e Single umbilical arteryQ • Not considered as an anomaly by itself
both • Normally there are 2 umbilical arteries and one vein (the left one is leftQ while right is obliteratedQ).
dneys, • Single umbilical artery – is seen in 1% of the fetuses and commonly seen with twins.
• But often associated with IUGRQ and sometimes trisomiesQ (18, 13 and 21), renal issues, sirenomelia
stic (mermaid syndrome).
tic Congenital cystic • CPAM (P for pulmonary).
adenomatoid • Though postnatally they appear as cysts in lungs in fetal USG due to very small cysts which are beyond
malformation (CCAM)Q resolution of USG there is diffuse increase in echogenicityQ of fetal lungs.
Congenital • It is mostly left sided (Bochdalek’s herniaQ) and stomach or bowel may be seen in thorax causing rightward
Diaphragmatic hernia displacement of heart.
(CDH) Q • When right sided, liver can be seen to lie in thorax.
• Obstruction to fetal swallowing can cause polyhydramnios.
• Associated with pulmonary hypoplasiaQ – commonest cause of death in these children
OmphaloceleQ • Results from failure of regression of normal embryonic midgut from the umbilical stalk.
• The abdominal contents including bowels, spleen, liver are herniated through the umbilicus but covered
with amniotic membraneQ
• The umbilical cord is inserted at the apex of the hernia sacQ
• Associated with trisomy 18 and 13Q
• It is a correctable lesion and in absence of other anomalies survival rate is good.
GastroschisisQ • It is herniation of bowel or other viscera through a paraumbilical defectQ in anterior abdominal wall.
• It is not commonly associated with chromosomal abnormalities
• Umbilical cord insertion is normal at the umbilicusQ
• However entry of amniotic fluid in fetal peritoneum can cause chemical peritonitis – poor prognosisQ.
Bladder exstrophyQ • Can be isolated or associated with epispadias.
• Cloacal exstrophy is associated with the OEIS syndrome - (omphalocele, exstrophy, imperforate anus and
spinal defects).Q
Duodenal obstruction • Causes can be atresia, midgut volvulus, annular pancreas.
• Presents as a USG Double bubble sign and is associated with Down’s syndrome.Q
Renal agenesis/Potter • It is incompatible with life and the baby dies in the first week after birth due to pulmonary hypoplasia.
syndromeQ • As urine constitutes significant amount of liquor, in these cases there is severe oligohydramnios which
causes typical flattened facies in the fetus along with pulmonary hypoplasia.
• Mnemonic POTTERQ:
Obstetric Imaging

ƒƒ Pulmonary hypoplasia
ƒƒ Oligohydramnios
ƒƒ Twisted wrinkly skin
ƒƒ Twisted abnormal face/Potter facies – Flattened face, low set ears, hypertelorism, retrognathia
ƒƒ Extremity deformities – Limb deformities like club feet
ƒƒ Renal agenesis
Multicystic Dysplastic • Most common abdominal mass in neonateQ
kidney (MCDK)Q • Diagnosed antenatally on USG – Enlarged kidney with multiple macrocysts withinQ, oligohydramniosQ,
• Non-visualization of renal pelvis and ureter
Contd…
255
Autosomal recessive • Most common of all the renal cystic diseasesQ, manifesting in infancy and childhood
polycystic kidney • Kidney & Liver – are always involved
disease (ARPCKD) Q • Seen antenatally as bilateral symmetrically enlarged echogenic kidneysQ, oligohydramnios and a non-
distended fetal UB.
CONCEPTUAL REVIEW OF RADIOLOGY

• In neonates and infants – Smooth, enlarged kidneys, with loss of corticomedullary differentiation on USG.
• Striated nephrogramQ – seen on conventional/CT urogram
• Liver abnormalities – Liver cysts and features of portal hypertension
Posterior urethral • It is exclusively disease of malesQ and important cause of bladder outlet obstruction in infants.
valveQ • Key hole/Spinning top appearanceQ: On antenatal USG the over-distended bladder and dilated prostatic
urethra give rise to a peculiar appearance
• Postnatally MCU will demonstrate the slit like valve in posterior urethra. Associated vesicoureteric reflux can
also be identified.

Lower
Antenatal Polycystic Kidneys – Do margin of
CO
NC E P T not Jump to a Logical Diagnosis! placental is
encroaching
• A polycystic kidney seen on an antenatal USG should logically on internal
be suggestive of Polycystic kidney disease, isn’t it? Os, suggests
• Well, life is not as simple and logical as it seems though ! So placental
STOP. Do not jump to a diagnosis as yet. previa
• Autosomal Recessive Polycystic Kidney Disease (ARPCKD):
ƒƒ Is seen in children and this is the type that can be diagnosed
antenatally.
ƒƒ Cysts are typically microcysts – hence usually not identified
on USG
ƒƒ Multiple cyst walls result in increased reflectivity of renal
tissue – hence USG appearance is that of bilateral enlarged
echogenic kidneys, with oligohydramnios.
• Multicystic Dysplastic Kidneys (MCDK): Multifetal Pregnancies
ƒƒ Can be diagnosed antenatally ƒƒ These are more commonly associated with in vitro
ƒƒ Has renal Macro-cysts fertilization proceduresQ.
• Thus the “Harsh Reality” is that: ƒƒ Early ultrasound (first trimester) will help to identify multife-
ƒƒ Antenatal Polycystic appearing kidney on USG – is actually tal gestation as well as their chorionicity.Q (Recent Pattern Jan 2018)
an MCDKQ ƒƒ Twin pregnancies can be either dizygotic (70%) or
ƒƒ Antenatal bilateral enlarged echogenic kidneys suggest – monozygotic (30%).
ARPCKDQQ ƒƒ Depending upon their age of separation chorionicity will be
Don’t go wrong now…..because all your competitors will! determined.
•• Dichorionic diamniotic twins: 0-4 daysQ
•• Monochorionic diamniotic twins: 4-8 daysQ
Third Trimester Scan •• Monochorionic monoamniotic: 1-2 weeksQ
ƒƒ Generally 2 scans are done one after completion of 28 weeks •• Conjoint (Siamese) twins: >2 weeksQ
and one just before term. ƒƒ Twin peakQ or lambda signQ is seen in dichorionic twins,
ƒƒ It is basically used to assess serial fetal growth, status while T signQ is seen in monochorionic twins.
of liquor (Amniotic fluid index), placental status, fetal ƒƒ The different complications of twins that can be identified
presentation, etc. on USG include twin-twin growth discordanceQ, twin-
twin transfusion syndrome (TTTS)Q, twin reversed arterial
Obstetric Imaging

ƒƒ Additionally this can be clubbed with Doppler examination.


Some of the salient features of abnormal Doppler findings perfusion sequenceQ (TRAPS) or twin anemia polycythemia
are: sequenceQ (TAPS).
•• Earliest signs of uteroplacental insufficiency is uterine
artery prediastolic notchQ Abnormalities of Amniotic Fluid
•• Early signs of fetoplacental insufficiency is reduced ƒƒ Measured by 4 quadrant method in third trimester which
diastolic flow in umbilical arteryQ, followed by absent gives amniotic fluid index.
end diastolic flow and finally reversed end diastolic ƒƒ Oligohydramnios: Usually diagnosed when AFI is <10 cm.
flow in the umbilical artery. Along with decrease in the The common causes are hypo or non-functioning bilateral
diastolic flow in the Umbilical artery – there is increase kidneys (polycystic kidneys, bilateral agenesis), urethral
in the diastolic flow in the fetal middle cerebral artery obstruction, premature rupture of membranes, placental
256 (Brain sparing effect)Q. insufficiency.
ƒƒ Polyhydramnios: It Is considered when AFI is more than ƒƒ On Doppler the tissue has increased vascularity.
25Q. The common causes include neural tube defects, ƒƒ In partial mole a gestational sac or fetal parts can be seen.
gastrointestinal obstructions, neck masses compressing ƒƒ MRI in modality of choice for invasive mole or choriocarci-
esophagus, maternal diabetes mellitus, fetal hydrops nomaQ.

WOMEN’S IMAGING
ƒƒ The metastases from choriocarcinoma are common in chest
Hydatidiform Mole/Molar Pregnancy (Refer and brain and are hemorrhagic type.
to Q. No. 35 in Image-Based Questions)
ƒƒ On USG on older machines - Snowstorm appearanceQ has
been described. The uterine cavity is filled with multiple
fluid filled thin walled cystic contents.

2
Clinical Quiz - Solution
So NT of 7.3 mm is grossly increased and abnormal. This may be (PAPP-A). The combination of Maternal age associated background
seen in all of the conditions mentioned in the options, namely risk + NT scan + Double marker test is known as Combined first
Down’s syndrome, Turners syndrome and Trisomy 13. It may trimester screening and is much more sensitive for detection of
also be associated with non-aneuplodic conditions – Congenital aneuploidy than any of the individual test components.
diaphragmatic hernia, congenital heart disease, omphalocele, TEST Detection rate
skeletal dysplasia, VACTREL association.Q for Trisomy 21
Maternal age only 30%
NT scan only 70%
Double marker test (Beta hCG and PAPP-A) 60%
only
Maternal age + NT scan 70 – 80%
Combined 1st trimester screening 85 – 95%
(Maternal age + NT scan + Double marker
test)
Triple test (HCG + AFP + Unconjugated 70%
estriol)
Quadruple test (HCG + AFP + 75%
Unconjugated estriol + Inhibin A)
Next step in Management will be Biochemical screening by
performing Double/Dual marker test – comprising estimation Serial integrated test (HCG + AFP + 85%
of Free Beta-HCG and Pregnancy associated plasma protein – A Unconjugated estriol + Inhibin A + PAPP-A)

Gynecologic Imaging
3 Gynecologic Imaging
CLINICAL QUIZ
4. A patient being treated for infertility was treated with injection
HMG following which she presented with abdominal pain and
vomiting. Most likely diagnosis based on this USG image is:
(NEET 2020 Pattern)
A. OHSS B. PCOD
C. Theca lutein cysts D. Ovarian torsion
5. The next best step in management of this patient would be:
A. Laparoscopy
B. Conservative management with IV fluids/colloids
C. 10000 IU of hCG IM injection
D. Low dose irradiation 257
UTERINE ABNORMALITIES
A HSG spot
image showing
Müllerian Duct Anomalies a concave
CONCEPTUAL REVIEW OF RADIOLOGY

ƒƒ MRI - is the best modality.Q impression on


ƒƒ USG/HSGQ - Initial/1st modality the uterine
fundus – s/o
Classification and imaging findings: Based on their prognosis
Arcuate uterus
for future fertility and surgical treatment
ƒƒ Class I: Uterine agenesis/Uterine hypoplasia:
•• This anomaly is part of the Mayer-Rokitansky-Küster-
Hauser syndromeQ
•• MRI is confirmatory.
ƒƒ Class II: Unicornuate uterus/Unicornis unicollis: Often
associated with recurrent abortions. Banana-shaped
appearance on HSG.Q Ipsilateral renal anomalies are
commonly associatedQ. Refer to Image 73 in “Imaging
signs in Radiology”
ƒƒ Class III: Uterus didelphysQ:
•• Duplication of uterine horns and cervix with no
communication between them
ƒƒ Class IV: Bicornuate uterus: Only failure of division of the
fundic region.
ƒƒ Class V: Septate uterus: Most common anomalyQ. Also, the
anomaly with highest rate of pregnancy lossQ
ƒƒ Class VI: Arcuate uterus: Clinically no significanceQ. Has
no effect of fertilityQ. A broad-based indentation is seen on A HSG spot
the fundal aspect of the endometrium. image showing
ƒƒ Class VII: in utero diethylstilbestrol (DES)Q exposure: On a Bicornuate
HSG it shows T-shaped uterusQ uterus

Uterine Fibroids
ƒƒ Most common solid benign uterine neoplasmsQ.
ƒƒ Location wise they are classified as   Clinical Pearls

Intrauterine – Outside inwards Extrauterine FIBROIDS – Jackpot of MCQs!


• Subserosal • Broad ligament Overall MC fibroid IntramuralQ
• Intramural – • Cervical
MC type of fibroid associated with Sub-mucosalQ
ƒƒ Most common typeQ. • Parasitic
Infertility/Recurrent miscarriages
ƒƒ May be treated with Uterine artery
Embolization (UAE)Q MC fibroid to undergo malignant Sub-mucosalQ
• Submucosal – change
• Least commonQ MC symptom of fibroid MenorrhagiaQ
• Associated with infertilityQ
Lantern on dome of St Paul Pedunculated cervical
appearance fibroidQ
Imaging Features MC type of degeneration in fibroids Hyaline degenerationQ
Gynecologic Imaging

ƒƒ Plain radiograph: Lest common type of change in Malignant change Q


•• Popcorn calcification/peripheral egg shell like fibroids
calcification in pelvisQ
ƒƒ Ultrasound: Most common investigationQ used clinically.
Appear as well defined and hypoechoicQ, but can be isoechoic, Adenomyosis
or even hyperechoic compared to normal myometrium
ƒƒ MRI: MRI is not generally required for diagnosis, except for It is ectopic endometrial tissue in the myometriumQ. Presents
complex or problem-solving cases. with menorrhagia and dysmenorrhea.
•• Non-degenerated fibroids appears to hypointense Imaging Features
on T1/T2 with variable enhancement.
•• T1 hyperintense rim around a centrally-located myoma ƒƒ Ultrasound
suggests red degeneration, which is caused by venous •• Usually the firstQ and often the only imaging modality
thrombosisQ employedQ.
258
•• Imaging findings include: ƒƒ Dermoid cysts – Fat (highly echogenic foci on USG), dense
 Round/Globular-shaped uterusQ calcification (hyperechoic with posterior shadowing),
 Heterogeneous appearing myometrium with fat fluid levels may be seen on USG. Fat can be easily
indistinct endo-myometrial interfaceQ demonstrated on MRI. Solid component may show variable

WOMEN’S IMAGING
 Thickening of the transition zone can sometimes enhancement on CE-MRI.
be visualized as a hypoechoic halo surrounding ƒƒ Endometriotic cysts/Chocolate cystsQ: Seen as cysts with
the endometrial layer of ≥12 mm thicknessQ uniform low level internal echoes withinQ- Ground glass
ƒƒ MRI Pelvic MRI is the modality of choiceQ sign on USGQ. They show hypointense signal on T2W
ƒƒ Ill-defined low-density masses on both T1 and T2W images images suggestive of - T2 shading signQ.
seenQ
Polycystic Ovarian Disease
Tubal Abnormalities ƒƒ Also known as Hyperandrogenic anovulationQ, is a chronic
ƒƒ Tubal block: anovulation syndrome associated with androgen excess.
•• HSG – Most commonly used modality ƒƒ Ultrasound: Usually sufficient for diagnosis.
•• Sonosalpingogram - checks for free intraperitoneal •• Rotterdam criteria 2003Q:
spillage of fluid instilled through cervical cannulation  12 folliclesQ measuring 2 – 9 mm in each ovary
without risk of radiation and gives idea about patency (Follicle Number Per Ovary – FNPOQ)
of tubes but exact anatomic localization not possible.  Ovarian volume >10 cm3Q
ƒƒ Hydrosalpinx/Pyosalpinx:  Any one ovary meeting these criteria is enough for
•• Appears as retort-shaped contrast/fluid filled adnexal diagnosis
structureQ (NEET 2020 pattern – Image-based question) •• New diagnostic criteria: “PCOS: New Diagnostic
seen either on USG or HSG. Incomplete septaeQ are Criteria Recommended - Medscape - Apr 17, 2013.”:
seen within it on USG  26 folliclesQ per ovary (FNPO)
•• MRIQ – Best investigation Refer to Image 74 in  Ovarian volume >10 cm3Q
“Imaging signs in Radiology”  Other useful morphological features:
•• Hyperechoic central stromaQ
•• Peripheral location of follicles: which can give a String
  Clinical Pearls of pearl appearance.Q

Tubal Factor Infertility Specifics Ovarian Hyperstimulation Syndrome (OHSS)Q


1 test to assess tubal patency
st
Hysterosalpingography (NEET Pattern PG 2020—Image-based Question)
(HSG)Q
ƒƒ Clinical symptom complex characterized by ovarian
Best time to perform HSG Day 6 – 10Q of menstrual enlargement and increased vascular permeability (due to
cycle increased vascular endothelial growth factorQ) resulting in
Most common cause of Physiological cornual loss of fluid.
bilateral cornual block on HSG spasmQ. Hence HSG cannot ƒƒ Seen in young females with PCOS and those administered
differentiate between with hCG injectionsQ
physiological spasm and ƒƒ Abdominal painQ and distensionQ – Most common symptom
true cornual block!
ƒƒ USG – enlarged ovaries with multiple folliclesQ – predisposed
Most common cause of Genital TBQ to torsion/hemorrhagic cysts/rupture of cysts. Mild to
pathological bilateral cornual moderate free fluid in abdomen +/- pleural effusions
block on HSG ƒƒ Treatment – Conservative with admission + IV fluid/colloids
Best investigation/Gold Laparoscopy with + Paracentesis of tense ascites + Prophylactic heparin –
standard for confirming tubal chemoperturbationQ to prevent thromboembolic phenomena secondary to
patency hemoconcentration Gynecologic Imaging
Ovarian Torsion
Ovarian Abnormalities ƒƒ Twisting of ovary over its pedicle – presents as Acute
abdomen.
Ovarian Cysts ƒƒ On USG the torsed ovary appears to be enlarged, edematous
ƒƒ USG – Initial modalityQ with peripherally displaced follicles. There may be free fluid
ƒƒ MRI – Best modalityQ in pelvis.
ƒƒ Simple cysts - on USG appear as anechoic, thin-walled well- ƒƒ On Doppler generally there is absence of venous flow with
defined round lesions. Fluid intensity on MRI, without any reduced arterial flow, absence of both arterial and venous
contrast enhancement flow indicates poor prognosis. The pedicle which has
ƒƒ Complex/Hemorrhagic cysts – presence of internal echoes/ undergone torsion may demonstrate twisting - Whirlpool
contents/debris/septae. Commonly painful. sign.Q
259
Sagittal MRI of USG image
uterus shows shows
multiple fibroids anechoic
CONCEPTUAL REVIEW OF RADIOLOGY

(white arrows) simple cyst


and endometrial in left ovary
collection (white
(black arrow) arrow)

Axial CECT Radiograph of


image shows pelvis shows
pelvic mass round calcified
containing fibroids (black
fat (white arrows)
arrow) and
calcification
(black arrow)
suggestive of
dermoid cyst

Post-contrast
sagittal MRI image -
USG image - Heterogeneous lesion
Hypoechoic in the endometrial
lesion with cavity – enhancing
thick internal less than the normal
echoes in ovary endometrium – on
– suggests a post-menopausal
Hemorrhagic woman – suggests
cyst Endometrial cancer

Ovarian Tumors •• Represents the transcoelomic spread to ovaries with


the primary being commonly in stomachQ or colonQ.
ƒƒ The ovarian neoplasms can present as either complex cysts
ƒƒ Ovarian fibromas:
or solid lesions.
•• Solid T1/T2 hypointense lesions which show mild to
ƒƒ Features favoring Neoplasm over complex cystsQ:
none enhancement
•• Presence of thick septations (>5 mm)Q
•• Meigs’ syndromeQ - Ovarian fibroma with ascites and
Gynecologic Imaging

•• Enhancing mural nodulesQ pleural effusion


•• Enhancing wallsQ ƒƒ Struma ovaryQ
ƒƒ CalcificationQ and fat componentQ - classical for Teratoma/
•• Ectopic thyroid containing tumor which will appear
Dermoid.
hyperdense on CT scan and show intense homogenous
ƒƒ Krukenberg tumorsQ/Carcinoma mucocellulareQ: post-contrast enhancement.
•• Both ovaries are enlarged ƒƒ It will also show uptake of thyroid specific iodine radionu-
•• Shows solid enhancement with smooth surfaces. clide study.Q

260
3
CLINICAL QUIZ - SOLUTION
These images look like Polycystic ovaries, don’t they? But look at the history given in the quiz question! Clearly the examiner wants

WOMEN’S IMAGING
to help you here by giving you a history of Infertility + Inj. HMG – used for follicle stimulation. Also the history of Abdominal pain,
distension and vomiting (not seen in PCOS) – points towards a diagnosis of OHSS. Theca lutein cysts are associated with Molar
pregnancy/Multifetal gestation and does not fit with this clinical picture. Torsion usually affects a single ovary – with severe pelvic/
lower abdominal pain and USG features of enlarged edematous ipsilateral ovary – with doppler changes. The other (contralateral)
ovary is usually normal.

Management of OHSS is usually supportive. Admission + IV fluid/colloids – to correct the hemoconcentration resulting from vascular
fluid loss. Paracentesis of tense ascites – to relieve mass effect. Prophylactic heparin – to prevent thromboembolic phenomena
secondary to hemoconcentration.

Multiple Choice Questions


Breast Imaging 8. A 55-year-old postmenopausal woman on Hormone
1. Breast lesions are described as per: replacement therapy presents with heaviness in both breasts.
A. BI-RADS B. BORDAC A screening mammogram reveals a high density, spiculated
C. BARIDS D. BEREED mass lesion with cluster of pleomorphic microcalcifications
2. Which is not sign of benign breast lesion? and ipsilateral enlarged axillary lymph nodes. This mass most
A. Wider than taller B. Spiculation likely represents:
C. Macrocalcifications D. No color flow on Doppler A. Cystosarcoma phyllodes
3. Digital tomosynthesis is modification of: B. Lymphoma
A. MRI Breast B. Mammography C. Fibroadenoma
C. Sonomammography D. CT Mammography D. Carcinoma breast
4. Linguine sign is seen in: 9. Lactating woman with painful breast, 1st investigation to be

Multiple Choice Questions


A. Infected fibroadenoma done should be: (AIIMS Nov 2013)
B. Ruptured breast implant A. USG
C. Necrotic breast carcinoma B. Mammography
D. Primary breast lymphoma C. CT
5. The anode material used in mammography tube is: D. MRI
A. Tungsten B. Copper 10. Mammography uses:  (JIPMER May 2016)
C. Molybdenum D. Aluminium A. Continuous spectrum radiation/Bremsstrahlung
6. Broken needle pattern of calcification on mammography is B. Characteristic radiation
seen in: C. Electrons
A. Duct ectasia D. Gamma rays
B. Fibroadenoma 11. Mean optical density for an acceptable mammographic
C. Carcinoma breast image is:
D. Breast abscess A. 0.5–1.0 B. 1.4–1.8
7. The most sensitive investigation for DCIS (Ductal carcinoma C. 2.0–2.3 D. 2.4–2.8
in situ):  (AI 2009) 12. Peak kilovoltage used in mammography is in the range of:
A. Mammography B. USG A. 26–30 B. 35–40
C. MRI D. PET scan C. 40–45 D. 45–50
261
13. The most commonly used target-filter combination in 27. Lemon sign is seen in:
mammography is: A. Spinal dysraphism B. Hydrocephalus
A. Molybdenum–molybdenum C. Holoprosencephaly D. Lissencephaly
CONCEPTUAL REVIEW OF RADIOLOGY

B. Molybdenum–rhodium 28. Double bubble sign can be seen in:


C. Molybdenum–palladium A. Annular pancreas B. Duodenal atresia
D. Rhodium–rhodium C. Midgut volvulus D. All of the above
14. For normal mammography, X-ray tube with a nominal focal 29. The earliest sign of uteroplacental insufficiency on Doppler
size of---mm is used: is:
A. 0.2–0.25 B. 0.3–0.35 A. Diastolic notch B. Reversal of diastolic flow
C. 0.4–0.45 D. 0.45–0.50 C. Blunting of systolic flow D. Absent mid diastolic flow
15. For mammography, the focus to film distance is -----cm. 30. Brain sparing effect on Doppler is seen in which vessel:
A. 30–35 B. 60–65 A. Umbilical artery B. Fetal MCA
C. 80–85 D. 90–95 C. Fetal ductus venosus D. Uterine artery
16. Coarse popcorn calcification in breast is seen in: 31. Nuchal translucency should be measured between:
A. Papilloma B. Phylloides tumor A. 9–11 weeks B. 11–13 weeks
C. Fibroadenoma D. Invasive ductal carcinoma C. 13–15 weeks D. 15–17 weeks
17. Spiculated mass is the commonest mammographic 32. Best trimester for performing USG for dating of pregnancy is:
appearance of: A. First B. Second
A. Fibroadenoma C. Third D. Post mature
B. Breast abscess
33. Lambda sign on ultrasound is seen in:
C. Invasive breast carcinoma
A. Ectopic pregnancy B. Twin pregnancy
D. Noninvasive breast carcinoma
C. Hydatidiform mole D. Fibroid
18. Mammographic features which help to distinguish between
34. Snowstorm appearance on ultrasound is seen in:
malignancy from a benign lesion include all except:
A. Ectopic pregnancy B. Hydatidiform mole
A. Architectural distortion B. Density of mass
C. Fibroid D. Abortion
C. Outline of mass D. Size of mass
19. Egg shell calcification is seen in which lesion: 35. Which of the following is false regarding the MRI in the
A. Arterial calcification B. Inside the cyst obstetric practice?
C. Fat necrosis D. Fibroadenoma A. It should be avoided in the first trimester of the pregnancy
B. Fetal anatomy is well displayed even in the presence of
20. ‘Breast within breast’ appearance is seen in”:
oligohydramnios
A. Lipoma B. Fibroadenoma
C. Spinal and intracranial anomalies are clearly defined
C. Hamartoma D. Intramammary lymph node
D. Placental abnormalities are not picked up
21. A 35-year-old woman presents with painless lump in the
outer upper quadrant of her left breast. She is referred for 36. Lemon sign is seen in which of the following:
an ultrasound examination of the left breast. Which one of A. Anencephaly
the following ultrasound findings would suggest a malignant B. Meningomyelocele
rather than a benign breast mass: C. Hydrocephalus
A. A larger transverse than anterior-to-posterior diameter D. Dandy Walker malformation
B. III-defined echogenic halo around the lesion 37. Normal looking gestational sac has all of the following
C. Less than 1 cm in greatest diameter features except:
D. Posterior acoustic enhancement A. Good decidual reaction is seen as double decidual sac sign
22. Microcalcifications in mammography measure less than: B. Regular-shaped gestational sac
C. Mean sac diameter >20 mm with yolk sac
Multiple Choice Questions

A. 0.5 mm B. 1 mm
C. 1.5 mm D. 2.0 mm D. Low lying gestational sac
38. Nuchal translucency is assessed in which of the following
Obstetric Imaging trimester:
23. Investigation of choice for placenta accreta is: A. Early first B. Late first
A. Ultrasound B. Doppler C. Second D. Third
C. MRI D. CT scan 39. Best parameter for assessment of IUGR is:
24. Spinning top appearance is seen in: A. FL B. Abdominal circumference
A. PUV B. PUJ obstruction C. CRL D. HC
C. Ectopic kidney D. Retrocaval ureter 40. Components of biophysical profile include:
25. Earliest congenital anomaly detected by ultrasound is: A. Fetal tone
A. Posterior urethral valves B. Fetal body movement
B. Anencephaly C. Fetal breathing movement
C. Ventricular septal defect D. All of the above
D. Hydrocephalus 41. Best parameter for predicting the gestational age in first
26. 1st structure seen within an intrauterine gestational sac is: trimester of pregnancy is:
A. Fetal pole B. Yolk sac A. Crown-rump length B. Femur length
C. Cardiac activity D. Double bleb C. Biparietal diameter D. Abdominal circumference
262
Gynecologic Imaging 48. According to recent criteria (2013) what is the Follicle
42. Banana shaped uterus is seen in: Number Per Ovary (FNPO) required for diagnosis of PCOD:
A. Uterus didelphys B. Unicornuate uterus A. >12 B. >15

WOMEN’S IMAGING
C. Septate uterus D. Bicornuate uterus C. >25 D. >26
43. Investigation of choice for evaluation of Müllerian duct 49. Most common primary site for Krukenberg tumor is:
anomaly is: A. Ovaries B. Stomach
A. HSG B. CE-CT C. Fallopian tubes D. Peritoneum
C. MRI D. USG 50. Pseudomyxoma peritonei is seen in:
44. Imaging modality of choice in adenomyosis is: A. Serous cystadenoma B. Dermoid cyst
A. USG B. CE-CT C. Sertoli cell tumor D. Mucinous cystadenoma
C. MRI D. Hysteroscopy
51. Retort-shaped structure on HSG is seen in:
45. Which complication of fibroid can be identified on imaging?
A. Hydrosalpinx B. Endometrial polyp
A. Red degeneration B. Hemorrhage
C. Cervical stenosis D. All of above
C. Torsion D. All of the above
52. Müllerian Duct Anomaly associated with the highest
46. Rotterdam criteria (2003) recommends the Follicle Number
Per Ovary (FNPO) for PCOD to be: incidence of pregnancy loss is:
A. >10 B. >12 A. Unicornuate uterus B. Bicornuate uterus
C. >14 D. >16 C. Septate uterus D. Arcuate uterus
47. String of Pearls appearance is seen in: 53. Meigs syndrome consists of:
A. PCOD B. Endometriosis A. Ovarian fibroma B. Ascites
C. Adenomyosis D. Fibroid C. Pleural effusion D. All of the above

Answers
1. A 8. D 15. B 22. B 29. A 36. B 43. C 50. D
2. B 9. A 16. C 23. C 30. B 37. D 44. C 51. A
3. B 10. B 17. C 24. A 31. B 38. B 45. D 52. C
4. B 11. B 18. D 25. B 32. A 39. B 46. B 53. D
5. C 12. A 19. C 26. B 33. B 40. D 47. A
6. A 13. A 20. C 27. A 34. B 41. A 48. D
7. C 14. B 21. B 28. D 35. D 42. B 49. B

Multiple Choice Questions

263
Explanations to Questions
CONCEPTUAL REVIEW OF RADIOLOGY

3. It is an advanced form of mammography, that uses a low- 24. Spinning top urethra is non-obstructive posterior urethral
dose X-ray system and computer reconstructions to create dilatation seen on MCU, mainly in females. It was initially
three-dimensional images of the breasts. considered as an indicator of distal urethral narrowing/
4. Linguine sign refers to the spaghetti-like appearance of stenosis. However, it is now believed to be due to functional
ruptured silicon breast implant seen on mammography. discoordinate voiding or bladder instability.
11. For the oblique mammograms, the mean OD in the main 29. The signs of uteroplacental insufficiency in increasing order
breast ranges between films from 1.25 to 2.24 with a mean of of severity are early/prediastolic notch, absent diastolic flow
1.69 +/– 0.02. In the craniocaudal mammograms, the mean and lastly reversal of flow during diastole.
OD in the main breast ROI ranges from 1.14 to 1.94 with a 30. Brain sparing effect is loss of normal high resistance flow
mean of 1.61 +/– 0.05. in the MCA secondary to shunting of blood to important
12. Peak kV used in mammography is 20–30 kV. The high voltage structures in fetal hypoxia.
generator converts the low voltage of 200–300 kV to 20–30 kV. 33. Twin peak sign (also known as the lambda (λ) sign) is a
13. Molybdenum filter produces low energy photons of 17.9 triangular appearance of the chorion insinuating between
and 19.5 which provide high contrast for breasts for average the layers of the intertwin membrane and strongly suggests a
thickness. For dense breasts rhodium is used. Filter used is dichorionic twin pregnancy.
also molybdenum which suppresses radiation above 20kv. 35. MRI though safe in pregnancy has theoretical chances of
14. In general smaller the focal spot sharper the image. Routinely inducing irreversible hearing loss in the developing fetal
focal spot size is 0.3 × 0.3 mm while for magnification auditory system, so fetal MRI is best avoided in first trimester.
technique small focus of 0.1 × 0.1 mm is used. For placental invasion abnormalities it is investigation of
15. FFD is one variable which controls image sharpness. Commer- choice.
cially available units have distances between 60 – 70 cm. 38. Read question carefully nuchal translucency is evaluated
22. The term microcalcification refers to calcifications of which in late first trimester while nuchal thickness is assessed in
diameter is inferior to 1 mm, knowing that current spatial second trimester.
resolution mammographs make small objects to be detected 52. Out of the given options septate uterus has highest risk of
without magnification for a size ranged between 100 and pregnancy loss because it hampers the implantation of the
200 μm. zygote.
Calcifications with a higher probability of malignancy are fine
pleomorphic, fine linear and fine linear branching.
23. Though USG is modality of choice to visualize position of
placenta, in suspected placental invasion abnormalities like
placenta increta/accreta MRI is imaging modality of choice.
Explanations to Questions

264
RADIONUCLIDE IMAGING
AND RADIOTHERAPY
C hapter O utline
• Radionuclide Imaging
• Radiotherapy
CONCEPTUAL REVIEW OF RADIOLOGY

• Multiple Choice Questions

Radionuclide Imaging
(Dear friends, we are going to deal with this topic in utmost ƒƒ Structural/Anatomic Imaging modalities:
detail here. The reason being that apart from being a very •• X-rays, USG, CT, MRI
important topic for your exams this topic is not covered well •• Provide information regarding the structure of an
in the available resources till date. Also as these topics are organ/lesion.
already so vast and full of clinical details, we have deliberately •• For example if a tumor is seen—these modalities will
not put up the Clinical Quiz type questions at the start of these tell us about the location/size/shape/structure–solid,
chapters. We have made these topics as comprehensive and cystic, etc.
clinical oriented as possible in view of recent NEET pattern ƒƒ Metabolic/Functional Imaging modalities:
MCQs. So let’s Rock Radionuclide Imaging & Radiotherapy •• Planar Scintigraphy/SPECT/PET
too !) •• Provide information regarding function/metabolism
Broadly speaking the imaging modalities we come across in •• For example–Cardiac viability, Renal perfusion, GFR
hospitals may be classified as: etc.

Structural/Anatomic Modalities versus Functional/Metabolic Modalities


CO
NC E P T
So what type of modalities are considered better? Renal imaging:
The answer is NONE!! • USG will give an idea about the renal appearance–suggesting
renal parenchymal disease/obstruction
Well, shocking as it may sound, it is impossible to choose one over
Tc DTPA scan will help us quantify the residual renal function
99m
the other. and hence it decides the management strategy
 We use the structural modalities most commonly in our Cardiac Imaging:
hospitals–X-rays/USG/CT/MRI–as they are readily available, • 2D Echocardiography (USG) will show us a wall motion
aesthetically easy to interpret and they work for us in most of our abnormality following a myocardial infarct
clinical problems. But that does not mean they are superior! • FDG-PET will tell us whether the affected myocardium is still
 The concept to understand here is that because these viable and thus influence Angioplasty/CABG decisions
2 types of modalities provide us with essentially different set Thus the key concept is –
Radionuclide Imaging

of information, they are not to be replaced by each other but Structural and Functional modalities are COMPLEMENTARY to
are in fact complementary to each other. Consider the following each other. Whenever one modality fails, the other modality
examples: helps us definitively!!

266
CONCEPT BASIS FOR RADIONUCLIDE Positron Emission Tomography (PET)
IMAGING ƒƒ FDG:
•• Fluoro-Deoxy-GlucoseQ or chemically 2 (18-F)

RADIONUCLIDE IMAGING AND RADIOTHERAPY


Fluoro-2-Deoxy-D-glucoseQ
•• Simply put, it is a Radioactive analogue of glucose, with
a half-life of 109 minutesQ
•• FDG is synthesized in a CyclotronQ by proton
bombardmentQ of 18O–Enriched waterQ causing a
Knockout reaction.Q

FDG–The Concept
CO
NC E P T
Did you read the Pathology–Neoplasia chapter from Robbins?
Well, if you did not, you missed it! However hard and boring the
chapter may seem at first it is indeed a treat of concepts regarding
the development of Cancer. It beautifully highlights each and
every step of how a perfectly normal cell undergoes its malignant
transformation. It is extremely important for your entrance exams
as well! If you have not read it till now, make a plan, and start today!
In humans, cell replication is an Active process requiring a lot of
energy. A neoplastic cell gets this energy by selectively upregulating
the production/overexpression of all the components required for
energy/ATP generation. For example:
IMPORTANT RADIONUCLIDE SCAN • GLUT transporters–on the cell surface are over-expressed so
RELATED TERMS that a neoplastic cell has a very large number of GLUT (mainly
type 2 and 4) on its surface.
Gamma CameraQ • Hexokinase upregulation–ensures rapid 6–phosphorylation of
glucose
ƒƒ It is a system of one or more detectors linked to a computer
• Warburg Effect - The Warburg Effect, described by Otto
system.
Warburg in the early 20th century describes how the cancer
ƒƒ Collimators guide the gamma rays from the patient’s body cells use aerobic glycolysis as a source of their energy, rather
onto the detectors (Sodium iodide crystalsQ), which in turn than oxidative phosphorylation which is the more efficient
emit light in proportion to the radiation received. process of cellular respiration.
ƒƒ This light is measured by photomultiplier tubesQ and is Thus if a lot of glucose is available in the blood stream, most of it
registered by the computer. will be selectively taken up by the tumor cell!

Dual Isotope ImagingQ We trick the body by administering FDG–which is very similar to
ƒƒ 2 isotopes are administered simultaneously and are also glucose.
imaged together using gamma cameras sensitive to different ↓
energy levels. So in the hurry-and-flurry of taking up all available glucose, all FDG
ƒƒ Used for lung ventilation–perfusion scan where signals is also taken by the neoplastic cell and is also 6-phophorylated by
from 81m-Kr gas for ventilation and 99mTc macroaggregated Hexokinase producing 6P-FDG.

Radionuclide Imaging
albumin–for perfusion are obtained simultaneously.
It is at this point that the cell realizes–Oh my god! I have been
Planar ScintigraphyQ tricked!!
But by now it has already phosphorylated the FDG. So the FDG is
ƒƒ Signals emerging from patient’s body are measured by using
now trapped inside the cell and cannot be thrown out.
gamma cameras in ONE planeQ only.

ƒƒ So it is similar to a X-ray spot radiograph, and has limited
Thus FDG selectively localizes to neoplastic cells and helps us in
sensitivity and resolution. detecting them.
What happens further is even more fun!!
Single Photon Emission Computed
TomographyQ (SPECT) ƒƒ Annihilation ReactionQ:
ƒƒ Gamma camera/detectors placed in a ring around the •• The trapped FDG now emits a Positron–the twin
patient’s body, so that a tomographic image acquisition is brother of electron with a positive charge
done like in CT •• This positron collides with an adjacent tissue electron
ƒƒ Better sensitivity and resolution than planar scintigraphy. and undergoes annihilation

267
This is Planar
scintigraphy – Thyroid
scan image – See
CONCEPTUAL REVIEW OF RADIOLOGY

the patchy, blot like


uptake seen in both
lobes of thyroid
with poor spatial
resolution

  Clinical Pearls
False Positive and False Negative on Pet Scan
PET-CT Image False Positive for Malignancy on PETQ:
demonstration • Because PET is based on FDG uptake of the cell based on its
a Primary lung metabolism, conditions like infection, active granulomatous
carcinoma – The processes can show false positive finding on PET. Such
lower set of images conditions include:
are HRCT images, ƒƒ SarcoidosisQ
while the upper row ƒƒ TuberculomaQ and TB LymphadenopathyQ
of images are PET-CT ƒƒ CryptococcosisQ
– Functional color ƒƒ Paragonimiasis
map superimposed ƒƒ Pneumocystis infectionQ
on HRCT Images ƒƒ Abscesses
– Fusion imaging ƒƒ Radiation fibrosisQ–due to secondary inflammation
technique ƒƒ Pneumoconiosis with Massive fibrosisQ
ƒƒ Sclerosing hemangioma of lung
ƒƒ Granulation tissue around tumors
• False Negative for Malignancy on PETQ–(AIIMS NOV 2017)
ƒƒ Tumors with low activity and <1 cm sized tumor may register
low uptake - thus constitute a False negative. These include:
 Bronchoalveolar carcinomaQ
 Small sized lesions <1 cm sizeQ
 Lung metastasis from a Mucinous extrapulmonary tumorQ
 Tumor response after chemotherapyQ
 Carcinoid tumorQ: Typical carcinoid (low grade
malignancy)Q is more likely to register as false negative
than Atypical carcinoid (intermediate grade malignancy)
 Hyperglycemic stateQ- due to competition of FDG with
excess blood glucose for uptake into the tumor.

•• Annihilation–means total destruction of matter


Radionuclide Imaging

•• So both the positron and electron are destroyed, and CO


converted into energy in the form of 2 Gamma energy NC E P T
photonsQ of 511 KeVQ each. The detectors in a PET machine detect?
•• These Gamma photons are emitted at a direction 1800 A. Positrons      B. Electrons
oppositeQ to each other C. Protons        D. Gamma photons
ƒƒ Coincidence ImagingQ: PET–is POSITRON Emission Tomography.
So most of your competitors will read the question, remember
•• Because the above 2 Gamma photons have started
the name and mark the answer as A. Positrons and move ahead.
from the same point, they will reach 2 detectors located
But that is WRONG.
180° apart on the detector ring at the same time. Such
If you too got it wrong, read Annihilation reaction again.
signals co-inciding in time and signal configuration
Positron Emission–happens. But immediately after getting emitted
are taken as true signals and are amplified. The other the Positron is annihilated along with an electron and 2 Gamma
signals are suppressed. energy photons (511keV each) are emitted 1800 apposite to each
other. It is these Gamma photons that are ultimately detected by
the detectors in the machine.
268 Thus the correct answer is D. Gamma photons!
Hybrid ImagingQ/Fusion ImagingQ/
Technetium–is the Most Common
99m
Functional-Anatomical MappingQ Radionuclide used in Diagnostic
ƒƒ Combination of Anatomical and Functional imaging CO T Imaging–Why?

RADIONUCLIDE IMAGING AND RADIOTHERAPY


NC E P
techniques like 99m
Tc can be readily bound to other atoms/molecules/cells/
•• PET-CTQ particles. The combination of Radioactive technetium (emitting
•• SPECT-CTQ gamma rays) and a Neutral atom/Particle–results in a radioactive
ƒƒ Since they give both anatomical and functional information, entity that can be used in specific organ imaging. For example:
these modalities are much superior than each individual • 99mTc labeled Iodine - can now be used for Thyroid imaging
technique • 99mTc labeled RBC’s - can be used to identify any site of bleeding
in the body
• 99mTc labeled WBC’s–Can be used to detect any focus of
  Clinical Pearls inflammation/Infection in the body
Standard-Uptake-Value (SUV)Q • 99mTc labeled Nanocolloid particles–Can be used for Nanocolloid
• Important term in Quantitative radionuclide imaging Lymphoscintigraphy for lymphatic abnormalities.
• SUVQ = Activity per unit mass of tissue/Injected Activity per This versatility of Technetium to be bound easily to various
unit body mass atoms/molecules/cells/particles, along with its short half-life
• The cut off between benign and malignant lesion/nodule is in (6 hours), predominant gamma ray emission–makes it the MOST
the SUV range of 2.0-2.5Q. COMMON to be used.
• Many infectious and inflammatory processes will also have
high SUV
Renal and Adrenal Radionuclide Imaging
99m
� Tc DMSA ScanQ:
•• DMSA–Dimercaptosuccinic acidQ
SYSTEMIC RADIONUCLIDE IMAGING •• Agent of choice for Cortical static scintigraphyQ–
Used for evaluation of renal Structure and AnatomyQ
Conceptual Approach (Remember as DMSA–S is Structure, A is anatomy)
We will discuss, wherever possible, and very briefly, WHY–a •• This is because it is concentrated in the renal
particular radio-isotope is used for that organ/function. Once parenchyma for a long time, thus enabling a detailed
you understand it–remembering it would never be a problem, anatomical mapping
ever! •• Investigation of choice for–Detection of Renal scars in a
Let us Start patient with vesicoureteric refluxQ
•• Also used for EctopicQ/Pelvic kidneyQ/HypoplasticQ/
Technetium 99mQ: Most common Radionuclide used in
Aplastic kidneyQ evaluation.
Diagnostic Imaging–so we must know everything about it!
99m
ƒƒ Atomic number - 43Q � Tc DTPA ScanQ:
ƒƒ Name means - is Artificial/man-made •• DTPA–Di-ethylene Tri-amine Penta-acetic AcidQ
ƒƒ Is synthesized from Molybdenum-99Q in a Cyclotron •• Was the Agent of choice for Dynamic renal scinti­
ƒƒ mQ–in its name stands for–Metastable nuclear isomerQ graphy–Not used for this purpose now
ƒƒ Half-life–is 6 hoursQ •• Excreted primarily by glomerular filtrationQ. Hence it
ƒƒ Emits: is Agent of choice for GFR estimationQ (Inulin–is Gold
•• Gamma raysQ–140.5 keV energy–Most commonly standard and Most accurate–is not radioactive)
emitted � 99m
Tc MAG3 ScanQ:
•• Beta rays/electrons–Small amount emitted •• MAG3–Mercapto-acetyl-triglycineQ
ƒƒ With time 99mTc decays as:

Radionuclide Imaging
99m
•• Most commonly used renal radiopharmaceuticalQ
Tc
•• Excreted through tubular transport

99–Tc
•• Agent of choice now for Dynamic renal scintigraphyQ–
for Renal clearanceQ and Effective renal plasma flowQ

(ERPF)
99–RutheniumQ

269
Other Important Radiopharmaceuticals 99m
Tc-DMSA
Scan showing a
Diuretic • Used for assessment of Intermittent Ectopic/pelvic
CONCEPTUAL REVIEW OF RADIOLOGY

RenographyQ obstructive hydronephrosisQ or Pelvi- Right kidney (red


(Using DTPA/ ureteric junction obstructionQ arrows)
MAG3)
Captopril • In patients with significant renal artery
RenographyQ/ stenosis. Discussed in detail in Renal
Captopril– Artery Stenosis in GUT Imaging
DTPA/MAG3
scanQ
Radionuclide • Used for detection of
cystography ƒƒ Vesicoureteric refluxQ (Micturating
radionuclide cystography)
ƒƒ UB rupture
ƒƒ UB diverticuli
I123–MIBG • Metaiodobenzylguanidine
ScanQ • Uptake is seen in Noradrenaline
transporters 99m
Tc-DMSA
• Hence used for detection of Adrenal Scan showing
medullary tumorsQ a small scarred
Left kidney (red
I123–iodo- • Intermediate of steroid hormone
arrows)
cholesterolQ synthesis
• Hence used for detection of Adrenal
cortical tumorsQ
•• As RBCs will fill-up the ventricle, the resultant
radioactivity data is used for functional/quantitative
Cardiac Radionuclide Imaging assessment–Ejection fractionQ (LVEF), Peak filling
rateQ, Time to peak fillingQ, Stroke volumeQ, etc.
ƒƒ Myocardial Perfusion Imaging (MPI)Q: Perfusion means •• ECG gatingQ used to reduce blurring
capillary blood flowQ •• Used in MI, Cardiomyopathy, Chemotherapy toxicity,
•• Thallium-201Q (TI-201)–most commonly used etc. May be used as a Quantitative shunt study in
•• Used because–it behaves like potassium (K), thus is Congenital heart disease
concentrated in the myocytes by the Na-K exchange •• MUGA ScanQ:
mechanism. Also undergoes continuous exchange  Stands for Multiple-Gated Acquisition scanQ
between myocardial and vascular blood pool–hence  Most accurate method for assessment of LV
can correlate with actual blood flow/perfusion. functionQ
•• Impaired perfusion–means a functionally significant � 99mTc–RBCsQ/99mTc–AlbuminQ–is used
coronary artery stenosis. ƒƒ Cardiac–PETQ:
•• Normal myocardium will show normal perfusion. An •• Gold standard for non-invasive assessment of
infarct will not have normal perfusion–hence it will Myocardial viabilityQ
appear as a Cold spot on perfusion imaging.Q •• Radiopharmaceuticals used:
•• Alternatives to Thallium –  18-FDGQ–Most common
Radionuclide Imaging

� 99mTc–Sestamibi/MIBIQ  Rubidium 82
� 99mTc–TetrofosminQ  N-13 Ammonia
•• Stress–MPIQ–Heart subjected to Physical/ •• Used because:
Pharmacologic stress (Dobutamine administration) to  Normal cardiac metabolism–Free fatty acid
detect functionally significant stenosis uptakeQ
•• MPI–has excellent Negative predictive valueQ for  Ischemic myocardium – Increased glucose
predicting low mortality and MI in those with normal uptakeQ (Hence FDG is useful)
results •• Also the Best investigation for assessment of
•• PET scan using Rubidium 82Q, N-13 AmmoniaQ can HibernatingQ/Stunned myocardiumQ and Myocardial
also be used for MPI scarringQ.
ƒƒ Radionuclide VentriculographyQ: QuantitativeQ cardiac ƒƒ Infarct scintigraphy/Avid Infarct imaging–99mTc–Stannous
evaluation pyrophosphate imagingQ–Infarct appears as a Hot spotQ.
• 99mTc pertechnetate labeled RBCsQ (patient’s own
RBCs)–Most commonly used

270
Pulmonary Radionuclide Imaging ƒƒ Gallium 67 ScanQ:
•• Half-life is 78 hoursQ
ƒƒ Pulmonary Ventilation–Perfusion scan:
•• Binds to inflammatory proteinsQ (like transferrinQ),
•• For ventilation radiolabeled gas containing either
hence gets localized to sites of infection/inflammation

RADIONUCLIDE IMAGING AND RADIOTHERAPY


Technetium-99m DTPA (TechnegasQ) or radiolabeled
•• Not specific for infection–most important disadvantage
Xenon are given to patient through a rebreathing mask
–positive also in conditions like Sarcoidosis, Lymphoma.
and for perfusion Technetium-99m Macro aggregated
albumin (MAA)Q is injected intravenously.
  Clinical Pearls
•• Multiple views of images are obtained using a gamma
camera. Positive Gallium scan in SarcoidosisQ
• Panda signQ: Due to bilateral involvement of parotid and
lacrimal glands in sarcoidosis, superimposed on the normal
  Clinical Pearls
uptake in the nasopharyngeal mucosa.
Pulmonary Ventilation–Perfusion scan - Specifics • Lambda sign of thoracic SarcoidosisQ: Bilateral hilar and right
• Matched defectsQ on V/Q scan: paratracheal lymph nodes are typically involved which can
ƒƒ Absent Ventilation–Absent perfusion–in the same segment resemble the Greek letter lambda (λ)
of lung
ƒƒ Seen in airway obstruction, parenchymal lung disease ƒƒ Radiolabeled Leukocyte scintigraphy:
• Mismatched defectsQ on V/Q scan: •• Patient’s own WBCs are labeled with 99mTcQ or 111–
ƒƒ Present ventilation–Absent perfusion–in the same segment IndiumQ
of lung •• Thus the leukocytes become radioactive and their
ƒƒ Typical of Pulmonary ThromboembolismQ localization to sites of infection/inflammation can be
ƒƒ In early cases segmental/lobar defects are seen, in later detected.
cases more peripheral defects become apparent. •• More accurateQ and specificQ than Gallium scans.
• Reversed mismatched defectsQ:
ƒƒ Absent ventilation–Present (or only mildly reduced)
Gastrointestinal Radionuclide Imaging
perfusion–in the same segment of lung
ƒƒ Seen in infective consolidation/atelectasis ƒƒ Evaluation of GI BleedingQ (AIIMS MAY 2013): Very important
• Normally the ventilation perfusion ratio (V/Q) is 1Q but in PTE topic. Let us quickly review the imaging of GI bleeding in
due to loss of perfusion with intact ventilation a ratio greater general before discussing the specific radionuclides for this
than 1 is obtainedQ. indication.
• Triple matchQ: AngiographyQ: Has low sensitivity
ƒƒ V – Q matched defect that matches exactly with an •• Threshold rate for detection of GI bleeding on a Flush
opacification on a CXR, s/o pulmonary infarct resulting from aortogram - >5 mL/minQ
PTE •• Threshold rate for detection of GI bleeding on a
selective arteriography - >0.5 mL/minQ
ƒƒ 18-FDG PET-CT for Solitary pulmonary NoduleQ (SPN):
•• Because the contrast remains in the blood stream only
•• Has sensitivity, specificity and accuracy of 90% or for a short time, it can detect only Active GI bleedQ–that
greater in the diagnosis of benign nodules
is if bleeding is occurring at the time of injection of
•• False positive results occur in infections/granulomatous contrast
disordersQ
Radionuclide studiesQ: Very high sensitivityQ - Excellent
•• False negative results may occur in smallQ (<10 threshold of 0.05–0.1 mL/minQ
mm)/slow growing lesions like bronchioloalveolar
• 99mTc–Sulfur colloid scanQ –
carcinomaQ.
 Can image only active GI bleedingQ
ƒƒ 18–FDG PET-CT for Lung Cancer:
• 99mTc–RBCsQ–
•• Useful for Primary stagingQ - Very effective for detection  Can detect active as well as Intermittent GI

Radionuclide Imaging
of distant metastasis–except in Brain as the normal
bleedingQ
cerebral cortex also has high FDG uptake
• 99m
Tc–pertechnetateQ scanning–
•• Also useful for Re-stagingQ  Used to detect bleeding occurring from a Meckel’s
•• Assessing response to therapyQ– diverticulumQ. Meckel’s bleeds because it has
 Differentiation of postsurgery changes from
ectopic gastric mucosa – secretes acid – damages
recurrence
the mucosa.
 Differentiation of post chemo/radiotherapy
 Most common presentation of Meckel’s: GI
changes from recurrence
bleedingQ with abdominal pain
 Most sensitive for diagnosisQ
Infection/Inflammation Radionuclide ƒƒ HIDA scanQ/IODIDA ScanQ/DISIDA ScanQ/
Imaging CholescintigraphyQ for Hepatobiliary abnormalities:
ƒƒ This application of radionuclide imaging may be used in •• Uses Iminodiacetic acid (IDA)Q derivatives–HIDA/
cases of Pyrexia of unknown origin/or may be applied to any IODIDA/DISIDA
infection/inflammation in any system •• HIDAQ–stands for Hepatic useQ of –IDA compounds
271
Reformatted PET-CT Other Important Radiopharmaceuticals
Done in a case of Lung
carcinoma – shows the Dysphagia • 99mTc- Sulfur colloid swallow studyQ–
CONCEPTUAL REVIEW OF RADIOLOGY

hypermetabolic lung done exactly similar to a routine Barium


mass and multiple bone swallow study. Also known as the Milk
metastasis scanQ
Diarrhea • 99mTc–Leukocyte scintigraphyQ–As
WBCs will selectively migrate towards
the focus if infection/Inflammation
Recurrent vomiting/ • Gastric emptying studies
Gastroparesis and � 99mTc- Sulfur colloid in EggQ or
Dumping syndrome ƒƒ 111- In- DTPA in orange juiceQ
Hepatic • 99mTc–RBCsQ–Being a mass com­pris­ing
hemangioma blood vessels, it will show progressing
pooling of RBCs
Abdominal sepsis/ • 99mTc–LeukocyteQ/111–In - Leukocyte
pyrexia of unknown scintigraphyQ–As WBCs will selectively
origin (PUO)/ migrate towards the focus if infection/
Inflammatory bowel Inflammation
disease
Ectopic splenic • 99mTc–Heat Damaged RBCsQ
tissue • Spleen has areas of Red and White
pulp. One of the functions of spleen is
to sequester the damaged/aged RBCs.
Hence if labeled Heat damaged RBCs
are injected they will be selectively
18-FDG PET- sequestered in the splenic tissue,
CT showing wherever it is in the body–thus
multiple detecting it
hypermetabolic
Liver metastatic
deposits
Musculoskeletal Radionuclide Imaging
ƒƒ Concept: Use of PolyphosphonateQ/BisphosphonateQ
compounds:
•• Read about drugs like Alendronate/Pamidronate
•• Concept–Once injected in the blood stream, these in Orthopedics? Yes, they are most commonly used
compounds are taken up selectively by the hepatocytes in management of Osteoporosis. These drugs bind
and then are secreted actively into bile. Thus within a to OsteoclastsQ and inhibit osteoclast mediated
short time of injection the entire biliary tree is full of bone resorptionQ. They may also have an action on
radioactive compounds and can be visualized. OsteoblastsQ. But how is this relevant here?
•• Calculous CholecystitisQ: •• It is, because herein lies the simplicity of this concept.
 Most common clinical indication for performing •• Now that we know how these drugs act, just imagine
CholescintigraphyQ that such a compound–example Methyl Di-
 Non-visualization of GB on HIDA scanQ - NEET
Radionuclide Imaging

phosphonate (MDP)Q–is labeled with 99mTc. Thus we


2017 pattern will have a radioactive bisphosphonate compound that
 This is because there is a calculus impacted at the if administered to the patient will bind selectively to the
GB neck/cystic duct, that prevents HIDA from bone cells. Thus almost the entire spectrum of Skeletal
entering the GB–hence it is not seen radionuclide imaging is performed using such labeled
 Rim signQ - Increased uptake of the 99mTc-HIDA Bisphosphonate compounds like 9m–Tc MDP.
agent in the liver adjacent to the gallbladder fossa, ƒƒ OsteomyelitisQ:
is a useful ancillary finding seen in patients with • 99mTc-MDP bone scanQ/In111-WBC ScanQ/Gallium
acute cholecystitis scanQ is used
•• Biliary atresiaQ, Biliary dyskinesiaQ, Postoperative ƒƒ Any skeletal pathology–FracturesQ/Stress fracturesQ/
biliary leakQ–can also be diagnosed. ArthritisQ/Avascular necrosisQ–99mTc–MDP may be used

272
ƒƒ Lymphedema: 99mTc–nanocolloid particlesQ used. These
Nanocolloid particles being very small escape from the 99m
Tc-MDP Bone
blood stream and drain via lymphatics–thus depicting scan showing a left
lymphatic abnormalities. Tibial diaphyseal

RADIONUCLIDE IMAGING AND RADIOTHERAPY


stress fracture – seen
  Clinical Pearls as a focal area of
increased uptake in a
99m
Tc – MDP SUPER-SCAN
symptomatic patient
Excessive skeletal radioisotope uptake in relation to soft tissues
along with absent or faint activity in the genitourinary tract on
bone scintigraphy is known as a ‘superscan’. Seen in
• Renal failure
• HPT
• Metabolic bone disease
• Paget’s disease
• Widespread metastatic disease
• Hyperthyroidism
• Fibrous dysplasia

CNS Radionuclide Imaging Neck Radionuclide Imaging


ƒƒ Stroke imaging - 99mTc–HMPAOQ: Hexamethyl propylene ƒƒ Thyrotoxicosis: 123I-Na-Iodide scanQ
amine oxime SPECT scanQ is done. Lipophilic compound ƒƒ Ectopic thyroid tissue/Thyroid nodule: Thyroid
that crosses the normal blood-brain barrier. Brain uptake is scintigraphy done using 99mTc–pertechnetateQ
proportional to the cerebral blood flow ƒƒ Parathyroid localization: Radionuclide studies are
ƒƒ Hydrocephalus/CSF Rhinorrhea–Intrathecal administra- Investigation of choiceQ AIIMS MAY 2017 pattern
tion of 99mTc DTPAQ–done, follows CSF flow pathway thus •• In Olden days–99mTc–Thallium subtraction scanQ
detecting CSF leaks. •• Recently–99mTc–Sesta-MIBI Scan–Agent of choiceQ
ƒƒ Encephalitis/Dementia-99mTc–HMPAO SPECT scanQ ƒƒ Dry mouth/Connective tissue disorders like
ƒƒ Epilepsy: Scleroderma/Sjögren’s syndrome: Salivary gland
•• Ictal SPECTQ: Using 99mTc–HMPAOQ: SPECT scan scintigraphy using 99mTc - pertechnetateQ
•• Inter-Ictal PETQ: Using 18 - FDG PET scanQ

Other Important Radiopharmaceuticals

Cancer Imaging
Thyroid cancerQ • Whole body iodine scintigraphy using 123I–Sodium Iodide
Skeletal metastasisQ • 99mTc–Polyphosphonate (MDP)
Soft tissue sarcoma Q
• 18-FDG PET scan
• 201–Thallium chloride scan
Tumor Staging–Recurrence–Response to treatment • 18-FDG PET scan
Brain tumors • 18-FDG PET scan
• 201–Thallium chloride scan
InsulinomaQ • Somatostatin receptor scintigraphy
• 111In-Octreotide/Pentetreotide scintigraphy
Radionuclide Imaging
Carcinoid tumorQ • Somatostatin receptor scintigraphy
• 111In-Octreotide/Pentetreotide scintigraphy
Sentinel lymph node detection • 99mTc–Nanocolloid lymphoscintigraphy
NeuroblastomaQ • 123I- MIBG scintigraphy
Tumor hypoxiaQ • Hypoxia scintigraphy–18 F–Fluoromisonidazole
Colorectal cancer • CEA–scan: using 99mTc–Arcitumomab
DVT–peptide imaging • 99mTc–Apcitide
Reticuloendothelial imaging (Liver/Spleen) • 99mTc–Albumin colloid
Pancreatic scan • 75–Selenium methionine scan

273
MIBG ScanQ: ƒƒ Used for imaging of:
ƒƒ I-131 metaiodobenzylguanidineQ (MIBG) is used as •• Neuroblastoma
radionuclide–is an guanethidine analog •• Carcinoid tumor
ƒƒ It is useful for Neuroendocrine tumors. Guanethidine and •• Pheochromocytoma
CONCEPTUAL REVIEW OF RADIOLOGY

its analogs are stored in vesicles within the cells of the •• Paraganglioma
sympathomedullary system. Hence, MIBG will concentrate •• Medullary thyroid cancer
in catecholamine-producing adrenal medullary •• Ganglioneuroma
tumors—both intra-adrenal (PHEOs) and extra adrenal •• Ganglioneuroblastoma
(paragangliomas)—and chromaffin cell tumors

Radiotherapy
HISTORICALS IN RADIOTHERAPY
1896 H. Becquerel Discovery of natural radioactivity from Uranium.
1898 M. Skƚodowska-Curie, P. Curie Discovery of polonium and radium.
1903 W.H. Bragg Discovered the Bragg peak.
1905 R. Abbe Cure of cervical cancer with radium sources.
1906 J. Bergonie, L. Tribondeau Cell radiosensitivity law.
1913 G. Forssell, J. Heyman, E. Berven, M. Strandqvist, Stockholm system of brachytherapy dosimetry.
R. Sievert, R. Thoraeus
1919 C. Regaud, A. Lacassagne Brachytherapy for cervical cancer.
1930 R.F. Mottram Oxygen effect on radiosensitivity.
1930 E. Quimby, G. Failla Quimby system of dosimetry.
1933 H. Crabtree, W. Cramer Oxygen effect in radiotherapy.
1934 I. Joliot-Curie, F. Joliot-Curie Discovery of artificial radioactivity.
1948 G. Fletcher, M. Lederman, L.F. Lamerton Fletcher’s system of gynaecological brachytherapy.
1951 I. Smith, H.E. Johns Cobalt-60 teletherapy.
1956 H.S. Kaplan, E. Ginzton Medical linear accelerator (Stanford).
1994 NOMOS Peacock Intensity modulated radiation therapy (IMRT).
1999 J.R. Adler Robotic radiotherapy.
2001 A.L. Boyer Volumetric modulated arc therapy (V-MAT).
2004 J.F. Dempsey, B.W. Raaymakers, J.J. Lagendijk MRI-linac.

RADIOACTIVITY BASICS
Basic Definitions
Atomic number (Z)Q: • Number of electrons/protonsQ in an atom
Atomic mass number/ • Total number of nucleonsQ that is (Protons + Neutrons)Q
Radiotherapy

Atomic weight (A)Q:


IsotoPes Q • Atoms with same number of Protons, but different neutronsQ
• Thus they have same atomic number (so they are atoms of same element), but different atomic mass number
IsotoNesQ • Atoms with same number of Neutrons, but different protonsQ
• Atoms of different elements with different atomic numbers and atomic mass numbers
IsobarsQ • Atoms with different protons, different neutrons, but with same atomic mass numberQ
IsomersQ • Same atomic number, same atomic mass number, but different nuclear energiesQ
RadioactivityQ • The property possessed by some elements (such as Uranium) of spontaneously emitting energy in the form
of radiation as a result of the decay (or disintegration) of an unstable atom.
274 • Discovered by Henri BecquerelQ
UNITS OF RADIOACTIVITY Law of Radioactive Decay
ƒƒ Conventional Units: dN
– = lN
•• Disintegration-per-second (dps)Q: Number of radio- dt

RADIONUCLIDE IMAGING AND RADIOTHERAPY


active decays per second
• dN/dT = Velocity of disintegrationQ
•• CurieQ (Named after Pierre and Marie Curie) • N = Original number of radioactive atoms presentQ
 I Curie (Ci) = 3.7 × 1010 dpsQ = Number of decays • Λ = Disintegration constant Q–determines the Half-life (t1/2)
per second in 1 gm of radiumQ of the element
ƒƒ SI Unit: BecquerelQ (Named after Henri Becquerel) • Remember–Radioactive decay is ExponentialQ
•• 1 Becquerel (Bq) = 1 dpsQ
Half-Life of Radioactive Element (T1/2)Q
Types of Radioactive Decay
ƒƒ The time required for half of the atoms in any given quantity
Alpha decay q
• The emission of an α particle, or 4He of a radioactive isotope to decay is the half-life of that isotopeQ
nucleusQ ƒƒ Each particular isotope has its own half-life.
• Alpha particle has ƒƒ The half-life of 14C is 5730 years, thus it is useful for dating
• Maximum Ionizing powerQ/Damaging archaeological materialQ
powerQ
• Least Penetrating powerQ SOURCES OF RADIATION EXPOSURE
Beta decayq/ • Occurs if the instability of a nucleus is due
isobaric decayq to excess neutronsQ
OCCURRING TO HUMANS
• β decay occurs when a neutron is changed Natural sources—Estimated total natural radiation exposure
into a proton within the nucleusQ is around 0.1 Rad/year
• Beta particles are negatively charged ƒƒ Cosmic raysQ:
electrons emitted by the nucleus. •• Radiation coming from outer space
• The mass number is unchanged–hence ƒƒ Environmental exposureQ:
called Isobaric decayQ •• Terrestrial–from ground related sources mainly
Gamma • Gamma rays are a type of electromagnetic Uranium, Thorium and Radium. KeralaQ–has
decayq/ radiation that results from a redistribution maximum terrestrial exposure in India
isomeric of electric charge within a nucleusQ •• Atmospheric–from radioactive gases like Radon
decayq • A γ ray is a high energy photon ƒƒ Background radiation exposureQ: Is a term used to define
• Neither the mass number nor the atomic radiation exposure occurring constantly from natural sources
number is changed when a nucleus emits a
γ ray in the reaction–hence called Isomeric Man-made SourcesQ
decayQ
• Gamma ray: ƒƒ Radiology equipment–X-ray machines, CT scanners, PET/
ƒƒ Least ionizing powerQ/Damaging powerQ SPECT
ƒƒ Maximum penetrating powerQ
RADIATION EFFECTS ON HUMAN BODY
Ionizing Radiation Emitted by
ƒƒ DNA damage–“DNA double stranded breaks”Q–result in cell
Radioisotopes death following radiation exposure
Pure beta emitters • Phosphorous 32 ƒƒ Cell exposure in:
• Strontium •• G1 phase–Chromosome abnormalitiesQ occur
• Yttrium 90 •• G2 phase–Chromatid aberrationsQ occur
Gamma emitters • Iridium 192 ƒƒ Steps of Radiation induced damage:
• Cobalt 60
• Cesium 137
• Xenon 133
• Iodine 123
Radiotherapy

• Iodine 132
• Technetium 99
• Gallium 70
• Radon 222
• Selenium ƒƒ Radiosensitivity of cells:
Beta and Gamma emitters • Radium 226 •• Maximum at: G2M junctionQ
• Gold (Au) 198 •• Followed by G1–G2 phase
• Iodine 131 •• Least at: S phaseQ and late G2 phase–because at this
Neutron and Gamma emitters • Tantalum 182 stage the cell has large amounts of synthesis enzymes
• Californium 252 that have the ability to repair DNA quickly 275
RADIOTHERAPY Types of Radiotherapy
According to Aim of Treatment
Fractionated Radiotherapy
CONCEPTUAL REVIEW OF RADIOLOGY

ƒƒ Curative RadiotherapyQ: Cure is defined by using


ƒƒ The total dose required for treatment of tumors cannot be radiotherapy only. Used in Early stage Hodgkin’s lymphoma,
administered in a single fraction–as it would be extremely Nasopharyngeal cancer.
damaging to the normal body tissues. ƒƒ Palliative RadiotherapyQ: Advanced disease making cure
ƒƒ Hence a common strategy used is fractionation. impractical. Hence the aim of treatment is to relieve cancer
ƒƒ The total dose is divided into fractions, that are administered symptoms by using low doses of radiation. Used in cases
in a particular manner with Brain/bone metastasis/IVC encasement.
ƒƒ Conventional fractionationQ: ƒƒ Prophylactic RadiotherapyQ: Prevention of possible
•• 1 Fraction per dayQ from Monday-to-Friday metastasis/recurrence by using radiotherapy. Examples are
•• Weekend holidayQ (Sat and Sunday) Whole body radiation in ALL/Small cell lung carcinoma.
ƒƒ HyperfractionationQ/Accelerated fractionationQ:
According to Timing
•• >1 fraction administered per dayQ
•• Used for Cerebral gliomasQ, Small cell carcinoma ƒƒ Neoadjuvant RadiotherapyQ: Radiotherapy given before
lungQ, Head and neck cancersQ any kind of treatment. Example–Preoperative radiotherapy.
ƒƒ HypofractionationQ: Example is systemic therapy for breast cancer that is given
•• <5 fractionsQ administered per week before surgery. The most common objective for neoadjuvant
•• May be associated with higher local recurrence/ therapy is to reduce the size of the tumor so as to facilitate
treatment failure more effective surgery.
•• Used for MelanomasQ, soft tissue sarcomasQ and in ƒƒ Adjuvant RadiotherapyQ: Radiotherapy given after any kind of
Palliative radiotherapyQ radical treatment. Example–Postoperative radiotherapy. Used
for colon cancer, lung cancer, pancreatic cancer, breast cancer,
ƒƒ Advantages of Fractionation:
prostate cancer, and some gynecological cancers. Some forms
•• Allows normal cells to recover in the holiday period.
of neoplasms fail to benefit from adjuvant therapy - renal cell
The tumor cells are generally less efficient in repair.
carcinoma, and certain forms of brain cancer.
•• As the tumor size decreases, better oxygenation of
ƒƒ RadiochemotherapyQ/ChemoradiotherapyQ:
remaining tumor cells occurs thus rendering them
Radiotherapy given concurrently with chemotherapy.
more susceptible to next fractions
•• Few cells in the S phase (most radioresistant phase According to Delivery Mode of
of cell cycle), may migrate to M phase/G2 phase
(radiosensitive phases) at the time of next fraction– Radiotherapy
thus increasing treatment efficacy.
ƒƒ Disadvantages: External Beam RadiotherapyQ/TeletherapyQ
•• Overall treatment time increases ƒƒ The radiation source is placed at a distance from the
•• Tumoral proliferation increases in the Holiday periods patient’s body
ƒƒ Cobalt 60 machinesQ:
  Clinical Pearls •• Co-60–is the most common radioisotopeQ used in
conventional Teletherapy
5 Rs of Fractionated Radiotherapy
1. RepopulationQ:
•• It is an artificial isotopeQ. Naturally occurring form of
cobalt is Co-59Q–is a stable, non-radioactive atom
ƒƒ Cells (Normal as well as Cancer cells) may proliferate as a
physiological response to cell death following Radiotherapy •• Half-life of Co-60 is 5.27 yearsQ
ƒƒ If Radiotherapy is prolonged, tumors enter into Accelerated •• Handling Co-60 machines is technically difficult and
repopulation. Hence Repopulation is associated with requires frequent periodic replenishing of Co-60,
decreased response on treatment prolongationQ hence the use is now declining
2. Repair/RecoveryQ: Also associated with decreased response ƒƒ Linear Accelerators (LINACs)Q:
on treatment prolongationQ •• The linear accelerator uses microwave technology
3. Reassortment/RedistributionQ: Increased response during a (similar to that used for radar) to accelerate electrons
Radiotherapy

Fractionated radiotherapy regimenQ in a part of the accelerator called the “wave guide”Q
4. ReoxygenationQ: Increased response during a Fractionated •• Output from LINACs–High energy X-raysQ and
radiotherapy regimen electronsQ
5. RadiosensitivityQ: •• Easy to operate, No replenishment required, hence
ƒƒ Basis of Fractionated radiotherapyQ frequently used nowadays
ƒƒ Law of Bergonié and TribondeauQ: Radiosensitivity is ƒƒ Types of Teletherapy based on Beam intensity/quality:
directly proportional to MitosisQ and inversely proportional •• Kilovoltage TherapyQ:
to differentiationQ  Contact therapy (40–50 kV)Q–Very superficial lesions
ƒƒ SF2Q = Surviving cell fraction after a 2 Gy radiation doseQ. As  Superficial therapy (50–150 kV)Q
SF2 increases, radiosensitivity decreasesQ.  Orthovoltage/Deep therapy (150–500 kV)Q
276
•• Supervoltage therapy (500–1000 kV)Q–Deeper located •• Temporary implants:
lesions/tumors  Cesium 137
•• Megavoltage therapy (1 Megavolts or higher)Q  Cobalt 60
 Radium

RADIONUCLIDE IMAGING AND RADIOTHERAPY


BrachytherapyQ/Internal RadiotherapyQ/ ƒƒ Mould BrachytherapyQ:
Endocrine TherapyQ/Sealed Source •• Radioactive source is moulded in the form of a cap and
RadiotherapyQ placed over a superficially located tumor, like Penile
Radiation source is placed inside the patient‘s’ body. Types include: carcinoma
ƒƒ Interstitial BrachytherapyQ: The radiation source is •• May be:
injected within the lesion/tumor itself. Example–insertion  Low dose rate (LDR)
of radiation source within a retroperitoneal tumor. Used for  Medium dose rate (MDR)
Head and Neck cancers, Skin cancer, Prostate cancer, Anal  Pulsed dose rate (PDR)
canal and breast cancers. Contraindications are–Tumor  High dose rate (HDR)
infiltrating bone, Infection involving the implant volume,
inaccessible tumor site. The Sealed source/Implant may be   Clinical Pearls
temporary or permanent– Remote afterloading in Brachytherapy (NEET 2020 Pattern)
•• Permanent ImplantsQ: • These systems automatically administer a radioisotope directly
 Cesium 131 to cancerous tissue, thereby minimizing the radiation dose to
 Gold 198 surrounding tissue and eliminating the radiation exposure to
 Radon 222 hospital staff.
 Palladium–103 • In afterloading procedures, an unloaded applicator is inserted
 Yttrium manually into the tissue to be irradiated first. These are empty
 Iodine–125 applicators.
• Then the radioactive sources are introduced later in an
•• Temporary ImplantsQ:
automated manner into these empty applicators by using
 Cesium 137
special equipment.
 Cobalt 60 • Thus the radiation exposure to the department staff from
 Iridium 192 – most common temporary source of handling of radioactive sources is minimized.
Brachytherapy
 Radium 226
 Tantalum Systemic RadiotherapyQ
 Californium Intravenous administration of radio-isotopes done, which gets
ƒƒ Intracavitatory BrachytherapyQ: localized to particular organs based on organ affinity and deliver
•• Radiation source is placed within a cavity in the body. the radiation dose.
Example placement of a radiation source within the Most important applications are:
cervical canal for cervical cancer irradiation ƒƒ Iodine 131Q–used in Papillary thyroid cancer
•• Permanent implants: ƒƒ Phosphorus 32Q–Polycythemia rubra vera
 Gold 198 ƒƒ Samarium 153Q/Strontium 89Q–Painful bone metastasis
 Iodine 131

Other Special Types of Radiotherapy/Radiotherapy Applications


Intraoperative • ElectronsQ (Most common) and Low energy X-raysQ are used
Radiotherapy • Stationary Megavoltage electron beamQ–Most commonly used
(IORT)Q • Delivered to tumor bed just after resection of tumor
• Followed-up with Teletherapy
• Used for:
ƒƒ Pancreatic carcinomaQ
ƒƒ Gastric carcinoma
Radiotherapy

ƒƒ Colorectal/Head and Neck carcinomas


Stereotactic • Stereotactic means accurate localizationQ
RadiotherapyQ • Most important application is CNS tumorsQ
• After placement of a rigid head frame, typically by use of four pins that penetrate the scalp and impinge the
outer table of the skull, an 3D - imaging study (CT, MRI) is performed to localize the target volume relative to
the head frame coordinates.
• Several radiation beams directed from different directions are precisely focussed on the tumor target–thus
delivering the optimum dose
• Stereotactic Body radiotherapy (SBRT)Q–used for extracranial sites
Contd…
277
• Gamma knifeQ–Contains 201 Co60 sources of around 30 Ci, placed in a circular array with beams emerging from
201 ports–producing High energy Gamma rays
• If dose is administered in 1 day (Large dose)–is called as Stereotactic Radiosurgery (SRS)Q
• If dose is administered as a fractionated dose (Low dose on multiple days)–is called as Stereotactic RadiotherapyQ
CONCEPTUAL REVIEW OF RADIOLOGY

• Applications–Brain tumorsQ, AV malformationQ, Trigeminal neuralgiaQ


3D Conformal • Technique that conforms (matches) the beam exactly to the target tissue by using data from 3D CT/MRI, thus
Radiotherapy minimizing exposure to adjacent normal tissues
(#d-CRT)
Intensity • Highly advanced form of 3D-CRT, that provides a highly conformal dose distribution around a target using non-
Modulated uniform beam intensities.
Radiotherapy • The dose can be changed according to the target’s shape by modulating the intensities of each subsegment
(IMRT)Q • Used mainly for Prostate cancerQ, Head and neck cancer, CNS tumors
• Not suitable for treatment of lung cancer currentlyQ
Image guided • Integrates various radiological and functional imaging techniques for high precision radiotherapy.
radiotherapy • Takes into account target volume changes during radiotherapy, such as tumor decrease or weight loss (Adaptive
(IGRT)Q radiotherapy)
Tomotherapy Q
• Serial tomotherapyQ–Has a special collimator system called as MiMIC, mounted on a classic LINAC gantry. Can
administer IMRT in several arcs
• Helical tomotherapyQ–Dedicated tomotherapy machine. Consists of a 6-Mv LINAC mounted on a CT machine.
Delivers IMRT with spiral movements similar to CT procedure – in a slice-by-slice manner.
Cyberknife/ • Type of Sterotactic Radiotherapy/Radiosurgery
Robotic • Provides frameless treatment for both cranial and extracranial tumors
RadiosurgeryQ • Consists of a 6-MV LINAC machine mounted on a robotic arm and has ability to deliver all types of radiotherapy
techniques
Boron Neutron • A Boron compound selectively taken up by Brain tumor cells, is administered IV
Capture therapy • The tumor cells that have taken up the Boron compound are irradiated with slow neutrons
(BNCT)Q • Boron atoms reacting with these neutrons create Alpha radiationQ, causing direct DNA damage to the tumor
cells
Craniospinal • Therapeutic–Treatment already established spinal tumors– MedulloblastomaQ
IrradiationQ • Prophylactic–Used in MedulloblastomaQ, GlioblastomaQ, Germinoma
• Also used prophylactically in management of tumors that have a high risk of CNS metastasis–ALLQ, Small cell
lung carcinomaQ, Non-Hodgkin’s lymphomaQ
• Hypothyroidism, Growth hormone disturbances are common
Mantle • Treatment of Supradiaphragmatic Hodgkin’s diseaseQ
radiotherapyQ
Inverted Y • Treatment of Infradiaphragmatic Hodgkin’s diseaseQ
radiotherapyQ
V-MAT • Volumetric modulated arc therapy (V-MAT) is a technique that delivers a precisely sculpted 3-D dose
distribution with a single 360 degree rotation of the linear accelerator gantry

Specific Phenomena and Terms in Radiotherapy


TD5/5Q • Defines the minimum tolerance dose
• It is the dose that has a less than 5% complication probability in
5 years
TD 50/5 Q
• Maximum tolerance dose
• It is the dose that has a 50% complication probability in 5 years
Radiotherapy

Oxygen effectQ • Increased radiosensitivity of oxic cells in comparison to hypoxic


cells.
• This is because the oxygen molecules react with free radicals in
the tissues and produce peroxy radicals – that cause further tissue
damage
• Oxygen enhancement ratio (OER)Q = Radiation dose at Hypoxic
conditions
• Radiation dose at Oxic conditions
• Maximum value of OER is 3Q
278 Contd…
Bystander effectQ • The occurrence of similar biological effects in cells that are not
themselves exposed to radiation, but their neighboring cells are
exposed.
• Occurs due to intercellular communication and cytokines

RADIONUCLIDE IMAGING AND RADIOTHERAPY


Radiation recall phenomenonQ • Severe reactions are seen in skin of areas now when chemotherapy
is administered, whereas these areas have been irradiated long
back
• This occurs due to interaction of basal layers of irradiated skin with
cytotoxic agents secreted from chemotherapy induced cell death
• Most frequently seen with Actinomycin, Doxorubicin, Methotrex-
ate, Fluorouracil, Paclitaxel
Radiation hormesisQ • Indicates stimulating effects of low dose ionizing radiationQ
• Defined as physiological benefits gained from low Linear Energy
Transfer (LET) radiation between the dose range of 1–50 cGy total
absorbed dose
Abscopal effectQ • Occurrence of systemic side effects in regions that are far away
from the irradiated site
• The increase of fraction size to more than 2 Gy per fraction increases
the Abscopal effect
Bragg’s peakQ – NEET 2018 Pattern • Seen with heavy and charged particles like Alpha particlesQ and
ProtonsQ
• The curve denotes the Dose delivered (Y – axis) with increasing
depth in the patient’s body (X axis)
• The dose delivered in case of protons and alpha particles is
inversely proportional to their velocity in tissues.
• So at site of entry, velocity is maximum – so dose deposited is
minimum.
• But at a certain depth when the velocity is minimum, the dose
deposited is MAXIMUM – forming a sudden sharp peak of
exposure – Bragg’s peak
Carbon ions
• Thus if the tumor is localized at this depth – precise dose delivery
to the tumor can be achieved
• The normal tissues beyond this point is also not exposed.

Integral doseQ • Total energy absorbed in the treated tissue volumeQ


• Joule = Kg x GyQ
Avalanche phenomenonQ • More rapid death of tissue cells exposed to radiation, after their
number decreases to a certain point (F point in the image)
• More prominent at high doses and along with additional
treatments (Chemotherapy/surgery)

Radiotherapy

Karnofsky performance scale indexQ • Allows patients to be classified as to their functional impairment.
May be used prior to radiotherapy/chemotherapy to compare
effectiveness of different therapies and to assess the prognosis
in individual patients. The lower is the Karnofsky score, the worse
the survival.
Inverse square lawQ • The intensity of radiation from any source decreases by the square
of the distance from the source
Contd… 279
CHART • Continuous Hyperfractionated Accelerated Radiotherapy.
• CHART may be given to patient suffering from Non-small cell lung
cancer (NSCLC). CHART is superior to conventional radiotherapy
in achieving local tumor control and survival in locally advanced
CONCEPTUAL REVIEW OF RADIOLOGY

NSCLC
CROSS • Chemoradiotherapy for Oesophageal Cancer followed by
Surgery Study (CROSS) trial was performed on patients of both
adenocarcinoma and squamous cell carcinoma of the esophagus
and gastroesophageal junction. It has become a standard-of-care
for such patients.
PORTEC • Postoperative Radiation Therapy in Endometrial Carcinoma
TARGIT • Targeted intraoperative radiotherapy
POET • PreOperative chemotherapy or radiochemotherapy in Esophago-
gastric adenocarcinoma Trial

Important Half-Lives (T1/2) Chemical Modifiers of Radiotherapy


RadiosensitizersQ
Iodine 132 2.3 hours They play a role of fixation of free radicals generated in the
Yttrium 90 2.5 days tissues as a result of radiation exposure, this minimizing
tissue repair.
Technetium 6 hours ƒƒ Hypoxic cell sensitizersQ: Enhance the tissue response to
Iodine 123 13 hours standard radiation, by mimicking the effects of oxygen,
which induces the formation and stabilization of toxic DNA
Gold 2.7 days
radicals. These include:
Thallous chloride 3.1 days •• Nitroimidazones–MetronidazoleQ, Misonidazole, etc.
Gallium 70 3.2 days •• Hyperbaric oxygenQ–Most potent radiosensitizer
•• Antineoplastic drugs–Actinomycin D, 5-FU, Hydro­
Radon 220 3.8 days xyurea, Bleomycin, Cisplatin, etc
Xenon gas 5.2 days ƒƒ Nonhypoxic cell sensitizers: Exact mechanism of action
is not known. Include Halogenated pyrimidines - BUdRQ,
Iodine 131 8 days IUdRQ
Radium 223 11.4 days ƒƒ CaffeineQ–also increases the damaging effect of radiation
P32 14.3 days RadioprotectorsQ
Iridium 74.5 days ƒƒ These are compounds that trap free electrons in tissues, thus
Tantalum 115 days making them unavailable for further chemical reactions
that may lead to cellular damage
Californium 252 2.6 years ƒƒ They increase the therapeutic ratio by promoting repair of
Cobalt 60 5.2 years irradiated normal tissues.
ƒƒ Examples include:
Strontium 90 28 years
•• Thiophosphate compounds containing sulfydryl
Cesium 137 30 years groups–Cysteine, Cysteamine
Radium 226 1622 years •• Amifostine–has no effect on CNS as it does not cross
blood-brain barrier
•• GM-CSF, IL-1, Pentoxifylline, Zinc oxide, Melatonin
Radiotherapy

Radioisotopes used for Treatment of etc.


Metastatic Bone PainQ
HyperthermiaQ—A Potential
ƒƒ Bone seeking radiopharmaceuticals are used including– Radiosensitizer
•• Samarium (Sm153)Q
ƒƒ Hyperthermia has an additive and synergistic effect with
•• Strontium (Sr89)Q
radiation
•• Rhenium (Re186)Q ƒƒ It selectively damages tumor vasculature
•• Tin (Sn177)Q ƒƒ Used for Head and neck carcinomas, Cervical cancer, Rectal
•• Phosphorus 32Q and Breast cancers
280
Radiosensitivity of Tumors Points Dose Location
Point A Q
8000 Rad Q
• Approximately at point in pelvis
Highly Sensitive Tumors (WELMS)Q
where the uterine artery and ureter

RADIONUCLIDE IMAGING AND RADIOTHERAPY


ƒƒ Wilms tumorQ cross
ƒƒ Ewings sarcomaQ–is the most radiosensitive bone tumor • Located 2 cm above and lateral
ƒƒ LymphomaQ–most radiosensitive brain tumor to base of radium tube in cervical
ƒƒ Multiple myelomaQ canal
ƒƒ SeminomaQ/DysgerminomaQ – most sensitive testicular/ Point BQ 3000 RadQ • Corresponds to location of
ovarian tumor respectively obturator nodes
• 3 cm lateral to point A
Highly Resistant Tumors (OMPH)Q
ƒƒ OsteosarcomaQ
Radiation Induced Cancers
ƒƒ MelanomaQ
ƒƒ Pancreatic carcinomaQ Neck irradiation in childhood for benign neck Thyroid cancer
ƒƒ HCCQ conditions
Brush licking by Radium dial painters in Bone
Emergency RadiotherapyQ: Used in
watch factories sarcomas
ƒƒ SVC compressionQ
Uranium mining Lung cancer
ƒƒ Acute metastatic spinal cord compressionQ
ƒƒ Cardiac tamponade due to tumor invasionQ In utero radiation exposure Leukemia
ƒƒ Severe hypercalcemiaQ
Breast irradiation for post-partum mastitis Breast cancer
ƒƒ Tumor lysis syndromeQ
ƒƒ Raised ICT due to brain metastasisQ Radiotherapy alone may be Rx of choice in:
ƒƒ Head and Neck cancers – Oral cavity/Lip/Tongue/Cheek/
Radiotherapy in Carcinoma Cervix Naso-oro-hypopharynx/Nasal cavity/Larynx cancers
ƒƒ It is usually a combination of Teletherapy (to treat the ƒƒ Skin cancers–except melanomas
regional nodes and shrink the primary tumor) and Intra- ƒƒ Cervical cancer
cavitatory Brachytherapy (for the central tumor core) ƒƒ Bladder cancer–except T1 stage
ƒƒ Intracavitary Brachytherapy – may be used in early ƒƒ Seminoma
cases when the probability of lymph nodal metastasis is ƒƒ Early Hodgkins and Non-Hodgkin’s lymphoma
negligible ƒƒ Retinoblastoma

Radiotherapy

281
Multiple Choice Questions
CONCEPTUAL REVIEW OF RADIOLOGY

Radionuclide Imaging
1. PET uses: (Recent Pattern 2016) 13. Which of the following is the incorrect statement regarding
A. 2–Fluorodeoxyglucose GI bleeding? (AIIMS May 2013)
B. Technetium A. The sensitivity of angiography for detecting GI bleeding is
C. Cobalt 60 about 10–20% as compared to nuclear imaging
D. Chromium B. Angiography can image bleeding at a rate of 0.05–0.1 mL/
2. Hot spots on Bone scan may be seen in all of the following min or less
except: (AI Dec 2014) C. 99m
Tc-RBC scan image bleeding at rates as low 0.05–0.1
A. Osteomyelitis B. Multiple myeloma mL/min
C. Hyperparathyroidism D. Metastasis D. Angiography will detect bleeding only if extravasation is
occurring during the injection of contrast
3. Hot spot in myocardial infarction is seen in: (AI 1998)
A. Myocardial perfusion imaging 14. IOC for accessory Spleen is: (JIPMER May 2016)
B. Gallium scan A. USG B. CT Abdomen
C. 99m
Tc–stannous pyrophosphate scan C. MRI D. Radioscintigraphy
D. Radionuclide ventriculography 15. Tc-99m is derived from:
4. Which of the following agent is used to measure GFR? A. Str–99 B. Mo–99
(AI 2008) C. Str–90 D. Mo- 90
A. 99m
Tc DTPA B. 99mTc DMSA 16. Which of the following creates a False negative finding for
C. 99m
Tc MAG3 D. Iodohippurate malignancy on a PET scan? (AIIMS Nov 2017)
5. Used for detection of ectopic kidney: A. Large cell tumor B. Small cell tumor
A. DTPA B. MAG3 C. Typical carcinoid D. Atypical carcinoid
C. DMSA D. I- OIH scan 17. Half-life of 99m–Technetium is:
6. Reversible myocardial ischemia may be detected by: A. 2 hours B. 4 hours
A. Coronary angiography B. 2D echocardiography C. 6 hours D. 8 hours
C. Thallium scan D. MUGA scan Radiotherapy
7. Best non-invasive method for detection of myocardial 18. True about Cobalt 60 is all except: (PGI Nov 2014)
viability is: (AIIMS May 2015)
A. Artificial radioactive substance
A. MUGA scan B. Natural radio-isotope
B. Thallium scan C. Gamma ray emitter
C. Stannous pyrophosphate scan D. Beta ray emitter
D. FDG PET scan 19. Brachytherapy is: (MH 2005)
8. Best investigation for detection of renal scars in a patient of A. Radiotherapy with the source of radiation outside the
vesicoureteric reflux is: (JIPMER May 2017) body well at a distance
A. DMSA scan B. DTPA scan B. External beam radiation therapy
C. MAG3 D. Inulin clearance C. Radiation source used in body cavities or implanted into
9. Which of the following phase of a 99mTc MDP Bone scan will tissues
Multiple Choice Questions

help differentiate between Osteomyelitis and Cellulitis? D. Whole body irradiation


A. Flow phase 20. A patient is receiving external beam radiation for metastatic
B. Blood pool phase endometrial carcinoma. Which of the following organs in the
C. Delayed phase field is most radiosensitive: (AIIMS Nov 2011)
D. None of the above A. Ovary B. Bladder
10. Best investigation for parathyroid localization is: C. Vagina D. Rectum
(AIIMS May 2017) 21. Atoms with same mass number and atomic number but with
A. CT B. USG different nuclear energies are called as: (JIPMER May 2016)
C. MRI D. Sestamibi scan A. Isomers B. Isotopes
11. Nonvisualization of Gallbladder on HIDA scan is suggestive C. Isobars D. Isotones
of: (Recent Pattern 2017) 22. Which of the following is the most ionizing radiation?
A. Acute cholecystitis B. GB perforation A. Alpha B. Beta (AI 2010)
C. Biliary leak D. Biliary atresia C. Gamma D. X-rays
12. Most common used radionuclide in nuclear medicine is: 23. Substances with same atomic number but with different
(JIPMER May 2016) atomic mass numbers are:
A. Radium B. Cobalt A. Isotope B. Isotone
282 C. Technetium D. Barium C. Isobar D. Isomer
24. Which of the following statements best describes background 40. Emergency radiotherapy may be used in:
radiation?  (AI 2010) (PGI June 2005)
A. Radiation in background of nuclear reactors A. SVC syndrome B. Pericardial tamponade

RADIONUCLIDE IMAGING AND RADIOTHERAPY


B. Radiation in background during radiological investigations C. Raised ICT D. All of above
C. Radiation present constantly from natural sources 41. About Bragg peak effect - all are true except:
D. Radiation from nuclear fall-out (Recent Pattern Jan 2018)
25. Which of the following creates a False negative finding for A. Applies to Electromagnetic radiation
malignancy on a PET scan? (AIIMS Nov 2017) B. Produces ionization in biological system
A. Large cell tumor B. Small cell tumor C. The dose at a single point is two to four times greater than
C. Typical carcinoid D. Atypical carcinoid along the path dose
26. Which of the following radioisotope is not used in D. Beam therapy for AV malformation of the brain
Brachytherapy? (Recent Pattern Dec 2016) 42. True about concomitant chemoradiotherapy: (PGI May 2017)
A. I-125 B. I-131 A. Chemotherapy and radiotherapy both given simultaneously
C. Co-60 D. Ir-192 B. Radiotherapy acts locoregionally and chemotherapy acts
27. Radionuclide used in both teletherapy and brachytherapy: against distant micrometastases
A. Iridium 127 B. Cobalt 60 (AIIMS 2011) C. Radiotherapy acts as sensitizing agent
C. Palladium D. Iodine 131 D. Radiotherapy with newer technology
28. All of the radioisotopes are used as systemic radiotherapy 43. Neoadjuvant chemotherapy stands for: (PGI May 2017)
agents except: A. Chemotherapy along with surgery
A. Phosphorus 32 B. Strontium 89 B. Chemotherapy before surgery
C. Samarium 153 D. Iridium 192 C. Chemotherapy after surgery
29. All of the following are radioresistant tumors except: D. Chemotherapy along with radiation therapy
(AIIMS May 2005) 44. Most radiosensitive organ in humans:
A. Pancreatic carcinoma B. Osteosarcoma (Recent Pattern June 2018)
C. HCC D. Ewing’s sarcoma A. Adrenal B. Pancreas
30. Which of the following mode is commonly employed in intra- C. Gonads D. Uterus
operative radiotherapy? (AIIMS 2003) 45. External beam radiation therapy (EBRT) used for treating
A. Electron B. Gamma rays prostate cancer is/are:  (PGI May 2019)
C. Alpha particles D. Neutrons A. Proton B. Neutron
31. Intraoperative radiotherapy is administered to the tumor C. Alpha rays D. Beta rays
bed in: (Recent Pattern Dec 2016) E. Gamma rays
A. Transitional cell carcinoma B. Brain tumors 46. Which of the following is/are used for brachytherapy in
C. Pancreatic carcinoma D. Lymphoma cervical cancer:  (PGI May 2019)
32. IMRT is currently used for: (AIIMS Nov 2005) A. Iodine-125 B. Cobalt-60
A. Prostate cancer B. Brain tumor C. Iridium-192 D. Palladium-103
C. Leukemia D. Stomach cancer E. Cesium–137
33. Stereotactic radiosurgery is used for treatment of: 47. Natural radio isotope(s) is/are:  (PGI May 2019)
(Recent Pattern Dec 2016) A. Radon-222 B. Radium-226
A. Brain tumors B. Trigeminal neuralgia C. Cobalt-60 D. Iridium-192
C. AV malformations D. All of above E. Iodine-129
34. Radiation damage to hypoxic cells may be potentiated by use 48. Radioactive iodine (I-131) is/are used in treatment of: 
of: (AIIMS 2003)  (PGI May 2019)

Multiple Choice Questions


A. Myostal B. Metronidazole A. Medullary Ca thyroid B. Papillary Ca thyroid
C. Alcohol D. Melphalan C. Follicular Ca thyroid D. Anaplastic Ca thyroid
35. Amifostine does not have a protective effect on: (AI 2009) E. Hashimoto thyroiditis
A. Kidneys B. Liver 49. All are true about FDG-PET scan except:  (PGI May 2019)
C. Brain D. GIT A. Half-life of F18 is 110 minutes
36. Most common radiation induced cancer is: (AI Dec 2014) B. Warburg effect best describes its mechanism of action
A. Lymphoma B. Leukemia C. Helps in identifying tumor recurrence in patients who have
C. HCC D. Thyroid cancer undergone brain surgery
37. Radiation exposure in infancy is linked to development of: D. Works on the principle of O2 and glucose uptake by tumor
A. Breast cancer B. Thyroid cancer (AI 2003) cell
C. Lung cancer D. Liver cancer E. Anatomic resolution better than MRI
38. Most radiosensitive period of cell cycle is: (AI 2014) 50. True statement regarding nitrates in myocardial perfusion
A. G2M junction B. G2 phase study:  (JIPMER May 2019 Pattern)
C. G1 phase D. S phase A. Stress agent
39. The cell is most resistant to radiation in which of the following B. Decreased myocardial flow in ischemia
phases of cell cycle: (DPG 2009/AI 2008) C. Enhances the detection of myocardial viability in
A. G2 phase B. Early G1 phase Thallium-201 radiopharmaceutical agent
C. M phase D. S phase D. Reduced collateral flow into ischemic zone 283
Answers
1. A 9. C 17. C 25. C 33. D 41. A 49. E
2. B 10. D 18. B 26. B 34. B 42. A,B,D 50. C
CONCEPTUAL REVIEW OF RADIOLOGY

3. C 11. A 19. C 27. B 35. C 43. B


4. A 12. C 20. A 28. D 36. B 44. C
5. C 13. B 21. A 29. D 37. B 45. A,E
6. C 14. D 22. A 30. A 38. A 46. A,B,C, E
7. D 15. B 23. A 31. C 39. D 47. A,B,E
8. A 16. C 24. C 32. A 40. D 48. B,C
Multiple Choice Questions

284
HIGH-YIELD TOPICS
C hapter O utline
• Tuberculosis
CONCEPTUAL REVIEW OF RADIOLOGY

• Hydatid Cyst
• Skeletal Trauma and Nonaccidental Injury/Battered Baby Syndrome
• Multiple Choice Questions with Explanations

Tuberculosis
1
CLINICAL QUIZ
1. Primary pulmonary tuberculosis is characterized by all except: 2. 45-year-old Ramlal presents with complaints of hematuria
A. Calcification for 2–3 months. Ultrasound of abdomen reveals multiple
B. Hilar lymphadenopathy renal parenchymal calcifications with few dilated calyces,
C. Cavitation calcification in the wall of urinary bladder with a small and
D. Apical involvement contracted bladder. Most likely diagnosis is:
A. Carcinoma bladder
B. Schistosomiasis
C. Primary amyloidosis
D. Tuberculosis

  Clinical Pearls   Clinical Pearls


Recent Concept in TB Imaging CavitationQ is Never seen in Primary Pulmonary TB
The host immune response is the sole predictor of radiographic Primary TB is the 1st exposure of the bacillus to the body. Hence
appearanceQ. Severely immunocompromised patients have the hypersensitivity reaction to the tubercular antigen has not
tendency to have primary form of TB, while immunocompetent yet developed. Hence there is minimal tissue destruction that
patients show propensity for re-activated form of TB. occurs in Primary TB. Hence cavitation is never seen.
Remember that cavitation is, in fact a hallmark of postprimary
TB–for obvious reasons!
ƒƒ Primary TB:
•• 1st infection/exposure.
ƒƒ Postprimary TB:
•• It is most common in infants and children and has the
highest prevalence in children <5 years of age.
•• Endogenous reactivation of latent infection develops
many years after the initial infection.
•• The lungs are the most common site of primary
•• Hypersensitivity to TB Antigen has already developed–
infection by Mycobacterium tuberculosisQ
so greater tissue destruction and CAVITATIONQ
•• Cavitation is not seenQ. •• Involves predominantly the apical and posterior
•• Ghon’s Focus/lesionQ: segments of the upper lobes and the superior segments
 Lung parenchymal evidence of primary TB of the lower lobesQ.
infection. •• Reactivation TB tends to progression and fibrosisQ.
Tuberculosis

•• Ghon’s complexQ: ƒƒ Primary progressive TB:


 Ghon’s focus with involvement of hilar lymph •• Due to inadequate immunity, primary infection
nodes and the intervening lymphatics continues to progress instead of becoming latent.
•• Ranke’s complexQ: •• No recovery from primary infection.
 Calcified Ghon’s focus is Ranke’s complex ƒƒ Miliary TB:
•• Healing is the ruleQ—eventual result is scar/calcification. •• Occurs due to hematogenous dissemination of bacillus.

286
IMAGING FINDINGS   Clinical Pearls
Miliary Shadows on CXR
Primary Tuberculosis • Infections: TB, varicella, histoplasmosis, bronchopneumonia,

HIGH-YIELD TOPICS
Lung Parenchymal Findings in Primary TB brucellosis, coccidioidomycosis, cryptococcosis, blastomycosis
• Cardiac causes: Mitral stenosis, pulmonary edema
ƒƒ Dense, homogeneous parenchymal consolidation in any • Neoplastic: Lymphangitis carcinomatosis, metastasis, leukemia,
lobe; more common in lower and middle lobes. lymphoma, alveolar cell carcinoma
ƒƒ Often indistinguishable from of bacterial pneumonia. • Pneumoconiosis: Silicosis, coal worker pneumoconiosis
Points in favor of TB are: • Allergic: Löffler’s syndrome
•• Radiographic evidence of lymphadenopathy • Interstitial pulmonary, fibrosis, sarcoidosis, right arthritis, his-
•• Lack of response to conventional antibiotics tiocytosis X, alveolar microlithiasis

Lymphadenopathy—CT Better than CXR


Pleural Effusion
ƒƒ Lymphadenopathy is typically unilateral and right sided,
involving the hilum and right paratracheal region–most ƒƒ Often the only manifestation—followed by pleural thicken-
common patternQ. ing/calcification.
ƒƒ Any nodes >2 cm in diameter generally have a low-
attenuation center (due to caseous necrosis) secondary to
necrosis at CT and are highly suggestive of active disease. POSTPRIMARY TB
Cavitation is hallmark of Postprimary TBQ.
Main distinguishing features as compared to Primary TB areQ:
  Clinical Pearls
Peripheral RIM Enhancement Pattern of TB Lymph Nodes on ƒƒ Predilection for upper lobe involvement
CE-CT ƒƒ Absence of lymphadenopathy
ƒƒ Presence of cavitation (is never seen in Primary TBQ)
The histopathological hallmark of TB lymph nodes is an area of
central caseous necrosis and a necrotic area never enhances on Imaging findings in postprimary TB may be broadly classified as:
a contrast study. So such LNs on CE-CT show a central hypodense ƒƒ Lung parenchymal findings in postprimary TB:
appearance while the peripheral viable portion enhances. •• Patchy, poorly defined consolidationQ- in apical and
This appearance is so peculiarly associated with TB that in India, posterior segments of the upper lobes.
whenever you see such LNs, anywhere in the body–you think •• Cavitation—the hallmark of postprimary-tuber­
of TB. culosisQ
•• The cavities typically have thick, irregular walls, which
become smooth and thin with successful treatment.
Miliary TBQ (Recent Pattern Jan 2019) Refer to •• Tree-in-bud opacitiesQ- visible in the lung periphery
Q. No. 23 in Image-Based Question and resemble a branching tree with buds at the tips of
the branches, are indicative of active tuberculosis with
ƒƒ The classic radiographic findings of evenly distributed
endobronchial spread of diseaseQ.
diffuse small 2–3 mm nodules, with a slight lower lobe
2. Airway involvement: Bronchial stenosis/Traction bron-
predominanceQ.
chiectasisQ
ƒƒ High-resolution computed tomography (HRCT) is more
3. Pleural effusion: Less common than in primary TB.
sensitive than conventional radiography, with nodules seen
in a random distributionQ.

  Clinical Pearls
Eponyms in Pulmonary TB
Across the spectrum of chest involvement in TB you will come across many named entities. We have studied a few like Ghon’s focus/
complex, Ranke’s complex above. Here is an attempt to cover all such eponyms in TB:
Eponyms in Tuberculosis–All Potential MCQs
Tuberculosis

Ghon’s focus/lesion – Ghon’s Complex – Ranke’s complex: Discussed above in primary TB


Ranke stages of TB/Ranke • Historical classification of TB by clinical and pathological aspects and with respect to the
dreistufenlehre immunological reaction
ƒƒ Stage I: Primary complex
ƒƒ Stage II: Generalization by hematogenous spread (stage of allergy)
ƒƒ Stage III: Isolated organ TB (necrotizing/fibrosing) with tendency to spread (stage of relative
immunity)

Contd...
287
Simon’s focus • Apical lung nodule due to hematogenous spread from an extrapulmonary Primary TB
• Heals by scarring
• Subpleural in location and appears as a fibronodular patch or ill-defined reticular shadow in
CONCEPTUAL REVIEW OF RADIOLOGY

the upper lung fields on Chest X-rays (CXR).


Puhl’s lesion/nodule or • Isolated lesion of chronic pulmonary TB
Aschoff-Puhl reinfection/ • Located in lung apex
exogenous reinfection
Assman’s foci • Reactivation of Simon’s focus
• Typical infraclavicular location
Weigert focus • Caseating focus in the wall of a pulmonary vein
• Subintimal location
Rasmussen’s aneurysm • Pulmonary artery aneurysm within/adjacent to a TB cavity
• May rupture and cause hemoptysis
Rich’s focus • TB granuloma in cerebral cortex
• Ruptures into the sub-arachnoid space causing TB meningitis
Simmond’s focus • TB focus in liver
Pott’s disease • TB involvement of Spine
Pott’s caries
Pott’s curvature
Pott’s syndrome
David’s disease

CXR PA View – Complications and Sequelae of


showing patchy
consolidation Pulmonary TB
in the left lower ƒƒ Aspergilloma in a TB cavity:
lung filed (red •• TB cavity may be colonized by Aspergillus forming an
arrow) with a “Aspergilloma”Q.
cavity (yellow •• Air crescent signQ–Spherical mass/nodule in depend-
arrow) in a case of ent portion with an overlying crescentic lucency of air
Postprimary TB
separating it from the wall.
ƒƒ Rasmussen’s aneurysmQ—Pseudoaneurysm of pulmonary
artery caused by erosion from an adjacent TB cavity. May
cause hemoptysis that may be massive and life threatening.

ABDOMINAL TB
The abdomen is the most common focus of extrapulmonary
tuberculosisQ, with the solid viscera being affected more often
than the gastrointestinal tract.
CT is the mainstay for investigating possible abdominal
tuberculosisQ.

HRCT - Multiple TB Lymphadenopathy


nodules scattered
Tuberculosis

ƒƒ Abdominal lymphadenopathy (with central non-


randomly in both enhancing caseous necrosis) is the most common
lungs – Miliary TB
manifestation of abdominal tuberculosisQ.

288
TB Peritonitis shrunken cecum retracted out of the right iliac fossa by
mesocolon retraction - Pulled up cecumQ.
ƒƒ Peritonitis is a common clinical manifestation of abdominal •• Pipe stem colonQ: Long segments of narrowing, rigidity
tuberculosis. and loss of colonic distensibility.

HIGH-YIELD TOPICS
ƒƒ It is subdivided into three main types: Wet–Dry and Fibrotic. •• Chicken intestineQ: Disturbances in tone and
ƒƒ US stellate signQ: Fixed loops of bowel and mesentery peristaltic contractions resulting in hypersegmentation
standing out as spokes radiating out from the mesenteric of barium column.
root. •• Fleischner’s sign/Inverted umbrella signQ: Thicken-
ƒƒ Club sandwich sign/Sliced bread appearanceQ: Localized/ ing of ileocecal valve lips/widely gaping valve with nar-
focal ascites in between radially-oriented bowel loops due rowing of terminal ileum.
to exudation from involved bowel/LNs. •• Goose neck deformity of terminal ileumQ: Dilated
ƒƒ Abdominal cocoon/Sclerosing encapsulating peri­ terminal ileum appears suspended and hanging down
tonitisQ: Small bowel loops localized at the center of from a retracted shortened cecum.
abdomen encased by a soft tissue density covering mantle. •• Purse-string stenosis at IC junctionQ
•• Stierlin’s signQ: Rapidly emptying ileum through a
GI Tract/Bowel TB gaping IC valve into a rigid contracted cecum.
ƒƒ Ileocecal junction is most common site of involvementQ. •• String signQ: Persistent narrow stream of barium
in bowel–indicated stenosis. (Also seen in Crohn’s
disease)
  Clinical Pearls
Why is the Ileocecal junction the most common site of involve-
  Clinical Pearls
ment in GI-TB?
This is considered to be due to: TB versus Crohn’s Disease
• Physiological stasis at this site Tuberculosis Crohn’s disease
• Abundant presence of lymphoid tissue Asymmetric, irregular wall Circumferential wall thickening
• Increased rate of absorption at this site thickening
• Closer contact of bacilli with the mucosa of this region
• Favorable pH environment Fleischner sign on Barium Cobblestone appearance on
study Barium study
ƒƒ The most common CT finding is mural thickeningQ - No creeping fat Creeping fat (abnormal
typically concentric excessive mesenteric fat seen)
ƒƒ Localized lymphadenopathy is seen with typical central Pulmonary findings may be Usually none
caseous necrosis seen on CXR
ƒƒ Barium Meal Follow Through (BMFT)/Barium Enema Enlarged LNs with low Enlarged LNs with soft tissue
studies: density centers due to density without necrosis
•• Spasm and hypermobility with edema of the valve–is central caseous necrosis
EARLIESTQ finding Omental and peritoneal Normal omentum and
•• Advanced gastrointestinal tuberculosis characteristi- thickening peritoneum
cally appears as Napkin ring stenosesQ, with a conical,

Multiple CXR PA View –


patchy areas of Marked upper lobe
consolidation and fibrosis, more on
centrilobular nodules right side (yellow
(yellow arrows) arrow), both hila
with tree in bud are pulled up and
Appearance (Inset trachea deviated
– red arrows) – to right side (red
Postprimary TB arrow). Some active
Tuberculosis

Thick-walled cavitary infiltrates also


lesion in posterior seen in left lower
segment of upper zone (blue arrow) –
lobe with traction Postprimary TB
bronchiectasis
anteriorly with few
fibrotic changes and
patchy consolidation
– Postprimary TB
289
Noncaseating Caseating
Tuberculoma granuloma
– Uniformly with a solid
hyperintense center –
CONCEPTUAL REVIEW OF RADIOLOGY

on T2W with Central iso/


enhancement hypointense
(yellow arrows) on T2W
images

Caseating granuloma
with central
liquefaction –
Central hyperintense
appearance on T2W
with peripheral rim
enhancement

CNS TUBERCULOSIS Spinal TB


ƒƒ More common in immunocompromised patients. On MRI
ƒƒ MRI with Gadolinium enhancement is BEST investiga- ƒƒ CSF loculation and obliteration of the spinal subarachnoid
tionQ. space, with loss of outline of the spinal cord in the
cervicothoracic spine and matting of the nerve roots in the
lumbar region.
TUBERCULOUS MENINGITIS (TBM) ƒƒ Contrast-enhanced imaging reveals nodular, thick, linear
intradural enhancement, which can completely fill the
ƒƒ TBM is the most common manifestation of CNS
subarachnoid space, sometimes giving the appearance of a
tuberculosisQ across all age groupsQ.
normal unenhanced MR image.
ƒƒ Typical radiographic finding is abnormal meningeal
enhancement, most pronounced in the basal cisternsQ.
ƒƒ Communicating hydrocephalusQ—most common
complication of tuberculous meningitis. CT scan is the HEAD AND NECK TB
best method for sequential follow-up of hydrocephalusQ. ƒƒ The most common location is within neck nodesQ, often
ƒƒ Basal ganglia ischemic infarctsQ manifesting as bilateral painless cervical lymphadenitis,
ƒƒ Cranial nerve involvement can occur rarely. also known as scrofula.
ƒƒ The involved nodes are initially homogeneous but later
Parenchymal Tuberculosis undergo central necrosis. They show peripheral rim
ƒƒ The most common CNS parenchymal lesion is enhancement.
tuberculomaQ (tuberculous granuloma)Q.
ƒƒ Most commonly occurs within the frontal and parietal
lobeQ MUSCULOSKELETAL TB
ƒƒ CE-CT/CE-MRI imaging:
•• Appear as round or lobulated masses with a TB Spondylitis
Homogeneous/Disc-like or ring enhancementQ ƒƒ Spine involvement is most common site involved in MSK
•• Rarely “target signQ” (i.e., central calcification or punc- TBQ
tate enhancement with surrounding hypoattenuation ƒƒ Lower thoracic and upper lumbar are the most commonly
Tuberculosis

and ring enhancement) - suggestive of, but not pathog- involved regionsQ.
nomonic for, tuberculosis. ƒƒ MRI with gadolinium enhancement is best imaging
ƒƒ Miliary CNS tuberculosis: techniqueQ
•• Usually associated with tuberculous meningitis ƒƒ The loose internal structure of the disk allows the infection
•• MR imaging: Multiple tiny (<2-mm), hyperintense to disseminate more widely into additional spinal segments,
T2 foci that homogeneously enhance on contrast- resulting in the classic pattern of involvement of more
enhanced T1-weighted images. than one vertebral body together with the intervening
discsQ.
290
ƒƒ Pott’s abscessQ - paravertebral TB abscess.
ƒƒ If left untreated, the infection eventually results in vertebral Postcontrast T1W
collapse and anterior wedging, leading to kyphosis and images – Enhancing
gibbus formationQ. disc and adjacent

HIGH-YIELD TOPICS
vertebral segments
with epidural extension
  Clinical Pearls – suggests Tubercular
Phemister’s triad in TB arthritis spondylodiscitis
Phemister’s triadQ: The triad of Phemister refers to three features
seen classically with tuberculous arthropathy:
• Juxta-articular osteopenia/osteoporosis
• Peripheral osseous erosions
• Gradual narrowing of joint space

TB OSTEOMYELITIS
ƒƒ It is relatively rare, metaphyses are involved, with
radiographic features that include osteopenia and poorly
defined lytic lesions with minimal surrounding sclerosis.
ƒƒ Penumbra signQ: Thin intermediate signal intensity rim •• Putty kidneyQ homogeneous/uniform/ground glass
along the periphery of a bone/soft tissue abscess on T1W like calcification representing calcified caseous tissue.
images on MRI–is due to layer of granulation tissue. •• AutonephrectomyQ: The final outcome of renal
tuberculosis is autonephrectomy, which represents a
small, shrunken, scarred, nonfunctioning kidney.
  Clinical Pearls
ƒƒ Thickened ureteric wall and strictures - most common in
Spina ventosa/TB dactylitis the distal third of the ureterQ.
• “Spina” means “short bone” and “Ventosa” means “expanded ƒƒ Beaded or Cork-screw appearance of ureterQ may be seen
with air”. ƒƒ Pipe-stem ureterQ: Severe wall thickening causes a rigid
• It is painless involvement of the short tubular bones of the shortened ureter with a narrow lumen.
hands and feet, is also more common in children. ƒƒ Thimble bladderQ: In advanced disease, there is eventual
• At radiography, pronounced fusiform soft-tissue swelling with scarring with long-term loss of cystic volume and a small,
or without periostitis is the most common finding. irregular, calcified bladder.
• Involved bone shows a diaphyseal expansile lesion
1
CLINICAL QUIZ - SOLUTION
GENITOURINARY TRACT TB Remember that cavitation is never seen in primary TB – we have
studied the concept in detail in this topic. Hence answer ti 1st Quiz
Renal TB question is C.
ƒƒ Sterile pyuriaQ is a characteristic feature. The 2nd Quiz question is tricky. See that the patient has complains
ƒƒ Intravenous Urography/pyelography (IVU/IVP) is the for 2-3 months – so a subacute to chronic condition. Also observe
BEST modality for early diagnosis of Renal TBQ. that the entire urinary tract from kidneys to UB is involved. The
ƒƒ Once the disease is established and in advanced cases– key to distinguish between Schistosomiasis and TB is the extent
CE-CT Abdomen and pelvis is betterQ. of involvement.
ƒƒ The most valuable radiologic feature of Genitourinary TB
is the multiplicity of abnormal findingsQ.
ƒƒ The most common CT finding is renal calcification.
ƒƒ On IVU study: CO Schistosomiasis versus Tuberculosis
NC E P T
•• “Moth-eaten” calyxQ due to erosions – earliest
Schistosomiasis and TB can have a very overlapping picture
Tuberculosis

finding and progresses to papillary necrosis.


both clinically and imaging wise especially when the latter
•• Hydronephrosis with irregular margins.
involves the ureters and bladder. The key concept is that
•• Hiked-up renal pelvisQ: The cephalic retraction of
Schistosomiasis involves only the urinary bladder and never
the inferior medial margin of the renal pelvis at the
the kidneys/ureters. This is in distinct contrast with TB wherein
ureteropelvic junction (UPJ). the entire urinary tract is usually involved–hence this can be
•• Kerr’s kinkQ: Deformity of pelvicalyceal system used to distinguish between them.
caused by traction from a strictured infundibulum/
parenchymal fibrosis. Thus the answer to this question is D. TB.
291
Hydatid Cyst
CONCEPTUAL REVIEW OF RADIOLOGY

2
CLINICAL QUIZ
3. A 40-year-old patient presented with nonspecific complaints of abdominal
discomfort and mild intermittent pain in the right hypochondriac region.
A CT Abdomen revealed a complex hepatic lesion with dense peripheral
circumferential rim of calcification. Most likely diagnosis is:
A. Liver abscess
B. Hydatid cyst
C. Simple cyst
D. Metastases

  Clinical Pearls
Hydatid Cyst Basics ƒƒ PericystQ–
Worldwide zoonosis produced by the larval stage of Echinococcus  Outermost layerQ
tapewormQ.  Represents the host response to the parasiteQ
• Two types of infection are known: ƒƒ EctocystQ–
ƒƒ Echinococcus granulosusQ—Most common type of Hydatid  Middle laminated acellular membraneQ
disease ƒƒ EndocystQ–
ƒƒ Echinococcus multilocularisQ—Less common, but is more  Innermost germinal layerQ–is metabolically the most
invasive, and may mimic a malignancy active layerQ

ƒƒ Hydatid Cyst structure: The cyst has 3 layers in its wall.

CO Named Signs in Hydatid Disease—How to Remember Them?


NC E P T

• All your books will enlist a large number of named signs and contents within the cyst–This appearance is given several fancy
appearances seen in Hydatid cyst. It indeed seems like a burden names such as–Daughter cysts in a Mother cyst sign, Onion peel
trying to remember each and every one of it! sign
• What you are not told is that almost each and every sign in ƒƒ The germinal layer gradually gets detached from the wall and
Hydatid cyst correlates with a particular step in the disease appears as floating membranes in the cyst–called as Serpent
pathophysiology. sign (Snake like).
• In order to remember the signs well, we must know a few basics ƒƒ As the cyst enlarges (especially in the lungs), it ruptures into
about hydatid cyst structure and the disease process. the trachea-bronchial tree, thus air enters into the cyst. The
• This is a perfect example of integrated approach towards patient starts expectorating the cyst contents.
radiology–not studying it in an isolated manner as a subject, but ƒƒ Cyst contents floating on an air-fluid level within the cyst–is
rather integrating it as a part of holistic learning! called the epic - Floating water lily sign!
• Brief pathophysiology of hydatid cyst: ƒƒ Finally the entire cyst is empty air filled–the so called Empty
Hydatid Cyst

ƒƒ Once a cyst is formed–it may appear as a Simple cyst in its cyst sign!
early stages without any contents • Thus if you now know the pathophysiology of Hydatid cyst–you
ƒƒ As the germinal layer proliferates to create brood capsules– will automatically remember most of the named signs seen in
Outpouchings are seen progressing to Daughter cysts/septae/ this disease!

292
ƒƒ Radiological types of hydatid cyst–based on internal SPECIFIC ORGAN INVOLVEMENT
architecture
•• Type IQ: Simple cyst with no internal architectureQ: Liver Hydatid Cyst Refer to Q. No. 13 in
 Initial stage of cyst Image-Based Questions

HIGH-YIELD TOPICS
 USG–Appears unilocular anechoic cystic lesion
ƒƒ Most common siteQ
with minimal echogenic hydatid sand that falls to
ƒƒ Right lobeQ is more commonly involved than left
the dependent part on changing patient position–
ƒƒ Type I cysts–are very similar to simple hepatic/biliary cysts.
Falling snowflakes signQ/Snowstorm signQ
On USG the movement of echogenic internal foci (hydatid
 Rim sign on T2W MRIQ–Thin peripheral hypoin-
sand) on changing patient position–Falling snowflakes
tense rim around the cyst on T2W images–formed
signQ/Snowstorm signQ - is diagnostic of Hydatid cyst.
by the Pericyst layer. Nonspecific sign as it is also
ƒƒ USGQ–Most sensitive modality to detect septae/
seen in Amoebic abscess, hematoma, HCC.
membranes/hydatid sand in a hydatid cystsQ
•• Type IIQ: Cyst with daughter cysts and matrixQ:
ƒƒ USG floating Water-lily signQ – Completely separated
 USG–will show echogenic walls of daughter cysts membranes in the dependent part of the cyst
within the large mother cyst
ƒƒ Cyst within cyst signQ on USG/CT–seen with Daughter
•• Type IIIQ: Calcified cystQ: cysts (Type II cyst)
 Dead cysts with total calcification ƒƒ Spoke wheel pattern on USGQ – Daughter cysts separated
•• Type IVQ: Complicated cystQ: by echogenic hydatid matrix
 Rupture–into the trachea-bronchial tree/Biliary ƒƒ Cyst calcification–usually involves the Pericyst–is curvilin-
tree ear/ring likeQ–may be seen on abdominal radiographs/CT.
 Superimposed infection ƒƒ Hydatid cyst may appear mildly hyperdense on NC-CTQ

  Clinical Pearls
2001 WHO Classification of Hepatic Hydatid Cyst: Based on USG AppearanceQ

CL • Unilocular anechoic cyst without any septae/echoes


• Active stage of the cystQ
CE 1Q • Unilocular cyst with fine echoes settled in dependent part of cyst–Hydatid sandQ
• Active stage of the cystQ
CE 2Q • Cyst with multiple septations giving rise to multivesicular/rosette like/honeycomb appearance
• Active stage of the cystQ PAIR is FeasibleQ
CE 3Q • Unilocular cyst with internal Daughter cysts and detached laminated membrane–Water lily
appearance
• Transitional stage of cystQ
CE 4 • Mixed hypoechoic and hyperechoic contents without Daughter cysts
• Ball of wool signQ/Yarn signQ/Concealed Water lily signQ
• Degenerating stage of cystQ
CE 5 • Partially/completely calcified cyst wall
• Dead/Inactive stage of cystQ
Another classification based on USG appearance is Gharbi classificationQ

Lung Hydatid Cyst •• Onion peel signQ: The air between the pericyst and
endocyst (after ectocyst has ruptured) has a laminated
ƒƒ 2nd Most common site of involvement in adultsQ
appearance. Sign of ruptured cyst wall–ectocyst layer
ƒƒ Most common site of involvement in childrenQ
•• Cumbo signQ: Air fluid level inside the endocyst and
Hydatid Cyst

ƒƒ Calcification–is very rare in Pulmonary hydatid cysts–


Onion peel sign together create this sign
almost never seenQ
•• Serpent signQ: After partial expectoration of cyst
ƒƒ X-ray findings
contents the membranes are seen within the mother
•• Seen as well-defined round radio-opacity in the lung cyst
fields.
•• Floating water lily signQ: Most of the cyst contents have
•• Crescent sign/Meniscus signQ: As small bronchioles been expectorated out, with the crumpled endocyst
are eroded, air enters between the pericyst and ectocyst
seen floating in the cyst fluid
appearing as a curvilinear lucency

293
•• Mass within a cavity signQ: All fluid is expectorated
out, with a small lump of membranes seen within the CE-CT – Liver
air-filled cyst cavity hydatid cyst
•• Whirl sign/Spin signQ: The adjacent pleural thickening showing
CONCEPTUAL REVIEW OF RADIOLOGY

around the cyst can be traced as a whirl along the cyst curvilinear
wall and separated membranes within the cyst peripheral
•• Empty cyst signQ: All contents are expectorated out, calcification
what remains is an empty lucent appearing cyst. (yellow arrow)
and central
Bone Hydatid Cyst hyperdense
membranes (red
ƒƒ Spine and Pelvis–are most common sites, followed by long arrow)
bones.
ƒƒ Pericyst formation does not occur in bone Hydatid cystQ
ƒƒ Therefore there is aggressive proliferation and branching
spread of lesion that occurs. Eventually the cortex is
Liver and
breached and the extra-osseous tissues are involved.
Splenic Hydatid
– Liver lesion is
indistinguishable
  Clinical Pearls
from a simple
Percutaneous Aspiration Injection and ReaspirationQ (PAIR)Q hepatic cyst
It is percutaneous drainageQ of echinococcal cysts mostly the
Liver (under USG/CT guidance) performed with a fine needle or
a catheter, followed by the killing of the protoscolices remaining Left hip joint
in the cyst cavity by a protoscolicide agent. Hydatid cyst –
• If a catheter is temporarily left in the cyst after the procedure Extensive Hydatid
for drainage (D), the acronym PAIRDQ is used. cyst with osseous
• If numerous and large daughter cysts are present, an alter- and Extraosseous
native percutaneous technique “Percutaneous Puncture with involvement
Drainage and Curettage” (PPDC)Q may be used. causing bone
• PAIR is feasible in types CE1Q, CE2Q and CE3Q of the WHO destruction
classificationQ of Cystic Echinococcosis cysts.
• Contraindications of PAIRQ:
ƒƒ Noncooperative patients and inaccessible or risky location
of the cyst in the liver
ƒƒ Cyst in spine, brain and/or heart Coronal and Axial
ƒƒ Inactive or calcified lesion STIR MRI images
ƒƒ Cysts communicating with the biliary tree – Extensive left
ƒƒ Cysts open into the abdominal cavity, bronchi and urinary hip involvement
tract with Hydatid cyst

2
CLINICAL QUIZ - SOLUTION
A complex cystic lesion in the liver with a peripheral rim of calcification is suggestive of a hydatid cyst. Remember that a liver abscess
usually presents with signs of sepsis and will never show peripheral calcification unless it is chronic. Whereas Hydatid cysts are usually
detected incidentally and do not present with signs of sepsis. Though peripheral rim of calcification is common in Liver/Splenic hydatid
cysts – it is never seen in Pulmonary hydatid cyst.
Hydatid Cyst

Hence the answer to this question is B. Hydatid cyst

294
Skeletal Trauma

HIGH-YIELD TOPICS
3
CLINICAL QUIZ
4. A patient presented with history of trauma followed by periorbital
swelling and diplopia. Diagnosis based on this CT image is:
(NEET 2019 Pattern)
A. Le Fort fracture B. Fracture zygoma
C. Tripod fracture D. Blow-out fracture

SPECIAL BONE AND JOINT INJURY Greenstick/Hickory Stick FractureQ–One


PATTERNS Cortex Broken–One Cortex Buckled/
Normal
Let us look at a few Special types of Injury patterns first:
ƒƒ Pediatric skeleton has a lot of woven/pliable bone with very
high elasticity
  Clinical Pearls ƒƒ Whenever a bending force is applied to a bone, the convex
Joint disruptions side of the bone is subjected to tension resulting in a
Joint Subluxation transverse fracture involving the outer/convex cortex
• Partial loss of contact between the articular surfaces at a ƒƒ The concave/inner side remains intact or may undergo
joint. The joint surfaces are incongruous but a significant cortical buckling, but does not essentially fracture/break
portion remains opposed. ƒƒ Usually heals well without any complications
Joint Dislocation
• Complete loss of contact between the articular surfaces at a Pathological Fracture
joint. If there is associated fracture–it is termed as a Fracture-
ƒƒ Fracture through a bone weakened by a localized/systemic
Dislocation.
disease.
Joint Diastasis
• Complete separation occurring at a syndesmosis/slightly ƒƒ Banana fractureQ–Transverse pathological fractures in long
movable joint bones with smooth margins
• Common sites are–Pubic symphysis, skull sutures, etc.
Stress Fractures
ƒƒ Bone injury due to repetitive stress of lower intensity than
Torus/Buckling FractureQ– Cortical that required for an acute traumatic fracture. These fractures
BucklingQ and Trabecular ImpactionQ – can be of 2 types:
Skeletal Trauma

without Cortical BreakQ ii. Fatigue fractureQ–Occurs secondary to Abnormal


stress–applied to a bone of normal strengthQ. Seen in
ƒƒ “Torus” in Latin means–to bulgeQ
military recruits, athletes and dancers
ƒƒ It is a special type of Greenstick fracture and is seen in ii. Insufficiency fractureQ–Occurs secondary to Normal
children due to elastic bone structure stress–applied to a weakened boneQ. Seen in osteopo-
ƒƒ Due to the compression forces applied across the bone the rosis, osteomalacia, Paget’s disease, osteopetrosis, fi-
cortex simply buckles/bulges without any obvious cortical brous dysplasia, osteogenesis imperfecta
break. The trabeculae inside the bone overlap each other ƒƒ MRI - is BEST investigationQ– identifies fracture and
giving rise to a linear radiodensity across the 2 cortical marrow edema
buckling on 2 sides of the bone. ƒƒ Bone scan–is 2nd Best modality – shows increased uptake

295
ƒƒ Common sites: ƒƒ Pain, induced by activity, relieved by rest–typical
•• Pars interarticularis fracture of lower Lumbar spineQ presentation.
- Most common site of stress fracture in entire skeleton ƒƒ Radiographic latent period (Time interval for appearance of
•• March fractureQ/Deutschlander's fractureQ–Mid X-ray features)–is 10–21 daysQ
CONCEPTUAL REVIEW OF RADIOLOGY

and distal portions of 2nd/3rd metatarsal shafts–due ƒƒ Radiographic features:


to fatigue of peroneus longus muscle. Most common •• Periosteal reactionQ–Most commonQ/Most reliable
lower limb site of stress fractures X-ray signQ. Solid patternQ of periosteal thickening
•• Rower’s fractureQ–Stress fracture of Ribs in boat creating an eccentric protuberance of cortexQ at the site
rowers of maximum stress
•• Sacral stress fracturesQ–Elderly women with osteopo- •• Fracture lineQ– Subtle oblique/transverse radiolucent
rosis line may be seen

Torus fracture Greenstick


involving the distal fracture
Radius – look at the involving the
cortical buckling on distal radius –
either sides and the yellow arrow
trabecular impaction points to a
seen across them cortical break
(red arrows). Ulnar involving a
fracture shows a single cortex
cortical break hence
cannot be termed as
a Torus injury

Coronal STIR and T1W


MRI Knee – shows a
Tibial stress fracture.
Hypointense transverse
defect is seen on T1W
image, Short-TI Inversion
Recovery (STIR) shows
hyperintense marrow
edema around it

Radio- Submento-
graph vertex view
lateral shows left
view zygomatic
Skeletal Trauma

shows a arch fracture


Nasal bone
fracture

296
Salter-Harris Classification of Epiphyseal Injuries: (Remem-
Le Fort Type I
ber the mnemonic–S-A-L-T-ER)

Type Mnemonic Features • Floating palateQ

HIGH-YIELD TOPICS
• Horizontal maxillary
Type I S–Slipped/ • 5–7%
frHorizontal maxillary
Straight • Fracture plane passes through the fractureQ separating
across growth plate, does not involve the
the upper teeth from
bone
the upper face
• Good prognosis • Fracture involves the
• Slipped capital femoral epiphysis– alveolar ridge, floor of
Most common presentation maxillary sinus, lateral
Type II A–Above • 70–75%—Most common type nose wall
• Fracture passes through the growth
plate and extends up into the
metaphysis Le Fort Type II
• Good prognosis
• Distal radius–Most common site • Floating maxillaQ
• Thurston–Holland fragment– • Pyramidal fractureQ,
Metaphyseal fragment in Type II with teeth forming the
fractures base of pyramid, naso-
frontal suture forming
Type III L–Lower or • 5–10%
its apex
beLow • Fracture passes through the growth • Fracture involves the
plate and extends below into the nasal bones, inferior
epiphysis–intraarticular fracture orbital rim lateral wall
• Poor prognosis–as the proliferative of maxillary sinus,
and reserve zones are affected posterior alveolar ridge
• Distal tibia–Most common site
Type IV T–Two or • 8–10%
Through • Lateral condyle of Humerus–Most Le Fort Type III
and common site in <10 years
through • Fracture starts in the metaphysis
extends through the growth plate into
• Floating faceQ
the epiphysis
• Craniofacial
• Poor prognosis–as the proliferative dysjunctionQ
and reserve zones are affected • Fracture involves the
• Distal tibia - Most common site in >10 nasofrontal suture,
years maxilla-frontal suture,
Type V ER– • Very uncommon <1% zygomatic arch
ERasure of • Crushing injury damages the growth
the growth plate
plate • Worst prognosis
• Distal tibia/femur–Most common site
Tripod FractureQ
FACIAL INJURIES
Nasal Bone Fracture—Most Common
Skeletal Trauma

Facial Fracture
ƒƒ Lateral projection of skull with slight under exposureQ–Best
radiographic projection
ƒƒ CTQ–Best investigation–100% sensitivity
Le Fort Fractures of Mid-face: Fracture of pterygoid plateQ is
mandatory to call a fracture as Le Fort fracture

297
•• Teardrop signQ: Polypoid opacity hanging from the
Axial CT Bone roof of maxillary sinus
window image •• Black eyebrow signQ: Intra-orbital emphysema–free
– shows a C1 air just above the maxillary sinus in the orbit
CONCEPTUAL REVIEW OF RADIOLOGY

ring fracture
– Jefferson’s
fracture
Mandible Fracture
ƒƒ Most common site is mandibular bodyQ
ƒƒ Mandible is slowest healing boneQ in the body with
radiographic union lagging well behind the clinical union

SPINE FRACTURES AND


DISLOCATIONS
Jefferson’s Fracture/Bursting Fracture of
Atlas (C1)
ƒƒ Bursting fracture of ring of atlasQ, with double part fractures
through anterior and posterior arches–thus classically being
a four part fractureQ
ƒƒ CT is Best investigationQ
X-ray Cervical
spine lateral Hangman’s FractureQ/Hangee’s
– Fracture of
spinous process FractureQ/Traumatic Spondylolisthesis of
of C6 – Clay AxisQ
Shoveler fracture
ƒƒ Fracture through the bilateral pedicle/pars interarticularis
of C2Q
ƒƒ Motor vehicle accidentQ–Most common cause
ƒƒ Also known as Zygomaticomaxillary complex fractureQ/ ƒƒ Seen in judicial hanging–Hangee’s fractureQ is more accurate
Trimalar fractureQ ƒƒ Usually not associated with any neurological findingsQ–due
ƒƒ Involves 3 components: to the large spinal canal dimensions at this level
iii. Zygomatic archQ ƒƒ CT- Best investigationQ
iii. Orbital processQ
iii. Maxillary process surrounding the superior and lateral
margins of maxillary sinusQ Teardrop Fracture
ƒƒ Results from a blow over the malar eminence and essentially ƒƒ Acute hyperextension causing avulsion of a triangular
involves the zygoma fragment from the anteroinferior cornerQ of C2 (Axis) body
ƒƒ C2 is most common siteQ
Orbital Blow-out FractureQ – (Recent
Pattern Jan 2019) (Refer to Q. No. 18 in
Image-Based Questions)
ƒƒ Fracture involving the infra-orbital plate of maxillary bone.
The inferior orbital rim may be intact
ƒƒ Result from blow of fist/ball directly over the globe or just
Skeletal Trauma

caudal to it
ƒƒ Orbital contents are pushed down into the fracture defect
and get trapped–most commonly Inferior rectus muscleQ
ƒƒ Patient complains of pain, swelling and diplopia
ƒƒ Imaging findings:
•• Opacification of maxillary sinus with/without a Odontoid Process Fractures
horizontal air/fluid level
ƒƒ Anderson and D’Alonzo classificationQ used

298
Type I Type II Type III

HIGH-YIELD TOPICS
Avulsion fracture odontoid tipQ Involves odontoid base (junction of Extension into C2 body
odontoid and body)Q
Stable fractureQ Unstable fracture with risk of non-unionQ Stable fractureQ
Treated with immobilization bracing Most common odontoid fractureQ Heals more readily than Type II
Surgical fixation may be required Q
Disruption of C2 ring/Harris’ ringQ is seen
on lateral cervical spine radiographs

Other Peculiar Cervical Spine Fractures


Carrot stick fractureQ • Fracture through the ankylosed discovertebral junction in Ankylosing spondylitis
Clay Shoveler’s fractureQ/Coalminer’s • Avulsion of a lower cervical spinous process–most commonly C7
fractureQ/Root puller’s fractureQ • Caused by pull of Trapezius and Rhomboid muscles on the spinous process
• Double spinous process sign seen
Pillar fracture • Fractured articular pillar
Wedge fracture • Compressed anterior vertebral body fracture
Whiplash syndrome /Cervical sprain-
Q
• Occurs most commonly following a rear-end motor vehicle collision
strain injuryQ • Follows a forced hyper-extension, hyper-flexion sequence
• Characterized by predominantly soft tissue abnormalities
• Widened retro-pharyngeal space/retro-tracheal space/Anteriorly displaced pre-vertebral
fat stripe
• Loss of spinal lordosis, acute kyphotic angulations may be seen
• Joint abnormalities also seen involving the cervical spine joints

Compression Clavicle fracture –


fractures of Allman type I
L1 and L3
vertebral
bodies
(yellow
arrows)
Skeletal Trauma

299
THORACIC AND LUMBAR SPINE FRACTURES
Burst fractureQ • Specific compression fracture type in which a posterosuperior bone fragment is displaced into spinal canal
• 50% cases have resultant neurological injury
CONCEPTUAL REVIEW OF RADIOLOGY

• Result of axial compression and flexion force


• Acquired coronal cleft vertebra–coronally oriented burst fracture
• MRI is best for assessment of neurological injury
Chance fractureQ/Lap • Fulcrum fracture of lumbar spineQ
seat belt fractureQ • Chance fracture–Horizontal splitting of spinous process and pedicle, continuing through the posterior
vertebral body up to the superior end plate
• Horizontal splitting fracture–Same as Chance fracture except involvement of end plate
Compression fractures • Occur most commonly at T11-T12-L1 levels
• Most common fractures of lumbar spine
• Grandma fractureQ–Spontaneous insufficiency fracture in an osteoporotic bone in elderly
• Step defect–buckling of anterior cortex of vertebral body near upper end plate
• Wedge deformity–may create a kyphosis at that site
Kummel’s diseaseQ/ • Due to a complicating avascular necrosis following trauma, resulting in a progressive compression
Delayed post-traumatic deformity
vertebral collapseQ • Intravertebral vacuum phenomenon–may be seen
Transverse process • 2nd most common fractures of lumbar spine
fractures • L2–L3 are most commonly involved

Glenohumeral Joint Dislocation Refer to


Normally the Q. No. 48 in Image-Based Questions
Acromion
(yellow arrow) Anterior Shoulder Dislocation
and lateral end ƒƒ Most common dislocation of shoulderQ
of clavicle (blue
ƒƒ Subcoracoid type is most common type of anterior
arrow) must
dislocationQ–humeral head lies in subcoracoid position
be perfectly
aligned – if
not as in this   Clinical Pearls
image suggests
– Acromio- Recurrent Anterior Shoulder Dislocation—Associated Injuries
clavicular Recurrent anterior dislocation is associated with:
dislocation • Hill-Sachs defectQ/Hatchet defectQ–Impaction injury on pos-
terolateral aspect of humeral head
• Bankart lesionQ–Avulsion injury of anteroinferior glenoid rim
• Greater tuberosity fracture
• HAGL lesionQ–Humeral avulsion of Glenohumeral ligament
SHOULDER GIRDLE–FRACTURE AND
DISLOCATIONS
Posterior Shoulder Dislocation
ƒƒ Less common
Clavicle Fracture ƒƒ Associated with Triple E SyndromeQ– Epilepsy - Electric
ƒƒ Most common skeletal injury during childbirthQ shock - Extreme trauma
ƒƒ AP projection with 15 degrees cephalad angulationQ–Best ƒƒ Important imaging findings:
Skeletal Trauma

radiographic projection •• Rim signQ—Widening of joint space >6 mm


ƒƒ Allman classification used in clavicular fractures. •• Trough line signQ–Double articular surface line
•• Light bulb signQ–Typical appearance of humeral head
due to posterior rotation
  Clinical Pearls
•• Vacant glenoid signQ–Lack of close contact at anterior
NEER ClassificationQ - In Humerus fractures joint margin
Two important components of classification are: •• Tennis racquet appearanceQ–Cystic appearance of
ii. Number of fracture parts and humeral head
ii. Fragments displacement Inferior shoulder dislocation/Luxatio erectaQ—Seen in hyper-
abduction injury
Classified as one part, Two part, Three part and Four part fractures
SLAP lesionQ–Superior Labrum Anterior to Posterior tear. MRI
300 One part fractureQ–is most common type
is best modality
ƒƒ Fracture line–may not be seen in many patients in the
Perilunate immediate post-trauma radiograph. In presence of strong
dislocation – clinical suspicion, the hand is immobilized and a repeat
The Capitate

HIGH-YIELD TOPICS
radiograph is obtained after 1–2 weeks.
(red arrow) •• Avascular necrosis:
is dorsal to  Scaphoid has dual blood supply. A small artery
the Lunate
supplies the distal pole and tuberosity, while a
(yellow
larger artery supplies the rest of the scaphoid
arrow) -
Apple is out  More proximal the fracture–higher probability of
of the cup! avascular necrosisQ
 Necrosed fragment will appear dense/sclerotic
with fragmentationQ
•• Non-union–may occur. Herbert screw techniqueQ
used for treatment
•• Scapholunate dissociation–
 Due to disruption of scapholunate ligament
 Terry-thomas signQ/Madonna signQ–widened
Acromioclavicular Joint Separation scapholunate joint space
 Ring signQ–shortened scaphoid due to rotation
ƒƒ The inferior and superior clavicular margin should be
perfectly aligned with the acromionQ. Any step-off suggests
malalignment.
Lunate Dislocation—MC Carpal
Dislocation
Scaphoid FractureQ (Recent Pattern 2018/ ƒƒ Results from a hyperextension injury
AIIMS May 2018) Refer to Q. No. 29 in ƒƒ Pie signQ– Triangular appearing lunate on AP radiograph
Image-Based Questions ƒƒ On lateral view–the lunate tilts forward and anterior

Most common carpal bone to be fracturedQ


ƒƒ
Perilunate Dislocation
ƒƒ Most common site for occult fractureQ–not apparent on
initial radiograph ƒƒ All carpal bones are displaced dorsally except LunateQ–
ƒƒ Waist of scaphoidQ–Most common site which maintains its normal alignment with radius
ƒƒ Frontal projection of wrist with Ulnar deviationQ of hand– ƒƒ Best seen on lateral projectionQ
Best radiographic projection ƒƒ Commonly associated with Scaphoid fractureQ.

CO Lunate versus Perilunate Dislocation: The Apple-Cup-Saucer Analogy


NC E P T

Normal–on a Lateral wrist radiograph Lunate dislocationQ Perilunate dislocationQ

Skeletal Trauma

Radius–Lunate: Capitate are in a straight Lunate dislocated anteriorly. The concavity Concavity of Lunate is empty. The capitate
line–like an apple- in a cup–sitting on a of the lunate is empty. Radius and Capitate is displaced posteriorly–thus the Apple is
saucer remain in a straight line. The cup spills! out of the cup. Commonly associated with
Scaphoid fracture

301
Gamekeeper’s Intra-articular
fracture – fracture involving
Avulsion fracture the proximal end
CONCEPTUAL REVIEW OF RADIOLOGY

of Ulnar collateral of 1st metacarpal


ligament at the 1st carpo-
involving the metacarpal joint –
proximal phalanx Bennett’s fracture
of thumb

Fracture involving
the distal third of
radius with typical
dorsal angulation
of distal fracture
fragments
Diffuse osteopenia is
noted in this elderly
patient along with
fracture of distal
end of ulna too.

Other Upper Limb Fractures and Dislocations

Bar room • 4th/5th metacarpal neck fracture with Gamekeeper’s • Avulsion fracture of Ulnar collateral
fractureQ anteriorly displaced metacarpal head fractureQ ligament attached to proximal phalanx of
Barton’s • Posterior rim fracture of distal radius thumb at 1st metacarpophalangeal joint
fractureQ/Rim Kocher’s • Osteochondral fracture of Capitellum
fractureQ fractureQ
Bennett’s • Intra-articular fracture through base of 1st Little Leaguer’s • Avulsion of medial epicondyle of Humerus
fractureQ metacarpal elbowQ
Boxer’s fractureQ • 2nd/3rd Metacarpal neck/shaft fracture Monteggia • Fracture of proximal ulnar shaft with
Chauffeur’s • Fracture of Radial styloid process fractureQ dislocation of radial head
fractureQ Moore’s • Colles’ fracture + Ulnar styloid fracture +
Chisel fractureQ • Vertical fracture of Radial head, extending fractureQ Ulnar dislocation
around 10 mm from articular surface Nightstick • Fracture of Ulna shaft
Colles fracture Q
• Fracture of distal end of Radius with fracture/Parry
posterior/Dorsal angulation of fracture fractureQ
fragment Reverse Barton’s • Anterior rim fracture of distal end Radius
fractureQ
De Quervain’s • Anterior dislocation of Lunate, along with
fracture- Scaphoid fracture Rolando’s • Comminuted Bennett’s fracture
dislocationQ fractureQ
Drooping • Inferior subluxation of humeral head due Separated • Acromioclavicular joint dislocation
Skeletal Trauma

shoulderQ to hemarthrosis/infection/joint effusion, shoulderQ


etc. Sideswipe • Fracture involving distal humerus,
fracture/Traffic proximal Radius and Ulna–occurs when
Essex-Lopresti • Radial head comminuted fracture with
fracture/Baby elbow protruded from a car’s window is
fractureQ distal radioulnar dislocation
car fractureQ hit by an object
Fisher’s fractureQ • Dorsal avulsion of Triquetrum Smith’s fracture/ • Fracture of distal end of Radius with
Galeazzi’s • Fracture of distal third of Radius with Reverse Colles’ anterior/Ventral angulation of fracture
fracture/ dislocated distal radio-ulnar joint fractureQ fragment
Piedmont’s • Refer to Q. No. 18 in Image-Based Volar plate • Fractured anterior articular margin at
fractureQ Questions fracture Q base of middle phalanx
302 Contd...
RIB FRACTURES – CT IS IOC
CT Pelvis Axial
ƒƒ Golfer’s fractureQ–Rib fracture at its lateral margin.
Bone window
Occasionally a golfer may hit the ground instead of the
image – shows

HIGH-YIELD TOPICS
ball. This abrupt termination of motion/jerk may cause this a comminuted
fracture fracture involving
ƒƒ Passion fracture/Bear hug fractureQ–Seen in the recipient the left iliac blade
of a over enthusiastic hug (forceful Jadu-ki-zappi)
ƒƒ Cough fracture/Post-tussive fractureQ–Stress fractures of
6th/7th anterior end of ribs may be seen following excessive
prolonged coughing
ƒƒ Rower’s fractureQ–Stress fractures of ribs in Boat rowers.
PBH AP radiograph
– Shows a right
superior & inferior
PELVIC FRACTURES pubic rami fracture
(yellow arrows).
Bucket handle Superior and inferior pubic ramus
Diffuse osteopenia
fractureQ fracture with separation of contralateral
is noted in the
SI joint
form of prominent
Dashboard fractureQ Posterior acetabular rim fracture trabecular markings
Duverney’s fracture Q
Iliac wing fracture (blue arrows) and
a calcified fibroid is
Explosion fractureQ Central acetabular fracture–Most seen (red arrow)
common acetabular fracture
Malgaigne’s Superior and inferior pubic ramus
fractureQ fracture with fracture/separation of
ipsilateral SI joint PBH AP
Prussian’s diseaseQ Myositis ossificans of Adductor muscles radiograph –
due to excessive horseback riding shows a Left neck
of femur fracture
Rider’s boneQ Avulsion of secondary bone center of – Subcapital type
Ischium, that subsequently enlarges (yellow arrow).
Sprung pelvis/open Separation of both sacroiliac joints and Note that the
book fractureQ Pubic symphysis Shenton’s curve
Straddle fractureQ Bilateral superior pubic rami and is disrupted (blue
ischiopubic fractures arrow)

Suicidal jumper’s Horizontal fracture through S1/S2 sacral


PBH AP
fractureQ segments
radiograph
– show
Intertrochanteric
LOWER LIMB FRACTURES femur fractures
(yellow arrows)
with disruption of
Fracture Neck of Femur/Intracapsular Shenton’s curve
Femur FractureQ – (AIIMS May 2017) (blue arrow)
ƒƒ According to location, they may be
•• SubcapitalQ–Most common
•• Midcervical ƒƒ X-ray Findings:
Skeletal Trauma

•• Basicervical •• Disruption of Shenton’s line/curveQ


•• Femur in flexion and external rotationQ
  Clinical Pearls •• Impaction of fragments may cause a linear hyperdensity
along the neck
Garden classificationQ of Sub-capital femur fractures: ƒƒ CTQ–Best investigation
• Stage 1–Undisplaced, Incomplete fracture ƒƒ Avascular necrosisQ:
• Stage 2–Undisplaced, complete fracture •• Due to disruption of medial and lateral femoral
• Stage 3–Complete fracture however with apposed surfaces circumflex arteriesQ along the femoral neck
and partial displacement
•• Usually manifests around few months after the fracture
• Stage 4–Complete fracture with full displacement
•• Delbert classificationQ–correlates with risk of AVN
303
Extracapsular Fracture Femur
ƒƒ They may be either trochanteric, intertrochanteric or sub-
trochanteric fractures
CONCEPTUAL REVIEW OF RADIOLOGY

ƒƒ Intertrochanteric fracturesQ:
•• Most common extracapsular fractureQ
•• Usually comminuted
ƒƒ Usually treated with a dynamic hip screw

Hip Dislocation
ƒƒ Anterior hip dislocation–
•• Less common
•• Caused by forced abduction and external rotation of
femur
•• Femoral head lies near the obturator foramen, caudal Important Lower Limb Fractures/
and medial to acetabulum
Injury Patterns
ƒƒ Posterior hip dislocationQ –
•• More commonQ Aviator’s • Fracture of neck of Talus
•• Occurs as a result of Dashboard injury in a flexed hip astragalusQ/ • Complicated with avascular necrosis
•• Associated with a posterior acetabular rim fractureQ Aviator’s
fractureQ
(90%)
•• Sciatic nerve paralysisQ (usually involving the peroneal BB fractureQ • Both Bones of leg/forearm fractured
branchQ)–Most common complication of posterior hip together–Tibia + Fibula/Radius + Ulna
dislocation Beak fractureQ • Avulsion of posterior calcaneal tubercle
Bedroom • Phalangeal fracture that results from
Slipped Femoral Capital Epiphysis (SFCE) – fractureQ striking of foot on an object
Bimalleolar • Medial and lateral malleolar fracture
Type I Salter-Harris Injury fractureQ
ƒƒ Slipping of neck of femur over the head as the head remains Bipartite patellaQ • Not a fracture
in the acetabulum • Accessory ossification centers seen
ƒƒ Occurs at 10–15 years age, in Obese children exclusively on the SUPERO-LATERAL
ƒƒ Knee pain presenting before hip painQ–due to irritation of aspect of Patella
genu branch of Obturator nerve • Most are bilateral
ƒƒ X-ray features – Boot-top • Tibial fracture in adults–adjacent to the
•• Bilateral APQ and Frog leg projectionsQ are useful fractureQ upper end of high-top boots
•• Posterior medial slippage of epiphysisQ–most common Bosworth • Fracture dislocation of ankle–with
finding fractureQ fracture of Fibula and posterior
•• Reduced epiphyseal heightQ dislocation of Talus. Now an out-dated
term
•• Medial epiphyseal beaking
•• Klein’s lineQ–A line drawn along the superior lateral Bumper/Fender • Medial/lateral tibial plateau fracture from
fractureQ severe varus/valgus stress
femoral neck cortex.
 Normally–it should intersect a small portion of Chopart’s • Mid-tarsal dislocation
lateral aspect of femoral epiphysis dislocationQ • Dislocation between the talonavicular
and Calcaneonavicular joint
 SFCE–Because the epiphysis has slipped medially–
the Klein’s line does not intersect it but passes Cotton’s fracture/ • Three part fracture of Ankle involving
Skeletal Trauma

tangentially–Trethowan’s signQ Trimalleolar the medial and lateral malleolus and


fractureQ the posterior aspect of Tibial plafond–
•• Herndon’s humpQ–lateral buttressing on the femoral
referred as Posterior malleolus
neck
•• Capener’s signQ–Metaphysis lateral to acetabulum Dupuytren’s • Distal fibula fracture around 5–7 cm
fractureQ proximal to the lateral malleolus,
•• Pistol-grip deformity of proximal femurQ–curved disruption of distal tibiofibular ligament,
contour of proximal femur lateral talar dislocation, up-and-out foot
ƒƒ Degenerative joint diseaseQ—Most common complication displacement
Flake fractureQ • Following patellar dislocation,
an osteochondral fragment that
separates from its medial articular facet
304 Contd...
Floating kneeQ • Supracondylar femur fracture + Proximal Tillaux’s fractureQ • Fractures of medial malleolus, anterior
tibia fracture tibial tubercle and lateral malleolus with
Lipohemarthro- • Fat-Blood horizontal fluid level seen in diastasis of distal tibiofibular joint

HIGH-YIELD TOPICS
sisQ the suprapatellar bursa of knee joint Toddler’s • Distal tibial metaphyseal fracture in an
due to an intra-articular fracture. Seen fractureQ infant
on a horizontal beam cross table lateral Trampoline • Proximal tibial metaphysis fracture in
radiograph fractureQ children
• FBI signQ–Fat-Blood Interface sign
Triplane/Triplanar • Distal tibial fracture in adolescent
• Associated with intra-articular
fractureQ patients. As the physeal plate fusion
fracture–100%
begins medially, the fracture involves the
Lisfranc’s • Tarsometatarsal fracture - dislocation lateral aspect of growth plate–considered
fracture/Lisfranc’s • Dorsal dislocation of metatarsal bases a Type IV Salter-Harris injury
dislocationQ with associated fractures
Wagstaffe–Le • Avulsion fracture of medial aspect of
Maisonneuve’s • Proximal fibular fracture due to inversion Forte fractureQ distal fibula at attachment of anterior
fractureQ and external rotation injury at ankle tibiofibular ligament
March fracture/ • Stress fracture of 2nd/3rd Metatarsals
Deutschlanders
fractureQ Patellar fracture
Pilon fractureQ • Fracture of distal tibia due to an axial – The fracture
force driving the talus into the tibial fragments are
plafond always widely
separated and
Pott’s fractureQ • Distal fibular fracture with disruption of
require treatment
distal tibiofibular ligament
by Tension band
Reverse segond • Avulsion fracture of medial proximal tibia wiring
fractureQ at the insertion of deep fibers of Medial
collateral ligament/Coronary ligament/
Meniscotibial ligament
Segond’s • Avulsion fracture of lateral tibial condyle at Lover’s fracture:
fractureQ insertion of tensor fascia lata Fracture involving the
• Always associated with ACL tearsQ, Calcaneum seen on
meniscal tearsQ lateral projection (inset)
Skier’s fracture/ • Spiral fracture of distal tibial and fibular and axial projections
Boot top diaphysis/metaphysis (yellow arrows).
Compare with normal
fractureQ
right Calcaneum (blue
Stieda fractureQ • Avulsion fracture at medial collateral arrow)
ligament insertion site at the medial
femoral condyle
Contd...

  Clinical Pearls
Jones’ and Pseudo-Jones’ fracture

Jones’ fracture Pseudo-Jones’ fracture


Affected zone Zone 2 Zone 1
Involves Neck of 5th metatarsal Avulsion fracture of tuberosity
of 5th metatarsal – due to pull
Skeletal Trauma

of peroneus brevis tendon


Radiograph

305
Ankle AP & Lateral A Postoperative Hip
radiographs – Trimalleolar replacement patient –
fracture – along with shows extensive soft
CONCEPTUAL REVIEW OF RADIOLOGY

the medial and lateral tissue ossification on


malleoli the posterior lateral aspect of hip
tibial cortex is also – Myositis ossificans
fractured – considered as (yellow arrow)
the 3rd malleolus

MYOSITIS OSSIFICANS •• CT Brain/USG through anterior fontanelle for


suspected SDH
ƒƒ Heterotopic bone formation in the soft tissues following •• Abdominal USG/CT to rule out contusions/free fluid
traumaQ ƒƒ Imaging findings from head-to-toe:
ƒƒ Do not touch lesionQ–It resembles aggressive neoplastic •• Skull fracturesQ:
lesion and biopsy may also have a sarcomatous appearance  A non-parietal region fracture suggests possibility
in early stages of abuse
ƒƒ Pellegrini-Stieda diseaseQ–Ossification at the distal torn  Fracture extending across sutures with diastatic
attachment of medial collateral ligament of knee sutures
ƒƒ Prussian’s diseaseQ–Ossification in Adductor magnus in  Egg-shell fractures of skull
horse riders, due to a constant pressure of the saddle on the •• Multiple subdural hemorrhagesQ - Most common in
adductors subtemporal regionsQ and in different stages
ƒƒ Lumbar ossified bridge syndromeQ–Ossification bridging •• Cerebral edemaQ
the gap between 2 lumbar transverse process, following a •• Cervical spine compression injuriesQ
psoas muscle hematoma •• Skeletal fractures in various stages of healingQ
ƒƒ Radiographic features: •• Multiple closely approximated fracturesQ
•• Hazy soft tissue mass with cloudy calcification–in early •• Multiple rib fracturesQ–Most common in the lateral
stages and posterior portionQ
•• Rounded/linear calcification develops •• Metaphyseal corner fracturesQ–avulsion fractures
•• Smooth dense outer borderQ, with a lucent centerQ, of metaphyseal corners. Usually the proximal tibia,
seen separate from the adjacent boneQ medial aspect of distal femur and bilateral. Hallmark of
•• Calcified mass may be seen adjacent to diaphysis of non-accidental injuryQ.
a long bone, separate from the bone, with the bone •• Bucket handle fracture of metaphysisQ–Similar to
cortex being intact. corner fractures but a rim of bone across the metaphysis
is avulsed.
•• Epiphyseal displacementsQ
NONACCIDENTAL INJURY/BATTERED •• Exuberant periosteal new bone formationQ
•• Abdominal visceral injuriesQ
BABY SYNDROME
ƒƒ Also known as Parent-Infant traumatic stress syndrome/ 3
ƒƒ
Whiplash shaken infant syndrome
Characterized by clinical and radiologic evidence of
CLINICAL QUIZ - SOLUTION
Skeletal Trauma

repeated injury, usually in the first 2–3 years of life. Presents This is the classical tear-drop sign of Orbital floor fracture—
most commonly at around 6 months of ageQ. fracture involving the infra-orbital plate of maxillary bone. The
ƒƒ Investigations done: inferior orbital rim may be intact. It result from blow of fist/ball
•• A simple babygram/infantogram is inadequate and directly over the globe or just caudal to it. Orbital contents are
may not reveal fractures pushed down into the fracture defect and get trapped–most
commonly Inferior rectus muscleQ
•• A detailed skeletal surveyQ should be done if suspected
including–Skull, Chest X-ray with oblique’s for ribs,
Spine X-ray, Limb X-rays AP-LAT

306
Multiple Choice Questions

HIGH-YIELD TOPICS
Tuberculosis 13. Most common complication of TB meningitis is:
1. The most common site of Primary infection by Mycobac- A. Basal meningeal enhancement
terium tuberculosis is: B. Hydrocephalus
A. Lungs B. Brain C. Basal ganglia infarcts
D. Convulsions
C. Abdomen D. Spine
14. Central T2 hypointensity is a characteristic imaging feature
2. Ghon’s complex involves:
of:
A. Lung granuloma
A. Neurocysticercosis
B. Hilar lymph nodes
B. Tuberculous granuloma with solid center
C. Intervening lung lymphatics
C. Tuberculous granuloma with liquefied center
D. All of the above
D. Tubercular abscess
3. Tree in bud appearance suggestive of endobronchial spread
15. The most commonly involved site in Musculoskeletal TB is:
of infection is seen on:
A. Spine B. Knee Joint
A. Chest radiograph B. HRCT
C. Hip joint D. Quadriceps muscle
C. MRI D. PET scan
16. Pott’s abscess is:
4. Miliary nodules on a CXR are seen in all except:
A. Popliteal fossa abscess B. Gluteal abscess
(NEE Pattern Dec 2012)
C. Paravertebral abscess D. Retropharyngeal abscess
A. TB B. Sarcoidosis
17. Spina ventosa is:
C. Klebsiella pneumonia D. Metastasis
A. Ventral spinal TB B. Ventricular TB
5. Which of the following imaging finding is considered the
C. Tuberculous dactylitis D. Ventral spina bifida
Hallmark of Postprimary TB?
18. Autonephrectomy is a feature of:
A. Lung granuloma B. Lymphadenopathy
A. Renal sarcoid
C. Cavitation D. Fibrosis
B. Renal TB
6. True regarding Simon’s focus seen in TB is:
C. Renal artery stenosis
A. Apical lung nodule
D. Xanthogranulomatous pyelonephritis
B. Calcified Ghon’s focus
C. Calcified hilar lymph node Hydatid Cyst
D. Liver parenchymal TB lesion
19. Floating Water lily sign on CXR is seen in:
7. Rich’s focus of TB is seen in: A. Lung abscess B. Hydatid cyst
A. Lungs B. Brain C. Pulmonary hamartoma D. Lung cancer
C. Liver D. Kidney
20. Metabolically the most active layer of the Hydatid cyst wall
8. Radiological features of Abdominal TB are all except: is:
 (AI 2001) A. Pericyst B. Ectocyst
A. Ileocecal junction is least commonly affected site. C. Endocyst D. Capule
B. Rapid emptying of narrowed terminal ileum into shortened 21. Most common site of involvement by Hydatid cyst in the
rigid caecum. body in adults is:
C. Rigid contracted cone-shaped caecum. A. Lungs B. Liver
D. Omental cake like mass with separation and fixation of

Multiple Choice Questions


C. Brain D. Heart
bowel loops
22. Most common site of involvement by Hydatid cyst in the
9. Pulled-up Cecum is seen in: (AI 2013) body in children is:
A. TB B. Ca cecum A. Lungs B. Liver
C. Intussusception D. Ca Colon C. Brain D. Heart
10. Most common manifestation of abdominal TB is: 23. Which of the following type of Hydatid cyst denotes a
A. Ileocecal junction thickening complicated cyst?
B. Lymphadenopathy A. Type I B. Type II
C. Ascites C. Type III D. Type IV
D. Intestinal obstruction 24. According to the WHO classification of Hydatid cyst based on
11. Ring enhancing lesion on CT brain is a feature of: USG appearance, which of the following is the active stage
 (Recent Pattern Dec 2016) of the cyst?
A. Tuberculoma B. Intracranial hemorrhage A. Stage CL B. Stage CE1
C. Cysts D. Brain hamartoma C. Stage CE2 D. Stage CE3
12. Best imaging modality for diagnosis of Tubercular meningitis 25. Which of the following layer of the cyst wall is absent in a
is: skeletal Hydatid cyst?
A. CE-CT B. CE-MRI A. Pericyst B. Ectocyst
C. PET D. SPECT C. Endocyst D. None of the above is absent 307
26. The PAIR technique can be used for treatment of the 43. Dancer’s fracture occurs due to avulsion at insertion site of:
following stage of Liver hydatid cyst except: A. Peroneus longus B. Peroneus brevis
A. CE1 B. CE2 C. Peroneus tertius D. Tibialis posterior
CONCEPTUAL REVIEW OF RADIOLOGY

C. CE3 D. CE4 44. Pellegrini-Stieda lesion is due to avulsion of:


A. Medial collateral ligament
Skeletal Trauma B. Lateral collateral ligament
27. Banana fracture is a: C. Tensor fascia lata
A. Traumatic fracture B. Pathological fracture D. Patellar tendon
C. Stress fracture D. Avulsion fracture 45. Trampoline fracture is a fracture of:
28. Best investigation for detection of stress fracture is: A. Tibia B. Fibula
A. X-ray B. CT scan C. Calcaneum D. Talus
C. Bone scan D. MRI 46. Slap lesion involves:
29. Rower’s fracture involves: A. Zygomatic process
A. Metacarpals B. Phalanges B. Superior labrum of glenoid
C. Ribs D. Clavicles C. Inferior labrum of glenoid
30. Pediatric growth plate region fracture involves both the D. Glenohumeral ligament
Metaphysis and Epiphysis: 47. Which modality is the mainstay in trauma imaging?
A. Salter-Harris type 1 B. Salter-Harris type 2 A. Plain radiograph B. CT
C. Salter-Harris type 3 D. Salter-Harris type 4 C. MRI D. Radionuclide imaging
31. Pond fracture is seen in: 48. Facial and skull fractures are best depicted by:
A. Children B. Infants A. Noncontrast CT
C. Adolescents D. Elderly B. Helical CT with 3D reconstruction
32. Tripod fracture of face involves: C. MRI
A. Zygomatic arch B. Orbital process D. Scintigraphy
C. Maxillary process D. All of above 49. Open book appearance is seen in:
33. Clay Shoveler fracture involves: A. Spinal injury B. Pelvic girdle injury
A. C5 transverse process B. C7 transverse process C. Shoulder girdle injury D. Hand injury
C. C7 pedicle D. C7 spinous process 50. Best investigation for traumatic paraplegia is:
34. Most common bone fractured during childbirth is: A. CT scan B. MRI
A. Skull B. Clavicle C. X-ray spine D. Myelography
C. Pelvis D. Humerus 51. Investigation of choice for a lesion of temporal bone is:
35. Neer classification is used for fractures of: A. CT B. MRI
A. Radius B. Ulna (Recent Pattern 2018) C. USG D. Plain X-ray
C. Humerus D. Scapula 52. The hallmark of nonaccidental injury is:
36. Terry Thomas sign is seen in: A. Diaphyseal fractures
A. Scaphoid fracture B. Lunate dislocation B. Metaphyseal corner fractures
C. Perilunate dislocation D. Scapholunate dissociation C. Epiphyseal fractures
37. A comminuted Bennett’s fracture is known as: D. Greenstick fractures
A. Bennett’s 2 fracture B. Smith’s fracture 53. Synonym or non-accidental trauma is:
C. Barton’s fracture D. Rolando’s fracture A. Shaken baby syndrome
38. A positive fat pad sign at elbow suggests: B. Whiplash shaken infant syndrome
A. Joint dislocation B. Intra-articular hemarthrosis
Multiple Choice Questions

C. Parent infant traumatic stress syndrome


C. Joint capsule rupture D. Cartilage loss D. All of the above
39. Suicidal jumper’s fracture involves: 54. Imaging findings of non-accidental injury include all except:
A. Calcaneum B. Tibia A. Posterior rib fractures
C. Femur D. Sacrum B. Skull fractures
40. Trethowans sign involving the Klein’s line is a feature of: C. Multiple epidural hematomas
A. Fracture neck of femur D. Metaphyseal corner fractures
B. Avascular necrosis femur head 55. All of the following fractures are highly suspicious of child
C. Slipped femoral capital epiphysis abuse except:
D. Patellar dislocation A. Bucket-handle metaphyseal fractures
41. Aviator’s astragalus involves: B. Rib fractures
A. Talus B. Calcaneus C. Occipital bone impression fractures
C. Navicular D. Cuboid D. Sacral fractures
42. Bipartite patella is a finding seen at what part of Patella? 56. Differential diagnosis of nonaccidental injury is all except:
A. Superomedial B. Superolateral A. Osteogenesis imperfecta B. Scurvy
C. Apex D. Mid portion C. Caffey’s disease D. Osteopetrosis

308
Answers
1. A 8. A 15. A 22. A 29. C 36. D 43. B 50. B
2. D 9. A 16. C 23. D 30. D 37. D 44. A 51. A

HIGH-YIELD TOPICS
3. B 10. B 17. C 24. C 31. B 38. B 45. A 52. B
4. B 11. A 18. B 25. A 32. D 39. D 46. B 53. D
5. C 12. B 19. B 26. D 33. D 40. C 47. A 54. C
6. A 13. B 20. C 27. B 34. B 41. A 48. B 55. D
7. B 14. B 21. B 28. D 35. C 42. B 49. B 56. D

Explanations to Questions
12. Basal cisternal enhancement is often seen in TB meningitis 28. Both MRI and Bone scan can identify stress fracture but
and CT scan has lower resolution for skull base as compared MRI has better image resolution and can also rule out many
to MRI. PET and SPECT have more role in neoplasms and similar disorders and hence is considered better.
degenerative disorders of brain. 47. Though not sensitive for subtle undisplaced or early fractures,
14. Central T2/ FLAIR hypointensity is a characteristic feature of plain radiographs even today due to their widespread
CNS tuberculoma with solid centre and caseation. It will show availability, less radiation (as compared to CT scan), mobility
ring enhancement on post contrast images. As the center (can be taken in any OPD/IPD/ICU) remain the mainstay of
becomes liquified – it appears hyperintense on T2Wimages trauma imaging.
(as fluid is hyperintense on T2W images) 50. In traumatic paraplegia we want comment on the status of
23. Type IV hydatid cysts are complicated. They can be due to spinal cord or nerve roots as well as anatomical relation of
Internal cyst rupture or external. It is probably related to osseous spinal structures. Both of which can be provided by
decreasing intracystic pressure, degeneration, host response, MRI.
trauma or response to medical therapy and percutaneous 51. HRCT Temporal bone is the preferred modality for temporal bone
drainage. Internal rupture causes death of the parasite. evaluation. MRI is better for evaluation for internal auditory
26. Percutaneous treatment of LHCs introduced in the mid-1980s canal and evaluation of cochlear nerve in congenital deafness
has become an attractive alternative to surgery and medical 56. D/D of NAI consists all pediatric diseases where there is mostly
management. It is used ideally in CE1 and CE3a. May be tried normal bone pattern and multiple fractures. Osteopetrosis
in CE2. However, it is never to be used in complicated hydatid consists of diffuse increase in sclerosis of all bones and
cysts – CE4. pathological fractures can be easily differentiated from NAI.

Explanations to Questions

309
_______________________________________________________________________________________________________________________

_______________________________________________________________________________________________________________________

_______________________________________________________________________________________________________________________

_______________________________________________________________________________________________________________________

Note
_______________________________________________________________________________________________________________________

_______________________________________________________________________________________________________________________

_______________________________________________________________________________________________________________________

_______________________________________________________________________________________________________________________

_______________________________________________________________________________________________________________________

_______________________________________________________________________________________________________________________

_______________________________________________________________________________________________________________________

_______________________________________________________________________________________________________________________

_______________________________________________________________________________________________________________________

_______________________________________________________________________________________________________________________

_______________________________________________________________________________________________________________________

_______________________________________________________________________________________________________________________

_______________________________________________________________________________________________________________________

_______________________________________________________________________________________________________________________

_______________________________________________________________________________________________________________________

_______________________________________________________________________________________________________________________

_______________________________________________________________________________________________________________________

_______________________________________________________________________________________________________________________

_______________________________________________________________________________________________________________________

_______________________________________________________________________________________________________________________

_______________________________________________________________________________________________________________________

_______________________________________________________________________________________________________________________

_______________________________________________________________________________________________________________________

_______________________________________________________________________________________________________________________

_______________________________________________________________________________________________________________________

_______________________________________________________________________________________________________________________

_______________________________________________________________________________________________________________________

_______________________________________________________________________________________________________________________
NORMAL IMAGING
ATLAS—BRAIN,
CHEST AND ABDOMEN
C hapter O utline
• Brain Sectional Anatomy: Representative—Flair MRI Images
CONCEPTUAL REVIEW OF RADIOLOGY

• CT Chest Anatomy
• CT Anatomy Abdomen

Brain Sectional Anatomy


REPRESENTATIVE FLAIR MRI IMAGE (LEFT)
AND NONCONTRAST CT SCAN IMAGE (RIGHT)

Axial Section at the Level above Corpus Callosum

Axial Section at the Level of Body of Corpus Callosum


Brain Sectional Anatomy

312
Axial Section at the Level of Corpus Callosum

NORMAL IMAGING ATLAS—BRAIN, CHEST AND ABDOMEN


Axial Section at the Level of Thalamus

Axial Section at the Level of Third Ventricle

Brain Sectional Anatomy

313
Axial Section at the Level of Midbrain
CONCEPTUAL REVIEW OF RADIOLOGY

Axial Section at the Level of Pons

Axial Section at the Level of Medulla


Brain Sectional Anatomy

314
Midsagittal Section of Brain Coronal Section of Brain at Pituitary Level

NORMAL IMAGING ATLAS—BRAIN, CHEST AND ABDOMEN


Midsagittal Section of Brain Venous Anatomy Coronal Section of Brain at Posterior Level

Brain Sectional Anatomy

315
CT Chest Anatomy
CONCEPTUAL REVIEW OF RADIOLOGY

Axial CT Lung Window at the Level of Trachea Axial CT Lung Window at the Level of Heart

Axial CT Lung Window at the Axial CT Lung Window at the


Level of Carina Level of Diaphragm
CT Chest Anatomy

Abbreviations: Ant, anterior; LLL, left lower lobe; LUL, left upper lobe; Post, posterior; RLL, right lower lobe; RML, right middle lobe;
RUL, right upper lobe; seg, segment

316
Coronal CT Lung Window Anteriorly Sagittal CT Lung Window Right

NORMAL IMAGING ATLAS—BRAIN, CHEST AND ABDOMEN


Coronal CT Lung Window Middle Sagittal CT Lung Window Left

Coronal CT Lung Window Posteriorly

CT Chest Anatomy

317
Axial CT Mediastinal Window Axial CT Mediastinal Window at
Just Above Aortic Arch the Level of Heart
CONCEPTUAL REVIEW OF RADIOLOGY

Axial CT Mediastinal Window at Midsagittal CT Mediastinal Window


the Level of Aortic Arch

Axial CT Mediastinal Window at the Level of Carina


CT Chest Anatomy

318
Coronal CT Mediastinal Window Anteriorly Coronal CT Mediastinal Window Posteriorly

NORMAL IMAGING ATLAS—BRAIN, CHEST AND ABDOMEN


Coronal CT Mediastinal Coronal CT Mediastinal Window
Window Midportion Posteriormost

CT Chest Anatomy

319
CT Anatomy Abdomen
CONCEPTUAL REVIEW OF RADIOLOGY

Axial Contrast Enhanced CT Scan of Axial Contrast Enhanced CT Scan of


Abdomen at Level of Diaphragm Abdomen at Level of Renal Arteries

Axial Contrast Enhanced CT Scan of Axial Contrast Enhanced CT Scan of


Abdomen at Level of Celiac Trunk Abdomen at Level of Aortic Bifurcation

Axial Contrast Enhanced CT Scan of Axial Contrast Enhanced


Abdomen at Level of SMA CT Scan of Male Pelvis
CT Anatomy Abdomen

320 Abbreviations: GB, gallbladder; IVC, inferior vena cava; LK, left kidney; SMA, spinal muscular atrophy; SMV, superior mesenteric vein;
RK, right kidney
Coronal Contrast Enhanced CT Scan of Coronal Contrast Enhanced CT Scan of
Abdomen – Anterior Section Abdomen at Level of Aorta

NORMAL IMAGING ATLAS—BRAIN, CHEST AND ABDOMEN


Coronal Contrast Enhanced CT Scan of Coronal Contrast Enhanced CT Scan of
Abdomen at Level of Portal Abdomen at Level of Spine

CT Anatomy Abdomen

321
Right Parasagittal Contrast Enhanced Left Parasagittal Contrast Enhanced
CT Scan of Abdomen CT Scan of Abdomen
CONCEPTUAL REVIEW OF RADIOLOGY

Midsagittal Contrast Enhanced


CT Scan of Abdomen
CT Anatomy Abdomen

322
Appendix
SCIENTISTS AND THEIR IMPORTANT DISCOVERIES IN RADIOLOGY
Remembering names and discoveries (that could be asked as MCQs)—is indeed a nightmare for students preparing for entrance
examinations. Here is a short and relevant list exclusively for Radiology. Each and every scientists enlisted here is a legend and has
contributed to society a great deal. We are indebted to them for their pioneering work.

• Wilhelm Röntgen • German physicist: Produced and detected X-rays (Röntgen rays) in 1895, an achievement that
earned him the first Nobel Prize in Physics in 1901.
• Antoine Henri Becquerel • French physicist and Nobel laureate: Discovered radioactivity.
• Thomas Alva Edison • American inventor and businessman: Discovered that calcium tungstate screens produced brighter
X-ray images than before and based on this, he invented the first commercially available fluoroscope
which enabled real-time moving images of internal structures of the body.
• Frédéric Joliot-Curie and • Artificially produced radionuclides (radioisotopes)—radioactive isotopes used in nuclear medicine
• Irène Joliot-Curie for diagnosis, treatment, and research.
(Daughters of Marie Curie)
• John Lawrence • American physicist and physician: Used phosphorus-32 (radioactive isotope of phosphorus) to
treat leukemia.
• David E Kuhl • American scientist: Invented positron emission tomography (PET)—an imaging technique that
produces a three-dimensional image of the inside of the body by using gamma rays.
• Ian Donald • Scottish physician: Invented the ultrasound and tested with it, one year later, a pregnant woman.
• Charles Dotter • American vascular radiologist: Known as the “Father of Interventional Radiology”; introduced
image-guided medical procedures. For pioneering this technique he was nominated for the Nobel
Prize in Physiology or Medicine in 1978.
• Godfrey Hounsfield and • English electrical engineer and South African-American physicist respectively, invented CT scanner
Allan Cormack and shared the Nobel Prize for Medicine in 1979 for the invention of CT scanning. Computed
tomography (CT) or computed axial tomography (CAT), is a medical imaging method employing
computer processing which is used to generate 3D images of the inside of an object from a large
series of 2D X-ray images.
• Felix Bloch and Edward • The Nobel Prize in Physics in 1952, which went to Felix Bloch and Edward Purcell, was for the
Purcell development of nuclear magnetic resonance (NMR), the scientific principle behind MRI. However,
for decades magnetic resonance was used mainly for studying the chemical structure of substances.
• Raymond Vahan Damadian • Armenian-American medical practitioner and inventor: Built the first commercial MRI scanner—
not awarded the Nobel Prize!
• Paul Lauterbur and Peter • The Nobel Prize in Physiology or Medicine 2003 was awarded jointly to Paul C Lauterbur and
Mansfield Sir Peter Mansfield “for their discoveries concerning magnetic resonance imaging”.
• Lauterbur used the idea of Robert Gabillard (developed in his doctoral thesis, 1952) of introducing
gradients in the magnetic field which allows for determining the origin of the radio waves emitted
from the nuclei of the object of study. This spatial information allows two-dimensional pictures to
be produced.
• Marie and Pierre Curie • Marie and Pierre Curie were married scientists who discovered radium (on December 21, 1898).
They announced their findings a week later, on the 28th of December.
• After forty-five months of additional work, the pair first isolated radioactive radium salts (from
mineral pitchblende) at their Paris laboratory (on the 20th of April, 1902). The following year, they
shared the Nobel Prize in science for their ground-breaking work.
• In 1910, with Debierne, Marie finally succeeded in isolating pure, metallic radium. For this achieve-
ment, she was the sole recipient of the 1911 Nobel Prize in Chemistry, making her the first person
to win a second Nobel Prize.
• Charles Dotter • Father of Interventional Radiology
• Ernest Rutherford • Father of Nuclear Physics
• John Caffey • Father of Pediatric Radiology
_______________________________________________________________________________________________________________________

_______________________________________________________________________________________________________________________

_______________________________________________________________________________________________________________________

_______________________________________________________________________________________________________________________

Note
_______________________________________________________________________________________________________________________

_______________________________________________________________________________________________________________________

_______________________________________________________________________________________________________________________

_______________________________________________________________________________________________________________________

_______________________________________________________________________________________________________________________

_______________________________________________________________________________________________________________________

_______________________________________________________________________________________________________________________

_______________________________________________________________________________________________________________________

_______________________________________________________________________________________________________________________

_______________________________________________________________________________________________________________________

_______________________________________________________________________________________________________________________

_______________________________________________________________________________________________________________________

_______________________________________________________________________________________________________________________

_______________________________________________________________________________________________________________________

_______________________________________________________________________________________________________________________

_______________________________________________________________________________________________________________________

_______________________________________________________________________________________________________________________

_______________________________________________________________________________________________________________________

_______________________________________________________________________________________________________________________

_______________________________________________________________________________________________________________________

_______________________________________________________________________________________________________________________

_______________________________________________________________________________________________________________________

_______________________________________________________________________________________________________________________

_______________________________________________________________________________________________________________________

_______________________________________________________________________________________________________________________

_______________________________________________________________________________________________________________________

_______________________________________________________________________________________________________________________

_______________________________________________________________________________________________________________________

You might also like